You are on page 1of 256

Download From https://iit-jeeacademy.blogspot.

com

MOCK
TESTS
SUPER

JEE MAIN
2018
EBD_7206
Download From https://iit-jeeacademy.blogspot.com

• Corporate Office : 45, 2nd Floor, Maharishi Dayanand Marg, Corner Market,
Malviya Nagar, New Delhi-110017
Tel. : 011-49842349 / 49842350

Compiled and Edited by :


D. P. Gupta (Mathematics)
Preetima Bajpai (Chemistry)
Sanjeev Kumar Jha (Physics)

Typeset by Disha DTP Team

DISHA PUBLICATION
ALL RIGHTS RESERVED

© Copyright Publisher

No part of this publication may be reproduced in any form without prior permission of the publisher and author. The author
and the publisher do not take any legal responsibility for any errors or misrepresentations that might have crept in. We
have tried and made our best efforts to provide accurate up-to-date information in this book.

For further information about the books from DISHA,


Log on to www.dishapublication.com or email to info@dishapublication.com
Download From https://iit-jeeacademy.blogspot.com

Page No.
TREND ANALYSIS FOR JEE MAIN (2007-2017) (i) - (iii)

MOCK TESTS

1. Mock Test - 1 MT-1–12

2. Mock Test - 2 MT-13–24

3. Mock Test - 3 MT-25–38

4. Mock Test - 4 MT-39–50

5. Mock Test - 5 MT-51–62

6. Mock Test - 6 MT-63–74

7. Mock Test - 7 MT-75–84

8. Mock Test - 8 MT-85–96

9. Mock Test - 9 MT-97–108

10. Mock Test - 10 MT-109–120


EBD_7206
Download From https://iit-jeeacademy.blogspot.com

SOLUTIONS

1. Mock Test - 1 MT-121–134

2. Mock Test - 2 MT-135–146

3. Mock Test - 3 MT-147–160

4. Mock Test - 4 MT-161–172

5. Mock Test - 5 MT-173–184

6. Mock Test - 6 MT-185–196

7. Mock Test - 7 MT-197–208

8. Mock Test - 8 MT-209–220

9. Mock Test - 9 MT-221–234

10. Mock Test - 10 MT-235–248


Download From https://iit-jeeacademy.blogspot.com

TREND ANALYSIS FOR


JEE MAIN
(2007-2017)

CHEMISTRY

Years
S.N. Chapters
17 16 15 14 13 12 11 10 9 8 7
1 Some Basic Concepts of Chemistry 2 2 2 3 1 1 1 1 1 2 1
2 Atomic Structure 1 1 1 1 1 1 1 2 2 2 1
3 Periodic Classification and Periodic Properties 1 1 1 2 1 1 2 1 1
4 Chemical Bonding and Molecular Structure 2 2 3 2 3 2 1 2 3
5 States of Matter 1 1 1 1 1 1 2 1 1
6 Equilibrium 1 1 2 2 1 2 1 2 4 1 4 3
7 Thermodynamics 2 1 1 1 1 1 3 2 2 1 3
8 Solid State 1 1 1 1 2 1 1 1 2 1 1 0
9 Solutions and Colligative Properties 1 1 1 2 3 2 2 2 2 3
10 Electrochemistry 1 1 1 4 1 1 1 1 1 1 1 2
11 Chemical Kinetics 1 1 1 1 1 1 1 1 1 1 1 2
12 Nuclear Chemistry 1 2
13 Surface Chemistry 1 1 1 1 1
14 s-block Elements 1 1 2 1 1 4
15 p-Block Elements-Ist 3 1 1 2 2
16 p-Block Elements-IIst 2 3 2 1 2 1 1 1 2
17 Transition and Inner-Transition Elements 1 2 2 1 2 1 3 1 1 2 1 1
18 Coordination Compounds 1 1 2 1 1 1 1 1 2 2
19 Extraction of Metals 2 1 1 1 2 1
20 Qualitative Analysis 1 1 1 1
21 Organic Chemistry Basics 1 1 1 1 4 2 2
22 Hydrocarbons 1 2 1 2 1 1 3 3 2
23 Alkyl Halides 3 3 1 1 1 1 1 1 1
24 Alcohols and Ethers 1 1 1 1
25 Aldehydes and Ketones 2 3 1 1 2 1 2 1
26 Carboxylic Acids and Derivatives 1 1 2 1
27 Aliphatic Compounds Containing Nitrogen 2 1 1
28 Benzene and Alkyl Benzene 2 1
29 Aromatic Compounds Containing Nitrogen 2 1 2 1 2 2
30 Aryl Halides and Phenols 1 1 2 3
31 Aromatic Aldehydes, Ketones and Acids 1
32 Biomolecules and Chemistry in Everyday Life 2 3 3 2 1 3 1 1 1
Total No. of Questions 30 30 30 30 30 30 30 30 30 30 35 40

(i)
EBD_7206
Download From https://iit-jeeacademy.blogspot.com

PHYSICS

Years
S.N. Chapters 11
17 16 15 14 13 12 (Include s 10 9 8 7
RS Paper)

1 Units, Dimensions and Vectors 1 2 1 2 1 1 – 2 1 1 2 0

2 Kinematics 1 – 1 1 2 1 2 3 1 2

3 Laws of Motion 0 1 1 2 2 1 1 1 0 0 1 1

4 Work, Power and Energy 2 1 2 2 1 1 1 1 0 2 1

5 Impulse and Momentum 0 2 2 1 2 1 2 1

6 Rotation 2 2 2 1 2 2 1 1 3

7 Gravitation 1 1 1 1 1 1 1 1 1

8 Simple Harmonic Motion 1 1 1 2 1 2 2 1 3

9 Solids and Fluids 1 – 1 1 1 1 3 2 1 1 3

10 Waves 0 2 1 1 2 1 2 1 1 2 3 2

11 Heat and Thermodynamics 4 3 3 3 2 4 3 4 1 5 1 4

12 Optics 3 3 3 3 4 3 5 3 3 2 4 2

13 Current Electricity 3 1 2 1 8 2 2 3 1 4 1

14 Electrostatics 2 2 3 1 3 2 3 3 4 4 4 5

15 Magnetics 2 3 3 1 2 2 1 1 2 2 4

16 Electromagnetic Induction & AC 1 1 2 1 1 2 2 1 1 1 2

17 Modern Physics 6 7 4 5 1 6 4 4 6 6 4 9

Total No. of Questions 30 30 30 30 30 30 30 30 30 30 35 40

(ii)
Download From https://iit-jeeacademy.blogspot.com

MATHEMATICS

Years
S.N. Chapters
17 16 15 14 13 12 11 10 9 8 7

1 Sets, Relations and Functions 1 1 1 1 1 2 3 1 3 2 2

2 Complex Numbers and Quadratic Equations 1 2 2 3 3 2 2 1 2 3 3 1

3 Permutations and Combinations 1 1 1 3 2 1 1 2 1 2 1

4 Mathematical Induction 1 1 1

5 Binomial Theorem and Its Applications 1 1 1 1 1 1 1 1 1

6 Sequences and Series 2 2 2 2 2 2 1 1 1 1 1 2

7 Exponential and Logarithmic Series 1

8 Matrices and Determinants 4 2 2 2 2 2 2 3 3 2 3 3

9 Statistics and Probability 3 2 2 2 1 2 3 2 3 4 3 3

10 Trigonometry 2 2 2 2 2 1 1 1 2 1 2 2

11 Mathematical Reasoning 1 1 1 1 1 1 1 1 1 2

12 Coordinate Geometry 4 5 5 5 7 5 4 5 3 4 5 7

13 Limits, Continuity and Differentiability 2 2 2 4 1 5 4 4 1 3 2 4

14 Integral Calculus 3 3 3 3 3 3 2 2 2 2 3 4

15 Differential Equations 1 1 1 1 2 1 1 1 1 1

16 Three Dimensional Geometry 2 2 2 2 2 2 2 2 2 1 2 3

17 Vector Algebra 1 1 1 1 1 2 2 2 2 2 2 2

18 Dynamics 1 2

19 Application of Derivatives 1 2 2 1

Total No. of Questions 30 30 30 30 30 30 30 30 30 30 35 40

(iii)
EBD_7206
Download From https://iit-jeeacademy.blogspot.com
Download From https://iit-jeeacademy.blogspot.com

1 JEE MAIN
MOCK TEST
Time : 3 hrs. Max. Marks : 360

INSTRUCTIONS
• Chemistry (120 marks) : Question No. 1 to 30 are of 4 marks each.

• Physics (120 marks) : Question No. 31 to 60 are of 4 marks each.

• Mathematics (120 marks) : Question No. 61 to 90 are of 4 marks each.


• Negative Marking : One fourth (¼) marks will be deducted for indicating incorrect
response of each question. :

H O –
CHEMISTRY
: :
| |
1. Ionization energy of gaseous Na atoms is C. H – C – C
| + O– H
: :

495.5 kJ mol–1. The lowest possible frequency


B
H
of light that ionizes a sodium atom is
(h = 6.626 × 10–34 Js, NA = 6.022 × 1023 mol–1) (a) A (b) B
(c) C
(a) 7.50 × 104 s–1 (b) 4.76 × 1014 s–1
15 –1 (d) All have same energy
(c) 3.15 × 10 s (d) 1.24 × 1015 s–1 3. Which of the following pairs have identical bond
2. Which of the following resonance structure is order ?
lowest in energy?
(a) N 2 , O 22+ (b) N 2 , O2–
:

H O : : H O– : :
| || | ||
A. H–C –C B. H – C – C (c) N 2– , O 2 (d) O 2+ , N 2
| | +
: :

O– H O– H
B
H HB
:
EBD_7206
Download From https://iit-jeeacademy.blogspot.com

MT-2 JEE MAIN

4. In a compound AOH, electronegativity of ‘A’ is ( + Z-1)


9. The energy levels for ZA can be given by
2.1, the compound would be
(a) Acidic (a) En for A(+Z–1) = Z2 × E n for H
(b) Neutral towards acid & base
(c) Basic (b) En for A(+Z–1) = Z × En for H
(d) Amphoteric 1
(c) En for A(+Z–1) = × En for H
5. Which of the following orders is wrong? Z2
1
(a) Electron affinity– N < O < F < Cl (d) En for A(+Z–1) = × En for H
Z
(b) Ist ionisation potential – Be < B < N < O
(c) Basic property– MgO < CaO < FeO < Fe2O3 10. Given, HF + H 2 O ¾K
¾®
¾ a H 3O + + F — ;
K
(d) Reactivity–Be < Li < K < Cs Cl F — + H 2 O ¾¾ ¾b ® HF + OH — .

Which relation is correct


6. The dipole moment of chlorobenzene is 1
(a) Kb = Kw (b) Kb =
Kw
Cl
Ka
Cl (c) Ka × Kb = Kw (d)= Kw
Kb
1.5 D. Thedipolemoment of is
Cl Cl 11. In an amino acid, the carboxyl group ionises at
(a) 2.86D (b) 2.25D pK a1 = 2.34 and ammonium ion at pK a 2 = 9.60.
(c) 1.5D (d) 0D The isoelectric point of the amino acid is at pH
7. The initial volume of a gas cylinder is 750.0 mL. (a) 5.97 (b) 2.34
If the pressure of gas inside the cylinder changes (c) 9.60 (d) 6.97
from 840.0 mm Hg to 360.0 mm Hg, the final 12. AB, A2 and B2 are diatomic molecules. If the bond
volume the gas will be: enthalpies of A2, AB and B2 are in the ratio 1:1 :0.5
(a) 1.750 L (b) 3.60 L and enthalpy of formation of AB from A2 and B2 is
(c) 4.032 L (d) 7.50 L –100 kJ mol–1 . What is the bond energy of A2 :
8. Consider the following equilibrium (a) 200 kJ mol–1 (b) 100 kJ mol–1
+ (c) 300 kJ mol–1 (d) 400 kJ mol–1
AgCl ¯ +2NH3 ƒ éëAg ( NH 3 ) 2 ùû + Cl - 13. An 1% solution of KCl (I), NaCl (II), BaCl2 (III) andurea
White precipitate of AgCl appears on adding (IV) have their osmoticpressure at the same temperature
which of the following? in the ascending order (molar masses of NaCl, KCl,
(a) NH3 (b) aqueous NaCl BaCl2 and urea are respectively 58.5, 74.5, 208.4
(c) aqueous HNO3 (d) aqueous NH4Cl and 60 g mol–1). Assume 100% ionization of the
electrolytes at this temperature
Download From https://iit-jeeacademy.blogspot.com

Mock Test-1 MT-3

(a) I < III < II < IV (b) III < I < II < IV OH SO2OH
(c) I < II < III < IV (d) III < IV < I < II
NO2
14. If the following half cells have the E° values as
(c) (d)
Fe +3 + e – ¾
¾® Fe +2 ; E° = + 0.77V and
Fe +2 + 2e - ¾
¾® Fe ; E° = – 0.44V. The E° of the 18. Although Al has a high oxidation potential it
+3 – resists corrosion because of the formation of a
half cell Fe + 3e ¾ ¾® Fe will be
(a) 0.33 V (b) 1.21 V tough, protective coat of
(c) 0.04 V (d) 0.605 V (a) Al(NO3)2 (b) AlN
15. The oxidation states of sulphur in the anions (c) Al2O3 (d) Al2(CO3)2
19. A metal which is not affected by conc. H2SO4,
SO32- , S2 O 24- and S2 O62- follow the order
HNO3 or alkalis forms a compound X. This
(a) SO 32- < S 2O 24- < S2 O 62- compound X can be used to give a complex which
finds its application for toning in photography?
(b) S 2O 24- < S2 O 62- < SO 32-
The metal is
(c) S 2O 62- < S2 O 24- < SO 32- (a) Au (b) Ag
(c) Hg (d) Cu
(d) S 2 O 24- < SO 32 - < S 2 O 62 -
20. If Cl2 gas is passed into aqueous solution of KI
16. Which of the following statements is not containing some CCl4 and the mixture is shaken,
correct? then”.
(a) C – Cl bond in vinyl chloride is less polar (a) Upper layer becomes violet
than in CH3Cl (b) Lower layer becomes violet
(b) C – Cl bond in vinyl chloride is stronger (c) Homogenous violet layer is formed
than in CH3Cl (d) None
(c) C – Cl bond in vinyl chloride is shorter than 21. In Lassaigne’s test, the organic compound is
in CH3Cl fused with a piece of sodium metal in order to
(d) Vinyl chloride undergo nucleophilic (a) increase the ionisation of the compound
substitution more readily than CH3Cl. (b) decrease the melting point of the compound
17. Which of the following will not be soluble in (c) increase the reactivity of the compound
sodium carbonate solution? (d) convert the covalent compound into a
OH mixture of ionic compounds
O2N NO2 COOH 22. An aqueous solution of colourless metal
(a) (b) sulphate M gives a white precipitate with
NH4OH. This was soluble in excess of NH4OH.
On passing H2S through this solution a white
NO2 ppt. is formed. The metal M in the salt is
EBD_7206
Download From https://iit-jeeacademy.blogspot.com

MT-4 JEE MAIN

(a) Ca (b) Ba
(c) Al (d) Zn (a) NH2 Cl
23. Which of the following oxidising reaction of
KMnO4 occurs in acidic medium?
(i) Fe2+ (green) is converted to Fe3+ (yellow). (b) C N Cl
(ii) Iodide is converted to iodate.
(iii) Thiosulphate oxidised to sulphate.
(iv) Nitrite is oxidised to nitrate. (c) N C Cl
(a) (i) and (iii) (b) (i) and (iv)
(c) (iv) only (d) (ii) and (iv)
24. Which of the following compound can not used (d) CH3–NH Cl
in preparation of iodoform?
(a) CH3CHO (b) CH3COCH3 27. Select the rate law that corresponds to the data
(c) HCHO (d) 2-propanol shown for the following reaction A + B ¾ ¾® C
25. The correct priorities for the substituents shown Expt. No. (1) (2) Initial Rate
below, according to the E-Z sequence rule is 1 0.012 0.035 0.10
I. – CN II. – CBr (CH 3 ) 2 2 0.024 0.070 0.80
3 0.024 0.035 0.10
O
|| 4 0.012 0.070 0.80
III. – COOH IV. – CH 2 - C - OCH3 (a) Rate = K[B]3 (b) Rate = K [B]4
(c) Rate = K [A] [B]3 (d) Rate = K [A]2 [B]2
O 28. An alkene upon ozonolysis yield
||
V. – C-H CHO – CH2– CH2– CH2 – CHO only. The alkene
is
(a) II, III, V, I, IV (b) V, II, I, IV, III (a) CH2= CH—CH2— CH2— CH2— CH2— CH3
(c) III, IV, I, II, V (d) II, V, I, IV, III
26. Identify X in the sequence given :
(b) (c)
NH2

CHCl3 HCl
(Y) X + methanoic acid
KOH (300 K) (d)

Cl
Mock Test-1 MT-5

29. 1 mole of N 2 and 3 mole of H 2 are placed in a 32. A body of mass 5 kg under the action of
®
closed container at a pressure of 4 atm. The constant force F = Fx ˆi + Fy ˆj has velocity at t =
pressure falls to 3 atm at the same temperature
when the following equilibrium is attained ®

N 2 (g) + 3H 2 (g ) 2 NH 3 (g ) .
( )
0 s as v = 6iˆ - 2ˆj m/s and at t = 10s as

® ®
The K p for the dissociation of NH 3 is v = +6jˆ m / s . The force F is:
3´ 3
atm - 2 0.5´ (1.5)3 atm2 æ 3 ˆ 4 ˆö
(a)
0.5 ´ (1.5)3
(b)
(a) ( -3iˆ + 4ˆj) N (b) ç - i + j ÷ N
è 5 5 ø
0.5 ´ (1.5)3 (1.5) 3
atm - 2 æ 3ˆ 4 ˆö
(c)
3´ 3
atm 2 (d)
0.5 (c) ( 3iˆ - 4ˆj) N (d) ç i - j ÷ N
è5 5 ø
30. Sodium Carbonate cannot be used in place of
33. The period of the satellite of the earth orbiting
(NH4)2CO3 for the identification of Ca2+, Ba2+ very near to the surface of the earth is T0. What
and Sr2+ ions (in group V) during mixture is the period of the geostationary satellite in
analysis because : terms of T0
(a) Mg2+ ions will also be precipitated. T0
(b) Concentration of CO32– ions is very low. (a) (b) 7T0
7
(c) Sodium ions will react with acid radicals.
(d) Na+ ions will interfere with the detection of (c) 7T0 (d) 7 7T0
Ca2+, Ba2+, Sr2+ ions. 34. In “Al” and “Si”, if temperature is changed from
normal temperature to 70 K then
PHYSICS (a) The resistance of Al will increase and that
of Si will decrease
31. Two balls of same mass and carrying equal charge (b) The resistance of Al will decrease and that
are hung from a fixed support of length l. At of Si will increase
electrostatic equilibrium, assuming that angles (c) Resistance of both decrease
(d) Resistance of both increase
made by each thread is small, the separation, x
35. A toy–car, blowing its horn, is moving with a
between the balls is proportional to :
steady speed of 5 m/s, away from a wall. An
(a) l (b) l 2 (c) l 2/3 (d) l 1/3 observer, towards whom the toy car is moving,
is able to hear 5 beats per second. If the velocity
of sound in air is 340 m/s, the frequency of the
horn of the toy car is close to :
EBD_7206
MT-6 JEE MAIN

(a) 680 Hz (b) 510 Hz 39. In a meter bridge experiment resistances are
(c) 340 Hz (d) 170 Hz connected as shown in the figure. Initially
36. Two rods of length d1 and d2 and coefficients of resistance P = 4 W and the neutral point N is at 60
thermal conductivities K1 and K2 are kept cm from A. Now an unknown resistance R is
touching each other. Both have the same area of connected in series to P and the new position of
cross-section the equivalent of thermal the neutral point is at 80 cm from A. The value of
conductivity is unknown resistance R is :
P Q
(a) K1 + K2 (b) K1d1 + K2d2
G
d1K1 + d 2 K 2 d1 + d 2
(c) (d) (d / K + d / K )
d1 + d 2 1 1 2 2 A B
N
37. Two long parallel wires P and Q are both
perpendicular to the plane of the paper with
( )
distance of 5 m between them. If P and Q carry E Rh K
current of 2.5 amp and 5 amp respectively in the
33
same direction, then the magnetic field at a point (a) W (b) 6 W
5
half-way between the wires is
20
3m 0 m0 (c) 7 W (d) W
(a) (b) 3
2p p 40. The radius of curvature of a thin plano-convex
3m 0 m0 lens is 10 cm (of curved surface) and the refractive
(c) (d) index is 1.5. If the plane surface is silvered, then
2p 2p
it behaves like a concave mirror of focal length
38. A certain amount of gas is taken through a cyclic (a) 10 cm (b) 15 cm
process (A B C D A) that has two isobars, one (c) 20 cm (d) 5 cm
isochore and one isothermal. The cycle can be 41. The circular head of a screw gauge is divided
represented on a P-V indicator diagram as :
into 200 divisions and move 1 mm ahead in one
P
B C B
C
revolution. If the same instrument has a zero error
P
of –0.05 mm and the reading on the main scale in
(a) D
(b) D measuring diameter of a wire is 6 mm and that on
A A

V V circular scale is 45. The diameter of the wire is


B C (a) 6.275 mm (b) 6.375 mm
P B
C
P
(c) 5.75 mm (d) 5.50 mm
(c) (d) 42. A goods train accelerating uniformly on a
A D
A D
V
straight railway track, approaches an electric pole
V
standing on the side of track. Its engine passes
Mock Test-1 MT-7

the pole with velocity u and the guard’s room 46. In case of a p-n junction diode at high value of
passes with velocity v. The middle wagon of the reverse bias, the current rises sharply. The value
train passes the pole with a velocity. of reverse bias is known as
u+v 1 2
(a) (b) u + v2 (a) cut off voltage (b) zener voltage
2 2 (c) inverse voltage (d) critical voltage
æ u 2 + v2 ö 47. Three resistors of 4 W, 6 W and 12 W are
(c) uv (d) ç ÷
è 2 ø connected in parallel and the combination is
connected in series with a 1.5 V battery of 1 W
43. A wheel is rotating at 900 r.p.m. about its axis.
When power is cut off it comes to rest in 1 minute. internal resistance. The rate of Joule heating in
The angular retardation in rad/s2 is the 4 W resistor is
(a) p/2 (b) p/4 (a) 0.55 W (b) 0.33 W
(c) p/6 (d) p/8 (c) 0.25 W (d) 0.86 W
44. Two springs of force constants 300 N/m 48. A photon materializes into an electron-positron
(Spring A) and 400 N/m (Spring B) are joined pair. The kinetic energy of the electron is found
together in series. The combination is to be 0.19 MeV. What was the energy of the
compressed by 8.75 cm. The ratio of energy photon?
E E
stored in A and B is A . Then A is equal (a) 0.38 MeV (b) 0.70 MeV
EB EB
to : (c) 1.40 MeV (d) None of these
4 16 49. If 10% of a radioactive material decays in 5 days,
(a) (b)
3 9 then the amount of the original material left after
3 9 20 days is approximately
(c) (d)
4 16 (a) 60% (b) 65%
45. A particle of mass m is acted upon by a force F (c) 70% (d) 75%
R 50. When white light passes through a dispersive
given by the empirical law F = 2 v(t). If this medium, it breaks up into various colours. Which
t of the following statements is true?
law is to be tested experimentally by observing (a) Velocity of light for violet is greater than
the motion starting from rest, the best way is to the velocity of light for red colour.
plot : (b) Velocity of light for violet is less than the
1 velocity of light for red.
(a) log v(t) against (b) v(t) against t2 (c) Velocity of light is the same for all colours
t
(d) Velocity of light for different colours has
1 nothing to do with the phenomenon of
(c) log v(t) against 2 (d) log v(t) against t
t dispersion
EBD_7206
MT-8 JEE MAIN

51. A bottle has an opening of radius a and length (a) 1023 A m2 (b) 1020 A m2
b. A cork of length b and radius (a + Da) where (c) 1016 A m2 (d) 1010 A m2
(Da < < a) is compressed to fit into the opening
55. A plate of mass (M) is placed on a horizontal
completely (see figure). If the bulk modulus of
frictionless surface and a body of mass (m) is placed
cork is B and frictional coefficient between the
on this plate. The coefficient of dynamic friction
bottle and cork is m then the force needed to
between this body and the plate is m. If a force
push the cork into the bottle is : 3mmg is applied to the body of mass (m) along the
a
horizontal, the acceleration of the plate will be
b mm mmg
(a) g (b)
M M+m
3mmg 2mmg
(c) (d)
M M+m
56. Lights of two different frequencies, whose
photons have energies 1 eV and 2.5 eV
(a) (pmB b) a (b) (2pmBb) Da respectively, successively illuminate a metal
(c) (pmB b) Da (d) (4 pmB b) Da whose work function is 0.5 eV. The ratio of the
52. An insect trapped in a circular groove of radius maximum speeds of the emitted electrons will be
12 cm. moves along the groove steadily and (a) 1 : 5 (b) 1 : 4
completes 7 revolutions in 100 sec. What is the (c) 1 : 2 (d) 1 : 1
linear speed of the motion 57. Which of the plots shown in the figure represents
(a) 2.3 cm/sec. (b) 5.3 cm/sec. speed (vn) of the electron in a hydrogen atom as
a function of the principal quantum number (n)?
(c) 0.44 cm/sec. (d) None of these
53. An experiment is performed to determine the 1–
V characteristics of a Zener diode, which has a
protective resistance of R = 100 W, and a maximum
power of dissipation rating of 1W. The minimum
voltage range of the DC source in the circuit is :
(a) 0 – 5V (b) 0 – 24 V
(c) 0 – 12 V (d) 0 – 8V
54. The magnetic field of earth at the equator is
approximately 4 × 10–5 T. The radius of earth is
(a) B (b) D
6.4 × 106 m. Then the dipole moment of the earth
(c) C (d) A
will be nearly of the order of:
Mock Test-1 MT-9

58. A particle starts S.H.M. from the mean position. (a) 1 (b) 4
Its amplitude is a and total energy E. At one (c) 3 (d) 0
instant its kinetic energy is 3 E/4, its
62. The probability that in the random arrangement
displacement at this instant is
of the letters of the word ‘UNIVERSITY’, the
a a two I’s does not come together is
(a) y= (b) y =
2 2
4
a (a) (b) 1/ 5
(c) y= (d) y = a 5
3/ 2
(c) 1/10 (d) 9/10
59. An engine has an efficiency of 1/6. When the
temperature of sink is reduced by 62°C, its 63. If 2 sec 2a = tan b + cot b then one of the values
efficiency is doubled. Temperatures of source of (a + b) =
and sink are p
(a) p (b)
(a) 99°C, 37°C (b) 124°C, 62°C 2
(c) 37°C, 99°C (d) 62°C, 124°C p
60. A sinusoidal voltage of peak value 283 V and (c) (d) None
4
angular frequency 320/s is applied to a series 64. The probability of getting the sum more than 7
LCR circuit. Given that R = 5 W, L= 25 mH and C when a pair of dice is tossed is
= 1000 mF. The total impedance, and phase
difference between the voltage across the source 1 1
and the current will respectively be : (a) (b)
9 4
(a) 10 W and tan–1 æç ö÷
5 7 5
è 3ø (c) (d)
12 12
(b) 7 W and 45°
5 n
æ8ö Cr
(c) 10 W and tan –1 ç ÷
è3ø
65. The value of å r
n
C r -1
=
r =1
(d) 7 W and tan ç ö÷ æ
–1 5
(a) 5 (n – 3) (b) 5 (n – 2)
è 3ø (c) 5n (d) 5 (2n – 9)

MATHEMATICS 66. 14
C7 +
3
å 17-i C6 =
61. If a1, a2, a3, ................. are positive numbers in i =1
G.P. then the value of (a) 16 (b) 17
C7 C7
(c) 17 (d) 16
log an log an+1 log an +2 C8 C8
log an+1 log an +2 log an +3
log an+ 2 log an +3 log an +4
EBD_7206
MT-10 JEE MAIN

71. If (2, 3, 5) are ends of the diameter of a sphere


d5y
67. If y = A sin wt then = x2 + y2 + z2 – 6x – 12y – 2z + 20 = 0. Then
dt 5
coordinates of the other end are
æ pö (a) (4, 9, –3) (b) (4, 3, 5)
(a) Aw5 cosç wt - ÷
è 2ø (c) (4, 3, –3) (d) (4, –3, 9)
pö 72. Three persons A, B, C throw a die in succession.
æ
(b) Aw5 sin ç wt - ÷ The one getting 'six' wins. If A starts then the
è 2ø
probability of B winning is
æ pö 36 25
(c) Aw5 cos ç wt + ÷ (a) (b)
è 2ø 91 91
æ pö 41 30
(d) Aw5 sin ç wt + ÷ (c)
91
(d)
91
è 2ø
68. Let ABC be a triangle with vertices at points A x2 y 2
73. If the foci of the ellipse + = 1 coincide
(2, 3, 5), B (–1, 3, 2) and C (l, 5, m) in three 16 b 2
dimensional space. If the median through A is 2 2
with the foci of the hyperbola x - y = 1 ,
equally inclined with the axes, then 144 81 25
(l, m) is equal to : then b2 is equal to
(a) (10, 7) (b) (7, 5) (a) 8 (b) 10
(c) (7, 10) (d) (5, 7) (c) 7 (d) 9
69. The angle between the two lines 74. If f(x) = | x – 2| and g(x) = f (f(x)) then for x > 10,
g'(x) equal
x +1 y + 3 z - 4 x - 4 y + 4 z +1
= = & = = is (a) –1 (b) 0
2 2 -1 1 2 2
(c) 1 (d) 2x – 4
æ4ö æ3ö
(a) cos -1 ç ÷ (b) cos -1 ç ÷ 75. If a, b, c are in A.P., b, c, d are in G.P. and c, d, e are
è9ø è9ø in H.P. then a, c, e are in
(a) A.P. (b) G.P.
æ2ö æ1ö
(c) cos -1 ç ÷ (d) cos -1 ç ÷ (c) H.P. (d) None
è9ø è9ø 10
76. The coefficient of x in the expansion of
70. The contrapositive of (p Ú q) Þ r is 15
æ 1 ö
çç 3x 2 - ÷÷ is
(a) r Þ (p Ú q) (b) ~ r Þ (p Ú q) è x2 ø
15! 10
(c) ~rÞ~pÙ~q (d) p Þ (q Ú r) (a) 3 (b) –
15! 10
3
10! 5! 10! 5!
15! 35 15! 8
(c) - (d) 7! 8! 3
10! 5!
Mock Test-1 MT-11

77. If the median and the range of four numbers 81. The domain of the function
{x, y, 2x + y, x – y}, where 0 < y < x < 2y, are 10
and 28 respectively, then the mean of the f (x) = exp( 5x - 3 - 2x 2 ) is
numbers is : (a) [3/2, ¥) (b) [1, 3/2]
(a) 18 (b) 10 (c) (–¥, 1] (d) (1, 3/2)
(c) 5 (d) 14 82. The value of the determinant
dx
78. ò (x - b) (x - a)(b - x )
is 1 a a2
cos(n –1) x cosnx cos (n + 1) x
is zero, if
2 x-α sin (n –1) x sinnx sin (n + 1) x
(a) +c
α -β β- x
(a) sin x = 0 (b) cos x = 0
2
(b) (x - α) ( β - x) + c 1 + a2
α -β (c) a = 0 (d) cos x =
2a
a-b
(c) ( x - a) b - x dx
2 83. If ò cos3 x2sin 2x
= (tan x)A + C(tan x)B + k ,
(d) None of these.
79. Consider the following planes where k is a constant of integration, then A + B + C
equals :
P : x + y – 2z + 7 = 0
Q : x + y + 2z + 2 = 0 16 27
(a) (b)
R : 3x + 3y – 6z – 11 = 0 5 10
(a) P and R are perpendicular 7 21
(c) (d)
(b) Q and R are perpendicular 10 5
(c) P and Q are parallel
æ 2p ö æ 2p ö
(d) P and R are parallel 84. If a = cos ç ÷ + i sin ç ÷ , then the quadratic
è 7ø è 7ø
1 1 1 p4 equation whose roots are a = a + a2 + a4 and b
80. If 4
+ 4
+ 4
+ ..... + ¥ = , then the value = a3 + a5 + a6, is
1 2 3 90
(a) x2 – x + 2 = 0 (b) x2 + x – 2 = 0
1 1 1 (c) x2 – x – 2 = 0 (d) x2 + x + 2 = 0
of + 4 + 4 + ......¥ is
1 34
5 85. If AB = 0, then for the matrices
é 2
cos q sin qù and
(a)
p4
(b)
p4 A = ê cos q ú
96 45 ëêcos q sin q sin 2 q ûú
89 4
(c) p (d) None of these
90
EBD_7206
MT-12 JEE MAIN

88. An inverted cone is 10 cm in diameter and 10 cm


é cos 2 f cos f sin fù deep. Water is poured into it at the rate of 4cm3/
B= ê ú , q – f is min. When the depth of water level is 6 cm, then
êëcos f sin f sin 2 f úû
it is rising at the rate
9 1
p (a) cm 3 / min . (b) cm 3 / min .
(a) an odd number of 4p 4p
2
(b) an odd multiple of p 1 4
(c) cm 3 / min . (d) cm 3 / min .
9p 9p
p 89. The equation of tangent to 4x2 – 9y2 = 36 which are
(c) an even multiple of
2 perpendicular to straight line 5x + 2y – 10 = 0 are
(d) 0 æ 117 ö÷
x(1 – x)
(a) 5 (y–3) = 2çç x - ÷
86. If f(x) = xe , x Î R , then f(x) is è 2 ø
(a) decreasing on [–1/2, 1]
(b) decreasing on R (b) 2y – 5x + 10– 2 18 = 0
(c) increasing on [–1/2, 1] (c) 2y – 5x – 10– 2 18 = 0
(d) increasing on R (d) None of these
87. The area enclosed by the curve y = x5, the x-axis
log p æ1 ö
and the ordinates x = –1, x = 1 is 90. òlog p2
e 2 x sec 2 ç e 2 x ÷ dx is equal to :
è3 ø
1
(a) 0 (b) 1
3 (a) 3 (b)
3
1 1
(c) (d) None 3 3
6 (c) (d)
2 2 3
Download From https://iit-jeeacademy.blogspot.com

2 JEE MAIN
MOCK TEST
Time : 3 hrs. Max. Marks : 360

INSTRUCTIONS
• Chemistry (120 marks) : Question No. 1 to 30 are of 4 marks each.

• Physics (120 marks) : Question No. 31 to 60 are of 4 marks each.

• Mathematics (120 marks) : Question No. 61 to 90 are of 4 marks each.


• Negative Marking : One fourth (¼) marks will be deducted for indicating incorrect
response of each question.

3. Which of the following compound is used for


CHEMISTRY preparation of melamine formaldehyde polymer ?
1. At certain temperature (T) if conductivity of pure NH2
water is 5.5 × 10–7 S cm–1, then calculate pOH of
pure water at temperature T. Given
(a)
l ¥ + = 350 S cm 2 eq -1 , l ¥ = 200 S cm 2 eq -1 N
H OH - N
N
(a) 5 (b) 6
(c) 7 (d) 8 (b) H N
2. What volume of 0.2 M RNH3Cl solution should 2 NH2
be added to 100 ml. of 0.1 M RNH2 solution to NH2
produce a buffer solution of pH = 8.7 ? Given : H2N NH2
pKb of RNH2 = 5, log 2 = 0.3 N
(a) 50 ml. (b) 100 ml. (c)
N N
(c) 200 ml. (d) None of these

NH2
EBD_7206
MT-14 JEE MAIN

NH2 (a) 0 (b) 1.5


(c) 1 (d) 2
9. 96.5gm. of oxidized as A ® A3+ + 3e–, when 2F
(d) charge is passed through solution with current
N efficiency of 90%. What is electrochemical
H2N
equivalent of A ?
4. When N2O5 is heated at temp. T, it dissociates (a) 53.61 (b) 48.25
as N2O5(g) ƒ N2O3(g) + O2(g), Kc =2.5. At the (c) 5.55 × 10–4 (d) 5.55 × 10–3
same time N2O3 also decomposes as N2O3(g) 10. pH of the solution :
ƒ N2O(g) + O2(g). If initially 4.0 moles of N2O5
are taken in 2.0 litre flask and allowed to attain æ 10 litre of 0.03 N X(OH)2 (strong diacidic base)ö
equilibrium, concentration of O2 was formed to ç + ÷
be 2.5M. Equilibrium concentration of N2O is – ç 5 litre of 0.08 M HNO3 ÷
ç + ÷
(a) 2.0 (b) 1.0 çè 485 litre of 0.01 M NaNO ÷ø
3
(c) 0.334 (d) None of these
5. What mass of H2(g) is needed to reduce 192gm. is –
of MoO3 to metal ? [At. wt. of Mo = 96] (a) 3.7 (b) 11.0
(a) 8 gm. (b) 16 gm. (c) 7.0 (d) 10.0
(c) 32 gm. (d) None of these 11. What is the name of the following reaction :
6. How many times solubility of CaF2 is decreased COOEt
in 4×10–3 M KF (aq). solution as compare to pure
water at 25°C. Given Ksp (CaF2) = 3.2 × 10–11. EtONa
(a) 50 (b) 100
(c) 500 (d) 1000
7. Choose the correct options –
(a) Tranquillizers are called psychotherapeutic COOEt
drugs. (a) Knovengel reaction
(b) Vitamin C, Vitamin E and b-carotene are (b) Perkin reaction
antioxidants. (c) Reformatsky reaction
(c) Sodium or potassium salt of a long chain (d) Dieckmann’s condensation.
fatty acid is called soap. 12. Choose the correct options –
(d) All of these (a) The change of atmospheric temperature
8. In a chemical reaction A is converted into with altitude is called the lapse rate.
B. The rates of reaction, starting with initial (b) The gases responsible for greenhouse
concentrations of A a s 2 × 10–3 M and effects are CO2, Water vapour, CH4 and
1 × 10–3 M, are equal to 2.40 × 10–4 Ms–1 and ozone.
0.60 × 10–4 Ms–1 respectively. The order of (c) The order of contribution to the acid rain is
reaction with respect to reactant A will be H2SO4 < HNO3 < HCl
(d) Both (a) and (b)
Mock Test-2 MT-15

13. The standard electrode potentials E


o
( M+ / M ) of (b) Sol A is positively charged and Sol B is
negatively charged.
four metals A, B, C and D are – 1.2 V, 0.6 V, 0.85 (c) Both the sols A and B are positively
V and – 0.76 V, respectively. The sequence of charged.
deposition of metals on applying potential is: (d) Sol A is negatively charged and sol B is
(a) A, C, B, D (b) B, D, C, A positively charged.
(c) C, B, D, A (d) D, A, B, C 19. Choose the correct option for
14. Which statement is/are correct about ICl 3
molecule – Cl
(a) All I – Cl bonds are equivalent
(b) Molecule is polar and non-planar HBr
I
(c) All adjacent bond angles are equal HBr II
(d) All hybrid orbitals of central atom having Perdoxide
equal s-character
(a) I and II are identical
15. Isopropyl alcohol is obtained by reacting which (b) I and II are different
of the following alkenes with concentrated (c) Mechanism of formation of I and II are same
H2SO4 followed by boiling with H2O? (d) None of these
(a) Ethylene (b) Propylene HgSO dil. OHQ
4 ® (A) ¾¾¾¾® (B)
(c) 2-Methylpropene (d) Isoprene 20. HC º CH ¾¾¾¾
H2SO 4 5ºC
16. In electro-refining of metal the impure metal is
Give the IUPAC name of “B” is –
used to make the anode and a strip of pure metal
(a) 2-butanal
as the cathode, during the electrolysis of an
(b) 3-hydroxy butanal
aqueous solution of a complex metal salt. This
(c) 3-formyl-2-propanol
method cannot be used for refining of
(d) 4-oxo-2-propanol
(a) Silver (b) Copper
(c) Aluminium (d) Sodium
17. Which of the following compounds does not
give nitrogen gas on heating – 21.
(a) NH4NO2 (b) (NH4)2SO4
(c) NH4ClO4 (d) (NH4)2Cr2O7
18. Flocculation value of BaCl2 is much less than Br
2 / hn ® Major (X) ¾¾¾¾® Alcoholic
¾¾¾¾
that of KCl for sol A and flocculation value of KOH / D
Na2SO4 is much less than that of NaBr for sol B. H - Br
The correct statement among the following is : Major (Y) ¾¾¾® Major (Z)
Peroxide
(a) Both the sols A and B are negatively Major final product (Z) is –
charged.
EBD_7206
MT-16 JEE MAIN

Product (Y) of this reaction is –


Br NO2 NO2

(a) (b)

Br
(a) (b)
Br OCH3 Br
Cl OCH3
Br
(c) (d) NO2 NO2

22. Choose the incorrect option – (c) (d)


(a) When a dilute solution of an acid is added
to a dilute solution of a base, neutralization OCH3
reaction takes place. OCH3
(b) In acid-base titrations, at the end point, the
amount of acid becomes chemically (i) NaOI SOCl C H NH
25. A ¾¾¾¾ 6 5
2 ® C ¾¾¾¾¾
® B ¾¾¾¾ 2®
equivalent to the amount of base present. (ii) HÅ NaOH
(c) In the case of a strong acid and a strong Conc. HNO
base titration, at the end point the solution
3 ® E (Major)
D ¾¾¾¾¾ E can be –
Conc. H 2SO4
becomes acidic. (1 equivalent)
(d) In acid-base titrations the end point is
determined by the hydrogen ion concentration
of the solution. O
23. Light of wavelength l strikes a metal surface
with intensity ‘x’ and the metal emits ‘y’ electrons NH
per second of maximum energy ‘z’ . What will (a)
happen to ‘y’ and ‘z’ if ‘x’ is halved?
(a) y will remain same
(b) y will be doubled NO2
(c) z will remain the same
NO2
(d) z will halved
O
NO2
24. NH
Br2 CH3ONa (b)
¾¾¾ ® X ¾¾¾¾ ®Y
Fe,D CH3OH, D

Cl
Mock Test-2 MT-17

(a) Statement-1 is false, Statement-2 is true.


O (b) Statement-1 is true, statement-2 is true and
statement-2 is correct explanation for
NH statement-1
(c) (c) Statement-1 is true, statement-2 is true and
NO2 statement-2 is NOT correct explanation for
statement-1
H (d) Statement-1 is true, Statement-2 is false.
29. If the elevation in boiling point of a solution of
(d) O2N NH2 non-volatile, non-electrolytic and non-
Cl associating solute in a solvent
(Kb = x K kg mol–1) is y K, then the depression in
26. The standard enthalpy of formation of NH3 is
freezing point of solution of same concentration
– 46.0 kJ/mol. If the enthalpy of formation of H2
would be (Kf of the solvent = z K kg mol –1)
from its atoms is – 436 kJ/mol and that of N2 is
– 712 kJ/mol, the average bond enthalpy of N - H 2xz yz
bond in NH3 is: (a) (b)
y x
(a) – 1102 kJ/mol (b) – 964 kJ/mol
xz yz
(c) + 352 kJ/mol (d) + 1056 kJ/mol (c) (d)
y 2x
27. The rate coefficient (k) for a particular reactions
is1.3 × 10–4 M–1 s–1 at 100°C, and 1.3 × 10–3 M–1 30. The standard redox potentials Eº of the following
s–1 at 150°C. What is the energy of activation systems are :
(EA) (in kJ) for this reaction? (R = molar gas System Eº(volts)
constant = 8.314 JK–1 mol–1) (i) MnO4– + 8H+ + 5e ® Mn2+ + 4H2O 1.51
(a) 16 (b) 60
(ii) Sn4+ + 2e ® Sn2+ 0.15
(c) 99 (d) 132
(iii) Cr2O72– + 14H+ + 6e ® 2Cr3+ + 7H2O 1.33
28. This question contains Statement-1 and
(iv) Ce4+ + e ® Ce3+ 1.61
Statement-2. Of the four choices given after the
The oxidising power of the various species
statements, choose the one that best describes
decreases in the order-
the two statements.
Statement 1 : Anhydrous copper (II) chloride is (a) Ce4+ > Cr2O72– > Sn4+ > MnO4–
a covalent while anhydrous copper (II) fluoride (b) Ce4+ > MnO4– > Cr2O72– > Sn4+
is ionic in nature. (c) Cr2O72– > Sn4+ > Ce4+ + MnO4–
Statement 2 : In halides of transition metals, the (d) MnO4– > Ce4+ > Sn4+ > Cr2O72–
ionic character decreases with increase in atomic
mass of the halogen.
EBD_7206
MT-18 JEE MAIN

5 GM 8 GM
PHYSICS (a)
6 x2
(b)
9 x2
31. A uniform sphere of weight W and radius 5 cm is
being held by a string as shown in the figure. 7 GM 6 GM
(c) (d)
The tension in the string will be : 8 x2 7 x2
W W 35. A uniform thin rod AB of length L has linear
(a) 12 (b) 5 8 cm
bx
5 12 mass density m (x) = a + , where x is
L
W W measured from A. If the CM of the rod lies at
(c) 13 (d) 13 æ 7ö
5 12 a distance of ç ÷ L from A, then a and b are
è 12 ø
32. N divisions on the main scale of a vernier related as :
callipers coincide with N + 1 divisions on the (a) a = 2b (b) 2a = b
vernier scale. If each division on the main scale (c) a = b (d) 3a = 2b
is of ‘a’ units, determine the least count of the 36. n moles of an ideal gas undergo a process A ®
instrument is B as shown in the figure. Maximum temperature
a a of the gas during the process is
(a) (b)
N +1 N -1
2P0 A
2a 2a
(c) (d)
N -1 N +1
33. A current of 4A produces a deflection of 30° in P0 B
the galvanometer. The figure of merit is
P V0 2V
(a) 6.5 A/rad (b) 7.6 A/rad V
0

(c) 7.5 A/rad (d) 8.0 A/rad


34. The gravitational field, due to the 'left over part' 9P0V0 3P0V0
(a) (b)
of a uniform sphere (from which a part as shown, nR 2nR
has been 'removed out'), at a very far off point, P, 9P0V0 9P0V0
located as shown, would be (nearly) : (c) (d)
2nR 4nR
37. Which of these nuclear reactions is possible –
23
Removed Mass of complete (a) 11 Na + 11H ¾¾ 20
® 10 Ne + 24 He
Part sphere = M
P 10 4
R R (b) ® 13
5 B + 2 He ¾¾
1
7 N + 1H
10 1 -
(c) 5 B + 0n ® 11
¾¾ 5 B+b + n
x
14 1 -
(d) 7 N + 1H ® 12
¾¾ 6 C+b + n
Mock Test-2 MT-19

38. For the determination of the focal length of a space by width ‘d’. If a be the angle of deviation
convex mirror, a convex lens is required because of proton from initial direction of motion (see
(a) it is not possible to obtain the image figure), the value of sin a will be :
produced by a convex mirror on the screen B
Bd
(b) a convex lens has high resolving power so (a) qV
2m
it helps to measure the focal length a
correctly B qd
(b)
(c) a convex mirror always forms a real image 2 mV d
which is diminished by the convex lens
(d) none of these B q q
(c) (d) Bd
39. What is the conductivity of a semiconductor d 2mV 2mV
sample having electron concentration of 5 × 1018 43. A series LR circuit is connected to an ac source
m–3, hole concentration of 5 × 1019 m–3, electron of frequency w and the inductive reactance is
mobility of 2.0 m2 V–1 s–1 and hole mobility of equal to 2R. A capacitance of capacitive
0.01 m2 V–1 s–1 ? reactance equal to R is added in series with
(Take charge of electron as 1.6 × 10–19 C) L and R. The ratio of the new power factor to the
(a) 1.68 (W – m)–1 (b) 1.83 (W – m)–1 old one is :
(c) 0.59 (W – m) –1 (d) 1.20 (W – m)–1
40. A transmitting antenna at the top of a tower has 2 2 3 5
height 32 m and height of the receiving antenna (a) (b) (c) (d)
3 5 2 2
is 50 m. What is the maximum distance between 44. Photons of an electromagnetic radiation has an energy
them for satisfactory communication in line of
11 keV each. To which region of electromagnetic
sight (LOS) mode?
spectrum does it belong ?
(a) 55.4 km (b) 45.5 km
(c) 54.5 km (d) 455 km (a) X-ray region (b) Ultra violet region
(c) Infrared region (d) Visible region
41. A body is executing simple harmonic motion. At a
45. A ball of mass 160 g is thrown up at an angle
displacement x from mean position, its potential of 60° to the horizontal at a speed of 10 ms–1. The
energy is E1 = 2J and at a displacement y from angular momentum of the ball at the highest point
mean position, its potential energy is E2 = 8J. of the trajectory with respect to the point from
The potential energy E at a displacement (x + y) which the ball is thrown is nearly (g = 10 ms–2)
from mean position is (a) 1.73 kg m2/s (b) 3.0 kg m2/s
(c) 3.46 kg m /s 2 (d) 6.0 kg m2/s
(a) 10J (b) 14J 46. A uniform wire (Young’s modulus 2 × 1011 Nm–2)
(c) 18J (d) 4J is subjected to longitudinal tensile stress of 5 ×
42. A proton (mass m) accelerated by a potential 107 Nm–2. If the overall volume change in the
difference V flies through a uniform transverse wire is 0.02%, the fractional decrease in the radius
magnetic field B. The field occupies a region of of the wire is close to :
EBD_7206
MT-20 JEE MAIN

(a) 1.0 × 10–4 (b) 1.5 × 10–4 electric field of magnitude E. The period of
(c) 0.25 × 10 –4 (d) 5 × 10–4 oscillation
47. A mass of 50g of water in a closed vessel, with (a) is proportional to E
surroundings at a constant temperature takes 2 (b) is proportional to 1/E
minutes to cool from 30°C to 25°C. A mass of
100g of another liquid in an identical vessel with (c) is p ml
3qE
identical surroundings takes the same time to
cool from 30° C to 25° C. The specific heat of the 1 ml
liquid is : (d) is proportional to but ¹ p
E 3qE
(The water equivalent of the vessel is 30g.) 51. A thin convex lens of focal length ‘f’ is put on a
(a) 2.0 kcal/kg (b) 7 kcal/kg plane mirror as shown in the figure. When an
(c) 3 kcal/kg (d) 0.5 kcal/kg object is kept at a distance ‘a’ from the lens -
48. An ideal monatomic gas with pressure P, volume mirror combination, its image is formed at a
V and temperature T is expanded isothermally to a
a volume 2V and a final pressure Pi. If the same distance in front of the combination. The
3
gas is expanded adiabatically to a volume 2V, value of ‘a’ is :
P 3
the final pressure is Pa. The ratio a is (a) 3f (b) f
Pi 2
(a) 2–1/3 (b) 21/3 (c) f (d) 2f
(c) 22/3 (d) 2–2/3
49. In a Young’s double slit experiment with light of
wavelength l, fringe pattern on the screen has
fringe width b. When two thin transparent glass 52. A sinusoidal wave is propagating in negative x-
(refractive index m) plates of thickness t1 and t2 direction in a string stretched along x-axis. A
(t1 > t2) are placed in the path of the two beams particle of string at x = 2m is found at its mean
respectively, the fringe pattern will shift by a position and it is moving in positive y-direction
distance at t = 1 sec. The amplitude of the wave, the
b ( m - 1) æ t1 ö mb t1 wavelength and the angular frequency of the
wave are 0.1 meter, p/2 meter and 2p rad/sec
(a) l çè t ÷ø
2
(b) l t2 respectively.
The equation of the wave is
b ( m - 1) l
(c) ( t1 - t2 ) (d) ( m - 1) ( t1 + t2 ) (a) y = 0.1 sin (4p (t – 1) + 8 (x – 2))
l b (b) y = 0.1 sin ((t – 1) – (x – 2))
50. A dipole consisting of two charges +q and –q (c) y = 0.1 sin (2p (t – 1) + 4 (x – 2))
joined by a massless rod of length l, is seen (d) None of the1se
oscillating with a small amplitude in a uniform
Mock Test-2 MT-21

53. In an experiment, a small steel ball falls through a


QE 0 1 QE 0
liquid at a constant speed of 10 cm/s. If the steel (a) 2 (b)
ball is pulled upward with a force equal to twice its mw 2 mw 2
effective weight, how fast will it move upward ? 2QE 0
(a) 5 cm/s (b) Zero (c) (d) None of these
(c) 10 cm/s (d) 20 cm/s mw 2
54. Six resistors of 10W each are connected as 57. In a photoelectric experiment, with light of
shown. The equivalent resistance between wavelength l, the fastest electron has speed v.
points X and Y is – If the exciting wavelength is changed to 3l/4,
the speed of the fastest emitted electron will
X become –
3 4
(a) v (b) v
4 3
Y
(a) 20 W (b) 5 W 4 4
(c) less than v (d) greater than v
(c) 15 W (d) 10 W 3 3
58. A particle moves along x-axis with initial position
55. Velocity–time graph for a body of mass 10 kg is x = 0. Its velocity varies with x-coordinate as
shown in figure. Work–done on the body in first shown in graph. The acceleration ‘a’ of this
two seconds of the motion is : particle varies with x as –
v
v (m/s)
-1
50 ms

a a

(a) x (b) x
(0,0) 10s t(s)

(a) – 9300 J (b) 12000 J


(c) –4500 J (d) –12000 J a a
56. A charged particle of mass m and having a charge
Q is placed in an electric field E which varies (c) x (d) x
with time as E = E0 sin wt. What is the amplitude
of the S.H.M. executed by the particle?
EBD_7206
MT-22 JEE MAIN

59. A ring of mass M and radius R lies in x-y plane


ì 2
with its centre at origin as shown. The mass ï 10 - x if - 3 < x < 3
distribution of ring is nun-uniform such that at 62. Given f (x) = í
any point P on the ring, the mass per unit length ï2 - e x -3 if x ³ 3
î
is given by l = l0 cos2q (where l0 is a positive
The graph of f (x) is –
constant). Then the moment of inertia of the ring
(a) continuous and differentiable at x = 3
about z-axis is –
(b) continuous but not differentiable at x = 3
y (c) differentiable but not continuous at x = 3
(d) neither differentiable nor continuous at x = 3
P 2 2
M
R 63. z1 + z2 + z1 - z2 is equal to

( )
q
2 ( z1 + z2 )
x 2 2
(a) (b) 2 z1 + z2

2 2
(c) z1 z2 (d) z1 + z2
64. The area of the triangle whose vertices are
complex numbers z, iz, z + iz in the Argand
1
(a) MR2 (b) MR 2 diagram is
2 (a) 2|z|2 (b) 1/2|z|2
2
1M 1 M (c) 4|z| (d) |z|2
(c) R (d) p l R 65. Box contains 2 one rupee, 2 five rupee, 2 ten rupee
2 l0 0 and 2 twenty rupee coin. Two coins are drawn at
60. The electric potential V is given as a function of random simultaneously. The probability that their
distance x by V = (5x2 + 10x – 9) volt. Value of sum is Rs. 20 or more, is
electric field at x = 1m is (a) 1/4 (b) 1/2
(a) –20 V/m (b) 6 V/m (c) 3/4 (d) 1/8
66. The equation (5x – 1)2 + (5y – 2)2 = (l2 – 4l + 4)
(c) 11 V/m (d) +20 V/m
(3x + 4y – 1)2 represents an ellipse if l Î
(a) (0, 1] (b) (–1, 2)
MATHEMATICS (c) (2, 3) (d) (–1, 0)
61. If two vertical poles 20 m and 80 m high stand 67. The value of the definite integral,
apart on a horizontal plane, then the height (in q2
dq 501p
m) of the point of intersection of the lines joining ò 1 + tan q = K
where
q1
the top of each pole to the foot of other is
(a) 16 (b) 18 1003p p
q2 = and q1 = . The value of K equals
(c) 50 (d) 15 2008 2008
Mock Test-2 MT-23

(a) 2007 (b) 2006 (a) 18, 50/9 (b) 18, 25/9
(c) 2009 (d) 2008 (c) 27, 50/9 (d) None of these
68. The straight line y = m (x – a) meets the parabola 75. The sum of the coefficient of all the terms in the
y2 = 4ax in two distinct points for – expansion of (2x – y + z)20 in which y do not
(a) all m Î R (b) all m Î [–1, 1] appear at all while x appears in even powers and
(c) all m Î R – {0} (d) None of these z appears in odd powers is –
69. The expansion of (1 + x)n has 3 consecutive terms 2 20 - 1
with coefficients in the ratio 1 : 2 : 3 and can be (a) 0 (b)
2
written in theform nCk : nCk+1 : nCk+2. The sum
of all possible values of (n + k) is – 320 - 1
(c) 219 (d)
(a) 18 (b) 21 2
(c) 28 (d) 32 76. All the five digit numbers in which each
70. The mean and standard deviation of 6 successive digit exceeds is predecessor are
observations are 8 and 4 respectively. If each arranged in the increasing order. The (105)th
observation is multiplied by 3, find the new number does not contain the digit
standard deviation of the resulting observations. (a) 1 (b) 2
(a) 12 (b) 18 (c) 6 (d) All of these
(c) 24 (d) 144
tan x tan x - sin x sin x
71. p Ú (p Ù q) is equivalent to – 77. lim equals
(a) q (b) p x ®0 x3. x
(c) ~p (d) ~q (a) 1/4 (b) 3/4
(c) 1/2 (d) 1
æ x cos3 x - sin x ö 78. Three people each flip two fair coins. The
72. Find ò esin x ç ÷ dx
è cos 2 x ø probability that exactly two of the people flipped
(a) x esin x – esin x sec x + C one head and one tail, is–
(b) x ecos x – esin x sec x + C (a) 1/2 (b) 3/8
(c) x2 esin x + esin x sec x + C (c) 5/8 (d) 3/4
(d) 2x esin x – esin x tan x + C r r r
79. If a, b, c are non-coplanar unit vector such that
73. The function f : [2, ¥) ® (0, ¥) defined by r r r 1 r r
f (x) = x2 – 4x + a, then the set of values of ‘a’ for a ´ (b ´ c) = (b + c) then the angle between
2
which f(x) becomes onto is r r
(a) (4,¥) (b) [4, ¥) the vectors a, b is
(c) {4} (d) f (a) 3p/4 (b) p/4
(c) p/8 (d) p/2
74. If a and b are the real roots of the equation
80. The greatest and the least value of |z1 + z2| if z1
x2 – (k – 2) x + (k2 + 3k + 5) = 0
(k Î R) . = 24 + 7i and |z2| = 6 respectively are
Find the maximum and minimum values of (a) 25, 19 (b) 19, 25
(c) –19, –25 (d) –25, –19
(a 2 + b 2 ) .
EBD_7206
MT-24 JEE MAIN

81. Let P = (–1, 0), Q = (0, 0) and R = (3, 3 3 ) be equation


three point. The equation of the bisector of the d2 y dy
(1 + t 2 ) + (2t + y tan -1 (t)) = k then k
angle PQR is dt 2 dt
is equal to
3
(a) x+ y =0 (b) x + 3 y = 0 (a) –2 (b) 2
2 (c) –1 (d) 0
3 87. Let f : R ® R and fn (x) = f (fn–1 (x)) n ³ 2, n Î N,
(c) 3x + y = 0 (d) x + y = 0. the roots of equation f3(x) f2(x) f (x) – 25f2(x) f (x)
2
+ 175 f (x) = 375. Which also satisfy equation
82. If ò 2 1 + sin xdx = -4 cos (ax + b) + C then f (x) = x will be
(a) 5 (b) 15
the value of (a, b) is :
(c) 10 (d) Both (a) and (b)
1 p p 88. A triangle ABC satisfies the relation 2 sec 4C +
(a) , (b) 1,
2 4 2 sin2 2A + sin B = 0 and a point P is taken on
(c) 1, 1 (d) None of these the longest side of the triangle such that it
83. If P(S) denotes the set of all subsets of a given divides the side in the ratio 1 : 3. Let Q and R be
set S, then the number of one-to-one functions the circumcentre and orthocentre of D ABC. If
from the set S = {1, 2, 3} to the set P(S) is PQ : QR : RP = 1 : a : b, then the value of a2 + b2.
(a) 24 (b) 8 (a) 9 (b) 8
(c) 336 (d) 320 (c) 6 (d) 7
84. If the tangent at any point on the curve x4 + y4 = 89. The value of
a4 cuts off intercepts p and q on the co-ordinate
axes then the value of p–4/3+q–4/3 is sin 2 x cos 2 x
(a) a –4/3 (b) a –1/2 ò0 sin -1 t dt + ò0 cos -1 t dt is
(c) a 1/2 (d) None
85. The area bounded by the curve y2 (2a – x) = x3 p
(a) p (b)
and the line x = 2a is 2
3pa 2 p
(a) 3pa2 sq. unit (b) sq. unit (c) (d) 1
2 4

3pa 2 6pa 2 90. If a is real and 2ax + sin By + cos Bz = 0 ,


(c) sq. unit (d) sq. unit
4 5 x + cos By + sin Bz = 0 , – x + sin By – cos Bz = 0,
86. If the substitution x = tan–1 (t) transforms the then the set of all values of a for which the system
differential equation of linear equations has a non-trivial solution, is
d2 y dy –
2
+ xy + sec 2 x = 0 into a differential (a) [1, 2] (b) [–1, 1]
dx dx
(c) [1, ¥] (d) [2–1/2, 21/2]
Download From https://iit-jeeacademy.blogspot.com

3 JEE MAIN
MOCK TEST
Time : 3 hrs. Max. Marks : 360

INSTRUCTIONS
• Chemistry (120 marks) : Question No. 1 to 30 are of 4 marks each.

• Physics (120 marks) : Question No. 31 to 60 are of 4 marks each.

• Mathematics (120 marks) : Question No. 61 to 90 are of 4 marks each.


• Negative Marking : One fourth (¼) marks will be deducted for indicating incorrect
response of each question.

3. Match List I with List II and select the correct


CHEMISTRY answer :
List I (Ions) List II (Shapes)
1. Let n1 be the frequency of the series limit of the
A. ICl –2 1. Linear
Lyman series, n2 be the frequency of the first line
of the Lyman series, and n3 be the frequency of the B. BrF2+ 2. Pyramidal
series limit of the Balmer series, then – C. ClF4- 3. Tetrahedral
1
(a) n3 = (n – n ) (b) n2 – n1 = n3 D. AlCl-
4 4. Square planar
2 1 3
5. Angular
(c) n1 – n2 = n3 (d) n1 + n2 = n3 A B C D
(a) 1 2 4 5
(b) 4 5 2 3
(c) 1 2 4 3
2. (d) 5 1 3 4
OH
N 4. Which one of the following is the correct
statement?
(a) Boric acid is a protonic acid
Number of a-H in alkene which is major product
(b) Beryllium exhibits coordination number of
in this reaction is – six
(a) 2 (b) 4 (c) Chlorides of both beryllium and aluminium
(c) 5 (d) 7 have bridged chloride structures in solid
phase
(d) B2 H 6 .2NH 3 is known as ‘inorganic
benzene’
EBD_7206
MT-26 JEE MAIN

5. At 675K, H2(g) and CO2(g) react to form CO(g) 9. The conductivity of a saturated solution of
and H2O (g), Kp for the reaction is 0.16. If a mixture Ag3PO4 is 9 × 10–6 Sm–1 and its equivalent
of 0.25 mole of H2(g) and 0.25 mol of CO2 is conductivity is 1.50 × 10–4 Sm2 equivalent–1. The
heated at 675K, mole % of CO(g) in equilibrium Ksp of Ag3PO4 is –
mixture is –
(a) 4.32 × 10–18 (b) 1.8 × 10–9
(a) 7.14 (b) 14.28
(c) 8.64 × 10–13 (d) None of these
(c) 28.57 (d) 33.33
10. Which statement is incorrect regarding thermite
6. Choose the correct option for
process –
Anhydrous (a) 3 part metal oxide +1 part Al is taken as
I
HI thermite mixture
O Conc. (b) Ignition mixture (Mg powder + BaO2) acts
II
HI as catalyst
(a) I and II are identical (c) Metals (Fe, Cr) are obtained in vapour form
(b) I and II are different (d) Reaction is exothermic due to highly +ve
(c) Mechanism of formation of I and II are not enthalpy of formation of Al2O3.
known 11. Which of the following option is having
(d) None of these maximum number of unpaired electrons –
7. For the extraction of Pb when impurity content (a) A tetrahedral d6 ion
is very high, then during roasting of galena, the (b) [Co(H2O)6]3+
formation of which compound (s) is/are
(c) A square planar d7 ion
prevented by lime –
(d) A co-ordination compound with magnetic
(a) PbSO4 (b) PbSiO3
moment of 5.92 B.M.
(c) Both (a) and (b) (d) CaSiO3
8. Structure of some important polymers are given. 12. pH of a saturated solution of silver salt of monobasic
Which one represents Buna-S? acid HA is found to be 9. Find the Ksp of sparingly
CH3 soluble salt Ag A (s). Given Ka (HA) = 10–10.
(a) 1.1 × 10–9 (b) 1.1 × 10–10
(a) (– CH2 – C = CH – CH2 –) n
(c) 1.1 × 10–11 (d) 10–12
(b) (– CH2 – CH = CH – CH2 – CH – CH2 –)
n 13. Which of the following carbide does not release
C6H5 any hydrocarbon on reaction with water.
(c) (– CH2 – CH = CH – CH2 – CH – CH2 –) n (a) SiC (b) Be2C
CN (c) CaC2 (d) Mg2C3
Cl

(d) (– CH2 – C = CH – CH2 –) n


Mock Test-3 MT-27

14. Amongst the following alcohols which would (a) XeO2F2 (b) XeOF4
react fastest with conc. HCl and ZnCl2 ? (c) XeO3 (d) XeO4
(a) pentanol aq. KOH
(b) 2-methyl butanol 20. Me – CH = CH2 + CHCl3 ¾¾¾¾ ® A (Major
excess, D
(c) 2-pentanol products) is –
(d) 2-methyl butan-2-ol
O
15. For the reaction takes place at certain temperature
NH4HS ƒ NH3 (g) + H2S (g), if equilibrium (a) OK
pressure is X bar, then DG° would be –
(a) – 2 RT ln X (b) – RT ln (X – ln 2)
OK
(c) – 2RT (ln X – ln 2) (d) None of these
(b)
16. Aqueous solution of (M) + (NH4)2S ® yellow
O
(NH 4 )2 S2
ppt (B) ¾¾¾¾¾ ® insoluble.
The cation present in (M) is –
(c)
(a) CdS (b) SnS2
O
(c) Cd 2+ (d) Sn2+
17. How many pairs of enantiomers are possible for
following complex compound. [M (AB) (CD) ef]n± (d) OH
(where AB, CD– Unsymmetrical bidentate ligand, e, OH
f–Monodentate ligands) 21. This question contains Statement-1 and
(a) 20 (b) 5 Statement-2. Of the four choices given after the
(c) 10 (d) 8 statements, choose the one that best describes
the two statements.
18. The instantaneous rate of disappearance of
Statement 1 : [HeH]+ is more stable as compared
MnO4– ion in the following reaction is
to H2+.
4.56 × 10–3 Ms–1
Statement 2 : Both the above species are having
2MnO4– + 10I– + 16H+ ® 2Mn2+ + 5I2 + 8H2O equal bond order.
The rate of appearance I2 is : (a) Statement-1 is false, Statement-2 is true.
(a) 4.56 × 10–4 Ms–1 (b) Statement-1 is true, statement-2 is true and
(b) 1.14 × 10–2 Ms–1 statement-2 is correct explanation for
(c) 1.14 × 10–3 Ms–1 statement-1
(d) 5.7 × 10–3 Ms–1 (c) Statement-1 is true,statement-2 is true and
19. Which of the following xenon-oxo compounds statement-2 is NOT correct explanation for
may not be obtained by hydrolysis of xenon statement-1
fluorides? (d) Statement-1 is true, Statement-2 is false.
EBD_7206
MT-28 JEE MAIN

22. The variation of concentration of A with time in


two experiments starting with two different initial
concentration of A is given in the following
graph. The reaction is represented as A (aq)® B O3 Conc. KOH
(aq). What is the rate of reaction 25. ¾¾® (A) ¾¾¾¾¾ ® (B)
Zn (1 mole)
(M/min) when concentration of A in aqueous
solution was 1.8 M ?

1.7 End product (B) of above reaction is –


1.6
1.5
1.4 CHO
Concentration (M)

1.3
1.2 (a) CH2OH
1.1
1.0
0.9
0.8 Experiment-1
0.7 Experiment-2
0.6
CO2 K
10
5 15 20
time (min.) (b)
(a) 0.072 M min–1 (b) 0.036 M min–1 CH2OH
(c) 0.1296 M min –1 (d) 1 M min–1
23. NH4ClO4 + HNO3 (dilute) ® X + HClO4
heat
X ¾¾¾
® Y (gas) Gas (Y) is –
CH2OH
(a) O2 (b) N2
(c) NO2 (d) N2O (c)
24. Which of the following is/are formed when CH2OH
ozone reacts with the unburnt hydrocarbons
in polluted air ?
(i) Formaldehyde (ii) Acrolein
(iii) Peroxyacetyl nitrate (iv) Formic acid
CO2 K
(a) (i) and (iv) (b) (ii) only
(c) (iii) only (d) (i), (ii) and (iii) (d)
CO2 K
Mock Test-3 MT-29

26. Precautions to be taken in the study of reaction 28. 1 mole of gas X is present in a closed adiabatic
rate for the reaction between potassium iodate vessel fitted with a movable frictionless piston.
(KIO3) and sodium sulphite (Na2SO3 ) using The initial temperature of gas X is 300 K. The
starch solution as indicator at different vessel is maintained at constant pressure of 1
concentrations and temperature – atm. Keeping the pressure constant at 1atm the
(a) The concentration of sodium thiosulphate reaction (3X (g) ® 2Y(g) ; DH = –30 kJ/mol) is
solution should always be less than the started with the help of negligible amount of
concentration of the potassium iodide electric energy.
solution.
If finally 75 mole % of X undergone reaction at
(b) Freshly prepared starch solution should be
constant pressure of 1 atm, find the final
used
temperature (in K) of reaction vessel. Given :
(c) Experiments should be performed with the Cp,m (X) = 40 J/K mole, Cp,m(Y) = 30 J/K mole.
fresh solutions of H2O2 and KI.
(a) 600 K (b) 300 K
(d) All of these
(c) 1200 K (d) 1000 K
27. Calculate the millimoles of SeO32– in solution on
the basis of following data : 29. In the given molecule choose the correct order
of their angle.
M
70 ml of solution of KBrO3 was added to O
60
SeO3 2– solution. The bromine evolved was y C x
removed by boiling and excess of KBrO3 was
H z O–H
M
back titrated with 12.5 mL of solution of (a) x > y > z (b) x < y < z
25
NaAsO2. (c) x = y > z (d) Can’t be predicted
The reactions are given below. 30. What would be the reduction potential of an
electrode at 298 K, which originally contained
(1) SeO32 - + BrO3- + H + ® SeO42 - + Br2 + H 2O 1M K2Cr2O7 solution in acidic buffer solution of
pH = 1.0 and which was treated with 50% of the
(2) BrO3- + AsO -2 + H 2 O ® Br - + AsO34- + H + Sn necessary to reduce all Cr 2O72– to Cr3+.
(a) 1.6 × 10–3 (b) 1.25 Assume pH of solution remains constant.
(c) 2.5 (d) None of these
EBD_7206
MT-30 JEE MAIN

33. A uniform wire of length l and radius r has a


Given : E 0 = 1.33 V , log2 = 0.3, resistance of 100 W. It is recast into a wire of
Cr2O72 - /Cr3+ ,H +
r
radius . The resistance of new wire will be :
2.303RT 2
= 0.06 (a) 1600 W (b) 400 W
F (c) 200 W (d) 100 W
(a) 1.285 V (b) 1.193 V 34. A piece of wood from a recently cut tree shows
(c) 1.187 V (d) None of these 20 decays per minute. A wooden piece of same
size placed in a museum (obtained from a tree
PHYSICS cut many years back) shows 2 decays per minute.
If half life of C14 is 5730 years, then age of the
31. A glass prism of refractive index 1.5 is immersed
4 wooden piece placed in the museum is
in water (refractive index ) as shown in figure. approximately:
3
A light beam incident normally on the face AB is (a) 10439 years (b) 13094 years
totally reflected to reach the face BC, if (c) 19039 years (d) 39049 years
A
35. Two blocks of mass M1 = 20 kg and M2 = 12 kg
B
q are connected by a metal rod of mass 8 kg. The
system is pulled vertically up by applying a force
of 480 N as shown. The tension at the mid-point
of the rod is :
C 480 N

M1
5 2
(a) sin q ³ (b) sin q ³
9 3
8 1
(c) sin q ³ (d) sin q ³
9 3
32. What is the minimum energy required to launch
a satellite of mass m from the surface of a planet M2
of mass M and radius R in a circular orbit at an
altitude of 2R? (a) 144 N (b) 96 N
(c) 240 N (d) 192 N
5GmM 2GmM 36. A particle falls freely near the surface of the earth.
(a) (b)
6R 3R Consider a fixed point O (not vertically below
the particle) on the ground. Then pickup the
GmM 3GmM incorrect alternative or alternatives.
(c) (d)
2R 2R
Mock Test-3 MT-31

(a) The magnitude of angular momentum of the 39. If the ratio of the concentration of electrons to
particle about O is increasing 7
that of holes in a semiconductor is and the
5
(b) The magnitude of torque of the 7
gravitational force on the particle about O ratio of currents is , then what is the ratio of
4
is decreasing their drift velocities?
5 4
(c) The moment of inertia of the particle about (a) (b)
O is decreasing 8 5
5 4
(d) The magnitude of angular velocity of the (c) (d)
4 7
particle about O is increasing 40. Two particles A and B of equal mass M are
moving with the same speed v as shown in the
37. Two spherical bubbles are in contact with each
figure. They collide completely inelastically and
other internally as shown. The radius of
move as a single particle C. The angle q that the
curvature of the common surface is R, then – path of C makes with the X-axis is given by:
Y
R1
R2 Common C
surface
q
X
A 45°
30°
(a) R > R1 (b) R1 > R > R2 B

(c) R < R2 (d) R = R1


38. Consider a spherical shell of radius R at 3+ 2
temperature T. The black body radiation inside it (a) tanq =
1- 2
can be considered as an ideal gas of photons
U 3- 2
with internal energy per unit volume u = µ (b) tanq =
V 1- 2
1 æ ö
U
T4 and pressure p = ç ÷ . If the shell now
3è V ø
1- 2
undergoes an adiabatic expansion the relation (c) tanq =
2(1 + 3)
between T and R is :
1 1 1- 3
(a) T µ (b) T µ (d) tanq =
3 1+ 2
R R
(c) T µ e–R (d) T µ e–3R
EBD_7206
MT-32 JEE MAIN

41. When the specific heat of a solid is measured by (a) 400 nm (b) 700 nm
the method of mixture, the heat is lost to (c) 500 nm (d) 667 nm
surroundings from the calorimeter by the process of 44. In the LC circuit, the current in the direction
(a) radiation shown and the charges on the capacitor plates
(b) conduction have the signs shown. At this time
(c) convection
(d) both conduction and convection. C
I
42. The fundamental frequency of a sonometer wire
of length l is n0. A bridge is now introduced at a +Q –Q
distance of Dl (<< l) from the centre of the wire. L
The lengths of wire on the two sides of the bridge
are now vibrated in their fundamental modes.
Then, the beat frequency nearly is –
(a) n0Dl/l (b) 8n0Dl/l (a) I is increasing and Q is increasing
(c) 2n0Dl/l (d) n0Dl/2l (b) I is increasing and Q is decreasing
43. In Young's double slit experiment shown in figure (c) I is decreasing and Q is increasing
S1 and S2 are coherent sources and S is the screen (d) I is decreasing and Q is decreasing
having a hole at a point 1.0mm away from the 45. A car, starting from rest, accelerates at the rate f
central line. White light (400 to 700nm) is sent through a distance S, then continues at constant
through the slits. Which wavelength passing speed for time t and then decelerates at the rate
through the hole has strong intensity? f
to come to rest. If the total distance traversed
2
is 15 S , then
S1 Centre of
screen 1 2
0.5mm (a) S = ft (b) S = f t
Screen 6
S2
1.0mm
hole 1 2 1 2
(c) S = ft (d) S = ft
4 72
50cm S
Mock Test-3 MT-33

46. Figure shows a network of capacitors where the 48. The current voltage relation of a diode is given
numbers indicates capacitances in micro Farad.
by I = ( e1000V T - 1) mA, where the applied
The value of capacitance C if the equivalent
capacitance between point A and B is to be 1 mF voltage V is in volts and the temperature T is in
is : degree kelvin. If a student makes an error
measuring ±0.01 V while measuring the current
C 1 of 5 mA at 300 K, what will be the error in the
A value of current in mA?
8 6 4 (a) 0.2 mA (b) 0.02 mA
(c) 0.5 mA (d) 0.05 mA
49. A diatomic molecule is made of two masses m1
2 2 12 and m2 which are separated by a distance r. If
we calculate its rotational energy by applying
B Bohr's rule of angular momentum quantization,
its energy will be given by : (n is an integer)
32 31
(a) mF (b) mF (m1 + m2 )2 n 2 h2
23 23 (a)
2m12 m22 r 2
33 34
(c) mF (d) mF
23 23 n2 h 2
(b)
47. The position of a projectile launched from the 2(m1 + m2 )r 2
r
( )
origin at t = 0 is given by r = 40iˆ + 50 ˆj m at t =
2 n2 h 2
2s. If the projectile was launched at an angle q (c)
(m1 + m2 )r 2
from the horizontal, then q is
(take g = 10 ms–2) (m1 + m2 )n 2 h2
(d)
2 3 2m1m2 r 2
(a) tan -1 (b) tan
-1
3 2 50. An ideal ammeter (zero resistance) and an ideal
4 voltmeter (infinite resistance) are connected as
7 -1
(c) tan -1 (d) tan shown. The ammeter and voltmeter reading for
4 5
R1 = 5 W, R2 = 15 W, R3 = 1.25 W and E = 20 V
are given as
EBD_7206
MT-34 JEE MAIN

R1=5W
R2 =5W
R3= 1.25W

E = 20 V
(a) 6.25 A, 3.75 V (b) 3.00 A, 5 V
+Q
(c) 3.75 A, 3.75 V (d) 6.25 A, 6.25 V
51. Consider an optical communication system
operating at a wavelength of 800 nm. Suppose, Q Q
only 1% of the optical source frequency is the (a) (b) 4e
16e 0 0
available channel bandwidth for optical
communication. How many channels can be Q
accommodated for transmitting audio signals (c) (d) None of these
8e 0
requiring a bandwidth of 8 kHz ?
(a) 4.8× 108 (b) 48 54. Two soap bubbles coalesce to form a single
(c) 6.2 × 108 (d) 4.8 × 105 bubble. If V is the subsequent change in volume
52. A monoatomic ideal gas is filled in a of contained air and S change in total surface
nonconducting container. The gas can be area, T is the surface tension and P atmospheric
compressed by a movable nonconducting pressure, then which of the following relation is
piston. The gas is compressed slowly to 12.5% correct?
of its initial volume. (a) 4PV + 3ST = 0 (b) 3PV + 4ST = 0
(c) 2PV + 3ST = 0 (d) 3PV + 2ST = 0
Find final temperature of the gas if it is T0 initially –
55. A point particle of mass 0.1 kg is executing SHM
(a) 4T0 (b) 3T0
of amplitude of 0.1m. When the particle passes
(c) 2/3 T0 (d) T0 through the mean position, its kinetic energy is
53. A point charge +Q is positioned at the center of 18 × 10–3 J. The equation of motion of this particle
the base of a square pyramid as shown. The flux when the initial phase of oscillation is 45° can be
through one of the four identical upper faces of given by –
the pyramid is –
æ pö
(a) y = 0.1 cos ç 6t + ÷
è 4ø
æ pö
(b) y = 0.1 sin ç 6t + ÷
è 4ø
Mock Test-3 MT-35

æ pö 59. The length of a magnet is large compared to its


(c) y = 0.4 sin ç t + ÷
è 4ø width and breadth. The time period of its
æp ö oscillation in a vibration magnetometer is 2s. The
(d) y = 0.2 sin ç + 2t ÷ magnet is cut along its length into three equal
è2 ø
56. What happens when the applied load increases parts and these parts are then placed on each
and upto breaking stress in the experiment to other with their like poles together. The time
determine the Young's modulus of elasticity ? period of this combination will be
(a) The area of wire goes on decreasing and 2
wire extends and breaks. (a) 2 3 s (b) s
3
(b) The area of wire goes on increasing and 2
wire breaks. (c) 2 s (d) s
3
(c) The wire extends and area remains constant.
(d) The area remains same and wire length is 60. Hot water cools from 60°C to 50°C in the first 10
also same. minutes and to 42°C in the next 10 minutes. The
57. Currents of a 10 ampere and 2 ampere are passed temperature of the surroundings is:
through two parallel thin wires A and B (a) 25°C (b) 10°C
(c) 15°C (d) 20°C
respectively in opposite directions. Wire A is
infinitely long and the length of the wire B is 2 m.
MATHEMATICS
The force acting on the conductor B, which is
situated at 10 cm distance from A will be 61. Let L1 be a straight line passing through the
origin and L2 be the straight line x + y = 1. If the
(a) 8 × 10–5 N (b) 5 × 10–5 N intercepts made by the circle x2 + y2 – x + 3y = 0
(c) 8p × 10–7 N (d) 4p × 10–7 N on L1 and L2 are equal, then which of the
58. An insect crawls up a hemispherical surface very following equation can represent L1 ?
slowly. The coefficient of friction between the (a) x + 7y = 0 (b) x – y = 0
insect and the surface is 1/3. If the line joining the (c) x – 7y = 0 (d) Both (a) and (b)
centre of the hemispherical surface to the insect 62. Let a1, a2 and b1, b2 be the roots of ax2 + bx + c =
makes an angle a with the vertical, the maximum 0 and px2 + qx + r = 0 respectively. If the system
possible value of a so that the insect does not of equationsa1y + a2z = 0 and b1y + b2z = 0 has a
slip is given by non-trivial solution, then
b2 ac c2 ab
(a) = (b) =
a q2 pr r 2 pq
2
(a) cot a = 3 (b) sec a = 3 a bc
(c) = (d) None of these
2 qr
(c) cosec a = 3 (d) cos a = 3 p
EBD_7206
MT-36 JEE MAIN

63. The equation sin x + x cos x = 0 has at least one 68. Let ‘a’ denote the roots of equation
root in –
æ 1+ x2 ö
æ p ö cos (cos–1 x) + sin–1 sin ç ÷ = 2 sec–1 (sec x)
(a) çè - , 0÷ø (b) (0, p) è 2 ø
2
æ 3p ö æ pö then possible values of [ | 10a | ] where [ . ]
(c) çè p , ÷ø (d) çè 0, ÷ø denotes the greatest integer function will be
2 2
(a) 1 (b) 5
64. The area above the x–axis enclosed by the
curves x2–y2 = 0 and x2 + y – 2 = 0 is (c) 10 (d) Both (a) and (c)
69. The two of the straight lines represented by the
5 7 equation ax3 + bx2y + cxy2 + dy3 = 0 will be at
(a) (b)
3 3 right angle if –
(a) a2 + c2 = 0
8 10 (b) a2 + ac + bd + d2 = 0
(c) (d) (c) a2c2 + bd + d2 = 0
3 3
65. Number of permutations 1, 2, 3, 4, 5, 6, 7, 8 and 9 (d) None of these
taken all at a time are such that the digit. 70. If x2 – 2x cos q + 1 = 0, then the value of x2n – 2xn
1 appearing somewhere to the left of 2, 3 cos nq + 1, n Î N is equal to –
appearing to the left of 4 and 5 somewhere to the (a) cos 2nq
left of 6, is (b) sin 2nq
(e.g., 815723946 would be one such permutation) (c) 0
(a) 9 . 7! (b) 8! (d) some real number greater than 0
(c) 5! . 4! (d) 8! . 4!
dx x6
66. If the function f : [0, 16] ® R is differentiable. If 0 71. If ò x + x7 = p( x) then, ò x + x7 dx is equal to:
16
< a < 1 and 1 < b < 2, then ò f (t) dt is equal to – (a) ln | x | – p (x) + c
(c) x – p (x) + c
(b) ln | x | + p (x) + c
(d) x + p (x) + c
0
(a) 4 [a3f (a4) – b3 f (b4)] 72. Vertices of a parallelogram taken in order are A
(2, –1, 4), B (1, 0, –1), C (1, 2, 3) and D. Distance
(b) 4 [a3f (a4) + b3 f (b4)]
of the point P (8, 2, –12) from the plane of the
(c) 4 [a4f (a3) + b4 f (b3)] parallelogram is –
(d) 4 [a2f (a2) + b2 f (b2)]
67. Three distinct points P (3u2, 2u3), Q (3v2, 2v3) 4 6 32 6
(a) (b)
and R (3w2, 2w3) are collinear then – 9 9
(a) uv + vw + wu = 0
16 6
(b) uv + vw + wu = 3 (c) (d) None of these
(c) uv + vw + wu = 2 9
(d) uv + ww + wu = 1
Mock Test-3 MT-37

73. Given equation of the locus of R is –


r r r (a) x2 + 2y2 – ax = 0 (b) 2x2 + y2 – 2ax = 0
A = 2iˆ + 3jˆ + 6k,
ˆ B = iˆ + ˆj - 2kˆ and C = iˆ + 2jˆ +. kˆ
r r r r r (c) 2x + 2y – ay = 0 (d) 2x2 + y2 – 2ay = 0
2 2
Compute the value of | A ´ [A ´ (A ´ B)].C | . 79. A box contains 6 red, 5 blue and 4 white marbles.
(a) 343 (b) 512 Four marbles are chosen at random without
replacement. The probability that there is atleast
(c) 221 (d) 243
one marble of each colour among the four
74. The value of the definite integral chosen, is –
3p / 4 48 44
ò [(1 + x)sin x + (1 - x) cos x ] dx is – (a)
91
(b)
91
0
88 24
3p p (c) (d)
(a) 2 tan (b) 2 tan 91 91
8 4 80. If z + 1/z = 2 cos q, then the value of |(z2n – 1)/
p (z2n + 1)|
(c) 2 tan (d) 0 (a) | tan n q | (b) tan n q
8
(c) | cot n q | (d) cot n q
75. Area of triangle formed by common tangents to 81. The two curves x3 – 3xy2 + 2 = 0 and 3x2y – y3 = 2
the circle (a) cuts at right angle
x2 + y2 – 6x = 0 and x2 + y2 + 2x = 0 is – (b) touch each other
(a) 3 3 (b) 2 3 p
(c) cut at an angle
(c) 9 3 (d) 6 3 3
76. The locus of the centres of the circles which cut p
the circles x2 + y2 + 4x – 6y + 9 = 0 and x2 + y2 – (d) cut at an angle
4
5x + 4y – 2 = 0 orthogonally is –
(a) 9x + 10y – 7 = 0 (b) x – y + 2 = 0 82. If the pair of lines ax 2 + 2 (a + b)xy + by 2 = 0 lie
(c) 9x – 10y + 11 = 0 (d) 9x + 10y + 7 = 0 along diameters of a circle and divide the circle
77. The sum to infinity of the series into four sectors such that the area of one of the
sectors is thrice the area of another sector then
1 1 1 1
+ + + + ....., is (a) 3a 2 - 10 ab + 3b 2 = 0
1 1 + 2 1 + 2 + 3 1+ 2 + 3 + 4
equal to – (b) 3a 2 - 2ab + 3b 2 = 0
(a) 3 (b) 1
(c) 2 (d) 3/2 (c) 3a 2 + 10 ab + 3b 2 = 0
78. The straight line joining any point P on the (d) 3a 2 + 2ab + 3b 2 = 0
parabola
83. Which of the following is a contradiction?
y2 = 4ax to the vertex and perpendicular from the
focus to the tangent at P, intersect at R, then the (a) (p Ù q)Ù ~ (p Ú q) (b) p Ú (- p Ù q)
(c) (p Þ q) Þ p (d) None of these
EBD_7206
MT-38 JEE MAIN

sin q + sin 2q 88. Set of values of m for which two points P and Q
84. Period of is lie on the line y = mx + 8 so that Ð APB = Ð AQB
cos q + cos 2q p
(a) 2p (b) p = where A º (– 4, 0), B º (4, 0) is –
2
2p p
(c) (d) (a) ( -¥, - 3) È ( 3, ¥ ) - {-2, 2}
3 3
85. If 1, w, w2 ,........ wn-1 are the n roots of unity,, (b) [ - 3, - 3] - {-2, 2}
(c) (-¥, -1) È (1, ¥) - {-2, 2}
then (1 - w) (1 - w2 )........(1 - wn-1 ) equals
(a) 0 (b) 2 (d) {- 3, 3}
(c) n (d) n2
86. If s, s' are the length of the perpendicular on a 89. The trace Tr(A) of a 3 × 3 matrix A = (aij ) is defined
tangent from the foci, a, a' are those from the by the relation Tr(A) = a11 + a22 + a33 (i.e., Tr(A) is
vertices is that from the centre and e is the sum of the main diagonal elements). Which of the
x2 y2 following statements cannot hold ?
eccentricity of the ellipse, + = 1 , then
a2 b2 (a) Tr(kA) = kTr(A) (k is a scalar)
ss ¢ - c 2 (b) Tr(A + B) = Tr(A) + Tr(B)
=
aa ¢ - c 2 (c) Tr(I3) = 3
(a) e (b) 1/e
(d) Tr(A2) = Tr(A)2
(c) 1/e2 (d) e2
87. One percent of the population suffers from a p /2
certain disease. There is blood test for this 90. Let a n = ò (1 - sin t)n sin 2t dt then
disease, and it is 99% accurate, in other words, 0
the probability that it gives the correct answer is n
an
0.99, regardless of whether the person is sick or
healthy. A person takes the blood test, and the
lim
n ®¥
å n is equal to
1
result says that he has the disease. The
probability that he actually has the disease, is – (a) 1/2 (b) 1
(a) 0.99% (b) 25% (c) 4/3 (d) 3/2
(c) 50% (d) 75%
Download From https://iit-jeeacademy.blogspot.com

4 JEE MAIN
MOCK TEST
Time : 3 hrs. Max. Marks : 360

INSTRUCTIONS
• Chemistry (120 marks) : Question No. 1 to 30 are of 4 marks each.

• Physics (120 marks) : Question No. 31 to 60 are of 4 marks each.

• Mathematics (120 marks) : Question No. 61 to 90 are of 4 marks each.


• Negative Marking : One fourth (¼) marks will be deducted for indicating incorrect
response of each question.

3. In a set of reactions p-nitrotoluene yielded a


CHEMISTRY product E.

1. A crystal of ZnO is heated evolve O2 and forms CH3


defect; the type of defects formed will be
Br Sn/HCl
(a) Metal excess (b) Metal deficiency ¾¾¾
2 ® B ¾¾¾¾
®C
FeBr3
(c) Impurities (d) None of these
2. Which of the following structures does not NO2
contain any chiral C atom but represent the
chirality in the structure. NaNO CuBr
¾¾¾¾
2 ® D ¾¾¾® E
(a) 2 – Ethyl – 3 – hexene HCl HBr
(b) 2, 3-Pentadiene The product E would be:
(c) 1,3 – Butadiene
(d) Pent – 3 – en – 1 – yne
EBD_7206
MT-40 JEE MAIN

6. Given
CH3 CH3
(A) n = 5, ml = + 1
Br Br Br
(B) n = 2, l =1, ml = –1, ms = –l/2
(a) (b) The maximum number of electron(s) in an atom
that can have the quantum numbers as given in
Br Br (A) and (B) are respectively:
(a) 25 and 1 (b) 8 and 1
CH3 CH2 Br (c) 2 and 4 (d) 4 and 1
Br 7. ‘a’ gm of element A reacts with ‘b’ gm of element
(c) (d) B. Also ‘b’ gm of element B combines with 2c gm
Br of C. If one gm equivalent of B weighs ‘b’ gm.
Then the mass ratio in which A and C combine is
Br
(a) a : c (b) a : 2c
4. van der Waal’s equation for a gas is stated as, (c) 2a : c (d) 3a : c
2
8. N2 and O2 are converted to mono cations N2+
nRT ænö and O2+ respectively, which of the following is
P= -aç ÷ .
V - nb èVø wrong?
This equation reduces to the perfect gas (a) In N2+, the N – N bond weakens
(b) In O2+, the O – O bond order increases
nRT (c) In O2+, paramagnetism decreases
equation, P = when,
V (d) N2+ becomes diamagnetic
(a) temperature is sufficient high and pressure 9. The reaction in which hydrogen peroxide acts
is low. as a reducing agent is
(b) temperature is sufficient low and pressure (a) PbS + 4H 2 O 2 ® PbSO 4 + 4H 2 O
is high.
(b) 2 Kl + H 2O 2 ® 2KOH + I 2
(c) both temperature and pressure are very
high. (c) 2 FeSO 4 + H 2SO 4 + H 2 O 2 ®
(d) both temperature and pressure are very low. Fe 2 (SO 4 ) 3 + 2H 2 O
5. Reduction with aluminium isopropoxide in excess
of isopropyl alcohol is called Meerwein (d) Ag 2 O + H 2 O 2 ® 2Ag + H 2 O + O 2
Ponndorff-Verley reduction (MPV). What will be 10. Reaction of CH — CH with RMgX leads to
2 2
the final product when cyclohex-2-enone is O
selectively reduced in MPV reaction ? formation of
(a) Cyclohexanol (b) Cyclohex-2-enol (a) RCHOHR (b) RCHOHCH3
(c) Cyclohexanone (d) Benzene R
(c) RCH2CH2OH (d) CHCH2OH
R
Mock Test -4 MT-41

11. Which reaction will not yield an amide? 15. When tert-butyl chloride is made to react with
O sodium methoxide, the major product is
||
(a) dimethyl ether
(a) C 2 H 5 – C – Cl + NH 3
(b) di-tert-butyl ether
O O (c) tert-butylmethyl ether
|| ||
(b) C 2 H 5 – C – O - C - Cl + CH 3 NH 2 (d) isobutylene
O 16. If s0, s1, s2 and s3 are the solubilities of AgCl in
|| water, 0.01 M CaCl2, 0.01 M NaCl and 0.05 M
(c) CH 3 – C – Cl + (CH 3 ) 3 N AgNO3 solutions, respectively, then
O O (a) s0 > s1 > s2 > s3 (b) s0 > s2 > s1 > s3
|| ||
(d) CH 3 – C – O – C – CH 3 (c) s0 > s2 > s3 > s1 (d) s0 > s1 = s2 > s3
+ CH 3 NH – CH 2 – CH 3 17. Which of the following compounds exhibit
tautomerism?
12. How many different dipeptides can be formed
by two different amino acids ? (a) Chloroethane (b) Ethanol
(a) 4 (b) 1 (c) Ethoxyethane (d) Nitroethane
(c) 3 (d) 2 18. An organic compound is treated with NaNO 2
13. If m and e are the mass and charge of the revolving and dil. HCl at 0°C. The resulting solution is
electron in the orbit of radius r for hydrogen atom,
added to an alkaline solution of b -naphthol
the total energy of the revolving electron will be:
where by a brilliant red dye is produced. It shows
1 e2 e2 the presence of
(a) (b) -
2 r r (a) – NO 2 group

me2 1 e2 (b) aromatic – NH 2 group


(c) (d) -
r 2 r (c) – CONH 2 group
14. Dinitrogen and dioxygen are main constituents
(d) aliphatic – NH 2 group
of air but these do not react with each other to
form oxides of nitrogen because __________. 19. The vapour pressure of benzene at a certain
(a) the reaction is endothermic and requires temperature is 640 mm of Hg. A non volatile and
very high temperature. non electrolyte solid weighing 2.175 g is added
to 39.08 g of benzene. If the vapour pressure of
(b) the reaction can be initiated only in
the solution is 600mm of Hg, what is the
presence of a catalyst.
molecular weight of solid substance?
(c) oxides of nitrogen are unstable.
(a) 49.50 (b) 59.60
(d) N2 and O2 are unreactive
(c) 69.60 (d) 79.87
EBD_7206
MT-42 JEE MAIN

20. Point out the incorrect statment among the 24. Which reagent can be used to identify nickel ion?
following : (a) Resorcinol
(a) The oxidation state of oxygen is +2 in OF2. (b) Dimethyl glyoxime
(b) Acidic character follows the order H2O < (c) Diphenyl benzidine
H2S < H2Se < H2Te. (d) Potassium ferrocyanide
(c) The tendency to form multiple bonds 25. When pink [Co(H2O)6 ]2Å is dehydrated the
increases in moving down the group from colour changes to blue. The correct explanation
sulphur to tellurium (towards C and N) for the change is :
(d) Sulphur has a strong tendency to catenate (a) The octahedral complex becomes square
while oxygen shows this tendency to planar.
alimited extent. (b) A tetrahedral complex is formed.
21. Removal of Fe, Cu, W from Sn metal after smelting (c) Distorted octahedral structure is obtained.
is by ............... because ............. (d) Dehydration results in the formation of
(a) Poling; of more affinity towards oxygen for polymeric species.
impurities 26. When CO2 dissolves in water, the following
(b) Selective oxidation; of more affinity towards equilibrium is established
oxygen for impurities
(c) Electrolytic refining; impurities undissolved CO2 + 2H2O H3O+ + HCO3–; for which
in elec-trolyte the equilibrium constant is 3.8 ´ 10–6 and pH = 6.0.
(d) Liquation; Sn having low melting point What would be the ratio of concentration of
compared to impurities. bicarbonate ion to carbon dioxide?
(a) 3.8 ´ 10–12 (b) 3.8
22. Among KO 2 , AlO-2 , BaO2 and NO +2 ,unpaired (c) 6 (d) 13.4
electron is present in 27. In the form of dichromate, Cr (VI) is a strong
oxidising agent in acidic medium but Mo (VI) in
(a) NO +2 and BaO 2 (b) KO 2 and AlO-2
MoO3 and W (VI) in WO3 are not because
(c) KO 2 only (d) BaO 2 only ____________ .
23. If a 0.1 M solution of glucose (Mol. wt 180) and (i) Cr (VI) is more stable than Mo(VI) and W (VI).
0.1 molar solution of urea (Mol. wt. 60) are placed
(ii) Mo (VI) and W(VI) are more stable than Cr(VI).
on two sided semipermeable membrane to equal
(iii) Higher oxidation states of heavier members
heights, then it will be correct to say that
of group-6 of transition series are more
(a) there will be no net movement across the
stable.
membrane
(b) glucose will flow across the membrane into (iv) Lower oxidation states of heavier members
urea solution of group-6 of transition series are more
stable.
(c) urea will flow across the membrane into
glucose solution (a) (i) and (ii) (b) (ii) and (iii)
(d) water will flow from urea solution to glucose (c) (i) and (iv) (d) (ii) and (iv)
solution
Mock Test -4 MT-43

28. A reaction rate constant is given by 32. The length of an elastic string is x when the tension
K = 1.2 ´ 1010 e-2500/RT . It means is 5N. Its length is y when the tension is 7N.
(a) log K vs T will give a straight line What will be its length, when the tension is 9N?
(b) log K vs 1/T gives a straight line with a (a) 2y + x (b) 2y – x
slope – 2500/2.303 R (c) 7x – 5y (d) 7x + 5y
(c) half life of reaction will be more at higher 33. A and B are two sources generating sound
temperature waves. A listener is situated at C. The frequency
(d) log K vs 1/T gives a straight line with a of the source at A is 500 Hz. A, now, moves
slope 2500/R towards C with a speed 4 m/s. The number of
29. The correct statement among the following is : beats heard at C is 6. When A moves away from
(a) The alkali metals when strongly heated in C with speed 4 m/s, the number of beats heard at
oxygen form superoxides. C is 18. The speed of sound is 340 m/s. The
(b) Caesium is used in photoelectric cells. frequency of the source at B is :
(c) NaHCO3 is more soluble in water than A C B
1KHCO3.
(d) The size of hydrated ions of alkali metals (a) 500 Hz (b) 506 Hz
increases from top to bottom. (c) 512 Hz (d) 494 Hz
30. If the following half cells have E° values as 34. A rod of length L is placed on x – axis between
A3+ + e– ––––® A2+, E° = y2V x = 0 and x = L. The linear density i.e., mass per
A2+ + 2e– ––––® A, E° = –y1V unit length denoted by r, of this rod, varies as, r
The E° of the half cell A3+ + 3e ––––® A will be = a + bx. What should be the dimensions of b?
2 y1 - y2 y2 - 2 y1 (a) M2L1T 0 (b) M1 L–2 T 0
(a) (b) –1
(c) M L T 3 1 (d) M–1 L2T 3
3 3
(c) 2y1 – 3y2 (d) y2 – 2y1 35. A body is thrown vertically upwards from the
surface of earth in such a way that it reaches
PHYSICS upto a height equal to 10Re. The velocity
imparted to the body will be
31. A car is standing 200 m behind a bus, which is (a) 10.6 km/s (b) 0.106 km/s
also at rest. The two start moving at the same
(c) 1.06 km/s (d) zero
instant but with different forward accelerations.
The bus has acceleration 2 m/s2 and the car has 36. A projectile can have the same range ‘R’ for two
acceleration 4 m/s2. The car will catch up the bus angles of projection. If ‘T1’ and ‘T2’ to be time of
after a time of : flights in the two cases, then the product of the
two time of flights is directly proportional to
(a) 110 s (b) 120 s 1
(a) R (b)
(c) 10 2 s (d) 15 s R
1
(c) (d) R 2
R2
EBD_7206
MT-44 JEE MAIN

37. A metallic wire of density d is lying horizontal 41. A pendulum made of a uniform wire of cross
on the surface of water. The maximum length of sectional area A has time period T. When an
wire so that it may not sink will be additional mass M is added to its bob, the time

2Tg 2pT period changes to TM. If the Young's modulus of


(a) (b) dg
pd 1
the material of the wire is Y thenis equal to:
2T Y
(c) (d) any length (g = gravitational acceleration)
pdg
38. Two points of a rod move with velocities 3v and é æ T ö2 ù A é æ T ö2 ù A
(b) ê1 - ç T ÷ ú Mg
M
v perpendicular to the rod and in the same (a) ê1 - ç ÷ ú
ë è T ø û Mg êë è M ø úû
direction, separated by a distance r. Then the
angular velocity of the rod is éæ T ö2 ù A éæ T ö2 ù Mg
M
(a) 3v/r (b) 4v/r (c) êç ÷ - 1ú (d) êç M ÷ - 1ú
ëè T ø û Mg ëè T ø û A
(c) 5v/r (d) 2v/r
39. Which logic gate with inputs A and B performs 42. A bucket full of hot water is kept in a room and it
the same operation as that performed by the cools from 75oC to 70oC in T1 minutes, from 70oC
following circuit? to 65oC in T2 minutes and from 65oC to 60oC in
A T3 minutes. Then
B (a) T1= T2 = T3 (b) T1 < T2 < T3
(c) T1 > T2 > T3 (d) T1< T3< T2
43. The equivalent capacity of the network, (with all
V capacitors having the same capacitance C)
Lamp
A

(a) NAND gate (b) OR gate


B
(c) NOR gate (d) AND gate
40. A non-conducting partition divides a container into (a) ¥ (b) zero
two equal compartments. One is filled with helium (c) C[(Ö3 – 1) / 2] (d) C[(Ö3 + 1) / 2]
gas at 200 K and the other is filled with oxygen gas 44. The load versus elongation graphs for four wires
at 400 K. The number of molecules in each gas is of same length and made of the same material
the same. If the partition is removed to allow the are shown in the figure. The thinnest wire is
gases to mix, the final temperature will be represented by the line
(a) 350 K (b) 325 K
(c) 300 K (d) 275 K
Mock Test -4 MT-45

Load D le 1 me le mp
(c) = (d) =2
C l p 2 mp lp me
B
47. A straight section PO of a circuit lies along the x-
A
axis from x = –a/2 to x = +a/2, and carries a steady
current ‘I’. The magnitude of magnetic field due
to the section PO at a point to y = + a will be
O Elongation
(a) proportional to a (b) proportional to a2
(a) OA (b) OC
(c) OD (d) OB (c) proportional to 1/a (d) equal to zero
45. Six equal resistances are connected between 48. A transformer is used to light a 140 W, 24 V bulb
points P, Q and R as shown in figure. Then net from a 240 V a.c. mains. The current in the main
resistance will be maximum between : cable is 0.7 A. The efficiency of the transformer is
P (a) 63.8 % (b) 83.3 %
(c) 16.7 % (d) 36.2 %
49. In the given circuit, the current drawn from the
r
source is
r V = 100x sin(100pt )
r
r

X C = 20W
X L = 10W
r
~ R = 20W
Q R
r

(a) P and R (b) P and Q (a) 20 A (b) 10 A


(c) Q and R (d) Any two points (c) 5 A (d) 5 2 A
46. Electrons are accelerated through a potential
difference V and protons are accelerated through 50. The surface charge density of a thin charged disc
a potential difference 4 V. The de-Broglie of radius R is s. The value of the electric field at
wavelengths are le and lp for electrons and
s
le the centre of the disc is . With respect to
protons respectively. The ratio of is given 2 Î0
lp the field at the centre, the electric field along the
by : (given me is mass of electron and mp is mass axis at a distance R from the centre of the disc :
of proton).
(a) reduces by 70.7% (b) reduces by 29.3%
le mp le me (c) reduces by 9.7% (d) reduces by 14.6%
(a) = (b) =
lp me lp mp
EBD_7206
MT-46 JEE MAIN

51. A flat plate P of mass ‘M’ executes SHM in a (a) will not be there
horizontal plane by sliding over a frictionless (b) will not be there if the intensity of light
surface with a frequency V. A block ‘B’ of mass reaching the screen from S1 and S2 are equal.
‘m’ rests on the plate as shown in figure. (c) will be there under all circumstances
Coefficient of friction between the surface of B (d) we will have only the central fringe
and P is m. What is the maximum amplitude of 54. What is the ratio of the circumference of the first
oscillation that the plate block system can have Bohr orbit for the electron in the hydrogen atom
if the block B is not to slip on the plate : to the de Brogile wavelength of electrons having
the same velocity as the electron in the first Bohr
orbit of the hydrogen atom?
(a) 1 : 1 (b) 1 : 2
(c) 1 : 4 (d) 2 : 1
55. In frequency modulation
mg mg
(a) (b) (a) the amplitude of modulated wave varies as
4p 2 V 2 4p 2 V frequency of carrier wave
m mg (b) the frequency of modulated wave varies as
(c) (d)
4p 2 V 2 g 2p 2 V 2 amplitude of modulating wave
52. A glass slab has the left half of refractive index (c) the amplitude of modulated wave varies as
n1, and the right half of n2=3n 1. The effective amplitude of carrier wave
refractive index of the whole slab is
(d) the frequency of modulated wave varies as
n1 frequency of modulating wave
(a) (b) 2n
2 56. The radioactivity of a sample is R1 at a time T1
3n 1 2n 1 and R2 at a time T2. If the half life of the specimen
(c) (d) is T, the number of atoms that have disintegrated
2 3
53. In the arrangement shown L1, L2 are slits and in the time (T2–T1) is proportional to
S1, S2 two independent sources on the screen, (a) (R1T1 – R2T2) (b) (R1 – R2)
interference fringes (c) (R1 – R2)/T (d) (R1– R2) × T
screen
57. The electric field associated with an e.m. wave
L1 r
in vacuum is given by E = iˆ 40 cos (kz – 6 ×
× × 108t), where E, z and t are in volt/m, meter and
S1 S2
seconds respectively. The value of wave vector
L2 k is
(a) 2 m–1 (b) 0.5 m–1
(c) 6 m –1 (d) 3 m–1
Mock Test -4 MT-47

58. P–V plots for two gases during adiabatic


processes are shown in the figure. Plots 1 and 2 MATHEMATICS
should correspond respectively to
61. The roots of the given equation
(p – q) x 2 + (q – r) x + (r – p) = 0 are :
p-q q-r
P (a) ,1 (b) ,1
1 r-p p-q
r-p
2 (c) ,1 (d) None of these
p-q
V 62. If a, b, c, d and p are distinct non zero real
(a) He and Ar (b) He and O2 numbers such that (a2 + b2 + c2) p2 –2(ab + bc
(c) O2 and N2 (d) O2 and He + cd)p + (b2 + c2+ d2) £ 0 then a,b,c,d are in
59. Two small equal point charges of magnitude q (a) A.P. (b) G.P.
are suspended from a common point on the
ceiling by insulating mass less strings of equal (c) H.P. (d) satisfy ab = cd
lengths. They come to equilibrium with each 63. Which of the following is correct?
string making angle q from the vertical. If the
(a) If a 2 + 4b 2 = 12ab , then
mass of each charge is m, then the electrostatic
potential
æ 1 at the ö centre of line joining them will be 1
ç =k÷ log(a + 2b) = (log a + log b)
è 4p Î0 ø. 2

log x log y log z


(b) If = = ,
(a) 2 k mg tan q (b) k mg tan q b-c c-a a-b

(c) 4 k mg / tan q (d) k mg / tan q then x a .y b .zc = abc

60. A magnetic needle lying parallel to a magnetic 1 1 1


(c) + + =2
field requiers W units of work to turn it through log xy xyz log yz xyz log zx xyz
600 . The torque needed to maintain the needle
(d) All are correct
in this position will be 64. If the number of 5-element subsets of the set
(a) 3W (b) W A= {a1, a2, ...., a20} of 20 distinct elements is k
times the number of 5-element subsets
3 containing a4, then k is
(c) W (d) 2 W
2 20
(a) 5 (b)
7
10
(c) 4 (d)
3
EBD_7206
MT-48 JEE MAIN

65. The value of cos360 cos420 cos780 is


p p
70. If w = cos + i sin , then value of
é 5 -1 5 + 1ù n n
ê Given : sin18 = and cos 36 = ú
ë 4 4 û
1 + w + w 2 + ... + w n-1 is
(a) 1/4 (b) 1/8
(c) 1/16 (d) [(Ö5 – 1)/4]2 (a) 1 + i (b) 1 + i tan (p/n)
66. If x =1/5, the value of cos (cos–1x + 2 sin–1 x) is (c) 1 + i cot ( p / 2n) (d) None of these
24 24 71. The circles x2 + y2 – 2x – 15 = 0 and x2 + y2 + 4y
(a) - (b)
25 25 + 3 = 0 have
1 1 (a) no common tangent
(c) - (d)
5 5 (b) one common tangent
p (c) three common tangents
67. If 0 < a, b, g < p/2 such that a + b + g = and cot
2 (d) four common tangents.
a, cot b, cot g are in arithmetic progression, then 72. Which of the following is correct?
the value of cot a cot g is (a) If A and B are square matrices of order 3
(a) 1 (b) 3 such that | A | = –1, | B | = 3, then the determinant
(c) cot 2 b (d) cot a + cot g of 3 AB is equal to 27.
68. Let P (3 sec q, 2 tan q) and Q (3 sec f, 2 tan f)
(b) If A is an invertible matrix, then det (A–1) is
p equal to det (A)
where q + f = , be two distinct points on the
2 (c) If A and B are matrices of the same order, then
x2 y2 (A + B)2 = A2 + 2AB + B2 is possible if AB = I
hyperbola - = 1 . Then the ordinate of the
9 4 (d) None of these
point of intersection of the normals at P and Q is: 73. If the system of linear equations :
11 11 x1 + 2x2 + 3x3 = 6
(a) (b) - x1 + 3x2 + 5x3 = 9
3 3
2x1 + 5x2 + ax3 = b
13 13
(c) (d) - . is consistent and has infinite number of solutions,
2 2
then :
69. If q1, q2 are the solutions of the equation
(a) a = 8, b can be any real number
2tan 2q – 4tanq + 1 = 0, then tan (q1 + q2) is equal
(b) b = 15, a can be any real number
to
(a) –4 (b) 4 (c) a Î R - {8} and b Î R - {15}
(c) 1 (d) 2 (d) a = 8, b = l5
Mock Test -4 MT-49

74. Let f(x) = | x – 1 |. Then 78. The set of points where f(x) = (x – 1)2 (x + | x –1 | )
(a) f(x2) = (f(x))2 (b) f(x + y) = f(x) + f(y) is thrice differentiable, is
(c) f(| x |) = | f(x) | (d) None of these (a) R (b) R – {0}
(c) R – {1} (d) R–{0,1}
2a 2b 79. Let f (x) = 1/(x – 1) and g (x) = 1/(x2 + x – 2). Then the
75. If sin -1 2
+ sin -1 2
= 2 tan -1 x, then
1+ a 1+ b set of points where (gof)(x) is discontinuous, is
x is equal to (a) {1} (b) {–2,1}
(c) {1/2, 1, 2} (d) {1/2 , 1}
a-b b
(a) (b) m
1 + ab 1 + ab 80. å n + r Cn is equal to :
b a+b r =0
(c) (d)
1 - ab 1 - ab (a)
n + m +1
Cn +1
76. If AB = 0, then for the matrices
n + m +2
(b) Cn
é 2
cos q sin qù
A = ê cos q ú n + m +3
C n -1
(c)
ëêcos q sin q sin 2 q ûú
(d) None of these
é cos f 2
cos f sin fù
and B = ê ú , q – f is x - {x + 1 }
êëcos f sin f sin 2 f úû 81. Let f ( x) = ; where {x} is the
x - {x + 2 }
p
(a) an odd number of
2 fractional part of x, then lim f ( x )
x®1 / 3
(b) an odd multiple of p (a) has value 0 (b) has value 1
p (c) has value –¥ (d) has value ¥
(c) an even multiple of
2 ( 2 + sin x ) dy = cos x
82. Let y (x) be a solution of .
(d) 0 (1 + y ) dx
2
77. Let f (x) = , g(x) = cos x and h(x) = x + 3 æ pö
x +1 If y (0) = 2, then y ç ÷ equals
è 2ø
then the range of the composite function fogoh,
is 5
(a) (b) 2
(a) R+ (b) R – {0} 2
(c) [1, ¥) (d) R+ – {1} 7
(c) (d) 3
2
EBD_7206
MT-50 JEE MAIN

88. The function f (x) = (x – 3)2 satisfies all the


1
83.
ò f (x)
dx = log(f (x))2 + C, then f(x) is conditions of mean value theorem in {3, 4}. A
point on y = (x –3)2, where the tangent is parallel
(a) x + a (b) 2x + a to the chord joining (3, 0) and (4,1) is
x æ 7 1ö æ 7 1ö
(c) +a (d) x 2 + a
2 (a) çè , ÷ø (b) ç , ÷
è 2 4ø
2 2
2a f (x) (c) (1, 4) (d) (4, 1)
84. Value of ò0 f (x) + f (2a - x)
dx is

1 a b
p 89. In a DABC, if 1 c a = 0, then
(a) 0 (b)
4
1 b c
(c) a (d) None of these

85. The value of sin 2 A + sin 2 B + sin 2 C =

p/4
9 4
(a) (b)
ò 9
3 2 4
(x | x | + sin x + x tan x + 1) dx is
-p / 4 (c) 1 (d) 3 3
(a) 0 (b) 1
2x 2y 2z
(c) p/4 (d) p/2 90. If x + y – z + xyz = 0, then + -
2 2
1- x 1- y 1- z2
86. Let (1 – x – 2x2)6 = 1+ a1x + a2x2 + …. + a12 x12. is equal to
a2 a4 a6 a12 xyz
Then 2
+ 4
+ 6
+ ...... + is equal to (a)
2 2 2 212 [(1 - x )(1 - y 2 )(1 - z 2 )]
2

(a) –1 (b) –1/2


- xyz
(c) 0 (d) 1/2 (b)
[(1 - x )(1 - y 2 )(1 - z 2 )]
2
87. The equation of a common tangent to y2 = 4x
and the curve x2 + 4y2 = 8 can be
8xyz
(a) x – 2y + 2 = 0 (b) x + 2y + 4 = 0 (c)
[(1 - x )(1 - y 2 )(1 - z 2 )]
2

(c) x – 2y = 4 (d) x + 2y = 4
-8 xyz
(d)
[(1 - x 2 )(1 - y 2 )(1 - z 2 )]
Download From https://iit-jeeacademy.blogspot.com

5 JEE MAIN
MOCK TEST
Time : 3 hrs. Max. Marks : 360

INSTRUCTIONS
• Chemistry (120 marks) : Question No. 1 to 30 are of 4 marks each.

• Physics (120 marks) : Question No. 31 to 60 are of 4 marks each.

• Mathematics (120 marks) : Question No. 61 to 90 are of 4 marks each.


• Negative Marking : One fourth (¼) marks will be deducted for indicating incorrect
response of each question.

2. Given
CHEMISTRY (A) n = 5, ml = + 1
(B) n = 2, l =1, ml = –1, ms = –l/2
1. In which of the following arrangements, the The maximum number of electron(s) in an atom
sequence is not strictly according to the property that can have the quantum numbers as given in
written against it? (A) and (B) are respectively:
(a) CO2 < SiO2 < SnO2 < PbO2: increasing (a) 25 and 1 (b) 8 and 1 (c) 2 and 4 (d) 4 and 1
oxidising power 3. Aluminothermy used for on the spot welding of
(b) NH3 < PH3 < AsH3 < SbH3: increasing large iron structure is based on the fact that-
(a) As compared to iron, aluminium has greater
basic strength affinity for oxygen.
(c) HF < HCl < HBr < HI: increasing acid (b) As compared to aluminium, iron has greater
strength affinity for oxygen.
(d) B < C < O < N: increasing first ionisation (c) Reaction between aluminium and oxygen is
endothermic.
enthalpy.
(d) Reaction between iron and oxygen is
endothermic.
EBD_7206
MT-52 JEE MAIN

4. Which of the following shows the tendency to 10. Which of the following ions are optically active?
form peroxide?
en en
(a) Lithium (b) Magnesium Cl + Cl +
(c) Beryllium (d) Radium
5. A 1.0 M solution with respect to each of the metal
Co Co
halides AX 3 , BX 2 , CX 3 and DX 2 is
electrolysed using platinum electrodes. If
Cl Cl
E° 3+ = 1.50V, E° 2+ = 0.3V, E° 3+ = –0.74V, en en
A /A B /B C /C
I II
E° 2+ = – 2.37 V. The correct sequence in which en
D /D 3+ en +
Cl
the various metals are deposited at the cathode is
(a) A, B, C, D (b) A, B, C
(c) D, C, B, A (d) C, B, A en Co Co
6. The correct sequence of decreasing number of
p-bonds in the structures of H2SO3, H2SO4 and Cl
en en
H2S2O7 is :
(a) H2SO3 > H2SO4 > H2S2O7 III IV
(b) H2SO4 > H2S2O7 > H2SO3 (a) I only (b) II only
(c) H2S2O7 > H2SO4 > H2SO3 (c) II and III (d) IV only
(d) H2S2O7 > H2SO3 > H2SO4
11. The pair of compounds having metals in their
7. An ideal gas undergoes isothermal expansion
at constant pressure. During the process : highest oxidation state is:
(a) enthalpy increases but entropy decreases. (a) MnO2 and CrO2Cl2
(b) enthalpy remains constant but entropy (b) [NiCl4]2– and [CoCl4]2–
increases.
(c) [Fe(CN)6]3– and [Cu(CN)2]2–
(c) enthalpy decreases but entropy increases.
(d) Both enthalpy and entropy remain (d) [FeCl4]– and Co2O3
constant. 12. Consider a 0.1M solution of two solutes A and
8. Among the reactions given below for B2H6, the B. A behaves as a non-electrolyte while 80% of
one which does not take place is
B dimerises. Which of the following statement
(a) B2H6 + HCl ¾¾ ® B2H5Cl + H2
is correct regarding these solutions?
D
(b) 2B2H6 + 6NH3 ¾¾® B3N3H6 (borazine) (a) The b.pt of A will be less than B
(c) B2H6 + 2N(CH3)3 ¾¾
® 2(CH3)3 NBH3
(b) The osmotic pressure of B will be more than
H 3O + that of A
(d) B2H6 + 6C2H4 ¾¾¾® 3C2 H5OH +
2B(OH)3 (c) The freezing point of solution A will be less
9. Copper crystallises in fcc with a unit length of than that of B
361pm. What is the radius of copper atom ?
(a) 157 pm (b) 128 pm (d) Boiling points of both solutions will be
(c) 108 pm (d) 181 pm same.
Mock Test-5 MT-53

13. A metal gives two chlorides A and B. A gives 18. The rate of SN1 reaction is fastest in the
black precipitate with NH4OH and B gives white. hydrolysis of which of the following halides ?
With KI, B gives a red precipitate soluble in (a) C6H5CH2Br (b) CH3Br
excess of KI. A and B are respectively (c) (CH3)2CHBr (d) (CH3)3CBr
(a) HgCl2 and Hg2Cl2 (b) Hg2Cl2 and HgCl2 19. Two elements A & B form compounds having
(c) HgCl2 and ZnCl2 (d) ZnCl2 and HgCl2 molecular formulae AB 2 and AB4 . When
dissolved in 20.0 g of benzene 1.00g of AB2
14. In which reaction, there is change in oxidation
lowers f.p. by 2.30C whereas 1.00g of AB4 lowers
number of N
f.p. by 1.30C. The molal depression constant for
(a) 2NO2 ® N2O4 benzene in 1000g is 5.1. The atomic masses of A
(b) NH4OH ® NH4+ + OH– and B are
(a) 52, 48 (b) 42, 25
(c) N2O5 + H2O ® 2HNO3 (c) 25, 42 (d) None
(d) 2NO2 + H2O ® HNO3 + HNO2 20. To detect iodine in presence of bromine, the
15. Potassium permanganate acts as an oxidant in sodium extract is treated with NaNO 2 + glacial
neutral, alkaline as well as acidic media. The final acetic acid + CCl4 . Iodine is detected by the
products obtained from it in the three conditions appearance of
are, respectively (a) yellow colour of CCl4 layer
(a) MnO42–, Mn3+ and Mn2+ (b) purple colour of CCl4
(b) MnO2, MnO2 and Mn2+ (c) brown colour in the organic layer of CCl4
(c) MnO2, MnO2+ and Mn3+ (d) deep blue colour in CCl4
(d) MnO, MnO2 and Mn2+ 21. An element (atomic mass =100 g/mol) having
bcc structure has unit cell edge 400pm. The
16. The number of p electrons present in 6.4 g of density (in g/cm3) of the element is
calcium carbide is – (NA = Avagadro’s number) (a) 10.376 (b) 5.19
(a) 4 NA (b) 0.4 NA (c) 7.289 (d) 2.144
(c) 0.1 NA (d) 0.2 NA 22. An organic compound A (C4H10O) has two
enantiomeric forms and on dehydration it gives
17. An organic compound ‘X’ on ozonolysis B(major product) and C (minor product). B and
followed by reduction with Zn/H2O gives 2 moles C are treated with HBr/ Peroxide and the
O O O compounds so produced were subjected to
|| || || alkaline hydrolysis then-
of H - C - H and H - C - CH 2 - C - H . ‘X’ is (a) B will give an isomer of A
(a) CH2 = CH – CH2 – CH = CH2 (b) C will give an isomer of A
(b) CH2 = CH–CH2–CH2–CH = CH2 (c) Neither of them will give isomer of A
(c) H– C º C –C ºC–H (d) Both B and C will give isomer of A
(d) CH2 = CH – CH = CH2
EBD_7206
MT-54 JEE MAIN

23. A reaction is found to be second order w.r.t. one of 25. Concentration of NH4Cl and NH4OH in a buffer
the reactants & has rate constant of 0.5 mol–1 dm3 solution is in the ratio of 1 : 1, Kb for NH4OH is
min–1. If initial concentration is 0.2 mol dm–3 then 10–10. The pH of the buffer is
t1/2 of reaction is
(a) 4 (b) 5
(a) 5 min (b) 10 min
(c) 9 (d) 11
(c) 15 min (d) 20 min
24. The major product expected from the following 26. Statement-1 : Amines are basic in nature.
reaction is : Statement-2 : Presence of lone pair of electron
CH2OH O on nitrogen atom.
HO2C (a) Statement-1 is true, Statement-2 is true,
HCl(g)/CCl4 Statement-2 is a correct explanation for
NH2
Statement -1
(b) Statement -1 is true, Statement-2 is true ;
OH Statement-2 is NOT a correct explanation
O for Statement - 1
(c) Statement - 1 is true, Statement-2 is false
HO2C O
(d) Statement -1 is false, Statement-2 is true
(a)
NH2
OH (i) NaNO2HCl
27. A. Compound ‘A’ is
CH2OH O (ii) CuCl
(b)
HO2C
NH2 Cl
NO2
Cl (a) (b)

O
O NO2
(c) Cl
O
NH2 (c) (d)

OH 28. The standard reduction potential of


O Li+/Li, Ba2+/Ba, Na+/Na and Mg2+/Mg are –3.05,
–2.73, –2.71 and –2.37 volts respectively. Which
(d) HO C
2 O one of the following is strongest oxidising agent?
(a) Na+ (b) Li+
(c) Ba2+ (d) Mg2+
Cl
Mock Test-5 MT-55

29. Allyl phenyl ether can be prepared by heating:


(a) C6H5Br + CH2 = CH – CH2 – ONa PHYSICS
(b) CH2 = CH – CH2 – Br + C6H5ONa 31. A person climbs up a stalled escalator in 60 s. If
(c) C6H5 – CH = CH – Br + CH3 – ONa standing on the same but escalator running with
constant velocity he takes 40 s. How much time is
(d) CH2 = CH – Br + C6H5 – CH2 – ONa
taken by the person to walk up the moving escalator?
Cº N (a) 37 s (b) 27 s
30. + C6H5MgBr (c) 24 s (d) 45 s
32. In the circuit shown, the total current supplied
Ether H3O + by the battery is
¾¾¾® A ¾¾¾¾
® Ba
B
O
C 6W
(a) 2W 3W
A C

6 volts
1.5W
N–MgBr
C
(b) D
(a) 2 ampere (b) 4 ampere
(c) 1 ampere (d) 6 ampere
N–H 33. Two periodic waves of intensities I1 and I2 pass
C through a region at the same time in the same
(c) direction. The sum of the maximum and minimum
intensities is
(a) 2 (I1 + I2) (b) I1 + I2
N–OH
C (c) ( I1 + I 2 ) 2 (d) ( I1 - I 2 ) 2
(d) 34. Three identical spheres, each of mass 1 kg are
kept as shown in figure, touching each other,
with their centres on a straight line. If their centres
are marked P, Q, R respectively, the distance of
centre of mass of the system from P is
EBD_7206
MT-56 JEE MAIN
y harmonic motion with frequency
1 AgP0 1 V0 MP0
(a) (b)
P Q R 2p V0 M 2p A 2 g
x

1 A 2 gP0 1 MV0
(c) (d)
PQ + PR + QR PQ + PR 2p MV0 2p AgP0
(a) (b)
3 3
38. A wooden block of volume 1000 cc is suspended
PQ + QR PR + QR from a spring balance. It weighs 12 N in air. It is
(c) (d)
3 3 then held suspended in water with half of it inside
35. Relative permittivity and permeability of a water. What would be the reading in spring
material e r and m r , respectively. Which of the balance now?
(a) 10 N (b) 9 N
following values of these quantities are allowed
(c) 8 N (d) 7 N
for a diamagnetic material?
39. The correct graph between the gravitational
(a) e r = 0.5, m r = 1.5 (b) e r = 1.5, mr = 0.5 potential (Vg) due to a hollow sphere and distance
from its centre will be
(c) e r = 0.5, m r = 0.5 (d) e r = 1.5, mr = 1.5
r
36. Let there be a spherically symmetric charge
distribution with charge density varying as
(a) Vg (b) Vg
æ5 rö
r(r ) = r0 ç - ÷ upto r = R , and r(r ) = 0
è 4 Rø r
for r > R , where r is the distance from the origin.
+Vg
The electric field at a distance r(r < R) from the +Vg r = Re r r = Re r
origin is given by
r0 r æ 5 r ö 4pr0 r æ 5 r ö (c) d)
(a) çè - ÷ø (b) ç - ÷
4e 0 3 R 3e 0 è 3 R ø
r0 r æ 5 r ö r0 r æ 5 r ö –Vg –Vg
(c) çè - ÷ø (d) ç - ÷
3ε 0 è 4 R ø
4ε0 4 R 40. In a cubical vessel are enclosed n molecules of a
37. An ideal gas enclosed in a vertical cylindrical gas each having a mass m and an average speed
container supports a freely moving piston of v. If l is the length of each edge of the cube, the
mass M. The piston and the cylinder have equal pressure exerted by the gas will be
cross sectional area A. When the piston is in
equilibrium, the volume of the gas is V0 and its n mv 2 n m2 v
(a) (b)
pressure is P0. The piston is slightly displaced l3 2 l3
from the equilibrium position and released.
Assuming that the system is completely isolated m nv 2 nmv
(c) (d)
from its surrounding, the piston executes a simple 3l 3
2l
Mock Test-5 MT-57

41. The half life of a radioactive substance is 20 45. A particle of mass m1 collides head-on with
minutes. The approximate time interval (t2 – t1) another stationary particle of mass m2 (m2 > m1).
2 The collision is perfectly inelastic. The fraction
between the time t2 when of it had decayed
3 of kinetic energy which is converted into heat in
1 this collision is
and time t1 when of it had decayed is :
3
(a) m2/(m1 + m2) (b) m1/(m1 + m2)
(a) 14 min (b) 20 min
(c) 28 min (d) 7 min (c) m1/(m1 – m2) (d) m2/(m1 – m2)
42. A sphere, a cube and a thin circular plate all made 46. The instantaneous values of current and voltage
of the same material and having the same mass, in an A.C. circuit are I = 4 sin wt and E = 100 cos
are initially heated to a temperature of 200oC.
Which of these objects will cool slowest when æ pö
ç wt + ÷ respectively. The phase difference
left in air at room temperature? è 3ø
(a) the sphere (b) the cube between voltage and current is
(c) the circular plate (d) all will cool at same rate
p 2p
43. An engine operates by taking n moles of an ideal (a) (b)
gas through the cycle ABCDA shown in figure. 3 3
The thermal efficiency of the engine is : (Take
5p 7p
Cv =1.5 R, where R is gas constant) (c) (d)
6 6
B C
2P0 47. Which of the following units denotes the
(a) 0.24
ML2
dimension , where Q denotes the electric
(b) 0.15 P Q2
P0
A
D charge?
(c) 0.32 (a) Wb/m2 (b) Henry (H)
(c) H/m2 (d) Weber (Wb)
V0 2V0
48. Wires 1 and 2 carrying currents i 1 and i 2
(d) 0.08 V respectively are inclined at an angle θ to each
44. An electromagnetic wave in vacuum has the other. What is the force on a small element dl of
r r
electric and magnetic field E and B , which are wire 2 at a distance of r from wire 1 (as shown in
figure) due to the magnetic field of wire 1?
always perpendicular to each other. The
r (a) m 0 i1i2 dl tan q
direction of polarization is given by X and that 2 pr 1 2
r
of wave propagation by k . Then m0
r r r r r (b) i1i2 dl sin q i
(a) X || B and k || B ´ E 2pr 1
r i2
m0
r r r r r (c) i1i2 dl cos q q dl
(b) X || E and k || E ´ B 2pr
r r r r r m0
(c) X || B and k || E ´ B (d) i1i2 dl sin q
4 pr
r r r r r
(d) X || E and k || B ´ E
EBD_7206
MT-58 JEE MAIN

49. In Young's double slit experiment, one of the slit is loop exists everywhere with half the loop outside
wider than other, so that amplitude of the light from the field, as shown in figure. The induced emf is
one slit is double of that other slit. If Im be the
maximum intensity, the resultant intensity I when
they interfere at phase difference f is given by :
L v
Im Im æ 2 fö
(a) (4 + 5 cos f) (b) ç 1 + 2cos ÷
9 3 è 2ø
Im æ 2 fö Im æ 2 fö
(c) ç 1 + 4 cos ÷ (d) ç 1 + 8 cos ÷
5 è 2ø 9 è 2ø (a) zero (b) RvB
50. A pendulum consists of a wooden bob of mass (c) vBL/R (d) vBL
m and length ‘l’. A bullet of mass m1 is fired
53. The current gain of a common emitter amplifier
towards the pendulum with a speed v1 . The
is 69. If the emitter current is 7.0 mA, collector
bullet emerges out of the bob with a speed v1/3,
current is :
and the bob just completes motion along a
(a) 9.6 mA (b) 6.9 mA
vertical circle. Then v1 is
(c) 0.69 mA (d) 69 mA
æ mö 3æ m ö 54. Range of frequencies alloted for commercial FM
(a) çè m ÷ø 5g l (b) 2 çè m1 ÷ø
5gl
radio broadcast is
1
(a) 88 to 108 MHz (b) 88 to 108 kHz
2 æ m1 ö æ m1 ö
(c) ç ÷ 5gl (d) çè ÷ø g l (c) 8 to 108 MHz (d) 88 to 108 GHz
3è m ø m
55. A ray of light is incident at an angle of 60° on
51. A boy playing on the roof of a 10 m high building one face of a prism of angle 30°. The ray emerging
throws a ball with a speed of 10m/s at an angle out of the prism makes an angle of 30° with the
of 30º with the horizontal. How far from the incident ray. The emergent ray is
throwing point will the ball be at the height of 10
(a) Normal to the face through which it emerges
m from the ground ?
(b) Inclined at 30° to the face through which it
1 3 emerges
[ g = 10m/s2 , sin 30o = , cos 30o = ]
2 2 (c) Inclined at 60° to the face through which it
(a) 5.20m (b) 4.33m emerges
(c) 2.60m (d) 8.66m (d) Inclined at 90° to the normal at face through
52. A conducting square loop of side L and which it emerges
resistance R moves in its plane with a uniform 56. The anode voltage of a photocell is kept fixed.
velocity v perpendicular to one of its sides. A The wavelength l of the light falling on the
magnetic induction B constant in time and space, cathode is gradually changed. The plate current
pointing perpendicular and into the plane at the I of the photocell varies as follows :
Mock Test-5 MT-59

60. A copper wire of length 1.0 m and a steel wire of


I I
length 0.5 m having equal cross-sectional areas
are joined end to end. The composite wire is
(a) (b) stretched by a certain load which stretches the
O l O l copper wire by 1 mm. If the Young’s modulii of
I I
copper and steel are respectively 1.0 × 1011 Nm–2
and 2.0 × 1011 Nm–2, the total extension of the
composite wire is :
(c) (d) (a) 1.75 mm (b) 2.0 mm
O l O l (c) 1.50 mm (d) 1.25 mm
57. A proton, a deutron and an a particle accelerated
through the same potential difference enter a
region of uniform magnetic field, moving at right
MATHEMATICS
angles to B. What is the ratio of their K.E.? 61. Let f be a composite function of x defined by
(a) 1 : 2 : 2 (b) 2 : 2 : 1 1 1
f (u ) = , u ( x) =
.
(c) 1 : 2 : 1 (d) 1 : 1 : 2 u +u-22 x -1
58. A hydrogen-like atom has one electron revolving Then the number of points x where f is
around a stationary nucleus. The energy discontinuous is :
required to excite the electron from the second (a) 4 (b) 3
orbit to the third orbit is 47.2 eV. The atomic (c) 2 (d) 1
62. If an equation of a tangent to the curve,
number of the atom is
y = cos(x + y), – 1 -1 £ x £ 1 + p, is x + 2y = k
(a) 3 (b) 4
then k is equal to :
(c) 5 (d) 6 (a) l (b) 2
59. In the figure battery B supplies 12 V. Take C1 = 1
mF, C2 = 2 mF, C3 = 3 mF, C4 = 4 mF. Charge on p p
(c) (d)
capacitor C1 when only1 S1 is closed is 4 2
C1 C3
(x 2 - 1)dx
63. If ò æ x2 +1ö
S2 (x 4 + 3x 2 + 1) tan -1 ç ÷
è x ø
C2 C4
S1 = log | tan -1 f (x) | + C, then
+ –
B x 2 +1
(a) 3 mC (b) 9 mC (a) f (x) = x2 + 1 (b) f ( x) =
2x
(c) 6 mC (d) 12 mC
x2 +1 1 2
(c) f (x) = (d) f ( x) = (x + 1)
x 2
EBD_7206
MT-60 JEE MAIN

68. The domain of the function


64. If the function f ( x ) = Pe 2 x + Qe x + Rx
f (x) = x - 1 - x 2 is
satisfies the con-ditions f (0) = -1, f ¢(log 2) = 31
log 4 39 é 1 ù é 1 ù
and ò0 (f (x) - Rx)dx = , then (a) ê -1, - úÈê ,1ú (b) [–1, 1]
2 ë 2û ë 2 û
(a) P = 5, Q = -6, R = 3 æ 1ù é 1 ö é 1 ù
(b) P = -5, Q = 6, R = 3 (c) çè -¥, - ú È ê , + ¥ ÷ (d) ê
ø
,1ú
2û ë 2 ë 2 û
(c) P = -5, Q = 6, R = 3 69. If the S.D. of a variable X is s, then the S.D. of
(d) P = 3, Q = 2, R = 3 aX + b
65. The minimum area (sq. units) of triangle formed (a,b,c are constant) is
c
x2 y 2 a a
by the tangent to the 2 + 2 = 1 & coordinate (a) s (b) s
a b c c
axes is
c c
a 2 + b2 (c) s (d) s
(a) ab (b) a a
2
70. Consider the system of equations in x, y, z as
( a + b) 2 a 2 + ab + b2 x sin 3q – y + z = 0, x cos 2q + 4y + 3z = 0,
(c) (d)
2 3 2x + 7y + 7z = 0. If this system has a non-trivial
solution, then for integer n, values of q are given by
x2
66. Area included between y= and
4a æ (–1)n ö æ (–1)n ö
(a) pçn + ÷ (b) p çè n + ÷
8a 3 è 3 ø 4 ø
y= is
x 2 + 4a 2
æ (–1)n ö np
a2 a2 (c) pçn + ÷ø (d)
(a) ( 6p - 4) (b) (4 p + 3) è 6 2
3 3
71. Which of the following is an empty set?
a2 (a) The set of prime numbers which are even
(c) (8p + 3) (d) None of these
3 (b) The solution set of the equation
r r
67. Given that a is ^ to b and p is a non zero scalar 2 (2x + 3) 2
r r r r r r – + 3 = 0; x Î R
if pr + (r .b) a = c then r equals x +1 x +1
r r r r (c) (A × B) Ç (B × A), where A and B are
(a) c / p - éë(b.c) a ùû / p 2 disjoint.
r r r r
(b) a / p - éë (c.a) b ùû / p2 (d) The set of real which satisfy x 2 + ix + i – 1 = 0
r rr r 2
(c) b / p - éë(a.b)c ùû / p
(d) None of these
Mock Test-5 MT-61

72. f (x) = x2 [x] 77. If f : R ® R be a function defined by f (x) = 4x3 –7.


(a) increases in (0,1)
Then
(b) decreases in (0 , 1)
(a) f is one-one -into
(c) increases in (–1,0)
(b) f is many-one - into
(d) None of these
(c) f is many-one onto
73. If y = ex + sin x , then d2x/dy2 is equal to (d) f is bijective
(a) ex – sin x 78. If g = {(1, 1), (2, 3), (3, 5), (4, 7)} is a function
(b) –(ex + cos x)–2 described by the formula, g (x) = a x + b then
(c) – (ex – sin x ) (ex + cos x)–2
what values should be assigned to a and b ?
(d) (sin x – ex) (cos x + ex)–3
(a) a = 1, b = 1 (b) a = 2, b = – 1
74. The sum to n terms of the series
(c) a = 1, b = – 2 (d) a = – 2, b = – 1
1 3 7 15 79. The roots a and b of the quadratic equation
+ + + + .............. is
2 4 8 16
px 2 + qx + r = 0 are real and of opposite signs.
(a) n – 1 – 2 –n (b) 1
The roots of a( x - b) 2 + b(x - a ) 2 = 0 are
(c) n – 1 + 2 – n (d) 1 + 2 – n (a) positive (b) negative
75. If f (x) = 3x10 – 7x8 + 5x6 – 21x3 + 3x2 – 7, then (c) of opposite signs (d) non-real
f (1 - a ) - f (1) 80. The value of x for which sin (cot –1 (1+ x)) = cos
lim is (tan–1 x) is
a®0 a3 + 3a (a) 1/2 (b) 1
53 53
(a) - (b) (c) 0 (d) –1/2
3 3 81. The equation k sin q + cos 2q = 2k – 7 possesses a
55 55 solution if :
(c) - (d)
3 3 (a) 2 £ k £ 6 (b) k > 2
76. A survey of 500 television viewers produced the (c) k > 6 (d) k < 2
following information, 285 watch football, 195
watch hockey, 115 watch basket-ball, 45 watch 1 - cos 3 x
football and basket ball, 70 watch football and 82. The value of lim is
x ® 0 x sin x cos x
hockey, 50 watch hockey and basket ball, 50 do
not watch any of the three games. The number 2 3
of viewers, who watch exactly one of the three (a) (b)
5 5
games are
(a) 325 (b) 310 3 3
(c) (d)
(c) 405 (d) 372 2 4
EBD_7206
MT-62 JEE MAIN

2x æ 2x ö
83. If y = sin -1 , then which of the following 87.
-1
If y = tan çç ÷, then dy at x = 0 is :
2 2 x +1 ÷
1+ x dx
is not correct? è1+ 2 ø

dy 2 3 2
(a) = for | x |< 1 (a) – log 2 (b) log 2
5 5
dx 1 + x 2
3
dy -2 (c) - log 2 (d) None
(b) = for | x | > 1 2
dx 1 + x 2
dy
(c) = 2 for x = -1 a a2 1+ a3
dx
2 3
dy 88. If b b 1 + b = 0 and the vectors
(d) does not exist at | x | = 1
dx c c2 1 + c3
1
84. If f (x ) = , th e number of points of A = (1,a,a2) ; B = (1, b, b2) ; C = (1,c, c2) are
1- x
discontinuity of f{f [f(x)]} is : non-coplanar then the product abc =
(a) 2 (b) 1 (a) 0 (b) 1
(c) 0 (d) infinite
(c) –1 (d) None
85. The value of cos 255° + sin 195° is
89. A and B are two independent witnesses (i.e. there
3 -1 3 -1
(a) (b) is no collusion between them) in a case. The
2 2 2
probability that A will speak the truth is x and
æ 3 -1ö 3 +1 the probability that B will speak the truth is y. A
(c) - çç ÷÷ (d)
è 2 ø 2 and B agree in a certain statement. The
86. If the slope of the tangent at (x, y) to a curve probability that the statement is true is
æ pö x–y xy
passing thr ough ç 1, ÷ is given by (a) (b)
è 4ø x+y 1 + x + y + xy
y æyö
- cos 2 ç ÷ , then the equation of the curve is x–y xy
x èxø (c) (d)
1 – x – y + 2 xy 1 – x – y + 2 xy
(a) y = tan -1 log (e / x )
90. Let x1 and y1 be real numbers. If z1 and z2 are
y = e1+ cot
( y / x)
(b) complex numbers such that |z1| = |z2| = 4, then
(c) y = x tan -1 log(e / x )
|x1z1 – y1z2|2 + |y1z1 + x1z2|2 =
(d) y = e1+tan(y/x) (a) 32(x12 + y12) (b) 16(x12 + y12)

(c) 4(x12 + y12) (d) (


8 x12 + y12 )
Download From https://iit-jeeacademy.blogspot.com

6 JEE MAIN
MOCK TEST
Time : 3 hrs. Max. Marks : 360

INSTRUCTIONS
• Chemistry (120 marks) : Question No. 1 to 30 are of 4 marks each.

• Physics (120 marks) : Question No. 31 to 60 are of 4 marks each.

• Mathematics (120 marks) : Question No. 61 to 90 are of 4 marks each.


• Negative Marking : One fourth (¼) marks will be deducted for indicating incorrect
response of each question.

3. Four species are listed below:


CHEMISTRY i. HCO3– ii. H3 O+
1. An azeotropic mixture of two liquids boil at a iii. HSO4– iv. HSO3F
lower temperature than either of them when Which one of the following is the correct
(a) It is saturated sequence of their acid strength?
(b) It does not deviate from Raoult’s law (a) iv < ii < iii < i (b) ii < iii < i < iv
(c) It shows negative deviation from Raoult’s law (c) i < iii < ii < iv (d) iii < i < iv < ii
(d) It show positive deviation from Raoult’s law 4. The enthalpy of combustion of H2(g), to give
2. Which of the following statement is wrong ? H2O(g) is –249 kJ mol–1 and bond enthalpies of
(a) Geometrical isomers are diastereomers H – H and O = O are 433 kJ mol–1 and 492 kJ mol–1
(b) Enantiomers have same chemical and respectively. The bond enthalpy of O – H is
physical properties
(c) Conformers are non inter convertable to (a) 464 kJ mol-1 (b) –464 kJ mol-1
each other (c) 232 kJ mol-1 (d) –232 kJ mol-1
(d) 3-methyl haxane shows optical isomerism
EBD_7206
MT-64 JEE MAIN

5. In an atom how many orbital(s) will have the 10. The process of the extraction of Au and Ag is
quantum numbers; n = 3, l = 2 and ml = + 2 ? based on their solubility in
(a) 5 (b) 3 (a) NH3 or NH4OH (b) KCN or NaCN
(c) HCl or HNO3 (d) H2SO4
(c) 1 (d) 7 11. Which of the following compounds having
6. Given highest and lowest melting point respectively ?

(i) HCN(aq) + H2O( l) H3O+ (aq) + CN (aq) (1) CsF (2) LiF
Ka = 6.2 × 10–10 (3) HCl (4) HF

Correct answer is
(ii) CN –(aq) + H 2O(l) HCN(aq) + OH (aq) (a) 2, 3 (b) 1, 4
–5
Kb = 1.6 × 10 . (c) 4, 3 (d) 2, 1
12. Silver bromide when dissolve in hypo solution
These equilibria show the following order of the
gives complex ..... in which oxidation state of
relative base strength, silver is ....
(a) OH– > H2O > CN– (a) Na3[Ag(S2O3)2], (I)
(b) OH– > CN– > H2O (b) Na3[Ag(S2O3)3], (III)
(c) H2O > CN– > OH– (c) Na3[Ag(S2O3)2], (II)
(d) Na3[Ag(S2O3)4], (I)
(d) CN– > H2O > OH– 13. The freezing point of a solution, prepared from
NH2 CH3 1.25 gm of a non-electrolyte and 20 gm of water,
is 271.9 K. If molar depression constant is 1.86 K
7. Correct IUPAC name of C C is mole–1, then molar mass of the solute will be
(a) 105.7 (b) 106.7
NO2 COOH (c) 115.3 (d) 93.9
14. Consider the reaction:
(a) E-3-amino - 3 - nitro - 2 - methyl propenoic acid
(b) Z -3- amino - 3 - nitro - 2 - methyl propenoic acid H 2SO3 (aq) + Sn 4 + (aq) + H 2 O(l )
(c) 3 - Amino - 3 - nitro - 2 - methyl propenoic acid ® Sn 2 + (aq) + HSO 4- (aq) + 3H + (aq)
(d) Trans - 3 - amino - 3 - nitro - 2 - methyl Which of the following statements is correct?
propenoic acid (a) Sn 4+ is the oxidizing agent because it
8. Which of the following undergoes hydrolysis undergoes oxidation
by S 1 mechanism : (b) Sn 4+ is the reducing agent because it
N undergoes oxidation
(a) CH3CH2CH2CH2Cl (b) CH3CH2CH2Cl (c) H2SO3 is the reducing agent because it
(c) CH3 – O – CH2Cl (d) CH3Cl undergoes oxidation
9. Which one of the following substituents at (d) H2SO3 is the reducing agent because it
para-position is most effective in stabilizing the undergoes reduction
15. An organic compound (A) reacts with sodium
metal and forms (B). On heating with conc.

O H2SO4, (A) gives diethyl ether, (A) and (B) are
phenoxide .ion?
(a) C2H5OH and C2H5ONa
(b) C3H7OH and CH3ONa
(a) – CH3 (b) – OCH3
(c) CH3OH and CH3ONa
(c) – COCH3 (d) – CH2OH (d) C4H9OH and C4H9ONa
Mock Test -6 MT-65

16. Which one of the following esters cannot 20. Alkali metals dissolve in liquid NH3 then which
undergo Claisen self-condensation? of the following observations is not true?
(a) CH 3 - CH 2 - CH 2 - CH 2 - COOC 2 H 5 (a) H2 gas is liberated
(b) Solution turns into blue due to solvated
(b) C6H5COOC2H5
electrons
(c) C6H5CH2COOC2H5
(c) It becomes diamagnetic
(d) C6H11CH2COOC2H5
(d) Solution becomes conducting
17. In recovery of silver from photographic film, you
21. Which of the following species involves sp3
have decided to dissolve the silver ion with dilute
hybridisation?
nitric acid. Addition of dilute HCl to precipitate (a) SbF6– (b) PCl5
AgCl seems to result in unacceptable losses. (c) N2Cl4 (d) SiF62–
You might improve recovery by addition 22. At very high pressures, the compressibility
of_______ in the latter step. factor of one mole of a gas is given by :
(a) NaNO3 (b) NaCl
(c) Ag2SO4 (d) sodium acetate Pb Pb
(a) 1+ (b)
18. An unknown alochol is treated with the “Lucas RT RT
reagent” to determine whether the alcohol is
Pb b
primary, secondary or tertiary. Which alcohol (c) 1- (d) 1 -
reacts fastest and by what mechanism : RT (VRT)
(a) secondary alcohol by SN1 23. The appearance of colour in solid alkali meta
(b) tertiary alcohol by SN1 halides is generally due to:
(c) secondary alcohol by SN2 (a) Schottky defect (b) Frenkel defect
(d) tertiary alcohol by SN2 (c) Interstitial position (d) F-centres
24. The electrode potential of Mg2+/Mg electrode,
19. In the reaction,
in which conc. of Mg2+ is 0.01M, is
HOCl B ( E o Mg2+ / Mg = -2.36V)
CH 2 = CH 2 ¾¾¾¾
® A ¾¾® CH 2 - OH
|
(a) 2.36V (b) –2.36V
CH2OH
(c) 2.42V (d) –2.42V
The molecule ‘A’ and the reagent ‘B’ are 25. Among the following compounds (I-III) the
(a) CH2 – CH2 and Hot water correct order of reactivity with electrophile is
OCH3 NO2
O
(b) CH3CH2Cl and NaOH
(c) CH3CH2OH and H2SO4
(d) CH2 – CH2 and NaHCO3
I II III
Cl OH (a) II > III > I (b) III < I < II
(c) I > II > III (d) I = II > III
EBD_7206
MT-66 JEE MAIN

26. One desires to prepare a positively charged sol


of silver iodide. This can be achieved by PHYSICS
(a) adding a little AgNO3 solution to KI
solution in slight excess 31. A 1 kg block attached to a spring vibrates with a
(b) adding a little KI solution to AgNO3 frequency of 1 Hz on a frictionless horizontal
solution in slight excess table. Two springs identical to the original spring
(c) mixing equal volumes of equimolar are attached in parallel to an 8 kg block placed
solutions of AgNO3 and KI on the same table. The frequency of vibration of
(d) none of these the 8 kg block is :
27. In the reaction of KMnO 4 with an oxalate in
1 1
acidic medium, MnO-4 is reduced to Mn 2 + and (a) Hz (b) Hz
4 2 2
C 2 O 24 - is oxidised to CO2. Hence, 50 mL of 1
(c) Hz (d) 2 Hz
0.02 M KMnO4 is equivalent to 2
(a) 100 mL of 0.05 M H 2C 2O 4 32. At ground level, there is a downard vertical
electric field of about 100 V/m. From this data
(b) 50 mL of 0.05 M H 2C 2O 4 calculate order of surface charge density on earth
(c) 25 mL of 0.2 M H 2C 2O 4 by assuming that earth is a conductor.
(a) 10–9 C/m2 (b) –10–9 C/m2
(d) 50 mL of 0.10 M H 2C 2O 4 +6 2
(c) 10 C/m (d) –10+6 C/m2
28. Lassaigne’s test for nitrogen is positive for
33. The magnetic field due to a current carrying
which compound?
circular loop of radius 3 cm at a point on the axis
(a) NH2OH (b) NH2NH2
at a distance of 4 cm from the centre is 54 mT.
(c) H2NCONH2 (d) All the three
What will be its value at the centre of loop?
29. Which of the following is true?
(a) Tollens’ reagent gives a positive result with (a) 125 mT (b) 150 mT
all aldehydes (c) 250 mT (d) 75 mT
(b) Tollens’ reagent gives a negative result with 34. The velocity of projection of a body is increased
all ketones by 2%, other factors remaining unchanged, what
(c) Both are true will be the percentage change in the maximum
(d) None is true height attained
30. Which of the following complexes will give white (a) 1 % (b) 2 %
precipitate with BaCl2 (aq)? (c) 4 % (d) 8 %
(a) [Co(NH3)4SO4]NO2 35. Hydrogen ( 1 H1 ) , Deuterium ( 1 H 2 ) , singly
ionised Helium ( 2 He4 ) and doubly ionised
(b) [Cr(NH3)4SO4]Cl +

(c) [Cr(NH3)5Cl]SO4
lithium ( 3 Li 6 ) all have one electron around
+ +

(d) Both (b) & (c)


the nucleus. Consider an electron transition from
n = 2 to n = 1. If the wavelengths of emitted
Mock Test -6 MT-67

radiation are l1, l 2 , l3 and l 4 respectively 38. The amplitude of velocityof a particle acted on
then approximately which one of the following by a force F cos w t along the x direction, is
is correct? 1
given by x =
(a) 4l1 = 2l 2 = 2l3 = l 4 [aw 2 - bw + c]
(b) l1 = 2l 2 = 2l3 = l 4 where a, b, c, are constants and b2 < 4 ac. For
(c) l1 = l 2 = 4l3 = 9l 4 what value of w does the resonance occur ?
(d) l1 = 2l 2 = 3l3 = 4l 4 b b
(a) w= (b) w =
36. A particle of mass m moves with a constant 2a a
velocity. Which of the following statements is (c) w=c (d) w = 0
not correct about its angular momentum ?
39. Three bars each of area of cross - section A and
Y length L are connected in series. The thermal
E
D conductivities of their materials are K, 2K, 1.5K.
C
A If the temperatures of the external ends of the
1m
first and last bar are 200 & 18ºC, then the
temperatures of both the junctions are
B
X (a) T1 = 116ºC, T2 = 74ºC
O
1m (b) T1 = 120ºC, T2 = 84ºC
(a) it is zero when it is at A and moving along OA.
(c) T1 = 132ºC, T2 = 98ºC
(b) the same at all points along the line DE.
(c) of the same magnitude but oppositely (d) T1 = 164ºC, T2 = 62ºC
directed at B and D. 40. In the following diagram the reading of the
(d) increases as it moves along the line BC ammeter is (when the internal resistance of the
battery is zero)
37. One end of a massless rope, which passes over a
massless and frictionless pulley P is tied to a hook
A 10V
C while the other end is free. Maximum tension
that the rope can bear is 360 N. With what value
of maximum safe acceleration (in ms-2) can a man 4W
5W
of 60 kg climb on the rope?
P
40 10
C
(a) A (b) A
29 9
(c) 5 (d) 2 A
A
3
(a) 16 (b) 6 (c) 4 (d) 8
EBD_7206
MT-68 JEE MAIN

41. A 2V battery is connected across AB as 3Io 31I o


shown in the figure. The value of the current (c) (d)
4 32
supplied by the battery when in one case
battery’s positive terminal is connected to A 44. In a uniform magnetic field of induction B a wire
and in other case when positive terminal of in the form of a semicircle of radius r rotates
battery is connected to B will respectively be: about the diameter of the circle with an angular
5W frequency w. The axis of rotation is perpendicular
D1
to the field. If the total resistance of the circuit is
R, the mean power generated per period of
D2 10 W rotation is
( B pr w )2 ( B pr 2 w ) 2
(a) (b)
2R 8R
A B B pr 2 w ( B pr w 2 ) 2
(c) (d)
(a) 0.4 A and 0.2 A (b) 0.2 A and 0.4 A 2R 8R
(c) 0.1 A and 0.2 A (d) 0.2 A and 0.1 A 45. The insulation property of air breaks down at
42. If a bar magnet of magnetic moment 80 units be E = 3 × 106 volts/ metre. The maximum charge
cut into two halves of equal lengths, the that can be given to a sphere of diameter 5 m is
magnetic moment of each half will be approximately
(a) 2 × 10–2 coulomb (b) 2 × 10–3 coulomb
(a) 80 units (b) 40 units
(c) 2 × 10–4 coulomb (d) 2 × 10–5 coulomb
(c) 160 units (d) 20 units 46. A compound microscope has an objective and
43. Moment of inertia of an equilateral triangular eye-piece as thin lenses of focal lengths 1 cm
lamina ABC, about the axis passing through its and 5 cm respectively. The distance between
centre O and perpendicular to its plane is Io as the objective and the eye-piece is 20 cm. The
shown in the figure. A cavity DEF is cut out distance at which the object must be placed in
from the lamina, where D, E, F are the mid points
front of the objective if the final image is located
of the sides. Moment of inertia of the remaining
at 25 cm from the eye - piece, is numerically
part of lamina about the same axis is :
C 95
95
(a) cm (b) 89 cm
6
F E
5 25
O
(c) cm (d) cm
6 6
A D B 47. When light of 3000 Å is incident on sodium
7 15 chloride, the stopping potential is 1.85 volt and
(a) Io (b) Io when light of 4000 Å is incident, then the stopping
8 16
Mock Test -6 MT-69

potential becomes 0.82 volt. The threshold (a) 35.2 × 10–10 J/m3 (b) 35.2 × 10–11 J/m3
wavelength for sodium is: (h = 6.6 × 10–34 J-sec.) –12
(c) 35.2 × 10 J/m 3 (d) 35.2 × 10–13 J/m3
(a) 5451 Å (b) 5154 Å 52. In a Young’s double slit experiment with light
(c) 5320 Å (d) 5211 Å of wavelength l the separation of slits is d and
48. A steel wire is suspended vertically from a rigid distance of screen is D such that D >> d >>
support. When loaded with a weight in air, it l. If the fringe width is b, the distance from
extends by la and when the weight is immersed point of maximum intensity to the point where
completely in water, the extension is reduced to intensity falls to half of maximum intensity on
either side is:
lw. Then the relatively density of material of the
weight is b b
(a) (b)
la 6 3
la
(a) (b)
lw la - l w b b
(c) (d)
4 2
lw lw
(c) (d) 53. In diagrams (1 to 4), variation of volume with
la - l w la changing pressure is shown. A gas is taken
49. In four complete revolution of the cap, the along the path ABCD. The change in internal
distance travelled on the pitch scale is 2 mm. If energy of the gas will be
there are 50 divisions on the circular scale, then
the least count of the screw gauge is V D V D
C C
(a) 0.001 mm (b) 0.01 mm
(c) 0.10 mm (d) 1.0 mm
50. A pendulum with time period of 1s is losing A A
energy. At certain time its energy is 45 J. If B B

after completing 15 oscillations, its energy has


P
become 15 J, its damping constant (in s–1) is : (1) (2) P

1 1
(a) (b) ln3
2 30 D C
D
C
1
(c) 2 (d) ln3
15 V V
51. An electromagnetic wave of frequency 1 × 1014 A B A
B
hertz is propagating along z-axis. The amplitude
of electric field is 4 V/m. If e0 = 8.8 × 10–12 C2/N- (3) P (4) P
m2, then average energy density of electric field
will be: (a) Positive in all the cases (1) to (4)
(b) Positive in cases (1), (2), (3) but zero in case(4)
EBD_7206
MT-70 JEE MAIN

(c) Negative in cases (1), (2), (3) but zero in case (4) cool from 30° C to 25° C. The specific heat of the
(d) Zero in all the cases liquid is :
54. Three masses m, 2m and 3m are moving in x-y (The water equivalent of the vessel is 30g.)
plane with speed 3u, 2u and u respectively as (a) 2.0 kcal/kg (b) 7 kcal/kg
shown in figure. The three masses collide at the
(c) 3 kcal/kg (d) 0.5 kcal/kg
same point at P and stick together. The velocity
57. An electric charge 10 m C is placed at the origin
–3
of resulting mass will be:
y (0, 0) of X – Y co-ordinate system. Two points A
and B are situated at ( 2, 2) and (2, 0)
2m, 2u
respectively. The potential difference between
the points A and B will be
60°
x (a) 4.5 volts (b) 9 volts
m, 3u P 60° (c) Zero (d) 2 volt
3m, u 58. A Zener diode is connected to a battery and a
load as show below:
(a)
12
(
u ˆ
i + 3jˆ ) (b)
12
(
u ˆ
i - 3jˆ ) 4 kW A IL

IZ
(c)
12
(
u ˆ
-i + 3jˆ ) (d)
12
(
u ˆ
-i - 3jˆ ) I
10 V = VZ RL = 2kW
60 V
55. A satellite of mass m revolves around the earth
of radius R at a height x from its surface. If g is
B
the acceleration due to gravity on the surface of
The currents, I, IZ and IL are respectively.
the earth, the orbital speed of the satellite is
(a) 15 mA, 5 mA, 10 mA
gR 2 gR
(a) (b) (b) 15 mA, 7.5 mA, 7.5 mA
R+ x R-x
(c) 12.5 mA, 5 mA, 7.5 mA
æ gR 2 ö 1/ 2
(c) gx (d) ç ÷ (d) 12.5 mA, 7.5 mA, 5 mA
è R + xø
59. An engine approaches a hill with a constant
56. A mass of 50g of water in a closed vessel, with speed. When it is at a distance of 0.9 km, it blows
surroundings at a constant temperature takes 2 a whistle whose echo is heard by the driver after
minutes to cool from 30°C to 25°C. A mass of 5 seconds. If the speed of sound in air is 330 m/
100g of another liquid in an identical vessel with s, then the speed of the engine is :
(a) 32 m/s (b) 27.5 m/s
identical surroundings takes the same time to
(c) 60 m/s (d) 30 m/s
Mock Test -6 MT-71

60. The Thallium-201 half-life is 74 hours. If the 1


sample has an activity of 80 millicuries initially, (c) ± (d) ± 2
2
what will be the activity after 9.25 days ?
y
(a) 2.5 mCi (b) 5 mCi dt d2y
65. If x = ò 2
, then
dx 2
is equal to :
(c) 10 mCi (d) 20 mCi 0 1+ t

MATHEMATICS (a) y (b) 1+ y 2


x
61. It has been found that is A and B play a game 12 (c) (d) y 2
2
times, A wins 6 times, B wins 4 times and they 1+ y
draw twice. A and B take part in a series of 3 66. The number of real roots of the equation
games. Find the probability that they will win 1 + a1 x + a2 x2 + …. + an xn = 0
alternately.
1
5 6 where |x| < and |an| < 2, is
(a) (b) 3
36 35
(a) n if n is even
4 7 (b) 0 for any natural number n
(c) (d)
35 36 (c) 1 if n is odd
x x (d) None of these.
3
62. Period of the function sin + cos5 is : p
2 5 - p
(a) 2 p (b) 10 p 67. If e 2 <q<
, then
2
(c) 8 p (d) 5 p
63. If a, b, c Î R and 1 is a root of equation ax2 + bx (a) cos log q > log cos q
+ c = 0, then the curve y = 4ax2 + 3bx + 2c, a ¹ 0 (b) cos log q < log cos q
intersect x-axis at (c) cos log q = log cos q
(a) two distinct points whose coordinates are 2
always rational numbers (d) cos log q = log cos q
(b) no point 3
(c) exactly two distinct points 68. If [x] denotes the greatest integer £ x, then
(d) exactly one point
64. Suppose q and f (¹ 0) are such that sec (q + f), 1
lim ([12 x] + [ 22 x] + [32 x] + ........+ [ n 2 x]) =
sec q and sec (q – f) are in A.P. If cos q x ®3 n 3

æ fö x x
= k cos ç ÷ for some k, then k is equal to (a) (b)
è 2ø 2 3
(a) ± 2 (b) ± 1 x
(c) (d) 0
6
EBD_7206
MT-72 JEE MAIN

69. The co-efficient of x in the expansion of 74. If w is a cube root of unity, then a root of the
5
æ 2 c ö is x +1 w w2
çè x + ÷
xø following equation w x + w2 1 =0 is
(a) 20 c (b) 10 c w2 1 x+w
(c) 10 c3 (d) 20 c3
70. Let Sp and Sq be the coefficient of xp and xq (a) x = 1 (b) x = w
respectively in (1 + x)p + q, then (c) x = w 2 (d) x = 0 .
q 75. The lines lx + my + n = 0, mx + ny + l = 0 and
(a) Sp ¹ Sq (b) Sp = p Sq nx + ly + m = 0 are concurrent if
(a) l – m – n = 0
p
(c) Sp = q Sq (d) Sp = Sq. (b) l + m – n = 0
(c) l – m + n = 0
71. Number of ways to distribute 10 distinct balls to
(d) l2 + m2 + n2 = lm + mn + nl
3 persons. One getting 2, 2nd getting 3 and 3rd
getting 5 balls is 76. If (x, y) are the co-ordinates of a point in the

10! 10! 3 4 2
(a) (b)
2!.3!.5! 2!.(3!) 2 .5! plane , then 5 8 2 = 0 represent
10! x y 2
(c) (d) 10! (a) a. st. line || to y-axis (b) a st. line || to x-axis
2!.5!
(c) a st. line (d) a circle
72. The general solution of the differential equation,
77. The equation of the circle whose radius is 5 and
æ dy ö which touches the circle x2 + y2– 2x – 4y–20 = 0 at
sin 2x ç - tan x ÷ - y = 0 , is :
è dx ø the point ( 5,5) is
(a) x2 + y2 + 18 x + 16 y + 120 = 0
(a) y tan x = x + c
(b) x2 + y2 – 18 x – 16 y + 120 = 0
(b) y cot x = tan x + c (c) x2 + y2 – 18 x + 16 y + 120 = 0
(c) y tan x = cot x + c (d) x2 + y2 + 18 x – 16 y + 120 = 0
æ 3ö
(d) y cot x = x + c 78. The normal at ç 2, ÷ to the ellipse,
è 2ø
sin B x2 y 2
73. In any triangle ABC, if cos A = , then + = 1 touch es a par abola, whose
2 sin C 16 3
(a) a = b = c (b) c = a equation is
(c) a = b (d) b = c (a) y2 = – 104 x (b) y2 = 14 x
2
(c) y = 26x (d) y2 = – 14x
Mock Test -6 MT-73

79. If p : I study and q : I fail (a) a = 1, b = –1 (b) a = 1, b = ± 1


Then negation of 'I study or I fail' is
(a) I do not study and I do not fail (c) a = –1, b = ± 1 (d) a = ± 1, b = 1
(b) I do not study or I do not fail 2 x -3
(c) Either I study and I do not rail or I study 84. If f(x) = , g (x) = and
x -3 x+4
and I do not fail
(d) I study and I do not fail 2(2 x + 1)
h(x) = – , then
80. The expression x 2 + x - 12
é æ 3p ö ù
3 êsin 4 ç - a ÷ + sin 4 (3p + a ) ú lim [f(x) +g(x) +h(x)] is equal to
ë è 2 ø û x ®3
(a) –2 (b) –1
é æp ö ù
-2 êsin 6 ç + a ÷ + sin 6 (5p - a ) ú is equal to 2
ë è2 ø û -
(c) (d) 0
(a) 0 (b) 1 7
85. p : Every quadratic equation has one real root
(c) 3 (d) sin 4a + cos6a
and q : Every quadratic equation has two real
81. The general solution of the trigonometric
roots, then truth value of p and q are
equation sin x – cos x = 1 is given by
(a) p is true and q is false
(a) x = 2n p, n Î I
(b) p is false and q is true
p p
(b) x = n p + (–1)n + , n Î I (c) p and q both true
4 4
(d) p and q both false
p 86. If (a, b), (c, d), (e, f) are the vertices of a triangle
(c) x =2n p + ,n ÎI
2 such that a, c, e are in G.P. with common ratio r
p and b, d, f are in G.P. with common ratio s then
(d) x = n ,n ÎI the area of the triangle is
2
r r r r r r ab
82. The value of [A - B, B - C, C - A] where (a) (r + 1) (s + 2) (s + r)
2
r r r
| A |= 1, | B |= 2 and | C |= 3 is
ab
(a) 1 (b) 6 (b) (r – 1) (s – 1) (s – r)
2
(c) 0 (d) 3
r r ab
83. If A = ˆi + ˆj + k,
ˆ B = 4iˆ + 3jˆ + 4kˆ and (c) (r – 1) (s + 1) (s – r)
2
r
C = ˆi + aˆj + bkˆ are linearly dependent vector (d) (r + 1) (s + 1) (s – r).
r
and C = 3 , then
EBD_7206
MT-74 JEE MAIN

87. If P and Q be two given points on the curve 89. If for a real number y, [y] is the greatest integer
1 uuur uuur less than or equal to y, then the values of the
y=x+ such that OP.iˆ = 1 and OQ.iˆ = -1
x
3p / 2
where î is a unit vector along the x–axis , then integral òp / 2 [2 sin x ]dx is

the length of vector 2OP + 3OQ is (a) –p (b) p


(c) p /2 (d) -p/2
(a) 5 5 (b) 3 5
90. If f(x) = ax2 + b, b ¹ 0 x £ 1;
(c) 2 5 (d) 5 = bx2 + ax + c, x > 1,
88. The area bounded by parabola y2 = x, st. line then f(x) is continuous and differentiable at x = 1 if
y = 4 and y-axis is
(a) c = 0, a = 2b (b) a = b, c arbitrary
16 64
(a) (b) (c) a = b, c = 0 (d) a = b, c ¹ 0
3 3
(c) 7 2 (d) None
Download From https://iit-jeeacademy.blogspot.com

7 JEE MAIN
MOCK TEST
Time : 3 hrs. Max. Marks : 360

INSTRUCTIONS
• Chemistry (120 marks) : Question No. 1 to 30 are of 4 marks each.

• Physics (120 marks) : Question No. 31 to 60 are of 4 marks each.

• Mathematics (120 marks) : Question No. 61 to 90 are of 4 marks each.


• Negative Marking : One fourth (¼) marks will be deducted for indicating incorrect
response of each question.

3. By what ratio the average velocity of the molecule


CHEMISTRY in gas changes when the temperature is raised
from 50 to 200ºC ?
1. Ethylene dicholoride and ethylidine chloride are
isomeric compounds. The false statement about 1.21 1.46
(a) (b)
these isomers is that they : 1 1
(a) react with alcoholic potash and give the 2 4
same product (c) (d)
(b) are position isomers 1 1
(c) contain the same percentage of chlorine 4. Which one of the following is an oxide ?
(d) are both hydrolysed to the same product (a) KO2 (b) BaO2
2. Addition of sodium hydroxide solution to a weak (c) SiO2 (d) CsO2
acid (HA) results in a buffer of pH 6. If ionisation 5. An ‘fcc’ in a unit cell of aluminium contains the
constant of HA is 10–5, the ratio of salt to acid equivalent of how many atoms ?
concentration in the buffer solution will be : (a) 1 (b) 2
(a) 4 : 5 (b) 1 : 10 (c) 3 (d) 4
(c) 10 : 1 (d) 5 : 4
EBD_7206
MT-76 JEE MAIN

6. Moissan boron is (c) exhibit precipitation of NaNO3


(a) amorphous boron of ultra purity (d) exhibit a marked color change
(b) crystalline boron of ultra purity 13. Extraction of copper by smelting uses silica as
(c) amorphous boron of low purity an additive to remove :
(d) crystalline boron of low purity (a) Cu2O (b) FeS
7. Which reaction characteristics are changed by
(c) FeO (d) Cu2S
the addition of a catalyst to a reaction at
14. Given the molecular formula of the hexa-
constant temperature ?
coordinated complexes (i) CoCl 3.6NH3, (ii)
(i) Activation energy
CoCl3.5NH3, (iii) CoCl3.4NH3
(ii) Equilibrium constant
If the number of co-ordinated NH3 molecules in
(iii) Reaction enthalpy
i, ii and iii respectively are 6, 5, 4, the primary
(a) (i) only (b) (iii) only valencies in (i), (ii) and (iii) are :
(c) (i) and (ii) only (d) all of these (a) 6, 5, 4 (b) 3, 2, 1
8. How many H-atoms are present in 0.046 g of
(c) 0, 1, 2 (d) 3, 3, 3
ethanol ?
15. Polyethylene is
(a) 6 × 1020 (b) 1.2 × 1021
21 (a) Random copolymer
(c) 3 × 10 (d) 3.6 × 1021
(b) Homopolymer
9. A metal M reacts with N2 to give a compound
‘A’ (M3N). ‘A’ on heating at high temperature (c) Alternate copolymer
gives back ‘M’ and ‘A’ on reacting with H2O (d) Crosslinked copolymer
gives a gas ‘B’. ‘B’ turns CuSO4 solution blue 16. The reason for “drug induced poisoning” is :
on passing through it. M and B can be : (a) Binding reversibly at the active site of the
(a) Al & NH3 (b) Li & NH3 enzyme
(c) Na & NH3 (d) Mg & NH3 (b) Bringing conformational change in the
10. Asthma patient use a mixture of .... for binding site of enzyme
respiration. (c) Binding irreversibly to the active site of
(a) O2 and CO2 (b) O2 and He the enzyme
(c) O2 and NH3 (d) O2 and CO (d) Binding at the allosteric sites of the enzyme
11. When ethanal reacts with CH3MgBr and 17. With a change in hybridization of the carbon
C2H5OH/dry HCl, the product formed are : bearing the charge, the stability of a carbanion
decreases in the order :
(a) ethyl alcohol and 2-propanol
(b) ehtane and hemi acetal (a) sp < sp2 < sp3 (b) sp < sp3 < sp2
3
(c) sp < sp < sp2 (d) sp2 < sp < sp3
(c) 2-propanol and acetal
(d) propane and methyl acetate 18. What is the molarity of H2SO4 solution if 25ml is
12. If the solutions of NaCl and NaNO3 are mixed in exactly neutralized with 32.63 ml of 0.164 M,
one beaker and the temperature adjusted to 383° NaOH ?
K, the contents of the beaker will most likely: (a) 0.107 M (b) 0.126 M
(a) freeze (b) boil (c) 0.214 M (d) – 0.428 M
Mock Test-7 MT-77

25. The electron in the hydrogen atom undergoes


19. In O2- , O 02 and O 22 - molecular species, the
transition from higher orbitals to orbital of radius
total number of antibonding electrons 211.6 pm. This transition is associated with :
respectively are :
(a) Lyman series (b) Balmer series
(a) 7, 6, 8 (b) 1, 0, 2
(c) Paschen series (d) Brackett series
(c) 6, 6, 6 (d) 8, 6, 8
20. Which of the following compounds are not 26. An organic amino compound reacts with aqueous
arranged in order of decreasing reactivity nitrous acid at low temperature to produce an
towards electrophilic substitution? oily nitrosoamine. The compound is:
(a) Methoxy benzene > Toluene > Bromo (a) CH3 NH2 (b) CH3CH2NH2
benzene (c) CH3CH2NH.CH2CH3 (d) (CH3 CH2)3N
(b) Phenol > N-Propylbenzene > Benzoic acid 27. In Kjeldahl’s method, nitrogen present in the
(c) Chlorotoluene > Para Nitro toluene > 2- organic compound is quantitatively converted into
chloro-4-Nitrotoluene (a) ammonium nitrite
(d) Benzoic acid > Phenol > N-Propylbenzene (b) ammonium sulphate
21. Which of the given sets of temperature and (c) ammonium phosphate
pressure will cause a gas to exhibit the greatest (d) ammonium nitrate
deviation from ideal gas behaviour ? 28. A white sodium salt dissolves readily in water to
(a) 100ºC & 4 atm (b) 100ºC & 2 atm
give a solution which is neutral to litmus. When
(c) –100ºC & 4 atm (d) 0ºC & 2 atm
silver nitrate solution is added to the solution, a
22. Which is not true for beryllium ?
(a) Beryllium is amphoteric white precipitate is obtained which does not
(b) It forms unusual carbide, Be2C dissolve in dil. HNO3. The anion could be
(c) Be(OH)2 is basic (a) CO 32 - (b) Cl -
(d) Beryllium halides are electron deficient
(c) SO 24 - (d) S2 -
23. Which of the following pairs show reverse
properties on moving along a period from left to 29. What quantity (in mL) of a 45% acid solution of
right and from up to down in a group : a mono-protic strong acid must be mixed with a
(a) Nuclear charge and electron affinity 20% solution of the same acid to produce 800
(b) Ionization energy and electron affinity mL of a 29.875% acid solution?
(c) Atomic radius and electron affinity (a) 320 (b) 325
(d) None of these (c) 316 (d) 330
24. The enthalpy change on freezing of 1 mole of 30. Excess of NaOH (aq) was added to 100 mL of
water at 5°C to ice at –5°C is : FeCl3 (aq) resulting into 2.14 g of Fe(OH)3. The
(Given DfusH = 6 kJ mol–1 at 0°C, molarity of FeCl3 (aq) is :
Cp(H2O, l) =75.3 J mol–1 K–1, (Given molar mass of Fe=56 g mol–1 and molar
mass of Cl=35.5 g mol–1)
Cp(H2O, s) = 36.8 J mol–1 K–1)
(a) 0.2 M (b) 0.3 M
(a) 5.44 kJ mol–1 (b) 5.81 kJ mol–1
–1 (c) 0.6 M (d) 1.8 M
(c) 6.56 kJ mol (d) 6.00 kJ mol–1
EBD_7206
MT-78 JEE MAIN

(a) 11.2 × 102 kms–1 (b) 112 kms–1


PHYSICS (c) 11.2 kms –1 (d) 11.2 × 10–2 kms–1
37. A coil having N turns is wound tightly in the
31. Two waves of wavelengths 99 cm and 100 cm form of a spiral with inner and outer radii a and b
both travelling with velocity 396 m/s are made respectively. When a current I passes through
to interfere. The number of beats produced per the coil, the magnetic field at the center is
second are
m o NI 2 m o NI
(a) 1 (b) 2 (a) (b)
(c) 4 (d) 8
b a
m 0 NI b m 0 IN a
32. In the isothermal expansion of 10g of gas from (c) ln (d) 2 b - a ln b
volume V to 2V the work done by the gas is 2 (b - a ) a ( )
575J. What is the root mean square speed of the 38. While determining the specific resistance of a
molecules of the gas at that temperature? wire using a metre bridge the formula used is
(a) 398m/s (b) 520m/s (where X, D, L and r denote unknown
(c) 499m/s (d) 532m/s resistance, diameter of the wire, the length of
33. An automobile travelling with a speed of 60 the wire and the specific resistance of the wire)
km/h, can brake to stop within a distance of 20m. XpD XpD 2
If the car is going twice as fast i.e., 120 km/h, the (a) r= (b) r =
4L 4L
stopping distance will be
(a) 60 m (b) 40 m X 2 pD 2 XpD 2
(c) 20 m (d) 80 m (c) r= (d) r =
4L 4L2
34. Using mass(M), length(L), time(T) and electric 39. A mass m hangs with the help of a string
current (A) as fundamental quantities the wrapped around a pulley on a frictionless
dimensions of permittivity will be bearing. The pulley has mass m and radius R.
(a) MLT–1A–1 (b) MLT–2A–2 Assuming pulley to be a perfect uniform circular
–1
(c) M L T A –3 4 2 (d) M2L–2T –2A2 disc, the acceleration of the mass m, if the string
does not slip on the pulley, is:
é pt ù
35. y = 2 (cm) sin ê + fú (a) g (b)
2
g
ë2 û 3
What is the maximum acceleration of the particle g 3
doing the SHM (c) (d) g
3 2
p p2 40. An ideal gas is initially at temperature T and
(a) cm/s2 (b) cm/s2 volume V. Its volume is increased by DV due to
2 2 an increase in temperature DT, pressure

(c)
p2
cm/s2 (d)
p
cm/s2 remaining constant. The quantity d = DV
4 4 V DT
varies with temperature as
36. The escape velocity for a body of mass 1 kg
from the earth surface is 11.2 kms–1. The escape
velocity for a body of mass 100 kg would be
Mock Test-7 MT-79

Y +Q –Q
Y

(a) (b) d a
d o X
o X T T + DT C
T T + DT Temperature (K)
Temperature (K)
–Q a +Q
Y (a) zero (b) kq/a 2
Y
(c) kq/ a 2 (d) None of these
(c) d
(d) d 45. At the centre of a circular coil of radius 5 cm
X o X
o T T + DT carrying current, magnetic field due to earth is
T T + DT
Temperature (K) Temperature (K)
0.5 × 10–5 W/m2. What should be the current
flowing through the coil so that it annuls the
41. A Hydrogen atom and a Li++ ion are both in the earth’s magnetic field
second excited state. If l H and l Li are their (a) 40A (b) 4 A
respective electronic angular momenta, and EH (c) 0.4 A (d) 0.2 A
and ELi their respective energies, then 46. In the arrangement shown in the Fig, the ends P
an d Q of an unstretchable string move
(a) l H > l Li and |EH| > |ELi|
downwards with uniform speed U. Pulleys A and
(b) l H = l Li and |EH| < |ELi| B are fixed.
Mass M moves upwards with a speed
(c) l H = l Li and |EH| > |ELi|
(a) 2Ucos q A B
(d) l H < l Li and |EH| < |ELi|
42. The temperature of reservoir of Carnot’s engine (b) U/ cos q
q q
operating with an efficency of 70% is 1000 kelvin.
The temperature of its sink is (c) 2U / cos q
P Q
M
(a) 300 K (b) 400 K
(d) U cos q
(c) 500 K (d) 700 K
43. A radar has a power of 1kW and is operating at 47. Two stars are 10 light years away from the earth.
a frequency of 10 GHz. It is located on a mountain They are seen through a telescope of objective
top of height 500 m. The maximum distance upto diameter 30 cm. The wavelength of light is 600
which it can detect object located on the surface nm. To see the stars just resolved by the
of the earth (Radius of earth = 6.4 × 106m) is : telescope, the minimum distance between them
(a) 80 km (b) 16 km should be (1 light year = 9.46 × 1015 m) of the
(c) 40 km (d) 64 km order of :
44. What is the electric potential at the centre of the (a) 108 km (b) 1010 km
11
(c) 10 km (d) 106 km
square?
EBD_7206
MT-80 JEE MAIN

48. A rectangular block of mass m and area of cross- 52. In a series LCR circuit R = 200W and the voltage
section A floats in a liquid of density r. If it is and the frequency of the main supply is 220V
given a small vertical displacemnet from and 50 Hz respectively. On taking out the
equilibrium it undergoes oscillation with a time capacitance from the circuit the current lags
period T. Then behind the voltage by 30°. On taking out the
1 1 inductor from the circuit the current leads the
(a) T µ (b) T µ voltage by 30°. The power dissipated in the LCR
A r
circuit is
1 (a) 305 W (b) 210 W
(c) T µ (d) T µ r
m (c) Zero W (d) 242 W
49. There are two wires of the same length. The 53. A parallel plate capacitor of area 60 cm2 and
diameter of second wire is twice that of the first. separation 3 mm is charged initially to 90 mC. If
On applying the same load to both the wires, the the medium between the plate gets slightly
extension produced in them will be in ratio of conducting and the plate loses the charge initially
at the rate of 2.5 × 10–8 C/s, then what is the
(a) 1 : 4 (b) 1 : 2
magnetic field between the plates ?
(c) 2 : 1 (d) 4 : 1 (a) 2.5 × 10–8 T (b) 2.0 × 10–7 T
50. A block is placed on a frictionless horizontal (c) 1.63 × 10 T–11 (d) Zero
table. The mass of the block is m and springs are 54. A sphere is placed in front of a convex lens of
attached on either side with force constants K1 focal length f. The radius of the sphere is much
and K2. If the block is displaced a little and left smaller compared to f. The image of the sphere
to oscillate, then the angular frequency of would look spherical if the object distance is
oscillation will be 3f
(a) f (b)
1 1 2
é K1K 2 ù 2 f
æ K1 + K 2 ö2 (c) 2f (d)
(a) çç ÷÷ (b) ê ú 2
è m ø ë m (K 1 + K 2 ) û th
æ1ö
1 1 55. A bullet looses ç ÷ of its velocity passing
é K 1K 2 ù2 é K12 + K 22 ù2 ènø
(c) ê ú (d) ê ú through one plank. The number of such planks
ë (K 1 - K 2 )m û êë (K1 + K 2 )m úû that are required to stop the bullet can be:
51. An air bubble of radius 0.1 cm is in a liquid n2 2n 2
having surface tension 0.06 N/m and density (a) (b)
2n - 1 n -1
103 kg/m3. The pressure inside the bubble is 1100 (c) infinite (d) n
Nm–2 greater than the atmospheric pressure. At 56. A liquid drop having 6 excess electrons is kept
what depth is the bubble below the surface of stationary under a uniform electric field of 25.5
the liquid? (g = 9.8 ms–2) kVm–1. The density of liquid is 1.26 × 103 kg m–
3. The radius of the drop is (neglect buoyancy).
(a) 0.1 m (b) 0.15 m
(c) 0.20 m (d) 0.25 m (a) 4.3 × 10–7 m (b) 7.8 × 10–7 m
–7
(c) 0.078 × 10 m (d) 3.4 × 10–7 m
Mock Test-7 MT-81

57. An ideal coil of 10H is connected in series


with a resistance of 5W and a battery of 5V. MATHEMATICS
2second after the connection is made, the
61. The function f (x) = tan –1(sin x + cos x) is an
current flowing in ampere in the circuit is
increasing function in
(a) (1 – e–1) (b) (1 – e)
p æ p pö
(c) e (d) e–1 (a) æç 0, ö÷ (b) çè - , ÷ø
58. The circuit shown here has two batteries of è 2ø 2 2
8.0 V and 16.0 V and three resistors 3 W, 9 W p p
and 9 W and a capacitor of 5.0 mF. (c) æç p , p ö÷ (d) æç - , ö÷
è 4 2ø è 2 4ø
I
62. The degree of differential equation
satisfying the relation
3W 5 mF 9W
1 + x 2 + 1 + y 2 = l( x 1 + y 2 - y 1 + x 2 )
9W is
8.0 V 16.0 V
(a) 1 (b) 2
(c) 3 (d) 4
I1 I2 63. The term independent of x in the expansion of
How much is the current (I) in the circuit in
[(t -1 - 1) x + (t -1 + 1)-1 x -1 ]8 is :
steady state?
(a) 1.6 A (b) 0.67 A 3 3
æ 1- t ö æ1+ t ö
(a) 56 ç ÷ (b) 56 ç ÷
(c) 2.5 A (d) 0.25 A è 1+ t ø è1- t ø
59. A beam of light of intensity 12 watt/cm2 is
incident on a totally reflecting plane mirror of 4 4
æ 1- t ö æ 1+ t ö
area 1.5 cm2, then the force (in newton) acting (c) 70 ç ÷ (d) 70 ç ÷
on the mirror will be è 1+ t ø è 1- t ø
(a) 2.4 × 10–6 (b) 1.2 × 10–7
x2 y 2
(c) 3.6 × 10–8 (d) 5.6×10–5 64. A tangent to the hyperbola - =1
60. A space craft of mass ‘M’, moving with 4 2
velocity ‘v’ suddenly breaks into two pieces. meets x-axis at P and y-axis at Q. Lines PR and
After the explosion mass ‘m’ becomes QR are drawn such that OPRQ is a rectangle
stationary. What is the velocity of the other (where O is the origin). Then R lies on :
part of the craft ? 4 2 2 4
Mv
(a) + =1 (b) - =1
2 2 2
(a) (b) v x y x y2
M-m
mv M-m 2 4 4 2
(c) + =1 (d) - =1
(c) (d) v 2 2 2
M m x y x y2
EBD_7206
MT-82 JEE MAIN

65. Given two independent events, if the probability (a) If R and S are transitive, then R È S is
26
transitive.
that exactly one of them occurs is and the (b) If R and S are transitive, then R Ç S is
49
transitive.
15 (c) If R and S are symmetric, then R È S is
, then
probability that none of them occurs is
49 symmetric.
the probability of more probable of the two (d) If R and S are reflexive, Then R Ç S is
events is : reflexive.
(a) 4/7 (b) 6/7
(c) 3/7 (d) 5/7
70. ò log 2x dx is
2
x2 x
ò [x ] dx , where [x] is the greatest integer £ x is
2
66. (a) x log 2x - (b) x log 2x -
2 2
0
x
(a) 5+ 2 + 3 (b) - 5 + 2 - 3 (c) x 2 log 2 x - (d) x log 2x - x + c
2
(c) 5- 2 - 3 (d) - 4 + 3 - 2 71. Area between curves y = x2, x = y2 is
67. The standard deviation of a variate x is s. The 1 1
(a) (b)
ax + b 9 3
standard deviation of the variable ; a, b,
c 1 2
c are constants, is (c) (d)
3 3
æaö a 72. The probability of A = Probability of B
(a) ç ÷s (b) s
ècø c 1
= Probability of C =
æa 2ö 4
(c) ç 2÷ s (d) None P(A) Ç P(B) Ç P(C) = 0, P(B Ç C) = 0 and
èc ø
68. a, b be the roots of x2 – 3x + a = 0 and g, d be the 1
P (A Ç C) = , P( A Ç B) = 0
roots of x2 – 12x + b = 0 and numbers a, b, g, d (in 8
order) form an increasing G.P. then the probability that atleast one of the events A, B,
(a) a = 3, b = 12 (b) a = 12, b = 3 C exists is
(c) a = 2, b = 32 (d) a = 4, b = 16 5 37
69. Assume R and S are (non-empty) relations (a) (b)
8 64
in a set A. Which of the following relation
given below is false 3
(c) (d) 1
4
Mock Test-7 MT-83

6 78. If a positive integer n is divisible by 9, then the


æ 1 ö sum of the digits of n is divisible by 9. So which
73. Coefficient of x6 in the expansion çç x + 2 ÷÷ is
è x ø statement is it contrapositive.
(a) 10 (b) 15 (a) (sum of digits of n is divisible by 9)
Þ (n is divisible by 9)
(c) 16 (d) None
(b) (sum of digits of n is not divisible by 9)
74. For a given integer k, in the interval Þ (n is not divisible by 9)
é p pù (c) (sum of digits of n is divisible by 9)
ê2pk + 2 , 2pk - 2 ú the graph of sin x is Þ (n is divisible by 9)
ë û (d) None of these
(a) increasing from –1 to 1 79. Fifteen coupons are numbered 1, 2 .....
(b) decreasing from –1 to 0 15,respectively. Seven coupons are selected at
(c) decreasing from 0 to 1 random one at a time with replacement. The
(d) None of these probability that the largest number appearing
dy on a selected coupon is 9, is
75. + y = 2e 2 x then y is
dx æ9 ö
6
æ8 ö
7

2 2x (a) ç ÷ (b) ç ÷
(a) ce - x + e è 16 ø è 15 ø
3
7
2 2x æ3ö
(b) (1 + x )e - x + e +c (c) ç ÷ (d) None of these
3 è5ø
2 2x
(c) ce - x + e +c 80. If a £ 0 then roots of x 2 - 2a x - a - 3a 2 = 0 is
3
(d) e - x + 2 e 2 x + c (a) ( -1 + 6)a (b) ( 6 - 1)a
3 (c) a (d) None of these
a+x a-x a-x 81. If n(A) = 1000, n(B) = 500 and if n(A Ç B) ³ 1 and
n(A È B) = p, then
76. If a - x a + x a - x = 0 then x is
(a) 500 £ p £ 1000 (b) 1001 £ p £ 1498
a-x a-x a+x (c) 1000 £ p £ 1498 (d) 1000 £ p £ 1499
(a) 0, 2a (b) a, 2a dy
(c) 0, 3a (d) None of these 82. If y = log 2 {log 2 ( x)} , then is
dx
77. The equation of a circle with origin as centre and
passing through the vertices of an equilateral log 2 e 2.3026
triangle whose median is of length 3a is (a) (b)
x ln x x ln x ln 2
(a) x 2 + y 2 = 9a 2 (b) x 2 + y 2 = 16a 2
1
(c) (d) None of these
(c) 2 2
x + y = 4a 2 2 2
(d) x + y = a 2
ln ( 2 x ) x
EBD_7206
MT-84 JEE MAIN

sin 3x 87. The least integral value a of x such that


83. f (x) = , when x ¹ 0
sin x x-5
2
> 0 , satisfies :
= k, when x = 0 x + 5x - 14
for the function to be continuous k should be (a) a2 + 3a – 4 = 0 (b) a2 – 5a + 4 = 0
(a) 3 (b) 0 (c) a2 – 7a + 6 = 0 (d) a2 + 5a – 6 = 0
(c) –3 (d) –1 88. The x satisfying
84. f ( x ) = sin x . f (x ) is not differentiable at sin -1 x + sin -1 (1 - x ) = cos -1 x are
(a) x = 0 only (b) all x (a) 1, 0 (b) 1, –1
p
(c) multiples of p (d) multiples of 1
2 (c) 0, (d) None of these
2
p/3
cos x + sin x 89. The S.D. of the following data is nearly
85. ò 1 + sin 2x
dx is
0 x i 140 145 150 155 160 165 170 175
4p 2p fi 4 6 15 30 36 24 8 2
(a) (b)
3 3
p (a) 8. 64 (b) 7. 26
(c) p (d) (c) 7. 05 (d) None
3
86. The angle between the pair of tangents drawn to 90. A line passes through (2,2) and is perpendicular
to the line 3x + y = 3 its y intercept is
the ellipse 3x 2 + 2y 2 = 5 from the point (1,2) is
1 2
(a) (b)
(a)
æ 12 ö
tan -1 ç ÷
è5ø
( )
(b) tan -1 6 5 3 3

4
(c) 1 (d)
(c)
æ 12 ö
tan -1 çç ÷÷ (
(d) tan -1 12 5 ) 3

è 5ø
Download From https://iit-jeeacademy.blogspot.com

8 JEE MAIN
MOCK TEST
Time : 3 hrs. Max. Marks : 360

INSTRUCTIONS
• Chemistry (120 marks) : Question No. 1 to 30 are of 4 marks each.

• Physics (120 marks) : Question No. 31 to 60 are of 4 marks each.

• Mathematics (120 marks) : Question No. 61 to 90 are of 4 marks each.


• Negative Marking : One fourth (¼) marks will be deducted for indicating incorrect
response of each question.

(a) (A)–(iii), (B)–(i), (C)–(iv), (D)–(ii)


CHEMISTRY (b) (A)–(iv), (B)–(i), (C)–(ii), (D)–(iii)
1. At 300 K, the density of a certain gaseous molecule (c) (A)–(ii), (B)–(i), (C)–(iv), (D)–(iii)
at 2 bar is double to that of dinitrogen (N2) at (d) (A)–(ii), (B)–(iv), (C)–(i), (D)–(iii)
4 bar. The molar mass of gaseous molecule is : 3. Which of the following has highest boiling point?
(a) 28 g mol–1 (b) 56 g mol–1 (a) H2Se (b) H2Te
(c) 112 g mol –1 (d) 224 g mol–1 (c) H2S (d) H2O
2. Match the items in Column I with its main use 4. Which of the following is non-aromatic?
listed in Column II:
(a) Benzene
Column I Column II
(A) Silica gel (i) Transistor (b) Cycloheptatrienyl cation
(B) Silicon (ii) Ion–exchanger (c) Cyclopentadienyl anion
(C) Silicone (iii) Drying agent
(D) Silicate (iv) Sealant (d) Cyclopentadienyl free radical
EBD_7206
MT-86 JEE MAIN

5. If the energy of an electron moving in orbit no. 1 9. p-Chloroaniline and anilinium hydrochloride can
is given by ‘E’, then energy of an electron in be distinguished by
orbit no. 2 of Li2+ (z = 3) is given by (a) Sandmeyer reaction (b) NaHCO3
(c) AgNO3 (d) Carbylamine test
9
E
10. ® I3- the Lewis acid is
In the reaction I 2 + I – ¾¾
(a) E (b)
4 (a) I2 (b) I–
(c) I3 – (d) None of these
E
(c) 9 E (d) 11. Which is the only compound which is
4
6. What is the value of x in the following complex? deactivating yet ortho-para directing when
attached to benzene ring?
OH (a) NO2 (b) CH3
(NH3)4 Co Co (NH3)2 (SO4)X (c) Cl2 (d) None of these
OH
C–CºH
Consider the oxidation state of Cobalt in the 12. +CH 3 - CH 2 MgBr gives
complex same as that of iron in K3[Fe(CN)6]?
(a) 1 (b) 2 C º CMgBr + CH3 - CH2
(c) 3 (d) 0 (a)
7. What is the structure of the major product when
phenol is treated with bromine water ? C º CMgBr + C2H6
(b)
OH
O
Br Br Br Br MgBr + CH3 - CH2 - C º CH
(a) (c)
(b)

Br Br C2H5 + HC º CMgBr
H
(d)
OH OH
13. Which of the following will be most reactive
Br towards nucleophillic substitution.
(c) (d)
Br Cl
Br
8. Reduction of esters with sodium and alcohol is (a) (b)
referred to as
(a) Bouvealt-Blanc reaction
(b) Merdius reaction OH OSO2CF3
(c) Clemensen’s reduction
(d) MPV reduction (c) (d)
Mock Test-8 MT-87

14. Ozone reacts with dry iodine to give 20. Which of the following compounds is most
(a) IO2 (b) I2O3 reactive to an aqueous solution of sodium
(c) I2O4 (d) I4O9 carbonate ?
15. The hydrogen electrode is dipped in a solution of
pH = 3 at 25ºC. The potential of the cell would be
(a) 0.177 V (b) –0.177 V (a) (b)
(c) 0.087 V (d) 0.059 V
16. 50 mL of 0.2 M ammonia solution is treated with
25 mL of 0.2 M HCl. If pKb of ammonia solution
is 4.75, the pH of the mixture will be : (c) (d)
(a) 3.75 (b) 4.75
(c) 8.25 (d) 9.25 21. AgBr dissolves in Na2S2O3 solution and forms
17. Dipole moment is shown by (A). On boiling an aquous solution of (A), a ppt.
(a) 1, 4-dichlorobenzene (B) is obtained. The colour of B is
(b) cis 1,2-dichloroethene (a) Red (b) White
(c) trans 1,2-dichloroethene
(c) Black (d) Colourless
(d) trans 2,3-dichloro-2 butene
22. A laboratory reagent imparts green colour to the
18. The relationship between the values of osmotic
flame. On heating with solid K2Cr2O7 and conc.
pressures of solutions obtained by dissolving
H2SO4 it evolves a red gas. Identify the reagent
6.00 g L–1 of CH3COOH (p1) and 7.45 g L–1 of
KCl (p2) is (a) CaCl2 (b) BaCl2
(a) p1 < p2 (c) CuCl2 (d) None of these
23. Which of the following does not give
(b) p1 > p2
effervescences of CO 2 with aq. NaHCO 3
(c) p1 = p2 solution?
p1 p2 OH
(d) =
p1 + p 2 p1 + p 2 NO2
19. In what manner will increase of pressure effect (a) H2CO3 (b)
the following equilibrium?
NO2
C(s) + H 2 O(g) CO(g) + H 2 (g)
OH
(a) Shift in forward direction
O2N NO2
(b) Shift in reverse direction
(c) (d) None of these
(c) Increase in yield of hydrogen
(d) No effect
EBD_7206
MT-88 JEE MAIN

24. In the preparation of p-nitro acetanilide from 29. For a reaction, A(g) ® A(l); DH= –3RT. The
aniline titration is not done by nitrating mixture (a correct statement for the reaction is :
mixture of conc. H2SO4 and conc. HNO3) because (a) DH = DU ¹ O (b) DH = DU = O
(a) on nitration it gives o-nitro acetanilide (c) |DH| < |DU| (d) |DH| > |DU|
(b) lt gives a mixture of o and p nitro aniline 30. The amount of arsenic pentasulphide that can
(c) –NH2 group gets oxidised be obtained when 35.5 g arsenic acid is treated
(d) it forms a mixture of o and p nitro acetanilide. with excess H2S in the presence of conc. HCl
25. KMnO4 reacts with oxalic acid as : (assuming 100% conversion) is :
MnO-4 + C2O24- + H+ ¾ ¾® Mn2+ + CO2 + H2O (a) 0.25 mole (b) 0.50 mole
Hence, 50 ml of 0.04 M KMnO4 is acidic medium (c) 0.333 mole (d) 0.125 mole
is chemically equivalent to
(a) 100 ml of 0.1 M H2C2O4
(b) 50 ml of 0.2 M H2C2O4
PHYSICS
(c) 50 ml of 0.1 M H2C2O4 Cp
(d) 25 ml of 0.1 M H2C2O4 31. A given ideal gas with g = = 1.5 at a
Cv
26. An alloy of copper, silver and gold is found to
have copper constituting the ccp lattice. If silver temperature T. If the gas is compressed
atoms occupy the edge centres and gold is adiabatically to one-fourth of its initial volume,
present at body centre the alloy has a formula the final temperature will be
(a) Cu4Ag2Au (b) Cu4Ag4Au (a) 2 2T (b) 4 T
(c) Cu4Ag3Au (d) CuAgAu (c) 2 T (d) 8 T
27. The rate of a reaction quadruples when the 32. Orbits of a particle moving in a circle are such
temperature changes from 300 to 310 K. The that the perimeter of the orbit equals an integer
activation energy of this reaction is : (Assume number of de-Broglie wavelengths of the
activation energy and pre-exponential factor are
independent of temperature; ln 2 = 0.693; particle. For a charged particle moving in a plane
R = 8.314 J mol–1 K–1) perpendicular to a magnetic field, the radius of
(a) 107.2 kJ mol–1 (b) 53.6 kJ mol–1 the nth orbital will therefore be proportional to :
(c) 26.8 kJ mol–1 (d) 214.4 kJ mol–1 (a) n 2 (b) n
OH (c) n 1/2 (d) n 1/4
TsCl
28. ¾¾¾® A, Then A is
LiAlH4 33. In an organ pipe, three successive resonance
OLi frequencies are observed at 425, 595 and 765Hz.
(a) (b) The length of organ pipe is (Velocity of sound
in air = 340 m/s).
(a) 2.0 m (b) 1.5 m
Cl
(c) (d) (c) 1.0 m (d) 2.5 m.
Mock Test-8 MT-89

34. Two bodies of mass m and 2m connected by an The speed of its image at the instant is
unextended spring of spring constant ‘k’ are (a) 4 cm/s towards the mirror
allowed to fall simultaneously in a uniform
gravitational field g. The extension ‘x’ in the spring (b) 9 cm/s towards the mirror
when the bodies A and B are falling (c) 4 cm/s away from the mirror
(d) 9 cm/s away from the mirror
m 38. An aluminium foil of relative emittance of 0.1 is placed
in between two concentric spheres at temperature
g 300 K and 200 K respectively. What is the temperature
of this foil when steady state is reached? Assume the
spheres to be black bodies. (s = 5.672 × 10–8)
2m (a) 264 K (b) 282 K
(c) 251 K (d) none
(a) varies with time as x = A sin (wt + f)
39. The space between the parallel plates of a
2mg capacitor is tightly filled with three dielectric
(b) is constant and has value
k slabs A,B, C of thickness 5mm, 3mm and 2mm
(c) is zero with dielectric constants 2, 3 and 5 respectively.
(d) increases linearly with time If a potential difference of 351 volts is applied to
35. An unstable nucleus decays in three different plates then electric field intensity in slab A is
modes, each mode having a different half-life
T1, T2, T3 (T1 >> T2 >> T3). The overall half-life (a) 4.5 × 104 V/m (b) 3 × 104 V/m
of the sample would be (c) 2.5 × 104 V/m (d) none of these
(a) T » T1 (b) T » T3 40. A small bar magnet is being slowly inserted with
constant velocity inside a solenoid as shown in
T + T2 + T3 figure. Which graph best represents the
(c) T» 1 (d) none of these.
3 relationship between emf induced with time
36. In young’s double slit experiment, if intensity of
maxima on screen is I (assume equal intensity of
the source). If the sources become incoherent,

average intensity on the screen is I2, then the


I1
ratio is
I2 (a) (b)
Time Time
1
(a) 2 (b) (c) 1 (d) 0
2
37. An object is moving towards a concave mirror
of focal length 24 cm. When it is at a distance of (c) (d)
Time
60 cm from the mirror its speed is 9 cm/s. Time
EBD_7206
MT-90 JEE MAIN

41. Given that radius of earth is 6.4 × 106 m and a l


transmitting antenna at the top of a tower has a wavelengths l and . If the maximum kinetic
height 40 m and the height of the receiving 2
antenna is 50 m. What is the maximum distance energy of the emitted photoelectrons in the
between them for satisfactory communication second case is 3 times that in the first case, the
in los mode? work function of the surface is :
(a) 45 km (b) 48 km hc hc hc 3hc
(a) (b) (c) (d)
(c) 50 km (d) 57 km 2l l 3l l
46. Consider two npn transistors as shown in figure.
42. While measuring the length of the rod by vernier If 0 Volts corresponds to false and 5 Volts
callipers the reading on main scale is 6.4 cm and correspond to true th en the output at C
the eight division on vernier is in line with corresponds to :
marking on main scale division. If the least count 5V
of callipers is 0.01 and zero error –0.04 cm, the
length of the rod is
C
(a) 5.25 cm (b) 6.52 cm
1
(c) 5.52 cm (d) 6.44 cm A
43. A structural steel rod has a radius of 10 mm and
length of 1.0 m. A 100 kN force stretches it along
its length. Young’s modulus of structural steel 2
is 2 × 1011 Nm–2. The percentage strain is about B
(a) 0.16% (b) 0.32%
(c) 0.08% (d) 0.24%
44. A ball is thrown from a point with a speed ' v0 '
at an elevation angle of q. From the same point (a) A NAND B (b) A OR B
and at the same instant, a person starts running (c) A AND B (d) A NOR B
' v0 ' 47. The figure of merit of a galvanometer is defined as
with a constant speed to catch the ball.
2 (a) the voltage required to produce unit
Will the person be able to catch the ball? If yes, deflection in the galvanometer
what should be the angle of projection q ? (b) the current required to produce unit
(a) No (b) Yes, 30° deflection in the galvanometer
(c) Yes, 60° (d) Yes, 45°
45. A photoelectric surface is illuminated (c) the power required to produce unit
successively by monochromatic light of deflection in the galvanometer
(d) none of these
Mock Test-8 MT-91

48. The activity of a radioactive element decreases 52. A moving particle of mass m, makes a head on elastic
to one-third of the original activity I0 in a period collision with another particle of mass 2m, which is
of nine years. After a further lapse of nine years initially at rest. The percentage loss in energy of the
its activity will be colliding particle on collision, is close to
2 (a) 33% (b) 67%
(a) I0 (b) I 0
3 (c) 90% (d) 10%
I0 I0 53. Which of the following graph represents the
(c) (d)
9 6 input characteristics of a common emmiter
49. In the X-rays tube before striking the target we transistor correctly ?
accelerate the electrons through a potential
difference of U volt. For which of the following Y
Y
value of U, we will have X-rays of largest

Base current
wavelength?

Base current
(a) 10 kV (b) 20 kV (a) (b)
(c) 30kV (d) 40 kV
Base voltage X X
Base voltage
50. Two bodies A and B of masses 10 kg and 5 kg are
placed very slightly separated as shown in figure. Y
Y

Base current

Base current
(c) (d)
A
F B Base voltage
X Base Voltage X

10kg 5kg
54. A charged ball B hangs from a silk thread S,
The coefficients of friction between the floor and which makes an angle q with a large charged
the blocks are as µs = µk = 0.4, g between block A
conducting sheet P, as shown in the figure. The
and ground breaks, block A will start pressing
block B and when welding of B also breaks, block surface charge density s of the sheet is
B will start pressing the vertical wall. If F = 20 N, proportional to
with how much force does block A presses the
block B
P
(a) 10 N (b) 20 N q
(c) 30 N (d) Zero S
51. The velocity of a particle is v = v0 + gt + ft2. If its
position is x = 0 at t = 0, then its displacement
after unit time (t = 1) is B
(a) v0 + g /2 + f (b) v0 + 2g + 3f (a) cot q (b) cos q
(c) v0 + g /2 + f/3 (d) v0 + g + f (c) tan q (d) sin q
EBD_7206
MT-92 JEE MAIN

55. From a sphere of mass M and radius R, a smaller 57. A tennis ball (treated as hollow spherical shell)
R starting from O rolls down a hill. At point A the
sphere of radius is carved out such that the ball becomes air borne leaving at an angle of 30°
2
cavity made in the original sphere is between with the horizontal. The ball strikes the ground
its centre and the periphery (See figure). For at B. What is the value of the distance AB ?
the configuration in the figure where the (Moment of inertia of a spherical shell of mass m
distance between the centre of the original 2
and radius R about its diameter = mR 2 )
sphere and the removed sphere is 3R, the 3
gravitational force between the two sphere is: O

2.0 m

30°
3R 0.2 m A B
41 GM 2 41 GM 2 (a) 1.87 m (b) 2.08 m
(a) 2 (b) 2
3600 R 450 R (c) 1.57 m (d) 1.77 m
59 GM 2
GM 2 58. There is a spherical cavity of radius R/2 in uniformly
(c) 2 (d) 2 charged spherical region having charge density
450 R 225 R
56. The efficiencyof an ideal gas with adiabatic exponent r and radius R. A small charged particle having
‘g’ for the shown cyclic process would be q0 charge is released at point O. It will collide with
the wall of cavity with kinetic energy
V C
2V0
R
V0 R/2
B A

r
T0 2T0 T

(2ln2 –1) (1 - 2ln2) rR 2 q 0 rR 2 q 0


(a) (b) (a) (b)
g /(g - 1) g /(g - 1) 3 Î0 6p Î0
(2 ln2 + 1) rR 2 q 0 rR 2 q 0
(c) (d) None of these (c) (d)
g /(g -1) 4p Î0 12 Î0
Mock Test-8 MT-93

59. The effective resistance between points A and


1
B of infinite network is (a) 3 (b)
3
1W 1W 1W
A
1W 4
(c) (d) 1
2W 2W 2W
3

B æ 2p ö æ 4p ö
64. If x cos q = y cos ç q + ÷ ³ cos ç q + ÷ ,
è 3 ø è 3ø
(a) 1W (b) 2W
(c) 3W (d) 4W then value of xy + yz + zx is
60. 500 g of water and 100 g of ice at 0°C are in a (a) 1 (b) 0 (c) 2 (d) 3
65. 25
Coefficient of x in the expansion of expression
calorimeter whose water equivalent is 40 g. 10 g
of steam at 100°C is added to it. Then water in 50
the calorimeter is : (Latent heat of ice = 80 cal/g, å 50 Cr (2x - 3)r (2 - x) n -r is
Latent heat of steam = 540 cal/ g) r =0

(a) 580 g (b) 590 g (a) 50 (b) -50 C 24


C 25
(c) 600 g (d) 610 g
(c) -50 C25 (d) -50 C30
MATHEMATICS 66. The equation x3 + 5x2 + px + q = 0 and x3 + 7x2 +
61. For what value of ‘p’ , y2 + xy + px2 – x – 2y + p = 0 px + r = 0 have two roots in common. If their third
represent 2 straight lines : roots are x1 and x2, then (x1, x2) =
2 (a) (–5, –7) (b) (5, 7)
(a) 2 (b)
3 (c) (5, –7) (d) (–5, 7)
1 1 67. The roots of the equation x5 – 40x4 + px3 + qx2 +
(c) (d)
4 2 rx + s = 0 are in geometric progression and the
62. If f (x) = 1 - 1 - x 2 , then at x = 0, sum of their reciprocal is 10. Then |S| is equal to
(a) 64 (b) 16
(a) f (x) is differentiable as well as continuous (c) 32 (d) None of these
(b) f (x) is differentiable but not continuous 68. The number of ways of selecting 4 cards out of
(c) f (x) is continuous but not differentiable 52 cards of an ordinary pack, so that exactly 3 of
(d) f (x) is neither continuous nor differentiable them are of same denomination is
63. If the length of a focal chord of parabola y2 = 4x is
25 (a) 52 × 48 (b) 13 C3 ´4 C3 ´ 48
and has positive slope, then the slope will be
4
52 52
(c) C3 ´ 48 (d) C 3 ´48 C 3
EBD_7206
MT-94 JEE MAIN

69. Let | a |= 2 , | b |= 5 . The values of k for which


ex sin 2x tan x 2
the vectors a + k b an d a - kb are 74. If D ( x) = ln(1 + x ) cos x sin x =A+ Bx + Cx2
perpendicular is cos x 2 e x - 1 sin x 2
2 3
(a) (b) + .... then B =
5 5
(a) 0 (b) 1
4 2 (c) 2 (d) None of these
(c) (d)
5 25 75. If in a DABC, ÐC = 90°, then value of tan(A – B)
70. The domain of the function
a 2 + b2 | a 2 - b2 |
2 is
f (x) = exp( 5x - 3 - 2x ) is 2ab 2ab
(a) [3/2, ¥) (b) [1, 3/2]
(c) (–¥, 1] (d) (1, 3/2) | a2 + b2 | | a 2 - b2 |
71. From a pack of 52 cards two cards are drawn at (a) (b)
2ab 2ab
random. The probability of both cards being
spades is c4 ca
(c) (d)
1 2 2 ab 2b
(a) (b)
17 17
(1 - iz )
76. If z = x + iy and w = , then | w | = 1, show
3 1 (z - i)
(c) (d)
17 15 that in complex plane
72. If f(A) = 3, f ¢(A) = –2, g(A) = –1, g¢(A) = 4, then (a) z is situated on imaginary axis
(b) z is situated on real axis
g ( x )f ( a ) - g ( a ) f ( x )
lim (c) z is situated on unit circle
x ®a x-a
(d) None of these
(a) –5 (b) 10
77. If z, wz and w z are the vertices of a triangle,
(c) –10 (d) 5
then the area of the triangle will be (where w is
73. Solution of differential equation cube root of unity) :
dy
= sin x + 2 x , is 3 | z |2 3 3 | z |2
dx (a) (b)
2 2
(a) y = x2 – cos x + c (b) y = cos x + x2 + c
3 | z |2
(c) y = cos x + 2 (d) y = cos x + 2 + c (c) (d) None of these
2
Mock Test-8 MT-95

78. From a point (h, 0) common tangents are drawn 82. A variable circle passes through the fixed point
to circles x2 + y2 = 1 and (x – 2)2 + y2 = 4, value A( p, q ) and touches x-axis. The locus of the
of h is
other end of the diameter through A is
(a) 2 (b) –2
(a) ( y - q)2 = 4 px (b) ( x - q)2 = 4 py
2 2
(c) - (d)
3 3 (c) ( y - p)2 = 4qx (d) ( x - p)2 = 4qy
79. The maximum area of a right angled triangle with
hypotenuse h is : é2 4 5 ù
ê ú
83. If A = ê 4 8 10 ú . Then rank of A is
h2 h2
(a) (b) êë- 6 - 12 - 15úû
2 2 2
equal to :
h2 h2
(c) (d) (a) 0 (b) 1
2 4
(c) 2 (d) 3
1 æx ö rr rr rr
80. If ò 1 + sin x dx = tan çè 2 + a ÷ø + b then 84. ( ) ( ) (
The vector i$ ´ a.b $i + $j ´ a.b $j + k$ ´ a.b k$ is )
equal to :
p p r r r
(a) a = - , bÎ R (b) a = , b Î R (a) b ´ a (b) a
4 4 r r r
(c) a ´ b (d) b
5p
(c) a = , bÎR (d) None of these 2 2
4 85. z1 + z2 + z1 - z2 is equal to

81.
ì1 - [ x]
ï
If f (x) = í 1 + x
, x ¹ -1
, then the value of
(a) 2 ( z1 + z2 ) (
(b) 2 z1 + z2
2 2
)
ïî1 , x = -1
2 2
(c) z1 z2 (d) z1 + z2
f ( 2 k ) will be (where [] shows the greatest 86. If a coin is tossed n times, the probability that
integer function] head appears odd no. of times is
(a) Continuous at x = –1
1 1
(b) Continuous at x = 0 (a) n (b) n -1
2 2
1
(c) Discontinuous at x = 1
2 (c) (d) None of these
2
(d) All of these
EBD_7206
MT-96 JEE MAIN

87. The complete solution set for sin–1(x) = 3 sin–1 (a) is


æ x 2 - y2 ö
89. If tan -1 ç 2 ÷ = e a where a is constant,
1 1 ç x + y2 ÷
(a) 0£a £ (b) - £a£0 è ø
2 2
dy
1 1 then
(c) - £a £ (d) None of these dx
2 2
x y
88. The equation of the plane through the line of (a) (b)
y x
intersection of planes
ax + by + cz + d = 0, a ' x + b' y + c' z + d' = 0 x2 y2
(c) (d)
and parallel to the line y = 0, z = 0 is y2 x2
(a) (ab'-a ' b) x + (bc '-b' c) y + (ad'-a ' d) = 0 90. The coordinates of the foot of perpendicular from
the point (1, 0, 0) to the line
(b) (ab'-a ' b) x + (bc'- b' c) y + (ad'-a ' d )z = 0
x - 1 y + 1 z + 10
(c) (ab'-a ' b) y + (ac'-a ' c)z + (ad'-a ' d) = 0 = = are
2 -3 8
(d) None of these (a) (2, – 3, 8) (b) (1, – 1, – 10)
(c) (5, – 8, – 4) (d) (3, – 4, – 2)
Download From https://iit-jeeacademy.blogspot.com

9 JEE MAIN
MOCK TEST
Time : 3 hrs. Max. Marks : 360

INSTRUCTIONS
• Chemistry (120 marks) : Question No. 1 to 30 are of 4 marks each.

• Physics (120 marks) : Question No. 31 to 60 are of 4 marks each.

• Mathematics (120 marks) : Question No. 61 to 90 are of 4 marks each.


• Negative Marking : One fourth (¼) marks will be deducted for indicating incorrect
response of each question.

(c) – 850 kJ mol–1 (d) + 120 kJ mol–1


CHEMISTRY 2. A hydrocarbon with five carbon atoms in the
molecule, decolourizes alkaline KMnO4, but does
1. Oxidising power of chlorine in aqueous solution can not give a precipitate with ammonical Cu2Cl2
be determined by the parameters indicated below:
1
solution. The hydrocarbon is possibly
o
1 D H D eg Ho
2 diss ® Cl(g) ¾¾¾¾ (a) 1-pentyne (b) 1, 3-pentadiyne
Cl 2 (g) ¾¾¾¾¾ ® Cl – (g)
2 (c) 2-pentyne (d) 1, 4-pentadiyne
D Ho 3. The threshold frequency of a metal is 1 ´ 1015
¾¾¾¾Hyd
® Cl – (aq) s–1. The ratio of maximum kinetic energies of
(using the data, the photoelectrons when the metal is irradiated
–1
D H° = 240 kJ mol ,
diss Cl 2 with radiations of frequencies 1.5 ´ 1015 s–1
–1
D eg H °Cl = –349 kJ mol , and 2 ´ 1015 s–1 respectively would be
o –1 (a) 3 : 4 (b) 1 : 2
D hyd H Cl – = –381 kJ mol ), will be
(c) 2 : 1 (d) 4 : 3
(a) + 152 kJ mol–1 (b) – 610 kJ mol–1
EBD_7206
MT-98 JEE MAIN

4. Bond distance in HF is 9.17 × 10–11 m. Dipole 9. Identify the feasible reaction among the
moment of HF is 6.104 × 10–30 Cm. The following:
percentage ionic character in HF will be : D
(a) K 2 CO3 ¾¾
® K 2 O + CO 2
(electron charge = 1.60 × 10–19 C) D
(a) 61.0% (b) 38.0% (b) Na 2 CO 3 ¾¾® Na 2 O + CO 2
(c) 35.5% (d) 41.5% D
(c) Li 2 CO 3 ¾¾
® Li 2 O + CO 2
5. In order to convert R – X to R – R’ which reaction
is most suitable (d) Rb 2 CO 3 ¾¾ ® Rb 2 O + CO 2
(a) Corey House reaction 10. Which of the following gives n-nitrosoamine
(b) Kolbe’s reaction on reaction with nitrous acid?
(c) Williamson’s synthesis CH3 CH3
(d) Wurtz reaction NH2 N
6. Lithium chloride has a cubic structure as shown
below. If the edge length is 400 pm., then the
radii of Cl– ions is (a) (b)

Cl
O
NHCH3 C–NH–CH3
+ a/2
Li (c) (d)
(a) 100pm (b) 200 pm
(c) 141.4pm (d) 282.8pm 11. Normality of a mixed solution of sulphuric acid
7. Which of the following order of root mean and hydrochloric acid is 0.6 N. 20 mL of this
squar e speed of different gases at same solution gives 0.4305 g of AgCl on reacting with
temperature is true? AgNO 3 solution. The strength of H 2SO 4 in
(a) (u rms ) H 2 > (u rms ) CH4 > ( u rms ) NH3 > ( u rms ) CO2 g/ L in the mixed solution is
(a) 42.05 g/ L (b) 22.05 g/ L
(b) (u rms )H 2 < ( u rms )CH 4 < ( u rms ) NH3 < (u rms )CO2 (c) 28.56 g/ L (d) 37.05 g/ L
(c) (u rms )H 2 < ( u rms )CH 4 > (u rms ) NH3 > (u rms )CO2 12. Consider the reaction
(d) (u rms )H 2 > (u rms )CH4 < (u rms ) NH3 < ( u rms ) CO2 CaCO3(s) CaO(s) + CO2(g) in a closed
container at equilibrium. What would be the
8. A solution containing a group–IV cation gives a
effect of addition of CaCO3 on the equilibrium
precipitate on passing H2S. A solution of this concentration of CO2 ?
precipitate in dil.HCl produces a white precipitate
(a) Increases
with NaOH solution and bluish–white precipitate
(b) Decreases
with basic potassium ferrocyanide. The cation is :
(a) Co2+ (b) Ni2+ (c) Data insufficient
(c) Mn 2+ (d) Zn 2+ (d) Remains unaffected
Mock Test-9 MT-99

13. Consider the reaction sequence below : 14. How many P–H and O–H bonds respectively,
are present in H4P2O7 molecule?
OCH3
(a) 1 and 3 (b) 0 and 4
(c) 4 and 0 (d) 2 and 3
Succinic anhydride Clemmenson 's
¾¾¾¾¾¾¾® A ¾¾¾¾¾¾® X 15. Br ,
AlCl3 reduction

CH–CH2Br
is Alc. KOH NaNH2
A B
OH NaNH2 CH 3CH 2Cl
OCH3
A and B are

(a) CH=CH2 CH2–CH3

(a) A= ;B=

OH C º CH C º CCH2CH3
OH
(b) A = ;B=
H3CO
(b) C º CH CH2CH2C º CH

(c) A= ;B=

(d) None of these


OH 16. The half life of a first order reaction is 10 min. If initial
OCH3 amount is 0.08 mol L-1 and concentration at some
instant is 0.01 mol L-1. What is the time elapsed ?
(a) 10 min (b) 20 min
(c) 30 min (d) 40 min
(c) 17. In the Freundlich adsorption isotherm equation
graph. The slope is equal to

H3CO
Slope = ?

(d)
log x/m
log p
EBD_7206
MT-100 JEE MAIN

(a) n (b) 1/n 24. The major product of the following reaction is :
(c) log k (d) log k – 1/n
18. Which one of the following on treatment with OH
50% aqueous sodium hydroxide yields the 1. K2CO3
corresponding alcohol and acid? 2. CH3I (1. eq.)
(a) C6H5CHO (b) CH3CH2CH2CHO OH
O OCH3
||
(c) CH 3 - C - CH 3 (d) C6H5CH2CHO
(a)
19. The complexes of Nickel (II) can be
OH
(a) Square planar, tetrahedral and octahedral O
(b) Square planar and octahedral
(c) Tetrahedral and octahedral (b)
(d) Square planar only
OH
20. Which of the following represents the correct
order of stability?
(c)
(a) H - C º C :- > H 2 C = CH :- > H 3C - CH 2 :-
OCH3
(b) H3C - CH 2 :- > H 2 C = CH :- H - C º C :- OCH3

(c) H 3C - CH 2 :- = H 2C = CH :- = H - C º C :- (d)
(d) None of these
21. Which of the metal is extracted by Hall-Heroult 25. The hybridization states of the central atoms in
process? the complexes [Fe(CN)6]3–, [Fe(CN)6]4– and
(a) Al (b) Cu [Co(NO2)6]3– are
(c) Ni (d) Zn (a) d2sp3, sp3 and d4s2 respectively
O (b) d2sp3, sp3d and sp3d2 respectively
NH OH H SO (c) d2sp3, sp3d2 and dsp2 respectively
22. ¾¾ ¾
2
¾® A ¾¾2 ¾
¾4
® B. The (d) all d2sp3
26. Among the acids given below
product B is CH3CH2COOH (X); CH2 = CHCOOH (Y) and
(a) Caprolactam (b) Lactone CH º C-COOH (Z)
(c) Perlone (d) Nylon-6 The correct order of increasing acid strength is
23. Which of the following is a high-spin (spin-free) (a) X < Y < Z (b) X < Z < Y
complex? (c) Y < X < Z (d) Z < Y < X
(a) [Co(NH3)6]3+ (b) [Fe(CN)6]4–
(c) [CoF6] 3– (d) [Zn(NH3)6]2+
Mock Test-9 MT-101

27. Which series of reactions correctly represents


NC OH
chemical reactions related to iron and its
compound? (b) and
(a) dil. H 2SO 4 H 2SO 4 , O 2
Fe ¾¾¾¾¾
® FeSO 4 ¾¾¾¾¾®

Fe 2 ( SO4 )3 ¾¾¾
heat NH2 OH
® Fe

(b) O , heat dil. H SO (c) and


Fe ¾¾¾¾
2 ® FeO ¾¾¾¾¾
2 4®

heat
FeSO 4 ¾¾¾
® Fe
OH NH2
(c) Cl2 , heat heat , air
Fe ¾¾¾¾® FeCl3 ¾¾¾¾®
(d) and
Zn
FeCl 2 ¾¾® Fe
O , heat CO , 600°C 30. What is the standard reduction potential (E°) for
(d) Fe ¾¾¾¾
2 ® Fe3O 4 ¾¾¾¾¾
® Fe3+ ® Fe ?
CO , 700°C Given that
FeO ¾¾¾¾¾
® Fe
28. The molar specific heat of air at room temperature Fe2+ + 2e– ® Fe; E°Fe2 + / Fe = – 0.47 V
and 1atm pressure is 25 J K–1mol–1. How much Fe3+ +e– ® Fe2+; E°Fe3+ / Fe2 + = + 0.77 V
heat is required to heat the room through 10
Kelvin at room temperature if 144g of air is (a) – 0.057 V (b) + 0.057 V
(c) + 0.30 V (d) – 0.30 V
present in room. Vapour density of air = 14.4.
(a) 1250 kJ (b) 25 J
(c) 50 J (d) 1250 J PHYSICS
29. A mixture of two aromatic compounds A and B is
separated by dissolving in chloroform followed 31. Radioactive material 'A' has decay constant '8 l'
by extraction with aq. KOH solution. The alkaline and material 'B' has decay constant 'l'. Initially
aqueous layer gives a mixture of two isomeric they have same number of nuclei. After what
compounds on treatment with carbon time, the ratio of number of nuclei of material 'B'
tetrachloride. The organic layer containing 1
to that 'A' will be ?
compound A gives an unpleasant odour on e
treatment with alcoholic solution of KOH.
1 1
Compounds A and B respectively are (a) (b)
7l 8l
CN OH
1 1
(c) (d)
(a) 9l l
and
EBD_7206
MT-102 JEE MAIN

32. Two batteries, two resistors and two condensers 35. A satellite is revolving round the earth with
are connected as shown. The charge on 2mF velocity v. The minimum percentage increase in
capacitor is
its velocity necessary for the escape of satellite
3m F will be
(a) 100% (b) 50%
E1 = 18V 15V=E2 (c) 82.3% (d) 41.4%
5W 4W
r1 = 1W 2W = r2 36. A 20 cm long capillary tube is dipped in water.
The water rises up to 8 cm. If the entire
2m F arrangement is put in a freely falling elevator the
(a) 30 mC (b) 20 mC length of water column in the capillary tube will be
(c) 25 mC (d) 48 mC (a) 10 cm (b) 8 cm
33. A ball projected from ground at an angle of 45° (c) 20 cm (d) 4 cm
just clears a wall in front. If point of projection is 37. A Carnot freezer takes heat from water at 0°C inside
4 m from the foot of wall and ball strikes the it and rejects it to the room at a temperature of
ground at a distance of 6 m on the other side of 27°C. The latent heat of ice is 336 × 103 J kg–1. If
the wall, the height of the wall is : 5 kg of water at 0°C is converted into ice at 0°C
by the freezer, then the energy consumed by the
(a) 4.4 m (b) 2.4 m freezer is close to :
(c) 3.6 m (d) 1.6 m (a) 1.51 × 105 J (b) 1.68 × 106 J
(c) 1.71 × 10 J 7 (d) 1.67 × 105 J
34. Two fixed frictionless inclined planes making an
angle 30° and 60° with the vertical are shown in 38. A particle of charge 16 × 10–16 C moving with
the figure. Two blocks A and B are placed on the velocity 10 ms–1 along x-axis enters a region
ur
two planes. What is the relative vertical where magnetic field of induction B is along
acceleration of A with respect to B ? the y-axis and an electric field of magnitude 104
Vm–1 is along the negative z-axis. If the charged
A particle continues moving along x-axis, the
B ur
magnitude of B is :
(a) 16 × 103 Wb m–2 (b) 2 × 103 Wb m–2
(c) 1 × 103 Wb m–2 (d) 4 × 103 Wb m–2
39. A slender uniform rod of mass M and length l
60° 30° is pivoted at one end so that it can rotate in a
vertical plane (see figure). There is negligible
(a) 4.9 ms–2 (in horizontal direction) friction at the pivot. The free end is held vertically
(b) 9.8 ms–2 (in vertical direction) above the pivot and then released. The angular
(c) Zero acceleration of the rod when it makes an angle
(d) 4.9 ms–2 (in vertical direction)
q with the vertical is
Mock Test-9 MT-103

43. When Uranium is bombarded with neutrons, it


undergoes fission. The fission reaction can be
written as :
235
92 U + 0 n1 ® 56 Ba141 + 36 Kr 92 + 3 x + Q(energy)
where three particles named x are produced and energy
3g 2g Q is released. What is the name of the particle x ?
(a) cos q (b) cos q
2l 3l (a) electron (b) a-particle
3g 2g
(c) sin q (d) sin q (c) neutron (d) neutrino
2l 2l 44. Given that K = energy, V = velocity, T = time. If
40. How many degree of freedom the gas molecules they are chosen as the fundamental units, then
have if under STP the gas density r=1.3 kg/m3 and what is dimensional formula for surface tension?
the velocity of sound propagation in it is 330 m/sec
(a) [KV–2T –2 ] (b) [K2 V2T–2 ]
(a) 3 (b) 5 2 –2
(c) [K V T ] –2 (d) [KV2T2]
(c) 7 (d) 8 45. Three charges of (+ 2q), (– q) and (– q) are placed
at the corners A, B and C of the equilateral triangle
41. The rectangular surface of area 8 cm × 4 cm of a
of side a as shown in the given figure. Then the
black body at temperature 127°C emits energy E per dipole moment of this combination is
second. If the length and breadth are reduced to
half of the initial value and the temperature is raised A + 2q
to 327°C, the rate of emission of energy becomes
3 81 a a
(a) E (b) E
8 16

9 81 D
(c) E (d) E –q B a C –q
16 64
(a) qa (b) zero
42. What should be the maximum acceptance angle
at the air core interface of an optical fibre if n1 2
and n2 are the refractive indices of the core and (c) qa 3 (d) qa
3
the cladding, respectively
46. The work function of a metallic surface is 5.01
(a) -1
sin (n 2 / n1 ) (b) sin -1
n12 - n 22 eV, photoelectrons are emitted when light of
wavelength 2000Å falls on it. The minimum
potential difference required to stop the fastest
é -1 n 2 ù é -1 n1 ù photoelectrons (h = 4.14 × 10–15 eV–s)
(c) ê tan ú (d) ê tan n ú
ë n1 û ë 2û
EBD_7206
MT-104 JEE MAIN

(a) 1.2 volts (b) 2.4 volt 51. The transverse displacement y (x, t) of a wave is
(c) 3.6 volt (d) 4.8 volt
47. The potential energy of a 1 kg particle free to given by y( x, t ) = e
(
- ax 2 + bt 2 + 2 ab ) xt )
move along the x-axis is given by
This represents a:
æ x 4 x2 ö b
V ( x) = ç - ÷ J. (a) wave moving in – x direction with speed
a
è 4 2ø
(b) standing wave of frequency b
The total mechanical energy of the particle is 2 J. 1
Then, the maximum speed (in m/s) is (c) standing wave of frequency
b
3 1 a
(d) wave moving in + x direction speed
(a) (b) 2 (c) (d) 2 b
2 2
52. A system of four gates is set up as shown. The
48. In an A.C. circuit with voltage V and current I ‘truth table’ corresponding to this system is :
the power dissipated
(a) depends on the phase between V and I A

(b) 1 (c) 1 (d) VI


VI VI
2 2

49. Torques t1 and t 2 are required for a magnetic Y

needle to remain perpendicular to the magnetic


fields at two different places. The magnetic fields
at the those places are B1 and B2 respectively;
B
B
then ratio 1 is
B2 (a) A B Y (b) A B Y
0 0 1 0 0 0
t2 t1 0 1 0 0 1 0
(a) (b)
t1 t2 1 0 0 1 0 1
1 1 1 1 1 0
t1 + t 2 t1 - t 2
(c) (d)
t1 - t 2 t1 + t 2
50. A stuntman plans to run across a roof top and (c) A B Y (d) A B Y
0 0 1 0 0 1
horizontally jumps on to another roof 4.9 m below 0 1 1
0 1 0
the first one and at a distance of 6.2 m away. 1 0 1 1 0 0
What is the minimum velocity, he must have 1 1 0 1 1 0
before the jump ?
(a) 3.1 m/s (b) 4.0 m/s
(c) 4.9 m/s (d) 6.2 m/s
Mock Test-9 MT-105

53. A particle of mass m is under a potential (a) 1/4 (b) 1/2


v(x) = (1/2)k1x2 for x > 0 and v(x) = k1x2 for x < 0. (c) 2/3 (d) 1/3
When disturbed a little from the position x = 0, it will 57. A hydrogen atom emits green light when it
(a) not execute SHM changes from n = 4 energy level to the n = 2
level. Which colour of light would the atom emit
m
(b) execute SHM with T = 2p when it changes from n = 5 level to the n=2 level?
3k1
(a) red (b) yellow
(c) execute SHM with T2 = 2p2m/(k12) (c) green (d) violet
58. The earth’s magnetic field lines resemble
p m that of a dipole at the centre of the earth. If the
(d) execute SHM with T =
æ 1 ö magnetic moment of this dipole is close to 8 × 1022
k1 çç1 + ÷÷
è 2ø Am2, the value of earth’s magnetic field near the
equator is close to (radius of the earth = 6.4 × 106 m)
54. A thin sheet of glass (m = 1.5) of thickness 6 (a) 0.6 Gauss (b) 1.2 Gauss
microns introduced in the path of one of
(c) 1.8 Gauss (d) 0.32 Gauss
interfering beams in a double slit experiment
59. If a tiny test charge, is placed in a cavity, a
shifts the central fringe to a position previously
region in between the molecules of a dielectric,
occupied by fifth bright fringe. Then the wave
will experience a force equal to its charge times
length of the light used is
(a) 6000 Å (b) 3000 Å (a) Electric Field E
(c) 4500 Å (d) 7500 Å (b) Electric displacement D
55. The radiation corresponding to 3 ® 2 transition of
(c) E + (4K/3) P
hydrogen atom falls on a metal surface to produce
photoelectrons. These electrons are made to enter a (d) A combination of E and P depending on
the shape of the cavity
magnetic field of 3 × 10–4 T. If the radius of the
60. A coil having n turns and resistance RW is
largest circular path followed by these electrons is connected with a galvanometer of resistance
10.0 mm, the work function of the metal is close to: 4RW. This combination is moved in time t
(a) 1.8 eV (b) 1.1 eV seconds from a magnetic field W1 weber to W2
weber. The induced current in the circuit is
(c) 0.8 eV (d) 1.6 eV
56. Inside a cylinder closed at both ends is a movable (W2 - W1 ) n (W 2 - W1 )
piston. On one side of the piston is a mass m of (a) - (b) - 5 Rt
Rnt
a gas, and on the other side a mass 2m of the
same gas. What fraction of volume of the (W2 - W1 ) n(W2 - W1 )
(c) - (d) -
cylinder will be occupied by the larger mass of 5 Rnt Rt
the gas when the piston is in equilibrium? The
temperature is the same throughout.
EBD_7206
MT-106 JEE MAIN

p+q
MATHEMATICS 66. If cosecq = ( p ¹ q ¹ 0) , then
p-q
r r æp qö
cot ç + ÷ is equal to:
61. If a = ˆi – 2ˆj + 3kˆ and b = –3iˆ + ˆj - kˆ and
è 4 2ø
r r r r r r r r
r ´ a = b ´ a, r ´ b = a ´ b p q
r (a) (b)
then a unit vector in the direction of r is q p
1 1
(a) (–2 î + ˆj - k̂ ) (b) (–2 î - ĵ + 2 k̂ ) (c) pq (d) pq
3 3
67. The differential equation of displacement of all
1
(c) (–2 î - ˆj - 2 k̂ ) (d) None of these "Simple harmonic motions" of given period 2p/n, is
3
62. The acute angle between two lines such that the d2 x d2 x
(a) + nx = 0 (b) + n2 x = 0
direction cosines l, m, n, of each of them satisfy the dt 2 dt 2
equations l + m + n = 0 and l2 + m2 – n2 = 0 is :
(a) 15° (b) 30° (c) 60° (d) 45° d2 x d2 x 1
(c) - n2 x = 0 (d) + x =0
63. In a two player game that always has a winner, dt 2
dt 2
n2
A beats B with probability 2/3; B beats C with
(log x + ax 2 )
probability 2/3 & C beats A with the same 68. òe cos(bx 2 + c) dx is equal to
probability. If B plays with C and then the
1 2 b
winner plays with A, the chance that A will be (a) e ax cos( bx 2 + c + tan -1 ) + A
the final winner is a 2 + b2 a
(a) 5/9 (b) 4/9 (c) 3/9 (d) None 1 2 b
(b) e ax cos( bx - c - tan -1 ) + A
64. If P = { x Î R :f (x) = 0 } and 2
2 a +b 2 a

Q ={ x Î R :g (x) = 0 } then P È Q is 1 2 b
(c) e ax cos( bx + c - tan -1 ) + A
a
(a) { x Î R : f (x) + g(x) = 0 } 2
a +b 2

{ x Î R : f (x) g(x) = 0 } 1 b
2
(b) (d) e ax cos( bx + c - tan -1 ) + A
2 a +b 2 2 a
(c) { x Î R : (f (x)) 2
+ ( g(x) ) = 0
2
} 69. Find the area lying between the curves y = tan x,
(d) None of these é pù
y = cot x and x-axis, x Î ê0, ú
r r r r r r r r 2û ë
65. If a = i + j+k , b = 4i + 3 j + 4k and
r r r r 1
log 2
c= i + a j + b k are linearly independent vectors (a) log 2 (b)
2
and | rc | = 3 , then-
æ 1 ö
(a) a = 1, b = – 1 (b) a = 1, b = ± 1 (c) 2 log ç (d) None of these
(c) a = – 1, b = ± 1 (d) a = ± 1, b = 1 è 2 ÷ø
Mock Test-9 MT-107

75. The centre of the ellipse


70. If ò 1 + sec x dx = 2(fog )( x ) + C, then
( x + y - 2) 2 ( x - y) 2
-1
+ = 1 is :
(a) f ( x ) = sec x - 1 (b) f ( x) = 2 tan x 16 9
(a) (0, 0) (b) (1, 1)
(c) g( x ) = sec x - 1 (d) None of these (c) (1, 0) (d) (0, 1)
71. If y2 = P (x) is a polynomial of degree 3, then 76. The point (2a, a) lies inside the region bounded
by the parabola x2 = 4y and its latus rectum. Then,
d é 3 d2y ù (a) 0 £ a £ 1 (b) 0 < a < 1
2 êy ú
dx ëê dx 2 ûú is equal to (c) a > 1 (d) a < 0
77. If DABC is right angled at A then the inradius of
(a) P(x) + P¢ (x) (b) P(x) + P¢¢¢ (x) the triangle is
(c) P(x) P¢¢¢ (x) (d) None of these (a) 2(a + b – 2) (b) 2(b + c – a)
72. Which of the following is an even function?
b+c-a b+c-a
(c) (d)
æ 1- x ö 2 4
(a) f ( x ) = log ç ÷
è1+ x ø
1 1 1
78. If , , are in A.P..
(b) f ( x ) = log ( x + 1 + x 2 ) b+ c c+ a a+ b

x x then 9 ax +1 ,9 bx +1 ,9cx +1 , x ¹ 0 are in :


(c) f ( x) = + +1
x
e -1 2 (a) G.P. (b) G.P. only if x < 0
(c) G.P. only if x > 0 (d) None of these
2x
(d) f ( x ) = e + sin x
p
73. A plane meets the coordinate axes in points A, sin (2k - 1) x
B, C and the centroid of the triangle ABC is 79. If a k = ò sin x
dx , then –
0
(a, b, g ) . The equation of the plane is (a) a1, a2, ....... are in A.P.
x y z (b) a1, a2, ....... are in G..P.
(a) + + =3 (b) ax+ by+ gz = 3abg
a b g (c) a1, a2, ....... are in H.P.
(d) a1, a2, ....... form a constant sequence
x y z 1
(c) + + = (d) None of these 80. If the roots of a(b – c) x2 + b(c – a) x + c(a – b) = 0
a b g 2
are equal, then a, b, c are in :
74. The eccentricity of the hyperbola (a) A.P. (b) G.P.
x 2 - 3y 2 = 2 x + 8 is (c) H.P. (d) None of these
81. If z1, z2, z3, z4 be the vertices of a quadrilateral
2 1
(a) (b) taken in order such that z1 + z3 = z2 + z4 and
3 3
2 æ z -z ö
3 |z1 – z3 | = | z2 – z4 | then arg ç 1 2 ÷ =
(c) (d) è z3 - z2 ø
3 2
EBD_7206
MT-108 JEE MAIN

p p 87. The function defined by f (x)


(b) ±
( )
(a)
2 2 ì 1 -1
ï 2
p p = í x + e 2-x , x ¹ 2, is
(c) (d) ï k x=2
3 6 î
82. Let f (x) = ex, g(x) = sin–1x and h (x) = f (g(x)), continuous from right at the point x =2, then k is
then h'(x)/h(x) = equal to
-1 (a) 0 (b) 1/4
(a) esin x (b) 1/ 1 – x 2
(c) sin–1 x (d) 1/(1 – x2) (c) –1/4 (d) None of these
83. Which of the following functions is differentiable é1 1 2 4 1 ù
88. Lim ê sec 2 2 + 2 sec 2 2 ............ + sec2 1ú
at x = 0? n ® ¥ ë n2 n n n n û
(a) cos(|x|) + |x| (b) cos (|x|) - |x| equals
(c) sin (|x|) + |x| (d) sin (|x|) - |x|
1 1
(a) sec 1 (b)
cosec 1
84. The largest interval lying in æç -p , p ö÷ for which 2 2
è 2 2ø 1
the function, (c) tan 1 (d) tan 1
2
2 æx ö 89. Consider a rectangle whose length is increasing
f ( x) = 4- x + cos -1 ç - 1÷ + log (cos x) i s at the uniform rate of 2 m/sec, breadth is
è2 ø
decreasing at the uniform rate of 3 m/sec and the
defined, is area is decreasing at the uniform rate of 5 m2/
(a) é - p , p ö é pö
(b) ê 0, ÷ sec. If after some time the breadth of the rectangle
ê 4 2÷ ë 2ø is 2 m then the length of the rectangle is
ë ø
(a) 2 m (b) 4 m
p p
(c) [0, p] (d) æç - , ö÷ (c) 1 m (d) 3 m
è 2 2ø 90. The differential equation of the family of curves
85. Let E = (1, 2, 3, 4) and F = (1, 2). Then the number 2 3
of onto functions from E to F is represented by c( y + c) = x is
(a) 14 (b) 16 2
d2y æ dy ö
(c) 12 (d) 8 (a) y - y 2 ç ÷ = 27 x
86. Which of the following functions is NOT one- dx 2 è dx ø
one ? 2 3
æ dy ö æ dy ö
(a) f : R ® R defined by f ( x ) = 6 x - 1. (b) 12 y ç ÷ = 8x ç ÷ - 27 x
è dx ø è dx ø
(b) f : R ® R defined by f ( x ) = x 2 + 7. 3 2
(c) f : R ® R defined by f ( x ) = x 3 . æ dy ö æ dy ö
(c) 8 y ç ÷ = 12 x ç ÷ - 27 x
è dx ø è dx ø
(d) 2x + 1 .
f : R - {7} ® R defined by f ( x) = 3 2
x -7 æ dy ö æ dy ö æ dy ö
(d) ç ÷ - ç ÷ + ç ÷ - y = 27 x .
è dx ø è dx ø è dx ø
Download From https://iit-jeeacademy.blogspot.com

10JEE MAIN
MOCK TEST
Time : 3 hrs. Max. Marks : 360

INSTRUCTIONS
• Chemistry (120 marks) : Question No. 1 to 30 are of 4 marks each.

• Physics (120 marks) : Question No. 31 to 60 are of 4 marks each.

• Mathematics (120 marks) : Question No. 61 to 90 are of 4 marks each.


• Negative Marking : One fourth (¼) marks will be deducted for indicating incorrect
response of each question.

5A 9A
CHEMISTRY (a)
9
(b)
5
36A 36A
1. Which one of the following is an example of co- (c) (d)
5 7
ordination isomerism ? 3. Oxidation of succinate ion produces ethylene
(a) [Co (NH3)5 Br] SO4 and [Co (NH3)5 SO4] Br and carbon dioxide gases. On passing 0.2
(b) [Co (NH3)5 NO2] Cl2 and [Co (NH3)5 Faraday electricity through an aqueous solution
ONO]Cl2 of potassium succinate, the total volume of
(c) [Cr (H2O)6] Cl3 and [Cr (H2O)5 Cl] Cl2 H2O gases (at both cathode and anode) at STP (1
(d) [Co (NH3)6] [Cr (CN)6] and (Cr (NH3)6] [Co atm and 273 K) is :
(CN)6] (a) 8.96 L (b) 4.48 L
2. If the shortest wavelength in Lyman series of (c) 6.72 L (d) 2.24 L
hydrogen atom is A, then the longest
wavelength in Paschen series of He+ is :
EBD_7206
MT-110 JEE MAIN

4. A wedding ring presented to a bride contains 788 mg 9. The repeating units of acrilan are
of gold and rest diamond. If the ring weighs 1 g
the bride receives (At wt : Au = 197, C = 12) H
(a) more number of gold atoms |
(a) H 2 C = C - COOCH 3
(b) more number of carbon atoms
(c) equal number of gold and carbon atoms
(d) gold and carbon atoms in ratio of 4:1 H
|
approximately (b) H 2 C = C - COOC 2 H 5
5. The solution containing 4.0 % PVC in one litre
of dioxane was found to have osmotic pressure H
of 6.0 ´ 10–4 atm at 300 K. The molecular mass |
of polymer is (c) H 2 C = C - CN
(a) 6.4 ´ 102 (b) 3.0 ´ 102
CH3
(c) 1.6 ´ 105 (d) 5.6´104 |
6. Urea and hydrazine react to form ammonia gas (d) H 2 C = C - COOCH3
along with compound X which reacts with
10. A metal X on heating in nitrogen gas gives Y.Y
aldehydes and ketones to form specific
on treatment with H2O gives a colourless gas
crystalline derivatives. X is
which when passed through CuSO4 solution
(a) phenylhydrazine (b) semicarbazide gives a blue colour. Y is
(c) biuret (d) acetylurea (a) Mg(NO3)2 (b) Mg3 N2
(c) NH3 (d) MgO
7. The IUPAC name of CH 2 - CH - C H 2 is
| | | 11. Chlorobenzne reacts with trichloro acetaldehyde
CN CN CN
in the presence of H2SO4.
(a) 1, 2, 3-propanetrinitrile
O
(b) 1, 2, 3 – tricyanopropane H2SO4
Cl + H – C – CCl3
(c) 3-cyano-1, 5-dinitrilepentane 2
(d) 3-cyano – 1, 5-pentanedinitrile
8. The catenation tendency of C, Si and Ge is in The major product formed is:
the order Ge < Si < C. The bond energies (in
kJmol– l) of C – C, Si – Si and Ge –Ge bonds are Cl
respectively; (a) Cl C Cl
(a) 348, 297, 260 (b) 297, 348, 260
(c) 348, 260, 297 (d) 260, 297, 348 Cl
Mock Test-10 MT-111

Cl 16. A gaseous mixture of three gases A, B and C has


a pressure of 10 atm. The total number of moles
(b) Cl C Cl of all the gases is 10. If the partial pressures of A
CH2Cl and B are 3.0 and 1.0 atm respectively and if ‘C’
has mol. wt of 2.0 what is the weight of ‘C’ in g
present in the mixture.
(c) Cl CH Cl
(a) 8 (b) 12
CCl3
(c) 3 (d) 6

(d) Cl CH Cl 17. The main product of the following reaction is


conc.H SO
Cl C6 H5CH 2CH(OH)CH(CH3 )2 ¾¾¾¾¾
2 4¾
®?
12. Which of the following has a shape different
from others? (a) H 5C 6 H
C=C
(a) [Zn(NH3)4]2+ (b) Ni(CO)4 H CH(CH3)2
(c) [Cd(CN)4]2+ (d) [Cu(NH3)4]2+
13. Di-n-propyl ether and diallyl ether can be (b) C6H5CH2 CH3
distinguished by C=C
H CH3
(a) Acetic acid (b) Sodium Metal
(c) Cold dilute KMnO4 solution (c) H5C6CH2CH2
(d) PCl5 C = CH 2
H 3C
14. In which of the following cases, the stability of
two oxidation states is correctly represented (d) C6H5 CH(CH3)2
(a) Ti3+ > Ti4+ (b) Mn2+ > Mn3+ C=C
2+
(c) Fe > Fe 3+ (d) Cu+ > Cu2+ H H
15. A and B in the reaction, 18. Zirconium phosphate [Zr 3(PO4)4] dissociates
into three zirconium cations of charge + 4 and
Mg 1.HCHO
CH 3 CH 2 CH 2 Cl ¾¾¾® A ¾¾ ¾¾® B four phosphate anions of charge – 3. If molar
ether
2.H 3O + solubility of zirconium phosphate is denoted by
(a) CH3CH2CH2MgCl, CH3CH2CH2CH2OH S and its solubility product by Ksp then which of
CH3CH2 the following relationship between S and Ksp is
(b) CH3CHMgCH2Cl, CHOH
H3C correct?
(a) S = {Ksp/ (6912)1/7}
H3C
(b) S = {Ksp/ 144}1/7
(c) CH3CH2CH2MgCl, H3C – C – OH (c) S = {Ksp/ 6912}1/7
H3C (d) S = {Ksp/ 6912}7
(d) CH3CH2CH2MgCl, CH3CH2OCH2CH3
EBD_7206
MT-112 JEE MAIN

19. For a chemical reaction X ® Y, the rate of 23. The intermediate formad during the addition of
reaction increases by a factor of 1.837 when the HCl to propene in the presence of peroxide is
concentration of X is increased by 1.5 times. The ·
order of the reaction with respect to X is (a) CH 3CHCH 2 Cl (b) CH3 CHCH3
(a) 1 (b) 1.5 · +
(c) 2 (d) 2.5 (c) CH 3CH 2CH 2 (d) CH3CH 2 CH 2
20. The final product of the following sequence of 24. Potassium dichromate when heated with
reactions concentrated sulphuric acid and a soluble
chloride, gives brown-red vapours of :
(a) CrO3 (b) CrCl3
NaOH , H O (c) CrO2Cl2 (d) Cr2O3
H ¾¾ ¾D¾ ¾
2¾®
25. In the following compounds,
LiCu (CH ) LiAlH
B ¾¾ ¾ ¾3¾
¾2®
C ¾¾ ¾¾

D OH OH OH OH
H 2O H 2O

(a) 3-ethyl-2-methyl-1-hexanol
NO2
(b) 2, 3-dimethyl-1-pentanol
CH3 NO2
(c) 2-ethyl-3-methyl-1-hexanol
(I) (II) (III) (IV)
(d) 3, 3-dimethyl-1-pentanol
21. A solution containing As3+, Cd2+, Ni2+ and Zn2+ the order of acidity is
is made alkaline with dilute NH4OH and treated (a) III > IV > I > II (b) I > IV > III > II
with H2S. The precipitate obtained will consist of
(c) II > I > III > IV (d) IV > III > I > II
(a) As2S3 and CdS
26. The electrolyte most effective for bringing about
(b) CdS, NiS and ZnS coagulation of Fe (OH)3 sol is
(c) NiS and ZnS (a) K4[Fe(CN)6] (b) BaCl2
(d) Sulphide of all ions (c) NaCl (d) None
22. M2SO4 (M+ is a monovalent metal ion) has a Ksp of
3.2 ´ 10–5 at 298 K. The maximum concentration 27. The reaction of CH3 CH = CH OH
of SO4–2 ion that could be attained in a saturated
solution of this solid at 298 K is with HBr gives
(a) 3 ´ 10–3 M (b) 7 ´ 10–2 M
(c) 2.89 ´ 10–4 M (d) 2´10–2 M (a) CH3CH = CH Br..
Mock Test-10 MT-113

efficient in converting electrical power to


(b) CH3CH2CHBr OH electromagnetic waves and consumes 100 W of
power. The amplitude of the electric field
associated with the electromagnetic radiation at
a distance of 5 m from the lamp will be nearly:
(c) CH3CHBrCH2 Br (a) 1.34 V/m (b) 2.68 V/m
(c) 4.02 V/m (d) 5.36 V/m
32. A neutron moving with a speed 'v' makes a head
on collision with a stationary hydrogen atom in
(d) CH3CH2CHBr Br ground state. The minimum kinetic energy of the
neutron for which inelastic collision will take
28. The standard reduction potential for Cu2+/Cu is place is :
+ 0.34. Calculate the reduction potential at (a) 20.4 eV (b) 10.2 eV
pH = 14 for the above couple. (Ksp Cu(OH)2 (c) 12.1 eV (d) 16.8 eV
= 1 × 10–19) 33. The time taken by a body in sliding down a rough
(a) – 0.22 V (b) + 0.22 V inclined plane of angle of inclination 45° is n
(c) – 0.44 V (d) + 0.44 V times the time taken by the same body in slipping
29. Final product (D) formed in the following reaction down a similar frictionless plane. The coefficient
sequence is
of dynamic friction between the body and the
plane will be
D 2 NBS
CH2=CHCH=CH2+ ¾¾® A ¾¾¾
¾® B
(a) 1/(1–n2) (b) 1 – (1/n2)
(c) Ö{1– (1/n2)} (d) Ö{1/(1–n2)}
alc.KOH Pd ,( -H )
2 ®D
¾¾ ¾¾® C ¾¾ ¾¾ ¾ 34. A mass M, attached to a horizontal spring,
(a) Naphthalene (b) Tetralin executes S.H.M. with amplitude A1. When the
(c) Benzene (d) Anthracene mass M passes through its mean position then a
30. A current of 2.0A when passed for 5 hours smaller mass m is placed over it and both of them
through a molten metal salt deposits 22.2g of move together with amplitude A2. The ratio of
metal of atomic weight 177.The oxidation state
æ A1 ö
of the metal in the metal salt is
çè A ÷ø is:
(a) +1 (b) +2 2

(c) +3 (d) +4 1
M +m æ M ö2
(a) (b) çè ÷
M M + mø
PHYSICS 1
æ M + mö 2 M
31. A lamp emits monochromatic green light (c) (d)
çè ÷ M +m
uniformly in all directions. The lamp is 3% M ø
EBD_7206
MT-114 JEE MAIN

35. Three particles of equal mass m are situated at electron to be a circular current loop, the
the vertices of an equilateral triangle of side L. magnetic moment of the hydrogen atom, when
What should be the velocity of each particle so the electron is in nth excited state, is :
that they move on a circular path without æ e n2 h ö æ e ö nh
changing L? (a) ç ÷ (b) ç ÷
ç 2m 2p ÷ è m ø 2p
è ø
(a) Ö(GM/2L) (b) Ö(GM/L)
(c) Ö(2GM/L) (d) Ö(GM/3L) æ e ö nh 2
æ e ön h
(c) ç ÷ (d) ç ÷
36. A charge Q is distributed over two concentric è 2m ø 2p è m ø 2p
hollow spheres of radii r and R (>r) such that the
39. From a tower of height H, a particle is thrown
surface densities are equal and placed on the vertically upwards with a speed u. The time taken
same axial points. Then the potential at the by the particle, to hit the ground, is n times that
common centre is taken by it to reach the highest point of its path.
Q(R 2 + r 2 ) The relation between H, u and n is:
Q
(a) (b) (a) 2gH = n2u2 (b) gH = (n – 2)2 u2d
4pe 0 (R + r ) R+r (c) 2gH = nu (n – 2) (d) gH = (n – 2)u2
2

Q(R + r) 40. At a specific temperature, the energy densities


(c) Zero (d) for a black body for three different wavelengths
4 pe 0 (R 2 + r 2 )
are 10, 19 and 7 units. For these wavelengths if
37. A wheel is rolling straight on ground without the absorption coefficient of a body is
slipping. If the axis of the wheel has speed v, the respectively 0.8, 0.3 and 0.9 then the emissive
instantenous velocity of a point P on the rim, powers of this body for these wavelengths are
defined by angle q, relative to the ground will be in the ratio
(a) 10:19:7 (b) 8:3:9
P
(c) 8:5.7:6.3 (d) 8:9.5:4.3
q 41. A uniformly tapering conical wire is made from a
material of Young's modulus Y and has a normal,
unextended length L. The radii, at the upper and
lower ends of this conical wire, have values R
and 3R, respectively. The upper end of the wire
æ1 ö æ1 ö is fixed to a rigid support and a mass M is
(a) v cosç q ÷ (b) 2 v cosç q ÷
è2 ø è2 ø suspended from its lower end. The equilibrium
extended length, of this wire, would equal :
(c) v(1 + sin q) (d) v(1 + cos q)
æ 2 Mg ö æ 1 Mg ö
38. In the Bohr model an electron moves in a circular (a) L çè1 + (b) L çè1 + 9
orbit around the proton. Considering the orbiting 9 pYR 2 ÷ø ÷
pYR 2 ø
Mock Test-10 MT-115

æ 1 Mg ö æ 2 Mg ö
(c) L çè 1 + 3 ÷ (d) L ç 1 +
pYR 2 ø è 3 pYR 2 ÷ø (c)
42. A Carnot’s engine works as a refrigerator
between 250 K and 300 K. If it receives 750
calories of heat from the reservoir at the lower
temperature, the amount of heat rejected at the
higher temperature is (d)
(a) 900 cal. (b) 625 cal.
(c) 750 cal. (d) 1000 cal.
43. A gas molecule of mass M at the surface of the 45. The electric field in a region of space is given by,
Earth has kinetic energy equivalent to 0°C. If it r
E = Eoiˆ + 2Eoˆj where Eo = 100 N/C. The flux of
were to go up straight without colliding with
any other molecules, how high it would rise? the field through a circular surface of radius 0.02
Assume that the height attained is much less m parallel to the Y-Z plane is nearly:
than radius of the earth. (kB is Boltzmann (a) 0.125 Nm2/C (b) 0.02 Nm2/C
constant). (c) 0.005 Nm /C2 (d) 3.14 Nm2/C
273k B 46. A cell of e.m.f. E is connected across a reistance
(a) 0 (b) R. The potential difference between the terminals
2Mg
546k B 819k B of the cell is found to be V. The internal
(c) (d) resistance of the cell must be
3Mg 2Mg
44. An electron beam is accelerated by a potential (a) [2(E–V)V]/R (b) [2(E–V)/V]R
difference V to hit a metallic target to produce X- (c) [(E–V)/V]R (d) (E–V)R
rays. It produces continuous as well as 47. A cylindrical vessel of cross-section A contains
characteristic X-rays.If lmin is the smallest water to a height h. There is a hole in the bottom
possible wavelength of X-ray in the spectrum,
of radius ‘a’. The time in which it will be emptied
the variation of log lmin with log V is correctly
is:
represented in :
2A h 2A h
(a) 2 (b)
(a) pa g pa 2 g

2 2A h A h
(b) (c) (d)
pa 2 g 2pa 2 g
EBD_7206
MT-116 JEE MAIN

48. Two straight long conductors AOB and COD 52. 56 tuning forks are so arranged in series that
are perpendicular to each other and carry each fork gives 4 beats per sec with the previous
currents I1 and I2 respectively. The magnitude one. The frequency of the last fork is 3 times
of the magnetic induction at a point P at a that of the first. The frequency of the first fork is
distance ‘a’ from the point O in a direction (a) 110 (b) 56
perpendicular to the plane ABCD is (c) 60 (d) 52
53. In a npn transistor 1010 electrons enter the
m0 m0
(a) (I1 + I 2 ) (b) (I1 – I 2 ) emitter in 10–6 s. 4% of the electrons are lost in
2 pa 2 pa the base. The current transfer ratio will be
m 0 æ I1I 2 ö (a) 0.98 (b) 0.97
m 0 é 2 2 ù 1/ 2
(c) I1 + I 2 (d) 2pa çç I + I ÷÷ (c) 0.96 (d) 0.94
2pa ë û è 1 2ø 54. Two slits separated by a distance of 1 mm are
49. The mutual inductance of a pair of coils, each of illuminated with red light of wavelength 6.5 ×
N turns, is M henry. If a current of I ampere in 10–7 m. The interference fringes are observed on
one of the coils is brought to zero in t second, a screen placed 1 m from the slits. The distance
the emf induced per turn in the other coil, in between third dark fringe & the fifth bright fringe
volt, will be is equal to
(a) 0.65 mm (b) 1.63 mm
MI NMI
(a) (b) (c) 3.25 mm (d) 4.88 mm
t t 55. Hysteresis loops for two magnetic materials A
MN and B are given below :
MI
(c) (d)
It Nt B B
50. In a series resonant circuit, having L, C and R as
its elements, the resonant current is i. The power
dissipated in the circuit at resonance is
H H
i 2R
(a) æ 1 ö (b) zero
ç wL - ÷ (A) (B)
è wC ø
(c) i2wL (d) i2R These materials are used to make magnets for
51. The co-ordinates of a moving particle at any time elecric generators, transformer core and
electromagnet core. Then it is proper to use :
‘t’are given by x = a t 3 and y = b t 3 . The speed
(a) A for transformers and B for electric
of the particle at time ‘t’ is given by generators.
(b) B for electromagnets and transformers.
(a) 3t a 2 + b 2 (b) 3t 2 a 2 + b2
(c) A for electric generators and trasformers.
(d) A for electromagnets and B for electric
(c) t 2 a 2 + b2 (d) a 2 + b2 generators.
Mock Test-10 MT-117

56. A lens is placed between a source of light and a 60. In an experiment the angles are required to be
wall. It forms images of area A1 and A2 on the measured using an instrument, 29 divisions of
wall, for its two different positions. The area of the main scale exactly coincide with the 30
the source of light is
divisions of the vernier scale. If the smallest
(a) (A1A 2 ) (b) (A1+A2)/2 division of the main scale is half- a degree
(= 0.5°), then the least count of the instrument is:
(a) half minute (b) one degree
(c)
( A1 + A 2 )2 é 1
(d) ê +
1 ù
ú
–1
(c) half degree (d) one minute
2 ë A1 A 2 û
57. For sky wave propagation of a 10 MHz signal,
what should be the minimum electron density in MATHEMATICS
ionosphere
61. n+4C – nC – 3.nC n n
(a) ~ 1.2 ´1012 m -3 (b) ~ 106 m -3 r r r–1 – 3. Cr–2 – Cr–3 is equal to
(a) n+1 Cr–1 (b) n+2 Cr–1
(c) ~ 1014 m-3 (d) ~ 10 22 m -3 (c) n+3Cr–1 (d) n+4Cr–1
58. The counting rate observed from a radioactive 62. The sum of the first three consecutive terms of
source at t = 0 second was 1600 counts per an A.P. is 9 and the sum of their squares is 35 .
second and at t = 8 seconds it was 100 counts The sum to n terms of the series is
per second. The counting rate observed, as
(a) 3n2 (b) 2n2
counts per second at t = 6 seconds will be
(a) 400 (b) 300 (c) 6n – 2n2 (d) 6n – n2
(c) 200 (d) 150 63. Let a/(a–1) and b/(b–1) be the roots of x2 + ax +
59. If a battery is connected across series b = 0. Then 1/a and 1/b are the roots of
combination of a capacitor and a resistor, at t = (a) bx2 + ax + 1 = 0
0. If at an instant t potential difference across (b) bx2 – ax + 1 = 0
the capacitor be ‘V’ and energy stored in it be U. (c) bx2 + (a + 2b)x + a + b + 1 = 0
Then which of the following graph is correct.
(d) bx2 – (a + 2b) x + a + b + 1 = 0
U U
64. For any real q, the maximum value of
(a) (b) cos2(cos q) + sin2(sin q) is
O V O V
(a) 1 (b) 1+sin21
(c) 1+cos21 (d) 1/2
U U
65. If tan(A/2), tan(B/2), tan(C/2) are in A.P. then sec
A, sec B, sec C are in
(c) (d) (a) A.P. (b) G..P.
O V O V (c) H.P. (d) None
EBD_7206
MT-118 JEE MAIN

66. The equation of the common tangent to the (a) 2007 (b) 2008
parabolas
(c) 20082 (d) 20072
y2 = 4ax and x2 = 4by is given by
(a) xa1/3 + yb1/3 + a2/3 b2/3 = 0 æ 1 -1 1 ö
(b) xb1/3 + ya1/3 + a2/3 b2/3 = 0 ç ÷
71. Let A = ç 2 1 - 3 ÷. and
(c) xa1/3 + yb1/3 – a2/3 b2/3 = 0 ç1 1 1 ÷ø
è
(d) None of these
67. If tangents be drawn to the cricle x2 + y2 = 12 at
æ 4 2 2ö
its points of intersection with the circle x2 + y2 – ç ÷
(10) B = ç - 5 0 a ÷.
5x + 3y – 2 = 0, then the tangents intersect at the ç 1 -2 3÷
point è ø
If B is the inverse of matrix A, then a is
æ 18 ö æ 18 ö
(a) ç – 6, ÷ (b) ç 6, ÷ (a) 5 (b) –1
è 5ø è 5ø (c) 2 (d) –2
72. The value of
æ 18 ö æ 18 ö
(c) ç – 6,– ÷ (d) ç 6,– ÷ 1
è 5ø è 5ø
lim (4 - 3 sin x - 2 cos 2 x ) 2 sin x -1 is
68. The locus of the mid points of the chords of the x ®p / 6
ellipse x2/a2 + y2/b2 = k, k > 0, making equal (a) 1 (b) e
intercepts on the coordinate axes, is (c) Öe (d) e–1/2
(a) x = y (b) x + y = 0
73. Find the number of critical points of f (x) =
(c) x/a2 = y /b2 (d) x/a2 + y/b2 = 0
69. Let R = {(x, y) : x, y Î N and x2 – 4xy + 3y2 = 0}, | x - 1|
.
where N is the set of all natural numbers. Then x2
the relation R is : (a) 1 (b) 2
(a) reflexive but neither symmetric nor
(c) 3 (d) 4
transitive.
74. The derivative of tan –1[(3x2 – 1)/(3x – x3)] with
(b) symmetric and transitive.
respect to sin–1[(x2 –1)/( x2 + 1)] is
(c) reflexive and symmetric,
(a) 2/3 (b) –2/3
(d) reflexive and transitive.
(c) 3/2 (d) –3/23
æ r r – 1ö
75. If 2a + 3b + 6c = 0 then at least one root of the
70. Matrix M r is defined as M r = çç ÷÷ , r
è r –1 r ø
equation ax 2 + bx + c = 0 lies in the interval
Î N value of det( M1 ) + det( M 2 ) + det( M 3 ) + (a) (0, 1) (b) (1, 3)
.... + det(M2007) is (c) (2, 3) (d) (1, 2)
Mock Test-10 MT-119

76. If Z1 ¹ 0 and Z2 be two complex numbers such (c) 2 2 + 2


(d) None of these
Z2 80. Let coordinates of a point ‘p’ with respect to the
that is a purely imaginary number, then
Z1 r r r
system of non-coplanar vectors a , b and c is
(3, 2, 1). Then coordinates of ‘p’ with respect to
2Z1 + 3Z 2 r r r r r r
the system of vectors a + b + c , a – b + c
2Z1 - 3Z 2 is equal to : r r
r
and a + b – c is
(a) 2 (b) 5
(c) 3 (d) 1 æ3 1 ö æ3 1ö
(a) ç , ,1÷ (b) ç 2 ,1, 2 ÷
p
è2 2 ø è ø
77. Value of ò 0 sin
n
x cos 2m+1 x dx
æ1 3 ö
(c) ç , ,1÷ (d) None
( where m, n Î N) is è2 2 ø
81. The three lines through the origin with direction
(2m + 1)! (2m + 1)! cosines l1, m1, n1; l2, m2, n2; and l3, m3, n3 are
(a) (b)
n! n2 coplanar, if
(a) l1 (l2 – l3) + m1 (m2 – m3) + n1 (n2 – n 3) = 0
p 2m +1
(c) ò 0 cos x dx (d) None of these (b) l1(m2n3 – m3n2) + m1(n2l3 – n3l2) + n1 (l2m3 –
l3m2) = 0
78. If f ¢( x ) = f ( x) and f (0) = 2, then
l1 m1 n1
f ( x) l2 m2 n2 = 0
ò 3 + 4f ( x )
dx = (c)
l3 m3 n3
(a) log(3 + 8e x ) + C (d) None of these
82. A plane makes intercepts OA, OB, OC whose
1 measures are a, b, c on the axes x, y, z respectively.
(b) log(3 + 8e x ) + C
4 The area of the triangle ABC is

1 1 2
(c) log(3 + 8e x ) + C (a) a + b2 + c2
2 2
(d) None of these 1
(b) b 2c 2 + c 2 a 2 + a 2 b 2
79. The area under the curve y = | cos x – sin x |, 2
p 1
0£ x£ , and above x-axis is : ab + bc + ac
2 (c)
2
(a) 2 2
1 2
(b) 2 2 -2 (d) a b + b 2 c + c 2a
2
EBD_7206
MT-120 JEE MAIN

83. If x + 2 = 6, then x = 4. So, which statement is its 87. A and B are two independent events. The prob-
converse? ability that both A and B occur is 1/6 and the
(a) If x ¹ 4, then x + 2 ¹ 6 probability that neither of them occurs is 1/3.
The probability of occurrence of A is :
(b) If x = 4, then x + 2 ¹ 6
5 1
(c) If x = 4, then x + 2 = 6 (a) (b)
6 6
(d) If, x ¹ 4 then x + 2 = 6
1
84. If y = f(x) be a monotonically increasing or (c) (d) None of these
decreasing function of x and M is the median of 5
variable x, then the median of y is
88. If X and Y are independent binomial variates
(a) f (M)
(b) M/2 æ 1ö æ 1ö
(c) f –1 (M) B çè 5, ÷ø and B çè 7, ÷ø , then P (X + Y = 3) is
2 2
(d) None of these
85. A fair die is tossed eight times. The probability 35 55
(a) (b)
that a third six is observed on the eighth throw is 47 1024

7 55 220 11
(a) C2 (c) (d)
8 512 204
6
æ 1 ö -1 æ 1 ö
7 53 89. S = tan -1 ç ÷ + tan ç 2 ÷ + ...
(b) C3 è n 2
+ n + 1 ø è n + 3 n + 3 ø
68
æ 1 ö
+ tan -1 ç ÷ , then tan S is
7 56 è 1 + ( n + 19)( n + 20) ø
(c) C6
68 equal to :
(d) None of these 20 n
uur uur uur (a) (b)
86. If a , b , c are non coplanar vectors and l is a 401 + 20 n 2
n + 20n + 1
uur uur uur uur 20 n
real number then [l (a + b ) l 2 b l c ] = (c) (d)
2
n + 20n + 1 401 + 20 n
uur uur uur uur
[a b + c b ] for 90. The middle term in the expansion of
n
(a) exactly one value of l æ 1ö
çè1 - ÷ø (1 - x ) in powers of x is
n
(b) no value of l x
(c) exactly three values of l (a) – 2nCn–1 (b) – 2nCn
(c) 2n Cn – 1 (d) 2n Cn
(d) exactly two values of l
Hints & Solutions
Mock Test-1
ANSWER KEY
1 (d) 16 (d) 31 (d) 46 (b) 61 (d) 76 (b)
2 (a) 17 (c) 32 (a) 47 (c) 62 (a) 77 (d)
3 (a) 18 (c) 33 (d) 48 (c) 63 (c) 78 (a)
4 (b) 19 (a) 34 (b) 49 (b) 64 (d) 79 (d)
5 (b) 20 (b) 35 (d) 50 (b) 65 (b) 80 (a)
6 (c) 21 (d) 36 (d) 51 (d) 66 (b) 81 (b)
7 (a) 22 (d) 37 (a) 52 (b) 67 (d) 82 (a)
8 (c) 23 (b) 38 (c) 53 (c) 68 (c) 83 (a)
9 (a) 24 (c) 39 (d) 54 (a) 69 (a) 84 (d)
10 (c) 25 (a) 40 (c) 55 (a) 70 (c) 85 (a)
11 (a) 26 (a) 41 (a) 56 (c) 71 (a) 86 (c)
12 (d) 27 (a) 42 (d) 57 (a) 72 (d) 87 (b)
13 (d) 28 (b) 43 (a) 58 (b) 73 (c) 88 (d)
14 (c) 29 (b) 44 (a) 59 (a) 74 (c) 89 (d)
15 (d) 30 (a) 45 (a) 60 (b) 75 (b) 90 (a)

Solutions
CHEMISTRY 5. (b) Correct order is B < Be < O < N.

1. (d) Energy = NA hn Cl 120°


495.5 = 6.023 × 1023 × 6.6 × 10–34 × n 1
6 Cl
2
6. (c) 3 Dipole moments
495.5 ´ 10 3 Cl 5 4 Cl
n= = 12.4 ´ 1014
23 -34
6.023 ´ 10 ´ 6.6 ´ 10 of 2Cl and 5Cl are vectorically cancelled.
= 1.24 × 1015 s–1. 2
It is due 1 Cl and 3 Cl m
2. (a) The order of stability of resonating
structures carrying no charge > carrying 2 2
= m1 + m 2 + 2m1 m 2 cos q
minimum charge and each atom having
octet complete.
= (1.5) 2 + (1.5) 2 + 2 ´1.5 ´1.5 cos 120 \
3. (a) Bond order in N2 and O 22+ is 3 (calculated m = 1.5 D
by energy level diagram) 7. (a) According to Boyle’s law
4. (b) In A — O — H, if EN of ‘A’ is 2.1 then it will V1 P2 750 360
be neutral, as XA – X0 = X0 – XH. (where X = ; =
V2 P1 V2 840
is EN)
V2 = 1750 ml = 1.750 L
EBD_7206
MT-122 JEE MAIN

8. (c) 2HNO3(aq) + [Ag(NH3)2]+ + Cl– 1/ 58.5


C NaCl = ´ 1000 = 0.17
100
® AgCl ( s ) ¯
¾¾ +2NH 4 + 2NO3-
\ pNaCl = 2 × 0.17 = 0.34
When nitric acid is added to amine solution,
solution is made acidic and the complex ion n
1
BaCl2 =
dissociates and liberate silver ion to 208.4
recombine with chloride ion. This is the 1/ 208.4
conformatory test for silver in group 1. CBaCl2 = ´ 1000 = 0.048
100
9. (a) EHe+ = EH × 22; ELi2+ = EH × 32 ..... up to Z
or p BaCl2 = 3 ´ 0.048 = 0.14
[H 3 O + ][F – ] 1
10. (c) Ka = and n urea =
[HF][H 2 O] 60
1/ 60
[HF][OH – ] Curea = ´ 1000 = 0.16
Kb = . 100
[F – ][H 2 O] \ purea = 1 × 0.16 = 0.16
Therefore, Ka × Kb = [H3O+] [OH–] = Kw. \ pBaCl2 < purea < pKCl < pNaCl
11. (a) Isoelectric point (pH) or III < IV < I < II
0.77 + 2( -0.44)
=
pKa1 + pK a 2 2.34 + 9.60 14. (c) E0 =
= = 5.97 3
2 2
12. (d) Let bond energy of A2 be x then bond 0.77 - 0.88 0.11
= =-
energy of AB is also x and bond energy of 3 3
B2 is x/2. » - 0.04
Enthalpy of formation of AB is – 100 kJ/mol 15. (d) The chemical bond method gives the O.N.
1 1 – O O O O
A 2 + B2 ® 2 AB; A 2 + B 2 ® AB; D = - 100 kJ O – – – –
2 2 – S = O: O – S – S – O : O–S–S–O
2x + x - 4x O +4 +3
æx xö +5 O O
or - 100 = ç + ÷ - x\- 100 = \x = 400 kJ
è2 4ø 4
d+ d– + –
13. (d) 1% solution contains 1 g of the solute in 16. (d) CH2 = CH– Cl CH 2 – CH = Cl
Vinyl chloride
100 g of solution.
Osmotic pressure, p = CRT 17. (c) o-Nitrophenol is not sufficiently strong acid
The value of R and T is same for all the so as to react with NaHCO3.
solute however, all of them undergo 100% 18. (c) Because the layer of Al2O3 (oxide) is inert,
dissociation insoluble and impervious.
\ pµ i×C 19. (a) Au, the gold is not attacked by acids and
iKCl = 2, iNaCl = 2, iBaCl = 3 and iurea = 1. alkalis. It forms AuCl3.AuCl3 further reacts
2
nKCl = 1/74.5 with HCl to form H[Au(Cl)4] which is used
in photography.
1/ 74.5
CKCl = ´ 1000 = 0.13
100 AuCl3 + HCl ¾¾
® H[Au(Cl)4 ]
or pKCl = 2 × 0.13 = 0.26 Complex

1
nNaCl=
58.5
Solutions-Mock Test-1 MT-123

20. (b) 2KI + Cl 2 ® 2KCl + I 2 H O-C


| |
I 2 + CCl 4 ® Violet Colour (iv) - C - C - OCH 3 ,
Note: The excess of Cl2 should be avoided. | |
The layer may become colorless due to H O

conversion of I 2 to HIO 3 O-C


| |
I 2 + 5Cl 2 + 6H 2O ® 2HIO3 + 10HCl - C - O
(v)
In case of Br 2 : |
H
Br2 + 2H 2 O + Cl 2 ® 2HBrO + 2HCl
21. (d) To convert covalent compounds into ionic Arrange (NNN), (BrCC), (OOO), (CHH),
(OOH) in increasing atomic number. The
compounds such as NaCN, Na2S, NaX, etc.
order is ii, iii, v, i, iv.
22. (d)
CHCl3
2+ 26. (a) Cl
Zn + 2NH 4 OH ® Zn(OH) 2 + 2NH 4+
¾¾ KOH
White ppt.

Zn(OH) 2 + 2NH 4 OH ¾¾
®(NH 4 ) 2 ZnO2 + 2H 2 O HCl
Cl NC [Y]
Soluble 300 K

(NH 4 )2 ZnO 2 + H 2S ¾¾
® ZnS + 2NH 4OH
White ppt. Cl + HCOOH

23. (b) 5Fe2+ + MnO-4 +8H+ ¾¾


® Mn2+ +4H2O + [X]
5Fe3+ 27. (a) Let the rate law be r = [A]x[B]y

5NO-2 + 2MnO-4 + 6H+ ¾®2Mn2+ + 5NO3- + 3H2O 0.10 [0.024]x [0.035] y


Divide (3) by (1) =
24. (c) Formaldehyde can not produce iodoform, 0.10 [0.012]x [0.035] y
as only those compound which contains \ 1 = [2]x, x = 0
either CH3 - CH - group or CH3 -CH - 0.80 [0.024]x [0.070] y
| || Divide (2) by (3) =
OH O 0.10 [0.024]x [0.035]y
group on reaction with potassium iodide \ 8 = (2)y , y = 3
and sod. hypochlorite yield iodoform. Hence rate equation, R = K[A]0[B]3 =K[B]3
CH2– C – H
N C CH 3
| | | O CH2– CH
25. (a) (i) - C - N , (ii) - C - CH 3 , 28. (b) CH2
Ozonolysis
CH2
| | | O CH2– CH
N C Br
CH2– C – H

29. (b) N 2 (g) + 3H 2 (g) 2 NH 3(g)


OH
| 1–x 3 – 3x 2x
(iii) - C - O , at equilibrium
| | Total moles,
O C
1 – x + 3 – 3x + 2x = 4 – 2x = 3 (given)
EBD_7206
MT-124 JEE MAIN

(Since, 4 moles = 4 atm given)


x q2
\ x = 0.5 Hence, 2l =
4p Î0 x 2 ´ mg
p N ´ p3H
K p for dissociation of NH 3 = 2 2 2q 2 l
2 Þ x3 =
p NH3 4p Î0 mg
1/3
3 æ q2l ö
æ 1 - 0 .5 ö é æ 3 - 3 ´ 0 .5 ö ù \ x = çç ÷÷
ç ´ 3 ÷ ´ êç ÷ ´ 3ú è 2p Î0 mg ø
è 3 ø ëè 3 ø û
= 2 Therefore x µ l1/3
é 2 ´ 0 .5 ´ 3 ù
ê 3 ú
ë û 32. (a) From question,
Mass of body, m = 5 kg
= 0.5 × (1.5) 3 atm 2 Velocity at t = 0, u = (6iˆ - 2 ˆj) m/s
30. (a) If Na2CO3 is used in place of (NH4)2CO3. It
Velocity at t = 10s, v = + 6 ĵ m/s
will precipitate group V radicals as well as
magnesium radicals. The reason for this is Force, F = ?
the high ionization of Na2CO3 in water into v -u
Na + and CO 3 2– . Now the higher Acceleration, a =
t
concentration of CO32 – is available which
exceeds the solubility product of group V 6 ˆj - (6iˆ - 2 ˆj) -3iˆ + 4 ˆj
= = m/s2
radicals as well as that of magnesium 10 5
radicals. Force, F = ma
PHYSICS ( -3iˆ + 4 ˆj )
= 5´ = ( -3iˆ + 4 ˆj ) N
5
2 3 2 3
31. (d) 33. (d) T 2 µ r 3 ; T0 µ R & Ts µ (> 7R) ;
2
q æ Ts ö
ç ÷ = 7 3 ; Ts = 7 7 T0
l çT ÷
è 0 ø
Tcos q
q 34. (b) T ¯ (300K to 70K )
Tsin q
q Fe T ¯ R metal ¯ R ­ semi - conductor
x q
(Al) (Si)
mg 35. (d) From Doppler's effect
æ 340 ö÷
f (direct) < f ççç ÷<f
In equilibrium, Fe = T sin q è 340 , 5 ø÷ 1
mg = T cos q
æ 340 ÷ö
f (by wall)=f çç
Fe q 2 çè 340+5 ÷÷ø = f2
tan q = =
mg 4p Î0 x 2 ´ mg Beats = (f1 – f2)
æ 340 340 ÷ö
5 < f ççç , ÷
also tan q » sin =
x/2
è 340 , 5 340 ∗ 5 ÷ø
l
Þ f < 170 Hz.
Solutions-Mock Test-1 MT-125

36. (d) When two rods are connected in series 38. (c) P-V indicator diagram for isobaric

A(T1 - T2 )t A(T1 - T2 ) t P
Q= =
d1 d 2 (d 1 + d 2 ) / K dP
+ slope =0
K1 K1 dV

d1 + d 2 d1 d 2 V
\ = + ;
K K1 K 2
P-V indicator diagram for isochoric process
(d1 + d 2 ) P
\K = dP
d1 d 2 slope =¥
+ dV
K1 K 2

37. (a) When current flow in both wire in same


direction then magnetic field at half way V
due to P wire.
P-V indicator diagram for isothermal
uur m I m I m process
BP = 0 1 = 0 1 = 0 (where I1= 5Amp)
5 p .5 p
2p P
2 dP - P
slope = =
dV V
The direction of Bp is downward

P Q
V
×

5 Amp 2.5 Amp 39. (d) In balan ce position of bridge,


C P l
=
× Q (100 - l )
Initially neutral position is 60 cm. from A,
5m so
4 Q 16 8
Magnetic field at half way due to Q wire = ÞQ= = W
60 40 6 3
m0I 2 m0 Now, when unknown resistance R is
BQ = = [upward × ]
5 2p connected in series to P, neutral point is 80
2p
2 cm from A then,
4+ R Q
[where I 2 = 2.5Amp. ] =
80 20
Net magnetic field at half way
4+ R 8
m m 3m =
B = BP + BQ = 0 + 0 = 0 80 60
p 2p 2p 64 64 - 24 40
R= -4= = W
(downward ) 6 6 6
Hence, the value of unknown resistance R
3m 0
Hence net magnetic field at midpoint = 20
2p is = W
3
EBD_7206
MT-126 JEE MAIN

40. (c) The silvered plano convex lens behaves as


a concave mirror; whose focal length is v2 - u 2
vc2 = u 2 +
given by 2
1 2 1
= + u 2 + v2
F f1 f m vc =
2
If plane surface is silvered 43. (a) Angular retardation,
R2 ¥ w2 - w1 2p(n 2 - n1 ) 2 p(0 - 900 / 60) p
fm = = =¥ a= = = = rad / s2 .
2 2 t t 60 2

1 æ 1 1 ö 44. (a) Given : k A = 300 N / m, k B = 400 N / m


\ = (m –1) ç – ÷
f1 è R1 R 2 ø Let when the combination of springs is
compressed by force F. Sprin g A is
æ 1 1 ö m –1
= (m – 1) ç – ÷ =
compressed by xA. Therefore compression
èR ¥ø R in spring B

1 2(m – 1) 1 2(m – 1) x B = (8.75 - x A ) cm


\ = + =
F R ¥ R F = 300 ´ x = 400(8.75 - x A )
R Solving we get, xA = 5 cm
ÞF =
2(m – 1) x B = 8.75 - 5 = 3.75cm
Here R = 20 cm, m = 1.5
1
20 k (x )2
\ F= = 20cm EA 2 A A 300 ´ (5)2 4
2(1.5 –1) = = =
EB 1 2 400 ´ (3.75) 2
3
k B (x B )
41. (a) Pitch = 1 mm 2
Number of divisions on circular scale = 200
Pitch
L.C = 45. (a) From F < R v(t) Þ m dv < R v(t)
Number of divisions on circular scale t2 dt t 2
1 mm
= = 0.005 mm = 0.0005 cm dv Rdt
ò dt ò mt 2
200 Integrating both sides <
Diameter of the wire = (Main scale reading
+ Circular scale reading × L.C.) – zero error
= 6 mm + 45 × 0.005 – (– 0.05) R
In v < ,
= 6 mm + 0.225 mm + 0.05 mm = 6.275 mm mt
42. (d) Let 'S' be the distance between two ends 1
'a' be the constant acceleration [ ln v ×
t
As we know v2 – u2 = 2aS
46. (b) In reverse bias on p-n junction when high
v2 - u2
or, aS = voltage is applied, electric break down of
2 junction takes place, resulting large increase
Let vc be velocity at mid point. in reverse current. This high voltage applied
2 2 S is called zener voltage.
Therefore, vc - u = 2a 47. (c) Resistors 4 W, 6 W and 12 W are connected in
2
parallel, its equivalent resistance (R) is given
vc2 = u 2 + aS by
Solutions-Mock Test-1 MT-127

1 1 1 1 12 N 2paΧa ≥ b
= + + Þ R= = 2W <B
R 4 6 12 6 (2pa)b pa 2 b
Again R is connected to 1.5 V battery
whose internal resistance r = 1 W. (2pa) 2 Χab 2
ÞN<B
Equivalent resistance now, pa 2 b
R¢ = 2W + 1W = 3W Force needed to push the cork.
V 1.5 1
= = A f < m N < m 4pbΧaB = (4pmBb)Da
Current, Itotal =
R' 3 2
1 1 52. (b) This is a example of uniform circular motion.
Itotal == 3x + 2x + x = 6x Þ x =
2 12 2p 7
w= = 2pn = 2p ´ = 0.44 rad / sec;
\ Current through 4W resistor = 3x T 100
1 1 V = Rw = 0.44 × 12 = 5.3 cm/sec.
= 3× = A
12 4 53. (c) The minimum voltage range of DC source
Therefore, rate of Joule heating in the 4W is given by
resistor V2 = PR
R = 100W
2
æ1ö 1 = 1×100 Q P = 1 watt,
= I2R = ç ÷ ´ 4 = = 0.25W \ V = 10 volt.
4
è ø 4
54. (a) Given, B = 4 × 10–5 T
48. (c) Conservation of Energy,
RE = 6.4 × 106 m
Photon ® e + + e - + energy Dipole moment of the earth M = ?
EP = E +E + K.E. of both e - + e + m0 M
re + re - B=
4p d3
We know that rest mass energy of e + + e -
4p´10-7 ´ M
is .5 MeV Hence E P = .5 + .5 + .19 ´ 2 = 4 ´10-5 =
( )
3
1.40MeV 4p´ 6.4 ´ 106
49. (b) Let initial amount be 100 gm.
disintegrated Left \ M @ 1023 Am2
5 days 100 ´ 10 mmg
100 gm ¾¾ ¾® 10 90 55. (a) a =
100 M
Next 5 days 90 ´ 10
90 ¾¾ ¾ ¾¾® 9 81 Body
100 m 3mmg
fr
Next 5 days 81´10
81 ¾¾ ¾¾¾® 8.1 » 73 M plate
100 fr

73 ´ 10
Next 5 days fr = mmg = Ma
73 ¾¾ ¾ ¾¾® 7.3 » 65
100 56. (c) Einstein equation KE max = E – Work
50. (b) As mv > mr therefore, vv < vr. function;
Normal force N N
51. (d) Stress < < < 1
mv2 = E - W
Area A (2p a)b 2
Stress = B×strain Using this concept,
EBD_7206
MT-128 JEE MAIN

1 1
1
mV12 max XC = = = 3.1 W
2 1 - .5 1 V max 1 wC 320 ´ 1000 ´ 10 -6
= = or 1 =
1 2 2.5 - .5 4 V2 max 2 Total impedance of the circuit :
V2 max
2
Z = R 2 + (XL - XC )2 = 25 + (4.9)2 = 7 W
57. (a) Velocity of electron in nth orbit of hydrogen
atom is given by : Phase difference between the voltage and
current
2pKZe2 X - XC
Vn = tan f = L
nh R
Substituting the values we get, 4.9
tan f = » 1 Þ f = 45°
2.2 ´ 106 1 5
Vn = m/s or Vn µ
n n MATHEMATICS
As principal quantum number increases,
61. (d) If the G.P be a, ar, ar2, .... then an = arn – 1
velocity decreases.
log a + ( n - 1) log r log a + n log r log a + (n + 1) log r
1
58. (b) Total energy, E = mw 2 a 2 ; D= log a + n log r log a + (n + 1) log r log a + ( n + 2) log r
2 log a + ( n + 1) log r log a + ( n + 2) log r log a + ( n + 3) log r

3E 1
= mw 2 (a 2 - y 2 ) . R3 ® R3 – R2 and R2 ® R2 – R1 gives,
K.E. =
4 2
log a + (n - 1)log r log a + n log r log a + (n + 1)log r
3 a2 - y2 a2 a log r log r log r
So, = 2
or y 2 = or y = . =
4 a 4 2 log r log r log r

T2 T2 1 5 = 0, since R2 and R3 are identical.


59. (a) From h = 1 - , = 1- h = 1- =
T1 T1 6 6 62. (a) Total no. of arrangements of the letters of
...(i) 10!
the word UNIVERSITY is .
In 2nd case : 2!
T2 - 62 2 2 No. of arrangements when both I's are
= 1 - h' = 1 - = .....(ii)
T1 6 3 together = 9!
So. the no. of ways in which 2 I’s do not
2 2 6 4
Using (i), T2 – 62 = T1 = ´ T2 = T2 10!
3 3 5 5 together = - 9!
2!
or 1 T2 = 62, T2 = 310 K = 310 – 273 = 37°C \ Required probability
5
10!
- 9!
6 6 2! 10!- 9! 2!
T1 = T2 = ´ 310 = 372K = =
5 5 10! 10!
= 372 - 273 = 99°C 2!
60. (b) Given,
V0 = 283 volt, w = 320, R = 5 W, L = 25 mH, C 10 ´ 9!- 9!.2! 9![10 - 2]
= =
= 1000 µF 10! 10 ´ 9!
XL = wL = 320 × 25 × 10–3 = 8 W 8 4
= =
10 5
Solutions-Mock Test-1 MT-129

2 tan 2 b + 1 1 d5 y æ pö
63. (c) = = \ = Aw5 cos wt = Aw5 sin ç wt + ÷
cos 2a tan b sin b cos b dx5 è 2ø
Þ sin2b = cos2a = sin (90–2a) Þ a + b 68. (c) Since AD is the median
p
= A (2, 3, 5)
4
64. (d) Sum of 7 can be obtained when (2,6), (3,5)
(3, 6), (4, 4), (4,5), (4, 6)(5, 3)(5, 4) (5, 5)
(5, 6)(6, 2)(6, 3)(6, 4)(6,5)(6, 6)
15 5
\ Probability of sum > 7 = =
36 12

n
Cr r. n r - 1. n - r + 1 n - r + 1
65. (b) = . = B
D C
n
Cr -1 r. n - r n n-r (–1, 3, 2) (l, 5, m)

n - r +1
= = n - r +1
n -r æ l -1 m+2ö
\ D=ç , 4, ÷
è 2 2 ø
5
Now, dR’s of AD is
\ å= n + (n -1) + (n - 2) + (n - 3) + (n - 4) æ l -1 ö l -5
r =1
a =ç - 2÷ =
= 5n -10 = 5(n - 2) è 2 ø 2

3 m+2 m -8
66. (b) 14
C7 + å 17-i C6 b = 4 – 3 = 1, c = -5 =
2 2
i =1
Also, a, b, c are dR’s
=14 C7 +14 C6 +15 C6 +16 C6 \ a = kl, b = km, c = kn where l = m = n
and l2 + m2 + n2 = 1
=15 C7 +15 C6 +16 C6 1
Þ l=m=n=
16
= C7 +16 17
C6 = C7 3
Now, a = 1, b = 1 and c = 1
dy Þ l = 7 and m = 10
67. (d) y = A sin wt. \ = Aw cos wt
dx 69. (a) a1 = 2, b1 = 2,c1 = –1 and a2 = 1, b2 = 2, c2 = 2
a1a 2 + b1b 2 + c1c 2
d2 y cos q =
= - Aw 2 sin wt
dx 2 a12 + b12 + c12 a 22 + b22 + c22

d3 y 2+ 4- 2 4
= -Aw3 cos wt = =± .
dx3 4 + 4 +1 1+ 4 + 4 9
70. (c) Contrapositive of p Þ q is ~ q Þ ~ p
d4 y
= + Aw 4 sin wt \ contrapositive of (p Ú q) Þ r is
dx 4
~ r Þ ~ (p Ú q) i.e. ~ r Þ (~ p Ù ~ q)
EBD_7206
MT-130 JEE MAIN

71. (a) Let the co-ordiante of other ends are (x, y, z).
K2 4
The centre of sphere is C(3, 6, 1) - =1 (Qb = ± 2)
9 4
x+2 Þ K2 = 18
Therefore, = 3Þ x = 4
2

y+3 z+5 74. (c) For x > 10, f (x) = x – 2.


= 6 Þ y = 9 and = 1 Þ z = -3 Therefore, g(x) = x – 2 – 2 = x – 4
2 2
\ g¢(x) = 1.
72. (d) P(E E) + P(E E E E E ) 75. (b) a, b, c in A.P. Þ a + c = 2b; b, c, d in G.P..
4 8
5 1 æ 5ö 1 æ 5ö 1 2ce
= ´ + ´ +ç ÷ ...¥ Þ bd = c2; c, d, e in H.P. Þ d =
6 6 çè 6 ÷ø 6 è 6ø 6 c+e
5 é æ 5ö ù 30
3
= ê1 + ç ÷ ......ú = a + c 2ce
36 êë è 6 ø \ ´ = c2 Þ (a + c)e = (c + e)c
úû 91 2 c+e

73. (c) Given equation of ellipse is Þ c 2 = ae.


x2 y 2 Therefore, in G.P.
+ =1
16 b 2 15
æ 1 ö
76. (b) çç 3x 2 - ÷÷
b2 è x2 ø
eccentricity = e = 1 -
16
r
15 2 15-r æ 1 ö
b2 Tr +1 = C r (3x ) çç - ÷÷
foci: ± ae = ± 4 1 - è x2 ø
16
=15 C r 315-r (-1) r x 30- 2 r - 2 r
2 2
Equation of hyperbola is x - y = 1 Therefore, 30 – 4r = 10 Þ r = 5.
144 81 25
-15! 10
x2 y2 Therefore, T6 = -15 C 5 310 = 3 .
Þ - =1 10! 5!
144 81
25 25 77. (d) Since 0 < y < x < 2y
x x
81 25 81 \ y> Þ x- y <
Eccentricity = e = 1 + ´ = 1+ 2 2
25 144 144
\ x – y < y < x < 2x + y
225 15 y+x
= = Hence median = = 10
144 12 2
12 15 Þ x + y = 20 ...(i)
foci: ± ae = ± ´ = ±3 And range = (2x + y) – (x – y) = x + 2y
5 12
But range = 28
Since, foci of ellipse and hyperbola coincide
\ x + 2y = 28 ...(ii)
b2 From equations (i) and (ii),
\ ± 4 1- = ± 3 Þ b2 = 7 x = 12, y = 8
16
Solutions-Mock Test-1 MT-131

\ Mean
a1 b1 c1 1
( x - y ) + y + x + (2 x + y ) 4 x + y Since, a = b = c = 3
= = 2 2 2
4 4 therefore P and R are parallel.
y 8 1 1 1 1 p4
= x+ = 12 + = 14 + + + + ....¥ =
4 4 80. (a)
14 24 34 44 90
dx
78. (a) I = ò ( x - b) ( x - a )(b - x )
1
+
1
+
1
+ ..... + ¥ +
1 æ1 1 1 1
4 çè 4
ö
+ 4 + 4 + 4 + ....¥÷
14 4
3 54
2 1 2 3 4 ø

Put x = a sin 2 q + b cos 2 q p4


=
[see the standard substitutions] 90
dx = 2(a - b) sin q cos q dq
1 1 1 1 1 p4 p4
2 + + + + .... + ¥ + ´ =
Also, ( x - a) = (b - a ) cos q 14 34 54 74 16 90 90
\
( x - b) = ( a - b) sin 2 q
1 1 1 1 p4 1 æ p4 ö
2(a - b) sin q cos q dq + + + + .... + ¥ = – ç ÷
\I = ò (a - b) sin 2 q (b - a) sin q cos q 14 34 54 74 90 16 è 90 ø

2 dq 2 15 æ p4 ö p 4
b - a ò sin 2 q b - a ò
= = cos ec 2 q dq = ç ÷= .
16 è 90 ø 96

=
2
b-a
(- cot q) + C =
2
a -b
cot q + C 81. (b) We have, f (x) = exp ( 5x - 3 - 2 x2 )
5 x - 3- 2 x 2
i.e. , f (x) = e
Now , x = a sin 2 q + b cos 2 q
For domain of f (x), 5 x - 3 - 2 x 2 should
Þ x cos ec 2 q = a + b cot 2 q be +ve.
Þ x (1 + cot 2 q) = a + b cot 2 q i.e., 5x - 3 - 2 x2 ³ 0

x-a Þ 2 x2 - 5x + 3 £ 0
\cot q = ; (By taking –ve sign common)
b-x
Þ 2 x( x - 1) - 3( x - 1) £ 0
2 x -a Þ (2 x - 3)( x - 1) £ 0
\I= +C
a-b b- x
Þ 2x - 3 £ 0 or x -1³ 0
79. (d) Given planes are 3
P : x + y – 2z + 7 = 0 Þ x£ or x ³1
2
Q : x + y + 2z + 2 = 0
3 é 3ù
and R : 3x + 3y –6z – 11 = 0 \ 1£ x £ i.e., x Î ê1, ú
Consider Plane P and R. 2 ë 2û
Here a1 = 1, b1 = 1, c1 = – 2 Hence, domain of the given function is
and a2 = 3, b2 = 3, c2 = – 6 3
[1, ].
2
EBD_7206
MT-132 JEE MAIN

82. (a) Given determinant


1 5 1
A= ,B= ,C=
1 a a2 2 2 5
c o s ( n - 1) x cos nx c os ( n + 1) x
=0 16
sin ( n - 1) x sin n x sin ( n + 1) x A+B+C=
5
1 + a 2 - 2 a cos x a a2 æ 2p ö æ 2p ö
84. (d) We have, a = cos ç ÷ + i sin ç ÷
Þ 0 co s n x co s ( n + 1) x è 7ø è 7ø
7
0 sin n x sin ( n + 1) x é æ 2p ö æ 2p ö ù
Þ a7 = ê cos ç ÷ + i sin ç ÷ ú
ë è 7 ø è 7 øû
=0
= cos 2p + i sin 2p = 1 …(i)
By applying C1 ® C1 + C3 – 2 cos x C2
Let S = a + b = (a + a2 + a4) + (a3 + a5 + a6)
By expanding
[Q a = a + a2 + a4, b = a3 + a5 + a6]
(1 + a2 – 2a cos x) [cos nx sin (n + 1) x
– sin nx cos (n + 1) x]= 0 Þ S = a + a2 + a3 + a4 + a5 + a6
Now, (1 + a2 – 2a cos x) sin (n + 1 – n) x = 0
a (1 - a 6 )
2
Þ (1 + a - 2a cos x)sin x = 0 =
1- a
a - a7 a - 1
1 + a2 Þ S= = = –1 …(ii)
sin x = 0 or cos x = 1- a 1- a
2a Let P = ab = (a + a + a )(a + a + a6)
2 4 3 5

= a4 + a6 + a7 + a5 + a7 + a8 + a7
æ 1 + a2 ö + a + a10
9
As a ¹ 1 \ç ÷ >1
è 2a ø = a4 + a6 + 1 + a5 + 1 + a + 1 + a2 + a3
[from Eq. (i)]
Þ cos x > 1 It is not possible. = 3 + (a + a2 + a3 + a4 + a5 + a6) = 3 + S
=3–1=2 [from Eqn. (ii)]
\ sin x = 0 Required equation is, x2 – Sx + P = 0
dx Þ x2 + x + 2 = 0
83. (a) ò cos3 x 4sin x cos x
85. (a) We have,
é cos 2 q cos q sin qù é cos 2 f cos f sin fù
dx AB = ê úê ú
= ò 2 cos4 x tan x
êëcos q sin q
2
sin q úû êëcos f sin f sin 2 f úû

Let tan x = t Þ sec2 x = 1 + t4


2
écos2 q cos2 f + cos q cos f sin q sin f
sec2 x dx = 2t dt =ê 2 2
êë cos q sin q cos f + sin q cos f sin f
sec 4 x dx sec 2 x (sec2 x dx )
= ò2 tan x
= ò 2 tan x cos2 q cos f sin f + cos q sin q sin2 fù
ú
cos q cos f sin q sin f + sin2 q sin2 f ûú
(1 + t 4 )2t dt
= ò 2t
= ò (1 + t 4 ) dt
écos q cos f cos q sin fù
= cos( q - f) ê ú
t5 ë sin q cos f sin q sin f û
= t+ +k
5 Since, AB = 0, \ cos( q - f) = 0

1 p
= tan x + tan 5 / 2 x + k ét = tan x ù \ q - f is an odd multiple of
5 ë û 2
Solutions-Mock Test-1 MT-133

86. (c) f ( x ) = xe x(1- x) , x Î R 1


= p (radius)2 × height
3
f ' ( x ) = e x(1- x) . é1 + x – 2 x 2 ù
ë û
1
x(1- x ) é 2 \ V = px 2 y. Let ÐBAD = a
= -e . 2 x - x –1ù 3
ë û
x(1- x ) éæ 1ö ù BD 5 1
= -2e . êç x + ÷ ( x - 1)ú Þ tan a = = = .
è 2 ø AD 10 2
ë û
Again, from right angled DAMR,
f ' ( x ) = -2e ( ) . A
x 1- x
we have
æ 1ö
where A = çè x + ÷ø ( x - 1) tan a =
MR x 1 x y
= ; Þ = ; \x = .
2 AR y 2 y 2
Now, exponential function is always +ve
and f ¢(x) will be opposite to the sign of A 2
1 1 æ yö p
é 1 ù \V = p.x 2 y = p.ç ÷ .y = y2 ......(1).
which is –ve in ê - ,1ú 3 3 è2ø 12
ë 2 û
By question, the rate of change of volume
é 1 ù
Hence, f ¢(x) is +ve in ê - ,1ú dV
ë 2 û = . = 4 cub.cm./min.
dt
é 1 ù
\ f(x) is increasing on ê - ,1ú We have to find out the rate of increase of
ë 2 û
water-level
87. (b) Required area dy
i.e. .
1 1 1 dt
ò ò
= | y | dx = | x 5 | dx = 2 | x 5 | dx ò Differentiating (1) with respect to t, we get
-1 -1 0
1 dV p 2 dy p dy dy 16
1
éx6 ù = .3y . ; \ 4 = y2. ; \ = .
2 1
ò
= 2 x 5 dx = 2ê ú = = dt 12 dt 4 dt dt py2

0
êë 6 úû 0 6 3 When y = 6 cm,
88. (d) Let y be the level of water at time t and x the
dy 16 4
radius of the surface and V, the volume of = = cub.cm. / min .
water. dt p6 2 9p
We know that the volume of cone 89. (d) Slope of the equations 4x2 – 9y2 = 36
D 4x
C 5cm B dy dy 4x
8x - 18y =0Þ = or m1 =
dx dx 9 y 9y

Slope of the straight line, 5x + 2y – 10 = 0 is


R 5
M m2 = -
10 cm 2
Therefore, for the perpendicularity, m1m2 = –1
y 4 x -5 10x
Now, ´ = -1 Þ y = .
9y 2 9
A x
EBD_7206
MT-134 JEE MAIN

10 x
Putting in 4 x 2 - 9 y 2 = 36
y= p1 2æ 1 ö
9 \I = ò p 2 sec ç t ÷ dt
è3 ø
gives imaginary roots resulting in no 2
tangents.
p
1 1é t ù
log p 2 æ 1 2x ö = . ê tan .ú
òlog
2x
90. (a) I= e sec ç e ÷ dx 2 1 ë 3 ûp2
p 2 è3 ø
3
Put e 2 x = t Þ 2 e 2 x dx = dt
3 é p pù
p2 = ê tan 3 - tan 6 ú
When x = log p 2 , t = e 2 log 2 ë û
log p 2 p
=e = 3é 1 ù
2 = ê 3- ú= 3
2ë 3û
p
When x = log p , t = e 2 log =p
Mock Test-2
ANSWER KEY
1 (b) 16 (d) 31 (d) 46 (c) 61 (a) 76 (a)
2 (b) 17 (b) 32 (a) 47 (d) 62 (b) 77 (b)
3 (c) 18 (b) 33 (b) 48 (d) 63 (b) 78 (b)
4 (b) 19 (a) 34 (c) 49 (c) 64 (b) 79 (a)
5 (a) 20 (b) 35 (b) 50 (c) 65 (b) 80 (a)
6 (b) 21 (c) 36 (b) 51 (d) 66 (c) 81 (c)
7 (d) 22 (c) 37 (a) 52 (c) 67 (d) 82 (a)
8 (d) 23 (c) 38 (a) 53 (c) 68 (c) 83 (c)
9 (c) 24 (b) 39 (a) 54 (b) 69 (a) 84 (a)
10 (a) 25 (b) 40 (b) 55 (c) 70 (a) 85 (b)
11 (d) 26 (b) 41 (c) 56 (a) 71 (b) 86 (c)
12 (d) 27 (b) 42 (d) 57 (d) 72 (a) 87 (d)
13 (c) 28 (b) 43 (d) 58 (a) 73 (d) 88 (a)
14 (c) 29 (b) 44 (a) 59 (a) 74 (a) 89 (c)
15 (b) 30 (b) 45 (c) 60 (a) 75 (a) 90 (b)

Solutions
CHEMISTRY
é æ 0.2 ´ V ö ù
ê çè 100 + V ÷ø ú 0.2 ´ V
k ´ 1000 5.3 = 5 + log ê ú\ 2=
1. (b) L¥
H 2O = ,
N ê 100 ´ 0.1 ú 10
êë 100 + V úû
5.5 ´ 10-7 ´ 1000
N= = 10-6 Þ V = 100 ml.
550
3. (c)
Kw = N2 or M2, pH = pOH = 6
4. (b) N2O5(g) ƒ N2O3(g) + O2(g)
2. (b) Let V ml of RNH3Cl added into RNH2 conn at eqm 2 – x x – y x + y
solution N2O3(g) ƒ N2O(g) + O2(g)
0.2 ´ V con n at eqm x – y y y + x
[RNH3Cl] in resultant solution = 2.5 ´ (x - y)
100 + V 2.5 =
(2 - x)
100 ´ 0.1
[RNH 2 ] = ; x – y = 2 – x or 2x – y = 2 and as per given
100 + V
[O2(g)] = x + y = 2.5
[RNH3+ ] x = 1.5 and [N2O(g)] = y = 1.0
pOH = pK b + log
[RNH 2 ]
EBD_7206
MT-136 JEE MAIN

M 144 11. (d) Ester of dicarboxylic acids undergo an


5. (a) Equivalent of MoO3 = = = 24 intramolecular version of the Classian
6 6
condensation when a five or six membered
equivalent of H2 : ring can be formed. This reaction is an
192 WH 2 example of a Dieckmann’s condensation
= Þ WH2 = 8gm. 12. (d) (a) The change of atmospheric
24 1
temperature with altitude is called the
6. (b) 4S3 = 3.2 × 10–11 lapse rate.
S = 2 × 10–4 (b) The gases responsible for
Let solubility of CaF2 is x in KF solution. greenhouse effects are CO2, Water
Ksp = x (2x + 4 × 10–3)2 ; 3.2 × 10–11 vapour, CH4 and ozone.
(c) The order of contribution to the acid
= x × 10–6 × 16 rain is
x = 2 × 10–6 i.e. 100 times H2SO4 > HNO3 > HCl
7. (d) (1) Tranquillizers are the substances used 13. (c) As the value of reduction potential
for the treatment of mental diseases. These decreases the reducing power increases i.e.
act on higher centres of the central nervous C < B < D < A
system. These are also called (0.85). (0.6). (–0.76) (– 1.2)
psychotherapeutic drugs. 14. (c)
µ2
(2) Vitamin C, Vitamin E and b-carotene are
antioxidants µ3
Cl
(substances that act against oxidants). x
(3) Sodium or potassium salt of a long chain µ1
fatty acid is called soap. I Cl
8. (d) A B x
Initial concentration Rate of reaction µ3 Cl
2 × 10–3 M 2.40 × 10–4 Ms–1
–3 µ2
1 × 10 M 0.60 × 10–4 Ms–1 (µD ¹ 0)
rate of reaction Molecule is polar and planar.
r = k[A]x Both Ð Cl I Cl are equal
where x = order of reaction Equatorial I – Cl bond has more s-character
than axial I–Cl bond.
hence 15. (b) Since the compound is formed by hydration
2.40 × 10–4 = k [2 × 10–3]x ......(i) of an alkene, to get the structure of alkene
0.60 × 10–4 = k [1 × 10–3]x ......(ii) remove a molecule of water from the alcohol.
On dividing eqn.(i) from eqn. (ii) we get -H O
CH3 C H CH 3 ¾¾¾®
2
CH 2 = CHCH 3
4 = (2)x | Propylene
\ x=2 OH
Isopropyl alcohol
i.e. order of reaction = 2
16. (d) Na reacts vigorously with water (exothermic
9. (c) w = Z × n × q process ).
96.5
Z= = 5.55 ´ 10-4 17. (b) (a) NH4NO2 ¾¾
® N2 + 2H2O
D
0.9 ´ 2 ´ 96500 D
10. (a) (b) (NH4)2SO4 ¾¾ ® NH3 + H2SO4
D
(c) 2NH4ClO4 ¾¾ ® N2 + Cl2 + 2O2 + 4H2O
5 ´ 0.08 - 10 ´ 0.03
[H + ]remaining = = 2 ´ 10-4 (d) (NH4)2Cr2O7 ¾¾
D
® N2 + Cr2O3 + 4H2O
10 + 5 + 485
pH = 3.7
Solutions-Mock Test-2 MT-137

18. (b) In first case the given compounds have 23. (c) Maximum kinetic energy of the
same anion but different cations having photoelectron is independent on intensity
of radiation. No. of electrons emitted per
different charge hence they will precipitate
second will be halved if the intensity of light
negatively charged sol i.e. ‘A’. striking on the surface halves.
In second case the given compounds have NO2
24. (b) NO2
similar cation but different anion with
different charge. Hence they will precipitate CH 3 ONa

positively charged sol. i.e. ‘B’. X=


SNAr
Br Br
19. (a)
Cl OCH3
Cl Cl Cl
Br Br 25. (b)
HBr O O
I II || ||
NaOH
X - C - CH 3 ¾¾¾® X - C - OH
HBr HÅ
Peroxide O
HgSO aldol SOCl2
||
20. (b) HC º CH ¾¾¾¾
4 ® CH - CHO ¾¾¾
3
H2SO4
® ¾¾¾¾ ® X - C - Cl
O
OH ||
| Ph - NH 2
¾¾¾¾ ® X - C - NH - Ph
CH3 - CH - CH 2 - CHO (3-hydroxy butanal)
3 2 1 (HNO + H SO )
¾¾¾¾¾¾¾
3 2 4 ®

Br O
Br 2 Alcoholic
21. (c) X – C – NH NO2
KOH/
(X) (Y)
1 3 ˆˆ† NH 3 ;
H 26. (b) Given N 2 + H 2 ‡ˆˆ
2 2
H – Br Br H Br
Peroxide
(Z) DHf = - 46.0 kJ / mol
22. (c) (a) When a dilute solution of an acid is ˆˆ† H 2 ; DH f = - 436 kJ / mol
H + H ‡ˆˆ
added to a dilute solution of a base,
neutralization reaction takes place. ˆˆ† N 2 ; DH f = - 712 kJ / mol
N + N ‡ˆˆ
(b) In acid-base titrations, at the end point,
1 3
the amount of acid becomes DH f ( NH 3 ) = DH N - N + DH H - H - DH N - F
2 2
chemically equilvalent to the amount
of base present. 1 3
(c) In the case of a strong acid and a
-46 = ( -712 ) + ( -436 ) - DH N - F
2 2
strong base titration, at the end point On calculation
the solution becomes neutral (i.e., pH DH N - F = - 964 kJ / mol
= 7)
(d) In acid-base titrations the end point is 27. (b) According to Arrhenius equation
determined by the hydrogen ion
k2 Ea æ 1 1 ö
concentration of the solution. log = -
k1 2.303R çè T1 T2 ÷ø
EBD_7206
MT-138 JEE MAIN

1.3 ´ 10-3 Ea 32. (a) Least count of vernier callipers


é 1 1 ù
log = - = value of one division of main scale – value
1.3 ´ 10 -4 2.303 ´ 8.314 ë 373 423 úû
ê
of one division of vernier scale
Now, N × a = (N + 1) a'
Ea é 1 1 ù
1= - (a' = value of one division of vernier scale)
2.303 ´ 8.314 ë 373 423 úû
ê
N
Ea = 60 kJ / mol a¢ = a
N +1
28. (b) Anhydrous copper (II) chloride is a
covalent while anhydrous copper (II) a
\ Least count = a - a ¢ =
fluoride is ionic in nature because in halides N +1
of transition metals, the ionic character 33. (b) Here I = 4A
decreases with increase in atomic mass of
the halogen. Fluorine being the most æ 30 ´ p ö p
q=30° = ç ÷=
electronegative, forms ionic salt with è 180 ø 6
copper.
I 4 4´6´7 2´6´7
29. (b) Given kb = x K kg mol–1 Now, k = = = =
DTb = kb × m q æpö 22 11
ç ÷
\ y= x × m è6ø
y 84
m= =
= 7.6 A / rad
x 11
We know 34. (c) Let mass of smaller sphere (which has to be
DTf = kf × m removed) is m
On substituting value of m, R
yz Radius = (from figure)
DTt = 2
x
30. (b) The redox system having the highest redox M m
=
potential in the most powerful oxidising 4 3 4 æRö
3
agent followed by the one with the lower pR pç ÷
3 3 è2ø
redox potential.
PHYSICS M
Þm=
8
31. (d) P
Mass of the left over part of the sphere
q M 7
8 cm M' = M - = M
8 8
T
Therefore gravitational field due to the left
over part of the sphere
Q 5 cm O
GM ' 7 GM
= 2
=
x 8 x2
w wcosq
35. (b) Centre of mass of the rod is given by:
PQ = OP 2 + OQ2 L
bx2
2
= 13 + 5 = 12 2 ò (ax + L
) dx
xcm = 0
Tension in the string L
bx
13
T = w cos q = W ò (a + L )dx
12 0
Solutions-Mock Test-2 MT-139

42. (d) From figure, sin a = d/R


aL2 bL2 L æ a + b ö
+ ç ÷
= 2 3 = è 2 3ø
bL b a R
aL + a+
2 2
a b a
+
7L 2 3 d
Now =
12 b
a+
2
On solving we get, b = 2a mv 2
And we know, = qvB
36. (b) Work done during the process A ® B R
= Area of trapezium (= area bounded by mv
Þ R=
indicator diagram with V-axis) qB
1 3 dqB
\ sin a =
=
2
( 2 P0 + P0 ) ( 2V0 - V0 ) = P0V0
2 mv
q é 1 2ù
êQ qV = 2 mv ú
Ideal gas eqn : PV = nRT sin a = Bd
2mV ë û
PV 3 P0V0
Þ T= = 43. (d) Power factor (old)
nR 2nR
R R R
37. (a) Charge conservation is violated in [b, c, d], = = =
nucleon conservation is violated in (d), (a) 2
R + XL 2 2
R + (2R) 2 5R
works. Power factor(new)
38. (a) A convex mirror always forms a virtual image
which can't be cast on a screen. Hence, a R R
= =
convex lens is used to find the focal lenght R 2 + (X L - XC )2 R 2 + (2R - R) 2
of a convex mirror.
39. (a) The conductivity of semiconductor R
=
s = e (heµe + hhµh) 2R
= 1.6 × 10 –19(5 × 1018 ×2 + 5 × 1019 × 0.01) R
= 1.6 × 1.05 = 1.68 New power factor 2R = 5
40. (b) Given : hR = 32 m \ =
Old power factor R 2
hT = 50 m
5R
Maximum distance, dM = ? hc hc
44. (a) E= Þ l=
Applying, dM = 2Rh T + 2Rh R l E
6.6 ´10-34 ´ 3 ´108
= 2 ´ 6.4 ´ 106 ´ 50 + 2 ´ 6.4 ´106 ´ 32 Þ l=
11´1000 ´1.6 ´ 10-19
= 45.5 km
= 12.4 Å
1 1 Increasing order of frequency
41. (c) E1 = kx 2 , E 2 = ky 2 ,
2 2
x-rays u-v rays visible Infrared
1
E= k(x + y)2 = E1 + E 2 + 2 E1E 2 wavelength range of visible region is 4000Å
2
to 7800Å.
= 2 + 8 + 2 16 = 18J 45. (c) Given : m = 0.160 kg
q = 60°; v = 10 m/s
® ®
Angular momentum L = r ´ m v
EBD_7206
MT-140 JEE MAIN

= H mv cos q 48. (d) For isothermal process :


PV = Pi .2V
v2 sin 2 q P = 2Pi ...(i)
= cos q
2g For adiabatic process
PVg = Pa (2V)g
é v2 sin 2 q ù
êH = ú (Q for monatomic gas g= 5 3 )
ëê 2g ûú
5 5
2 2 or, 2Pi V 3 = Pa (2V) 3 [From (i)]
10 ´ sin 60°´ cos 60°
=
2 ´10 -2
= 3.46 kg m2/s P 2 Pa
Þ a = 5 Þ =2 3
Pi Pi
46. (c) Given , y = 2 ´ 1011 Nm-2 23
æ Fö b ( m - 1) b ( m - 1)
Stress ç ÷ = 5 ´ 107 Nm -2 49. (c) Shift = t1 - t2
è Aø l l
DV = 0.02% = 2 × 10–4 m3 b ( m - 1)
= ( t1 - t2 )
Dr l
=?
r
ml 2
50. (c) I= ;
stress æ Dl ö g 12
g= Þ strain ç ÷ = … (i) Restoring troque
strain è l 0 ø stress
d 2q
DV = 2prl 0 Dr - pr 2 Dl … (ii) = qElsin q » qEl q =
dt 2
From eqns (i) and (ii) putting the value of
Dl , l 0 and DV and solving we get qEl qEl ´12 2p
\w = = =
Dr
I ml 2 T
= 0.25 ´10-4
r ml
47. (d) As the surrounding is identical, vessel is \T = p
3qE
identical time taken to cool both water and
liquid (from 30°C to 25°C) is same 2 minutes, 51. (d) When object is kept at a distance ‘a’ from
therefore thin covex lens

æ dQ ö æ dQ ö O
çè ÷ =ç ÷
dt ø water è dt ø liquid I1 v

(mw C w + W)DT (ml Cl + W) D T a


or, =
t t 1 1 1
By lens formula : v – u = f
(W = water equivalent of the vessel)
or , m w C w = m l C l 1 1 1
– =
mWCW V (– a) f
\ Specific heat of liquid , Cl = ml
1 1 1
or, = – ...(i)
50 ´ 1 v f a
= = 0.5 kcal / kg
100
Solutions-Mock Test-2 MT-141

Mirror forms image at equal distance from As in case of upward motion upward force
mirror is twice its effective weight, therefore, it will
move with same speed 10 cm/s

54. (b) D

I1 v v I2
X Y

Now, again from lens formula


C

I3 I2
Equivalent resistance = 5W
a/3
v–4
v 55. (c) Acceleration (a) = =
t
3 1 1
– = (0 , 50)
< ,5 m / s 2
a V f (10 , 0)
3 1 1 1 u = 50 m/s
– + = [From eqn. (i)] \ v = u + at = 50 – 5t
a f a f
Veocity in first two seconds t = 2
Hence, a = 2f
v(at t <2) < 40 m / s
52. (c) The equation of wave moving in negative
x-direction, assuming origin of position at x From work-energy theorem,
= 2 and origin of time (i.e. initial time) at t = 1
1 sec. ΧK.E. < W < (402 , 502 )´10 <,4500 J
2
y = 0.1 sin (2pt + 4x)
Shifting the origin of position to left by 2m, 56. (a) For a particle undergoing SHM with an
that is, to x = 0. Also shifting the origin of amplitude A and angular frequency w, the
time backwards by 1 sec, that is to t = 0 sec. maximum acceleration = w2A
y = 0.1 sin (2p (t – 1) + 4 (x – 2)) Here the maximum force on the particle = QE0
QE 0
53. (c) Weight of the body \ maximum acceleration = = w2A
m
4 3 QE 0
W = mg = pr rg
3 \ A=
T (upthrust) mw 2
F
1 hc
(viscous r 57. (d) mv2 = - f
force) 2 l
4 3
T = pr sg 1
mn '2 =
hc
-f=
4hc
- f . Clearly,,
3
W(weight) 2 (3l / 4) 3l
and F = 6phvr
When the body attains terminal velocity 4
net force acting on the body is zero. i.e., v¢ > v
3
W–T –F=0
2 r 2 (r - s)g
And terminal velocity v =
9 h
EBD_7206
MT-142 JEE MAIN

58. (a) The linear relationship between v and x is


80
v = – mx + C where m and C are positive Þ y= x ...(1)
constants. a
\ Acceleration,
æ 20 - 0 ö
dv Eqn of AB: y = çè ÷ ( x - a)
a=v = - m ( -mx + C) 0-a ø
dx
\ a = m2x – mC -20
Hence the graph relating a to x is
Þ y= ( x - a) ...(2)
a
a At M: (1) = (2)
80 -20
x Þ x= ( x - a)
a a
80 -20 a
Þ x= x + 20 Þ x =
a a 5
59. (a) Divide the ring into infinitely small lengths
80 a
of mass dm1. Even though mass distribution \ y= ´ = 16
is non-uniform, each mass dm1 is at same a 5
distance R from origin.
\ MI of ring about z-axis is + f (3 + h) - f (3)
= dm1R2 + dm2 R2 + ...........+ dmnR2 = MR2 62. (b) f ¢(3 ) = lim
h ®0 h
-dV
60. (a) E = V = 5x2 + 10x –9,
dx (2 - e h ) - 1 æ e h - 1ö
= lim = - lim ç ÷ = -1
-d h®0 h h®0 è h ø
E= (5x2 + 10x – 9) = – (10x + 10)
dx f (3 - h) - f (3)
On putting value of x in it we get, f ¢(3- ) = lim
h®0 -h
E = –(10 × 1 + 10) = –20 V/m
MATHEMATICS 10 - (3 - h)2 - 1 1 + (6h - h 2 ) - 1
= lim = - lim
h ®0 -h h ®0 -h
61. (a)
C(a, 80) -6
6h - h 2 = = -3
= lim
A (0, 20) h ®0 2
- h( 1 + 6h - h 2 + 1)
M Hence, f ' (3+) ¹ f (3–)
2 2
63. (b) z1 + z2 + z1 - z2
2 2 2 2
O(0, 0) N B (a, 0) = z1 + z2 +2 z1 z2 + z1 + z2 -2 z1 z2

= 2 éê z1 + z2 ùú
2 2 2 2
= 2 z1 + 2 z2 ë û
We put one pole at origin.
64. (b) Vertices of triangle in complex form is
BC = 80 m, OA = 20 m
z, iz, z + iz
Line OC and AB intersect at M. In cartesian form vertices are
To find: Length of MN. (x, y), (– y, x) and (x – y, x + y)
æ 80 - 0 ö
Eqn of OC: y = ç x
è a - 0 ÷ø
Solutions-Mock Test-2 MT-143

x y 1 æ y + am ö
y2 = 4a ç 2 2
1
-y
68. (c) è m ÷ø i.e., my – 4ay – 4a m = 0
\ Area of triangle = x 1
2 m ¹ 0; 16a2 + 16a2m2 > 0 which is true m.
x- y x+ y 1
\ m Î R – {0}
1 n
Ck 1
= [x (x – x – y) – y (– y – x + y) + 1 69. (a) =
2 n
C k +1 2
(– yx – y2 – x2 + xy)]
n! (k + 1)!(n - k - 1)! 1
1 1 Þ =
= [– xy + xy – y2 – x2] = (x2 + y2) k! (n - k)! n! 2
2 2
(Q Area can not be negative) k +1 1
or =
n-k 2
=
1 2
2
z (2
Q z = x + iy , z = x 2 + y 2 ) 2k + 2 = n – k
n – 3k = 2 ...... (1)
65. (b) Let A be the event such that sum is Rs. 20
n
or more Ck +1 2
Similarly, =
\ P (1) = 1 – P (Total value is < 20) n
Ck + 2 3
6 2
C2 - C2 14 1 1 n! (k + 2)!(n - k - 2)! 2
= 1- 8
=1- =1 - = . =
C2 28 2 2 (k + 1)! (n - k - 1)! n! 3
k+2 2
=
n - k -1 3
3k + 6 = 2n – 2k – 2
2n – 5k = 8 ...... (2)
66. (c) From (1) and (2)
2 2 2 n = 14 and k = 4
æ 1ö æ 2ö 2 æ 3x + 4y - 1ö
çè x - ÷ø + çè y - ÷ø = (l - 4l + 4) çè ÷ø \ n + k = 18
5 5 5
70. (a) Let the observations be x1, x2, x3, x4, x5 and
2 2 x6, so
æ 1ö æ 2ö 3x + 4y - 1
i.e., çè x - ÷ø + çè y - ÷ø = | l - 2 | 6
5 5 5
is an ellipse. å xi 6
If 0 < | l – 2 | < 1 i.e., l Î (1, 2) È (2, 3) their mean x = i =1
= 8 Þ å xi = 48
6 i =1
p On multiplying each observation by 3, we
67. (d) q1 + q2 =
2 get the new observations as 3x1, 3x2, 3x3,
q2 q2 3x4, 3x5 and 3x6.
dq tan q dq Now, their mean
\ I= ò = ò 1 + tan q
æp ö
q1 1 + tan ç - q÷ q1 6
è2 ø å 3x i
3 ´ 48
dq i =1
and also I = ò
q2 =x = = 24 =
6 6
q1 1 + tan q
q2 Variance of new observations
1002p 501p
\ 2I = ò dq = q2 - q1 = 2008
ÞI=
2008
6 6
q1 å (3x i - 24) 2 32 å (x i - 8) 2
Hence, K = 2008. i =1
= i =1
=
6 6
EBD_7206
MT-144 JEE MAIN

9 20! 20! p
= ´ 1 Variance of old observations 75. (a) (2x)p (- y)q (z)r = 2 (-1)q x p yq z r
1 p!q!r! p!q!r!

= 9 × 42 = 144 p + q + r = 20, q = 0
p + r = 20 (p is even and r is odd).
Thus, standard deviation of new
even + odd = even (never possible)
observations Coefficient of such power never occur
= Variance = 144 = 12 \ coefficient is zero
71. (b) p Ú (p Ù q) is equivalent to p. 76. (a)
Starting with 1 23456789
æ x cos3 x - sin x ö
72. (a) ò esin x ç
ç cos2 x
÷÷ dx = 8C4 = 70
è ø
Starting with 2 3 4 56 7 8 9
òe òe
sin x sin x
= x cos x dx – tan x sec x dx = 7C4 = 35
Total = 105
ò x d (e )–òe
sin x sin x (105)th number 26789
= d (sec x)
(tan x)3/ 2 [1 - (cos x)3/ 2 ]
(b) lim
{
= x esin x - ò esin x dx } 77.
x ®0 x 3/ 2 .x 2
.

1 - cos3 x 1
{
– esin x sec x - ò esin x sec x cos x dx } = 1×
lim
x ®0 x 2
.
1 + (cos x)3/2
= x esin x – esin x sec x + C
1 1 3
73. (d) f (x) = x2 – 4x + a always attains its minimum = . (1 + cos x + cos 2 x) =
value. 2 2 4
So its range must be closed. 78. (b) n = 3, P (success) = P (HT or TH) = 1/2
So, a = {f} 1
Þ p=q= and r = 2
74. (a) For real roots D ³ 0 2
(k – 2)2 – 4 (k2+ 3k + 5) ³ 0 2
æ 1ö 1 3
(k2 + 4 – 4k) – 4k2 – 12k – 20 ³ 0 P (r = 2) = 3C2 ç ÷ . =
è 2ø 2 8
– 3k2 – 16k –16 ³ 0 ; 3k2 + 16k + 16 £ 0
1 ˆ 1
æ 4ö 79. ˆ ˆ ˆ - (a.b)c
(a) (a.c)b ˆˆ ˆ= b+ cˆ
çè k + ÷ø (k + 4) £ 0 2 2
3
1 1
Now E = a 2 + b2 ; ˆˆ=
\ a.c and â.bˆ = -
2 2
E = (a + b) 2 - 2ab p
Þ angle between aˆ and cˆ = and angle
E = (k – 2)2 – 2 (k2 + 3k + 5) = –k2 – 10k – 6 4
E = – (k2 + 10k + 6) = – [(k + 5)2 – 19]
3p
= 19 – (k + 5)2 between â and bˆ =
4
\ Emin occurs when k = – 4/3
80. (a) |z1 + z2| £ |z1| + |z2| = |24 + 7i| + 6 = 25 + 6 = 31
121 171 - 121 50 Also, |z1 + z2| = |z1 – (–z2)| ³ ||z1| – |z2||
\ Emin = 19 - = =
Þ |z1 + z2| ³ | 25 – 6| = 19
9 9 9
Hence the least value of |z1 + z2| is 19 and
Emax occurs when k = – 4
the greatest value is 25.
Emax = 19 – 1 = 18
Solutions-Mock Test-2 MT-145

81. (c) The coordinates of points P, Q, R are Now, number of one-to-one functions from
(–1, 0), (0, 0), (3,3 3) respectively.. 8!
S ® P(S) is 8P3 = = 8 × 7 × 6 = 336.
5!
Slope of QR = 3 Y R (3, 3 3 ) 84. (a) 4x + 4y (dy/dx) = 0 Þ dy/dx = –x3/y3
3 3

Þ tan q = 3 M Equation of tangent, Y – y = – x3/y3 (X–x)


Þ y3Y + x3X = x4 + y4 = a4
p
Þ q= X Y
3 X'
2p / 3 p /3
X
Þ + =1
4 3
P (–1, 0) Q (0, 0) a /x a / y3
4
p
Þ ÐRQX = Y' Here, p = a4/x3, q = a4/y3
3
p 2p Þ p -4 / 3 + q -4 / 3 =
a -16 / 3
-4
+
a -16 / 3
y -4
(
= a -16 / 3 x 4 + y 4 )
\ ÐRQP = p - = x
3 3
= a -16 / 3 (a 4 ) = a -4 / 3
Let QM bisects the ÐPQR ,
85. (b) Let the equation of curve
2p y2(2a – x) = x3 ...(i)
\ Slope of the line QM = tan =– 3
3 and equation of line x = 2a ...(ii)
\ Equation of line QM is (y – 0) = – 3 (x – 0) The given curve is symmetrical about x-axis
and passes through origin.
Þ y= – 3x Þ 3 x + y= 0
x3
82. (a) Given
ò 2 1+ sin x dx = -4cos(ax + b) + C From (i) we have, y2 =
2a - x

æ x xö x3
Þ ò 2 ç sin + cos ÷ dx
è 2 2ø
But
2a - x
< 0 for x > 2a and x < 0

So, curve does not lie in the portion x > 2a


= -4cos(ax + b) + C
and x < 0, therefore curve lies in 0 £ x £ 2a .
æ 1 x 1 xö \ Area bounded by the curve and line
Þ ò 2. 2ç
è 2
sin +
2 2
cos ÷ dx
2ø 2a 2a
x3 / 2
= –4 cos(ax + b) + C = ò ydx =
ò 2a - x
dx
æ p x p xö 0 0
Þ ò 2 çè cos 4 sin 2 + sin 4 cos 2 ÷ø dx Put x = 2a sin2q and dx = 4a sin q cosq dq
= –4 cos(ax + b) + C p/2
é3 1 pù
ò 8a
2
\ I= sin 4 q dq = 8a 2 ê . . ú
æ x pö ë4 2 2û
Þ ò 2 sin ç + ÷ dx = –4 cos(ax + b) + C 0
è 2 4ø
æ p/2
æx pö
Þ -4 cos ç + ÷ = -4 cos(ax + b) + C
è2 4ø
ç using
çè ò
sin m x cos n x dx
0
1 p
Þ a= , b= m +1 n +1ö
2 4 G G
83. (c) Let S = {1, 2, 3} Þ n(S) = 3 2 2 ÷ 3pa 2
= = sq. unit
Now, P (S) = set of all subsets of S m+n+2 ÷ 2
2G ÷
2 ø
total no. of subsets = 23 = 8
\ n[P(S)] = 8
EBD_7206
MT-146 JEE MAIN

86. (c) cos 2 x


87. (d) f2 (x) = f (f (x)) = f (x) = x Let I2 = ò0 cos -1 t dt
f3 (x) = f (f2 (x)) = f (x) = x
Put t = cos 2 v Þ dt = -2 cos v sin vdv
Þ x3 – 25x2 + 175x – 375 = 0
(x – 5) (x2 – 20x + 75) = 0 Þ dt = - sin 2 v dv
(x – 5)2 (x – 15 ) = 0 Þ x = 5, 15 x x
ò
\I 2 = p v ( - sin 2 v )dv = - p v sin 2 vdv ò
88. (a) 2 sec 4C + sin22A + sin B = 0 2 2
A = 45°, B = 90° and C = 45° x
C ò
= - p u sin 2 udu [change of variable]
2
P \ I = I 1 + I2
x x
Q = ò0 ò
u sin 2 udu - p u sin 2 udu
2
p
R,B A 2 x x

Let AQ = a, then BP =
a
,
ò òp
= u sin 2udu + u sin 2udu - u sin 2udu òp
0
2
2 2
a p
PQ = and QR = a
2 2
p
2
ò
= u sin 2udu =
4
[Integrate by parts]
a 5a 0
\ PR = a 2 + =
4 2
90. (b) For non-trivial solution,
a 5a
\ 1: a : b = : a : = 1: 2 : 5 2a sin B cos B
2 2
\ a = 2 and b = 5 \ a 2 + b2 = 9 D= 1 cos B sin B =0
-1 sin B - cos B
sin 2 x
89. (c) Let I1 = ò sin -1 t dt
0
Þ a 2 [- cos 2 B - sin 2 B] - sin B [ - cos B + sin B]
Put t = sin 2 u Þ dt = 2 sin u cos udu
Þ dt = sin 2udu + cos B [sin B + cos B] = 0
x Þ -a 2 + sin 2B + cos 2B = 0 Þ a Î[-1,1]
\I1 = ò0 u sin 2u du
Mock Test-3
ANSWER KEY
1 (c) 16 (c) 31 (c) 46 (a) 61 (d) 76 (c)
2 (a) 17 (c) 32 (a) 47 (c) 62 (a) 77 (c)
3 (c) 18 (b) 33 (a) 48 (a) 63 (b) 78 (b)
4 (c) 19 (d) 34 (c) 49 (d) 64 (b) 79 (a)
5 (b) 20 (b) 35 (d) 50 (b) 65 (a) 80 (b)
6 (a) 21 (d) 36 (b) 51 (a) 66 (b) 81 (a)
7 (c) 22 (a) 37 (c) 52 (a) 67 (a) 82 (d)
8 (b) 23 (d) 38 (a) 53 (c) 68 (d) 83 (a)
9 (a) 24 (d) 39 (c) 54 (b) 69 (b) 84 (c)
10 (a) 25 (b) 40 (a) 55 (b) 70 (c) 85 (c)
11 (d) 26 (d) 41 (a) 56 (a) 71 (a) 86 (d)
12 (c) 27 (c) 42 (b) 57 (a) 72 (b) 87 (c)
13 (a) 28 (a) 43 (c) 58 (a) 73 (a) 88 (a)
14 (d) 29 (a) 44 (b) 59 (b) 74 (a) 89 (d)
15 (c) 30 (c) 45 (d) 60 (b) 75 (a) 90 (a)

Solutions
CHEMISTRY 3. (c) ICl,
2 Þ 2 bp + 3lp
(thus, sp3d hybridisation) = linear
æ 1 1 ö
1. (c) n=RcZ2 ç 2 - 2 ÷ BrF2∗ Þ 2 bp + 2lp
çn ÷
è 1 n2 ø
(thus, sp3 hybridisation) = pyramidal
æ1 1 ö
n1 =RcZ2 ç 2 - 2 ÷ =RcZ2 ClF4, Þ 4 bp + 2lp
è1 ¥ ø
(thus, sp3d2 hybridisation) = square planar
2
n2 =
æ 1 1 ö 3RcZ
RcZ2 ç 2 - 2 ÷ = AlCl,
4 Þ 4 bp + 0lp
è1 2 ø 4 (thus sp3 hybridisation) = tetrahedral
4. (c) The correct formula of inorganic benzene
æ 1 1 ö RcZ2
n3 =RcZ2 ç 2 - 2 ÷ = is B3N3H6 so (4) is incorrect statement.
è2 ¥ ø 4
OH
\ n1 - n2 =n3 |
Boric acid (H3BO3 or B - OH ) is a lewis
2. (a) |
OH
3° 3° acid so (1) is incorrect statement.
The coordination number exhibited by
+ (2 H) beryllium is 4 and not 6 so statement (2) is
N N incorrect.

Both BeCl 2 and AlCl 3 exhibit bridged
structures in solid state so (3) is correct
statement.
EBD_7206
MT-148 JEE MAIN

5. (b) 11. (d)


H2(g) + CO2(g) ƒ CO(g) + H2O (g) (a) For tetrahedral d6 ion,
At eqm 0.25–x 0.25–x x x
Kp = 0.16 =
x2 x
Þ 0.4 = Þ 0.1 - 0.4x = x 4 unpaired electrons
(0.25 - x) 2 0.25 - x
(b) For [Co(H2O)6]3+,
x = 0.0714
0.0714
Mole% of CO (g) = ´ 100 = 14.28
0.50 0 unpaired electrons
(c) For square planar d7 ion,
Anhydrous HI
6. (a) (I) ,
O SN2 I
HO
1 unpaired electrons
Conc. HI (d) B.M. = n(n + 2) , n = unpaired electrons
SN2 (II)
I
OH 5.92 B.M. = n(n + 2)
7. (c) By intermittently filled lime the blown air in n = 5 unpaired electrons
the sinterer gets diffused and PbS particles 12. (c) Ag (1) ƒ Ag+ (aq) + A– (aq)
come under limited supply of air therefore x x–y
formation of PbSO4 is prevented.
Ag (1) + H2O (l) ƒ HA (aq) + OH– (aq)
3 x–y y y
PbS + O ® PbO + SO2
2 2 Ksp = x (x – y)
PbO + SiO2 ® PbSiO3
gangue Kw y2 10-14 (10-5 ) 2
Kh = = ; =
PbSiO3 + CaO ® CaSiO3 + PbO K a (x - y) 10-10 (x - y)
more basic less basic
x – y = 10–6
8. (b) nCH 2 = CH - CH = CH 2 + n x = 10–5 + 10–6
1, 3-Butadiene
x = 1.1 × 10–5
CH = CH2 Ksp = 1.1 × 10–5 × 10–6
Ksp = 1.1 × 10–11
Na, Heat
Polymerisation 13. (a) (a) SiC Þ Covalent carbide
Styrene
(b) Be2C + 4H2O ¾¾ ® 2Be(OH)2 + CH4 ­
(c) CaC2 + 2H2O ¾¾ ® HC º CH ­ + Ca(OH)2
(– CH2 – CH = CH – CH2 – CH – CH2 –)n (d) Mg2C3 + 2H2O ¾¾ ® 2Mg (OH)2 + H3C
Butadiene - – C º C H­
Styrene copolymer
14. (d) ZnCl2 + conc. HCl is Lucas reagent. Lucas
(SBR or BUNA - S)
reagent reacts fastest with tertiary alcohol.
9. (a) 3° alcohol + Lucas reagent = Immediate
10. (a) (Mg powder + BaO2) acts as ignition mixture turbidity
Fe, Cr are obtained in molten state
Enthalpy of formation of Al2O3 is negative.
Solutions-Mock Test-3 MT-149

15. (c) PNH3 = PH 2S =


P X
=
dI2 4.56 ´ 10-3 ´ 5
2 2
\ = = 1.14 × 10–2 M/s
dt 2
2
æ Xö 19. (d) XeF6 + H 2 O ¾¾¾¾
Partial
® XeOF4 + 2HF
DG° = - RT ln K p = - RT ln ç ÷
è 2ø
Partial
XeF6 + 2H 2 O ¾¾¾¾
® XeO2 F2 + 4HF
æ Xö
= -2RT ln ç ÷ = – 2RT (ln X – ln 2)
è 2ø Compute
XeF6 + 3H 2 O ¾¾¾¾® XeO3 + 6HF
16. (c)
20. (b)
(NH ) S
(a) CdS ¯ +(NH 4 )2 S ¾¾
® CdS ¯ ¾¾¾¾¾
4 2 2 ® CdS ¯
COOK
Yellow Yellow Insoluble COOH
aq. KOH
Me – CH = CH2 + CHCl3 CH3CH – CH3
(b) SnS2 ¯ + (NH4 )2 S ¾¾
® SnS2 ¯ ¾¾¾¾¾
® (NH 4 )2 SnS3
(NH 4 ) 2 S2
CH3 – CH – CH2
Yellow Soluble CCl2

(NH ) S OH O
Cd 2 + + (NH4 )2 S ¾¾
HO
(c) 4 2 2 ® CdS ¯
® CdS ¯ ¾¾¾¾¾ Cl Cl OH
Yellow Insoluble C C
aq. KOH OH
CH3 – CH – CH2 CH3 – CH – CH2 CH3 – CH – CH2
(d) Sn 2+ + (NH4 ) 2S ¾¾® (NH ) S
4 2 2 ® (NH ) SnS
SnS ¯ ¾¾¾¾¾ 4 2 3
–H2O
Brown (Amm. thiostannate)
Soluble
21. (d) [HeH]+, M.O. configuration s1s 2
17. (c) B.O. = 1.0
e e f f H2+, M.O. configuration s1s1
A f A f A e A e
M
B.O. = 0.5
M M M
B D B C B D B C Higher the B.O. more is the stability.
C D C D 22. (a) Rate law r = k [A]m
e e f f
æ 0.3 ö
f f e e r1 = ç M / min from graph (exp. 1)
è 5 ÷ø
B B B B
M M M M
A D A C A D A C
C D C D æ 0.2 ö
r2 = ç M / min from graph (exp. 2)
e e è 5 ÷ø
A C A D m m
r1 3 ì [A]1 ü æ 1.5 ö
M M = =í ý = ç ÷ Þ m =1
B D B C r2 2 î[A]2 þ è 1ø
f f
0.3 1
= k (1.5) Þ k = min -1
dMnO -4 5 25
18. (b) Given - = 4.56 × 10–3 Ms–1
dt 1
r = k × [A] Þ ´ 1.8 = 0.072 M min –1
From the reaction given, 25
23. (d) NH4ClO4 + HNO3 (dilute) ® NH4NO3 +
1 dMnO 4 – 4.56 ´ 10 -3 HClO4
- = Ms -1
2 dt 2 (X)
NH4NO3 ¾¾¾ heat
® N2O + 2H2O
1 dMnO-4 1 dI2 (X) (Y)
- =
2 dt 5 dt 24. (d) 3CH4 + 2O3 ® 3CH2= O + 3H2O
Formaldehyde
5 dI dMnO-4
\ - = 2 CH2 = CHCH = O CH3COONO 2
2 dt dt
Acrolein Perxyacetyl nitrate(PAn)
On substituting the given value
EBD_7206
MT-150 JEE MAIN

25. (b) 7500


DT = = 300
40 30
+
– + 4 2
CH = O CO2 K
Conc. KOH Final temperature = 300 + 300 = 600 K
Cannizaro
CH = O 29. (a)
CH2OH
O O
(A) (B)
y x y x
26. (d) (a) The concentration of sodium C C
H + H
thiosulphate solution should always H z O H z O
be less than the concentration of the (more stable structure) (less stable structure)
potassium iodide solution. hence less contribution
(b) Freshly prepared starch solution Therefore, repulsion between both C – O bonds
should be used
will be greater than the repulsion between C – H
(c) Experiments should be performed with
the fresh solutions of H2O2 and KI. bond and C – OH bond or between C – H and C
27. (c) (a) = O bond. Repulsion between C – H and C = O
+4 +5 +6 0 bond is greater than that of between C – H and
SeO32- + BrO3- + H+ ® SeO42- + Br2 + H2O C– OH bond, because double bond character in
(b) C– OH bond will always be less than C = O.
+5 +3 -1 +5 30. (c) Cr2O72– + 14H+ + 6e– ® 2Cr3+ +
BrO3- + AsO-2 + H2O ® Br - + AsO34- + H+ 7H2O
In reaction (2) Initial conn 1 0.1 0
gm. eq. of BrO3– = gm. eq. of AsO2– After reaction 0.5 0.1 M 1M
12.5 1
n - ´6 = n - ´2 = ´ ´ 2 = 10-3 0.06 [Cr 3+ ]2
BrO3 AsO2 1000 25 E RP = E 0RP - log
6 [Cr2 O 72 - ] [H + ]14
10 -3
n = 0.06 1
BrO3- 6 E RP = 1.33 - log
6 (0.5) (0.1)14
In reaction (1)
moles of BrO3– consumed 0.06
= 1.33 – log (2 ´ 1014 ) = 1.187V
6
70 1 10 -3
= ´ - = 10 -3 PHYSICS
1000 60 6
gm eq. of SeO32– = gm. eq. of BrO3– 31. (c) For total internal reflection on face AC
n ´ 2 = 10 -3
´5 ; q > critical angle (C)
SeO32 - and sinq ³ sinC
n = 2.5 ´ 10 -3 1
SeO 32 - sin q ³
wm g
28. (a) Mole of X reacted = 0.75 mole
4
Heat liberated at constant pressure = 0.75 ×
mw
10 kJ = 7.5 kJ sin q ³ Þ sin q ³ 3
mg 3
1
Mole of X remaining = mole 2
4 8
3 2 1 \ sin q ³ .
Mole of Y formed = ´ = 9
4 3 2
æ1 1 ö
çè C p,m(X) + C p,m(Y) ÷ø ( DT) = q = 7500
4 2
Solutions-Mock Test-3 MT-151

32. (a) As we know, 2.303 ´ T1/ 2 N


-GMm = ´ Log10 0
Gravitational potential energy = 0.693 N
r
dN 0
and orbital velocity, v0 = GM / R + h N 20
But dt = 0 = = 10
1 GMm 1 GM GMm dN N 2
Ef = mv02 - = m -
2 3R 2 3R 3R dt
GMm æ 1 ö - GMm 2.303 ´ 5730
= ç - 1÷ = \ t= ´1
3R è 2 ø 6R 0.693
= 19039 years
-GMm
Ei = +K 35. (d) Acceleration produced in upward direction
R
F
Ei = E f a=
M1 + M 2 + Mass of metal rod
Therefore minimum required energy,
5GMm 480
K= = = 12 ms -2
6R 20 + 12 + 8
33. (a) Given, R1 = 100 W, r' = r/2, R2 = ? Tension at the mid point
rl æ Mass of rod ö
Resistivity of wire, R = Q Area × T = ç M2 + ÷a
A è 2 ø
length = volume = (12 + 4) × 12 = 192 N
rV 36. (b) If student will use angular momentum =
Hence, R = 2
A mvr.
Since, r ® constant, V ® constant He/she may conclude answer (a) as r is
1 decreasing angular momentum must
Rµ 2 decrease hence (a) is incorrect.
A
m
1
or R µ Q A = pr2 v
4 q
r mg r
R2
= 16 Þ R2 = 16 ´ 100 = 1600 W
R1 /////////////////////////////////////////
O
d
dN0
34. (c) Given: = 20 decays/min The magnitude of angular momentum of
dt particle about O = mvd
dN Since speed v of particle increases, its
= 2 decays/min angular momentum about O increases.
dt
Magnitude of torque of gravitational force
T1/2 = 5730 years
about
As we know, O = mgd Þ constant
N = N 0 e -lt Moment of inertia of particle about O = mr2
Hence MI of particle about O decreases.
N0 Angular velocity of particle about
Log = lt
N v sin q
O=
1 N r
\ t= Log 0 Q v and sin q increases and r decreases.
l N
\ angular velocity of particle about O
increases.
EBD_7206
MT-152 JEE MAIN

37. (c) Pressure difference is largest between 41. (a) The surface of the calorimeter is exposed
atmosphere and smaller bubbles. Hence radius to the air. So heat is lost from the surface by
of curvature (R) is smallest. radiation.
v
1æUö 42. (b) n 0 = ,
38. (a) As, P = ç ÷ 2l
3èVø
v v
U n1 = , n2 =
But = KT 4 2 ( l / 2 - Dl) 2 ( l / 2 + Dl)
V
Beat frequency = n 1 – n 2
1 4
So, P = KT é 1 1 ù
3 Þ vê -
ë l - 2Dl l + 2 Dl úû
uRT 1
or = KT 4 [As PV = u RT] (l + 2Dl) - (l - 2Dl) ù
V 3 = v éê ú
ë l 2 - 4 Dl 2 û
4 4D l 8 Dlv 8Dln 0
pR 3T 3 = constant =v 2 = =
3 l - 4Dl 2 l 2l l
1 43. (c) Wavelength for which maximum obtained at
Therefore, Tµ the hole has the maximum intensity on pass-
R
ing. So,
Ie ne eAve 7 7 ve v 5
39. (c) = Þ = ´ Þ e = nlD
Ih nh eAvh 4 5 vh vh 4 x=
d
40. (a) For particle C,
According to law of conservation of linear xd 1 ´ 10-3 ´ 0.5 ´ 10-3
l= =
momentum, verticle component, nD n ´ 50 ´ 10-2
2 mv' sin q = mv sin 60° + mv sin 45°
1 ´ 10-6 1000nm
mv mv 3 ...... (i) = =
2mv 'sin q = + n n
2 2
n = 1, l = 1000 nm ® Not in the given range
Horizontal component, n = 2, l = 500nm
2 mv' cos q = mv sin 60° – mv cos 45°
I C q
mv mv ...... (ii) 44. (b)
2mv'cos q = + I
I
2 2

Y A L
Y' I
B
v sin 60°

I I

As it can be easily seen by the direction of


30° v sin 45°
I that Q is decreasing thus, energy of
60° 45° capacitor is decreasing and hence, energy
X X'
v cos 60° – v cos 45° of inductance is increasing or æç 1 LI 2 ö÷
For particle A For particle B è2 ø
gives that I is increasing.
Dividing eqn (i) by eqn (ii),
1 2
45. (d) Distance from A to B = S = ft 1
1 3 2
+
2 2+ 3
tan q = 2 = Distance from B to C = (ft 1 ) t
1 1 1- 2
-
2 2
Solutions-Mock Test-3 MT-153

u2 (ft ) 2 1
Distance from C to D = = 1 Þ uy × 2 + (–10) ×4
2 a 2 (f / 2 ) 2
or, 50 = 2uy – 20
= ft 12 = 2S 70
or, uy = = 35m / s
A f B C f /2 D 2
t1 t 2t 1 u y 35 7
\ tan q = = =
u x 20 4
15 S
7
Þ S + f t1t + 2S = 15 S Þ Angle q = tan–1
4
Þ f t1t = 12 S ............. (i) 48. (a) The current voltage relation of diode is
1 2 I = (e1000 V /T - 1) mA (given)
f t1 = S ............ (ii)
2 When, I = 5mA, e1000 V /T = 6mA
t
Dividing (i) by (ii), we get t1 = 1000
6 Also, dI = (e1000 V /T

T
2 (By exponential function)
1 ætö f t2
Þ S= fç ÷ = 1000
2 è6ø 72 = (6 mA) ´ ´ (0.01) = 0.2 mA
300
46. (a) Capacitors 2mF and 2mF are parallel, their
equivalent = 4 mF 49. (d) The energy of the system of two atoms of
6mF and 12 mF are in series, their equivalent 1 2
= 4 mF diatomic molecule E = I w
2
Now 4mF (2 and 2 mF) and 8mF in series
where I = moment of inertia
3
= λF w = Angular velocity =
L
,
8 I
And 4mF (12 & 6 mF) and 4mF in parallel L = Angular momentum
= 4 +4 = 8mF
1
1 8 I = (m1r12 + m2 r22 )
8mF in series with 1mF = ∗1 Þ λF 2
8 9 1 2 2 2
8 8 32 Thus, E = (m1r1 + m2 r2 )w … (i)
2
Now Ceq = ∗ <
9 3 9
1 L2
1 1 9 32 E= (m1r12 + m2 r2 2 ) 2
With C, C < C ∗ 32 < 1 Þ C < 23 μF
2 I
eq L= nh
47. (c) From question, (According to Bohr's Hypothesis)
Horizontal velocity (initial),
40 1 L2
ux = = 20m/s E= (m1r12 + m2 r22 )
2 2 (m1r12 + m2 r22 )2
1 2
Vertical velocity (initial), 50 = uy t + gt 1 L2
2 E=
2 (m1r12 + m2 r22 )
EBD_7206
MT-154 JEE MAIN

n2 h 2 æ V0 ö
2/3
=
\ T = T0 ç = 4T0
8p2 (m1r12 + m2 r22 ) è V0 / 8 ÷ø
(m1 + m2 )n 2 h 2 Q
E= 53. (c) Flux going in pyramid = 2e
8 p2 r 2 m1m2 0

R 1R 2 Which is divided equally among all 4 faces


50. (b) Req = ( Parallel combination); Q
R1 + R 2
\ Flux through one face = 8e
Req = R1+R2(Series combination) 0
Req of the circuit 54. (b)
55. (b) A = 0.1m, m = 0.1 kg, KEmax = 18 × 10–3 J,
5 ´ 15 125 75 5
= + = + =5 W p
5 + 15 100 20 4 f=
4
R1 = 5 W
I1 36 ´ 10 -3
A k= = 3.6 ;
R 2 = 15W 4A R 3 = 1.25W (0.1) 2
4A 4 – I1 k 3.6
w= = = 6 rad / s
m 0.1
æ pö
\ Eqn. y = 0.1 sin ç 6t + ÷
E è 4ø
We know that I = = 20 = 4 A
R eq 5 56. (a) First, the length of wire goes on increasing
Potential difference across R1 and R2 are i.e., area decreases and finally at breaking
same stress the wire breaks.
(Parallel combination) I1R1 = (4 – I1)R2 57. (a) Force acting on conductor B due to
conductor A is given by relation
Þ 5I1 = (4 – I1) ´ 15 Þ I1 = 12–3I1 m 0 I1 I 2 l
Þ I1 = 3A F=
2 pr
Thus reading of ammeter = 3A l-length of conductor B
Voltage across 1.25W = I ´ R = 4 ´ 1.25 = 5V r-distance between two conductors
(Reading of voltmeter)
51. (a) Optical source frequency 4p´ 10-7 ´ 10 ´ 2 ´ 2
\F= = 8 × 10–5 N
c 2 ´ p´ 0.1
f = = 3 ´ 108 /(800 ´ 10-9 ) = 3.8 ´ 1014 Hz
l 58. (a) O r
Bandwidth of channel (1% of above) = 3.8
× 1012Hz a
Number of channels = (Total bandwidth of F1
R y Bowl
channel) / (Bandwidth needed per channel)
Number of channels for audio signal A h
12 3 8
= (3.8 ´ 10 ) /(8 ´ 10 ) ~ 4.8 ´ 10 mg cos a
52. (a) Let initial temperature and volume be T0 mg sin a
mg
and V0.
Since the process is adiabatic, the first The insect crawls up the bowl upto a certain
temperature and volume is height h only till the component of its
weight along the bowl is balanced by
TV g -1 = T0 V0g -1 (g = 5/3 for monoatomic limiting frictional force.
gas) For limiting condition at point A
Solutions-Mock Test-3 MT-155

R = mg cosa ...(i) 10 55 - q 0
F1 = mg sina ...(ii) =
8 46 - q 0
Dividing eq. (ii) by (i)
460 - 10q0 = 440 - 8q0
1 F
tan a = = 1 = m [ As F1 = mR ] 2q0 = 20
cot a R
q0 = 10°C
1é 1 ù
Þ tan a = m = Q m = ( Given ) ú
3 êë 3 û MATHEMATICS
\ cot a = 3
æ 1 3ö
61. (d) Centre of the circle is çè , - ÷ø .
I I 2 2
59. (b) T = 2p = 2p where
M ´B MB Its distance from the line x + y – 1 = 0 is 2
1 Let the required line be mx – y = 0
I= ml 2
12 m 3
+
When the magnet is cut into three pieces \ 2 2 = 2 Þ m = 1, –1/7
the pole strength will remain the same and
m2 + 1
1 æ mö æ l ö I
M.I. (I¢) = çè ÷ø çè ÷ø ´ 3 = \ The lines are x – y = 0, x + 7y = 0
12 3 3 9
We have, Magnetic moment (M) 62. (a) Since a1, a2 and b1, b2 are the roots of ax2 +
= Pole strength (m) × l bx + c = 0 and px2 + qx + r = 0 respectively,
therefore
\ New magnetic moment,
-b c
æ lö a1 + a2 = , a1a2 = ...(1)
M ' = m ´ ç ÷ ´ 3 = ml = M a a
è 3ø
-q r
T2 and b1 + b2 = , b1b2 = ...(2)
\T'= = s. p p
9 3
Since the given system of equation has a non-
60. (b) By Newton’s law of cooling trivial solution
q1 - q2 é q + q2 ù
= -K ê 1 - q0 ú a1 a 2
t ë 2 û \ = 0 i.e. a1b2 – a2b1 = 0
b1 b 2
where q0 is the temperature of surrounding.
Now, hot water cools from 60°C to 50°C in a1 a 2 a1 + a 2 a1a 2
10 minutes, or = = =
b1 b 2 b1 + b 2 b1b 2
60 - 50 é 60 + 50 ù
= -K ê - q0 ú ...(i)
10 ë 2 û pb pc b 2 ac
Þ = Þ =
Again, it cools from 50°C to 42°C in next 10 qa ra q 2 pr
minutes. 63. (b) Let f (x) = sin x + x cos x
50 - 42 é 50 + 42 ù Consider g (x) =
= -K ê - q0 ú ...(ii) x
10 ë 2 û
ò (sin t + t cos t) dt = t sin t]0 = x sin x
x
Dividing equations (i) by (ii) we get
0
1 55 - q0 g (x) = x sin x which is differentiable
=
0.8 46 - q0 Now, g (0) = 0 and g (p) = 0, using Rolles
Theorem
$ atleast one c Î (0, p) such that g '(c) = 0
i.e. c cos c + sin c = 0 for atleast one c Î (0, p)
EBD_7206
MT-156 JEE MAIN

64. (b) two places for 3 and 4 in 7C2 ways and from
y the remaining 5 places select any two for 5
and 6 in 5C2 ways
y = –x B (0, 2) y=x Now, the remaining 3 digits can be filled in
3! ways
(–1, 1) A C (1, 1) \ Total ways = 9C2 . 7C2 . 5C2 . 3!
9! 7! 5!
x = . . .3! = 9.7!
O 2!.7! 2!.5! 2!.3!
16

We first draw the given curves


66. (b) I= ò f (t) dt
0
The first curve x 2 - y 2 = 0 Þ y = ± x x4
represents a pair of straight lines with slopes 1 Consider g(x) = ò f (t) dt Þ g(0) = 0
and –1 passing through origin. The second curve 0
LMVT for g in [0, 1] gives, some a Î (0, 1)
Þ x 2 + y - 2 = 0 Þ x 2 = -y + 2
g(1) - g(0)
such that = g ¢(a ) ..... (1)
Þ x 2 = -( y - 2) 1- 0
represents a parabola with vertex (0,2) axis as y– Similarly, LMVT in [1, 2] gives, some b Î
axis and concavity dawnwards (see the chapter g(2) - g(1)
of parabola in coordinates). Both the curves are (1, 2) such that = g ¢ (b) .... (2)
2 -1
plotted in the figure and the required area is Eq. (1) + Eq. (2)
shown by the shaded region.
g ¢(a ) + g ¢ (b) = g(2) - g(0) ; but g' (x) = f
The points A and C are the points of intersection {
zero
of y 2 = x 2 with x 2 + y - 2 = 0 . (x4) . 4x3
Solving the two equations, we get 16

y 2 + y - 2 = 0 [putting value of x2 = y2]


\ 4 éëa 3f (a 4 ) + b3f (b4 ) ùû = ò f (t) dt
0
Þ ( y + 2)( y - 1) = 0
3u 2 2u 3 1
giving y = –2 and 1, but y = –2 is discarded as
2 3
the required area is above the x-axis. 3v 2v 1 =0
67. (a)
2 3
\y = 1 Þ x = ± 1 3w 2w 1
The points A and C are respectively (–1, 1) and
R1 ® R1 – R2 and R2 ® R2 – R3
(1, 1) now due to symmetry
Area of the bounded region OABCO u+v u 2 + v 2 + vu 0

[ ]
1 2 2
Þ v+w v + w + vw 0 = 0
= 2 × Area OBCO = 2x ò ( 2 - x 2 ) - x dx
0 w2 w3 1
[Since y = 2 –x2 is the upper curve and y = x is the R1 ® R1 – R2
lower curve]
1 u+w+v 0
1
é x3 x 2 ù é 1 1ù 7 2 2
= 2ê2x - - ú = 2 ê2 - - ú = Þ v+w v + w + vw 0 = 0
êë 3 2 úû ë 3 2û 3 2
0 w w3 1
65. (a) Number of digits are 9 Þ (v2 + w2 + vw) – (v + w) [(v + w) + u] = 0
Select 2 places for the digit 1 and 2 in 9C2 Þ uv + vw + wu = 0
ways from the remaining 7 places select any
Solutions-Mock Test-3 MT-157

68. (d) x Î [–1, 0]


1 1
1+ x 2 = ò x
dx – ò
x + x7
dx
x+ = -2x
2 = ln | x | – p(x) + c
x2 + 6x + 1 = 0 r
72. (b) n = 7iˆ + 2jˆ - kˆ is normal to plane
x = 2 2 - 3 Þ |10a | = [| 20 2 - 30 |] = 30 - 20 2
(Assuming n = aiˆ + bjˆ + ckˆ and using
x Î [0, 1]
r uuur r uuur r uuur
1 + x2 n.AB = 0, n.BC = 0, n.AC = 0 )
x+ = 2x P = (8, 2, – 12)
2
1 + x2 = 2x Þ x = 1 Þ | 10a | = 10 P(8,2,–12)

| 10a | = 10, | 20 2 - 30 | n

Þ [ | 10a | ] = 1, 10
69. (b) Let y = mx be any line represented by the
equation
ax3 + bx2y + cxy2 + dy3 = 0 A(2,–1,4)
Þ ax3 + bx2(mx) + cx (m2x2) + dm3x3 = 0 uuur
Þ a + bm + cm2 + dm3 = 0 which is a cubic AP = 6iˆ + 3jˆ - 16kˆ
equation. \ Distance
It represents three lines out of which two uuur r
are perpendicular hence AP.n 42 + 6 + 16 64 64 64 6 32 6
d= r = = = = =
|n| 49 + 4 + 1 54 3 6 18 9
a a
m1m2 = –1 and m1m2m3 = - Þ m3 = 73. (a)
d d r r r r r r r r r
and m3 is the root of the given equation V = A ´ éë (A.B)A - (A.A)B ùû .C
2 3
æ aö æ aö æ aö æ r r r r r r r rö r r r r r
Hence, a + b ç ÷ + c ç ÷ + d ç ÷ = 0 = çA ´(A.B)A- (A.A)A´B÷.C = – | A |2 [A B C]
è dø è dø è dø ç 14243 ÷
2 2 è zero ø
Þ d + bd + ca + a = 0
r 2
70. (c) x2 – 2x cos q + 1 = 0, Now, | A | = 4 + 9 + 36 = 49
\
2 3 6
2 cos q ± 4 cos 2 q - 4 r r r
x= = cos q ± i sin q [A B C] = 1 1 -2
2 =7
1 2 1
Let x = cos q + i sin q
\ x2n – 2xn cos nq + 1 r 2 r r r
\ - | A | [A B C] = 49 ´ 7 = 343
= cos 2nq + i sin 2nq – 2 (cos nq + i sin nq)
cos nq + 1 74. (a)
= cos 2nq + 1 – 2 cos2 nq + i (sin 2nq – 2 sin 3p /4 3p /4
nq cos nq) I = ò (sin x + cos x) dx + ò x{ (sin x - cos x) dx
14243
=0+i0=0 0 0 I II

x6 x6 3p /4
71. (a) ò x+ x 7
dx = ò
x(1 + x ) 6
dx
= ò (sin x + cos x) dx + x(- cos x - sin x) |30p /4
144424443
0 zero
6
(1 + x ) - 1
= ò x (1 + x 6 )
dx
EBD_7206
MT-158 JEE MAIN

3p / 4 P(E) = P(R R B W or B B R W or W W R B)
+ ò (sin x + cos x) dx
n (E) = 6C2 · 5C1 · 4C1 + 5C2 · 6C1 · 4C1 +
0 4C · 6C
2 1 · 5C1
3p / 4
n (S) = 15C4
=2 ò (sin x + cos x) dx = 2 ( 2 + 1)
0 720 × 4! 48
75. (a) A divides C1C2 externally in the ratio 1 : 3. \ P (E) = =
15 ×14 ×13 × 12 91

1 3 1
A(–3,0) q 80. (b) z + = 2 cos q
C1(–1,0) B C2(3,0) z
Þ z2 – 2 cos qz + 1 = 0
\ coordinate of A are (–3, 0)
We have sin q = 1/2 \ q = 30° 2cos q ± 4 cos 2 q - 4
Þ z= = cos q + i
Area = 3 × 3 tan 30° = 3 3 2
76. (c) Given circles are x2 + y2 + 4x – 6y + 9 = 0 sin q
Taking positive sign,
and x2 + y2 – 5x + 4y – 2 = 0
\ locus of centres is z 2 n - 1 2i sin nq
\ = = i tan nq
9x – 10y + 11 = 0 z 2n + 1 2 cos nq
1 1 1 1 Taking negative sign, we get
77. (c) + + + + .....,
1 1 + 2 1 + 2 + 3 1+ 2 + 3 + 4 z 2 n - 1 -2i sin nq
= = –tan nq
z 2n + 1 2 cos nq
1 2 é1 1 ù
tn = = = 2ê - z 2n - 1
1 + 2 + 3 + .....n n(n + 1) ë n n + 1 úû Þ = | ± i tan q| = tan nq
z 2n + 1
\ Sum =
81. (a) Two curves cuts at right angle if product of
éæ 1 1 ö æ 1 1 ö æ 1 1 ö ù their slopes is –1.
S = 2 êç - ÷ + ç - ÷ + ç - ÷ + .......ú = 2
ëè 1 2 ø è 2 3 ø è 3 4 ø û Two given curves are
78. (b) x3 – 3xy2 + 2 = 0 ..... (i)
N
2
P(at , 2at) and 3x2y – y3 – 2 = 0 ...... (ii)
R(h,k) Differentiate equ. (i),
O S(a,0) 2 2
dy 3(x - y )
Þ m1 = =
dx 6xy
T : ty = x + at2 ..... (1) Differentiate equ. (ii),
Line perpendicular to (1) through (a, 0)
dy -2xy
tx + y = ta ..... (2) Þ m2 = =
dx (x 2 - y 2 )
2
Equation of OP : y -x =0 ...... (3)
t (x 2 - y 2 ) –2xy
From equations (2) and (3) eliminating t we \ m1 ´ m 2 = ´
get locus 2xy (x 2 - y 2 )
6R Þ m1 ´ m 2 = -1
79. (a) Box 5B
4W
Solutions-Mock Test-3 MT-159

82. (d) \ ( x n - 1) = ( x - 1) ( x - w ) ( x - w 2 )
......( x - w n -1 )
q
3q
xn -1
Þ( x - w ) ( x - w 2 )......( x - w n -1 ) =
x -1
Taking lim on both side
Let one angle be q then other = 3q x ®1
lim ( x - w ) ( x - w 2 )......( x - w n -1 )
Clearly q + 3q = 180 Þ q = 45o x ®1
\ Angle between the diameters represented xn - 1
by combined equation = lim
x ®1 x - 1
-
ax 2 + 2 ( a + b ) xy + by 2 = 0 is 45o Þ (1 - w )(1 - w 2 )......(1 - w n 1) = n
86. (d) Let th e equation of tangent
2
2 h - ab
\ Using tan q = y= mx + a 2 m 2 + b 2
a+b
Foci º (± ae, 0), vertices º (± a, 0), C º (0, 0)
2 a 2 + b2 + ab
Þ1= mae + a 2 m 2 + b 2
a+b \s= ,
1 + m2
(
Þ ( a + b) = 4 a 2 + b2 + ab
2
)
- mae + a 2 m 2 + b 2
Þ 3a 2 + 3b 2 + 2 ab = 0 s¢ =
83. (a) 1 + m2

p q p Ù q p Ú q ~ (p Ú q) (p Ù q) Ù ~ (p Ú q)
ma + a 2 m 2 + b 2
T T T T F F a= ,
T F F T F F 1 + m2
F T F T F F
F F F F T F -ma + a 2 m 2 + b 2
a¢ = ,
1 + m2
\ (p Ù q) Ù (~ (p Ú q)) is a contradiction.

a 2 m 2 + b2
c=
84. (c) 1 + m2

m2 a 2e2
æ 3q ö æqö -
2 sin ç ÷ cos ç ÷ ss¢ - c 2
1 + m2 = e2
sin q + sin 2q è 2 ø è 2 ø = tan æ 3q ö \ =
= ç ÷ aa ¢ - c 2
m2a 2
cos q + cos 2q æ 3q ö æ qö è 2 ø -
2 cos ç ÷ cos ç ÷
è 2 ø è2ø 1 + m2
2p 87. (c) A: blood result says positive about the
Hence period =
3 disease
85. (c) Since, 1, w , w2 , ................. wn -1 are the n B1: Person suffers from the disease
roots of unity 1
\ P(B1) =
n 100
Consider x = 1
EBD_7206
MT-160 JEE MAIN

99 p
B2 : person does not suffer \ P(B2) = then Ð APB = Ð AQB = does not formed.
100 2
99 1 \ m¹ ±2
P(A/B1) = , P(A/B2) = 89. (d) (a) Tr (kA) = k (a11 + a22 + a33) = kTr(1)
100 100
P(B1 ).P(A / B1 ) (b) Tr (A + B) = a11 + b11 + a22 + b22 + a33
P (B1 / A) = + b33 = Tr (1) + Tr (2)
P(B1).P(A / B1 ) + P(B2 ).P(A / B2 )
(c) Tr (I3) = 1 + 1 + 1 = 3
1 99
× (d) Tr(A2 )
100 100 99 1
= = = = 50%
1 99 99 1 2 × 99 2 2 2
× + × = S a11 + S a12 ¹ (a11 + a 22 + a 33 ) 2
100 100 100 100
p /2

88.
p
(a) Since, Ð APB = Ð AQB = so y = mx + 8 90. (a) an = ò (1 - sin t) n sin 2t dt
2 0
intersect the circle whose diameter is AB. Let 1 – sin t = u Þ – cos t dt = du
Equation of circle is x2 + y2 = 16 CD < 4 1 æ1 1 ö
= 2ò u n (1 - u) du = 2 ç ò u n du - ò u n +1du ÷ = 2 æç 1 - 1 ö÷
Q è0 ø è n +1 n + 2ø
0 0
D
an æ 1 1 ö
P A Hence, n = 2 çè n(n + 1) - n(n + 2) ø÷
C
B
n
an æ æ1 1 öö 1 æ 1 1 ö
lim å = 2 çåç - ÷÷ - åç - ÷
8 n ®¥ 1 n è è n n +1 ø ø 2 è n n + 2 ø
Þ < 4 Þ 1 + m2 > 4
1+ m 2 æ n 1 1 öö n æ 1
= 2 ç å æç -
1 ö
ç è n n + 1 ÷ø ÷÷ å çè n n + 2 ÷ø
- -
Þ m Î (-¥, - 3) È ( 3, ¥) è 1 ø 1
If the line passing throw the point A (–4, 0), éæ 1ö æ1 1ö æ 1 1ö ù 3 1
= 2(1) - êç 1 - ÷ + ç - ÷ + ç - ÷ + .....ú = 2 - =
B (4, 0) ëè 3 ø è 2 4 ø è 3 5 ø û 2 2
Mock Test-4
ANSWER KEY
1 (a) 16 (b) 31 (c) 46 (d) 61 (c) 76 (a)
2 (b) 17 (d) 32 (b) 47 (c) 62 (b) 77 (c)
3 (b) 18 (b) 33 (c) 48 (b) 63 (c) 78 (c)
4 (a) 19 (c) 34 (b) 49 (d) 64 (c) 79 (c)
5 (b) 20 (c) 35 (a) 50 (a) 65 (b) 80 (a)
6 (b) 21 (d) 36 (a) 51 (a) 66 (c) 81 (b)
7 (b) 22 (c) 37 (d) 52 (c) 67 (b) 82 (c)
8 (d) 23 (a) 38 (d) 53 (b) 68 (d) 83 (c)
9 (d) 24 (b) 39 (b) 54 (a) 69 (b) 84 (c)
10 (c) 25 (b) 40 (c) 55 (b) 70 (c) 85 (d)
11 (c) 26 (b) 41 (c) 56 (d) 71 (a) 86 (b)
12 (a) 27 (b) 42 (b) 57 (a) 72 (d) 87 (b)
13 (d) 28 (b) 43 (c) 58 (d) 73 (d) 88 (b)
14 (a) 29 (b) 44 (a) 59 (c) 74 (d) 89 (a)
15 (d) 30 (b) 45 (b) 60 (a) 75 (d) 90 (c)

Solutions
CHEMISTRY 2
4. (a) Given P = nRT - a æç n ö÷
v - nb è vø
heat 1
1. (a) ZnO ¾¾¾
® Zn2+ + O + 2e - Which can also be written as
2 2
Excess Zn 2+ ions are trapped by vacant é n2 a ù
ê P + 2 ú (V - nb ) = nRT
interstitial sites and the electrons are in êë V úû
neighbouring interstitial sites. At low pressure and high temperature the
2. (b) The molecule 2,3 - pentadiene does not
have any chiral C but at the same time it a
effect of and b is negligible hence PV
does not have any mirror plane which makes V2
the molecule chiral. = nRT.
CH3 CH3 CH 3 CH3
5. (b)
Br Br Br

3. (b) Br 2
Sn/HCl NaNO2

+(CH3 ) 2 CHOH ¾¾¾¾¾¾¾®


FeBr3 HCl

NO2 NO2 NH2 CH3 N = NCl


[(CH3 )2 CHO]3 A1
Br HBr CuBr O

Br

+ (CH3)2C = O
HO
EBD_7206
MT-162 JEE MAIN

6. (b) (i) n = 5 means l = 0, 1, 2, 3, 4


16. (b) s0 = K sp ; s1 = K sp / 0.02 M;
since m = + 1
hence total no. of electrons will be s2 = Ksp / 0.01 M; s3 = K sp / 0.05 M
= 0(from s) + 2(from p) + 2(from d) + 2(from f) Obviously s0 > s2 > s1 > s3
+ 2(from g)
17. (d)
=0+2+2+2+2=8
H O
(ii) n = 2, l = 1, ml = –1, ms = –1/2 represent OH
2p orbital with one electron H3C C N H3C C N
7. (b) 1g equivalent of B reacts with 1 g equivalent O
H O
(i.e. ‘a’ g) of A. H
1 g equivalent of B will also react with 1g Rearrangement
equivalent (i.e. 2c g) of C. 18. (b)
Therefore, ratio of gm equivalents of A and C6H5 NH2 + NaNO2 + HCl ¾¾¾
®C6H5N = NCl
0°C
C is a : 2c
8. (d) In N2+ , there is one unpaired electron hence OH
it is paramagnetic. N 2 Cl + ¾
¾®
9. (d) In Ag2O (O.N. of Ag +1) in Ag the O.N. is
O. There is gain of electrons, hence H2O2 OH
is lacting as reducing agent.
N=N
10. (c) We know that
H2 O CH2 – CH 2 Red dye
H 2C – CH 2+RMgX CH 2 – CH2
–Mg(OH)X
O OH R
19. (c) P° - PS w / m ; (640–600)/640 = wM/mW
OMgX R =
P° W/M
11. (c) Tertiary amine do not have hydrogen
40/640 = 2.175 ´ 78/m ´ 39.08
atom to replace Cl. Hence it will not form
m = 2.175 ´78 ´640 / 39.08 ´ 40 = 69.458 @ 69.60
an amide with acid chloride.
20. (c) Multiple bonds formation tendency with
12. (a) No. of dipeptides = 2n ; n= no. of amino carbon and nitrogen decreases from
acids sulphur to tellurium.
i.e., 22 = 4 dipeptides can be formed. CS2 (S = C = S) is moderately stable,
13. (d) Total energy of a revolving electron is the CSe2 (Se = C = Se) decomposes readily
sum of its kinetic and potential energy. whereas,
Total energy = K.E. + P. E. CTe2 (Te = C = Te) does not exist
21. (d) Liquation is the principle based on
e æ e
2 2ö 2 difference in melting points.
= +ç- ÷ = - e 22. (c) In NO2+ odd (unpaired) electron is removed.
2r è r ø 2r In peroxides (O22–) no unpaired electrons
are persent as the antibonding pi M.O.’s
14. (a) acquired one more electron each for pairing.
15. (d) Tertiary halides on treatment with base, AlO-2 containing Al3+ (2s2p6)
such as sodium methoxide, readily undergo configuration and 2 oxides (O2–) ions each
elimination resulting in the formation of of which does not contain unpaired
alkenes. (Williamson’s Synthesis) electron. Superoxide O2– has one unpaired
CH3 electron in pi antibonding M.O.
| +– E2
CH3 — C — Cl + N a O CH2CH3 CH3 — C = CH2
| |
CH3 CH3
Solutions-Mock Test -4 MT-163

23. (a) The two solutions are isotonic hence there • The increasing order of size of
will be no movement of H2O. hydrated ions of alkali metals is Li+ >
24. (b) Nickel salts reacts with dimethyl glyoxime Na+ > K+ > Rb+ > Cs+
in presence of NH4OH to give red ppt. of • Cesium used in photoelectric cells due
nickel dimethyl glyoxime. to its low I.E. Hence statements (b) is
the only choice correct.
CH3 – C = NOH 30. (b) (i) A3+ + e– ––––® A2+, DG1 = – 1 F y2
2 + NiCl2 + 2NH4OH
CH3 – C = NOH
(ii) A2+ + 2e– ––––® A, DG2 = –2F(–y1)
= 2Fy1
Add, (i) and (ii), we get
OH O A3+ + 3e– ––––® A ;
CH3 – C = N N = C – CH3 DG3 = DG1 + DG2
Ni + 2NH4Cl + 2H2O –3FE° = –Fy2 + 2Fy1
CH3 – C = N N = C – CH3 –3FE° = –F (y2 – 2y1)
O OH y2 - 2 y1
E° =
Nickel dimethylglyoxime (red ppt.) 3
PHYSICS
25. (b) Hydrated CoCl2. 6H2O is pink coloured and
contains octahedral [Co(H2O)6]2+ ions. If 31. (c) 4 m/sec2 2 m/sec2
this is partially dehydrated by heating, then Car Bus
blue coloured tetrahedral ions
[Co(H2O)4]2+ are formed. 200 m
Given, uC = uB = 0, aC = 4 m/s2, aB = 2 m/s2
[Co(H 2 O) 6 ]2+ ƒ [Co(H 2 O)4 ]2 + + 2H 2O hence relative acceleration, aCB = 2 m/sec2
pink blue 1
Now, we know, s = ut + at 2
2
é H 3 O+ ù é HCO 3- ù 1
K= ë ûë û As pH = 6.0[H O]+ 2
26. (b) 3
200 = ´ 2t Q u = 0
[ 2 ][ 2 ]
CO H O
2 2
Hence, the car will catch up the bus after
=10–6 time

é H 3 O+ ù é HCO 3- ù t = 10 2 second
K=ë ûë û
(H2O is in excess, 32. (b) We have, F = kx
[ CO 2 ][ H 2 O]2
where, F, x and k are force, length and
constant respectively.
therefore its conc. remains constant)
\ 5 = kx ......(1)
éHCO 3- ù and 7 = ky ......(2)
ë û K 3.8 ´ 10 -6 Multiplying eq (2) by 2
= = = 3.8
[CO 2 ] é H 3 O+ ù 10-6 14 = 2ky ......(3)
ë û Subtracting eq (1) from (3),
14 – 5 = 2ky – kx or 9 = k(2y – x)
27. (b)
Hence, required length = 2y – x
28. (b)
29. (b) • Li does not form peroxide or 33. (c) f = 500 Hz
A 4 m/s C B
superoxide due to it small size.
• Solubility of carbonates and Listener
biocarbonates increases on moving
down the group.
EBD_7206
MT-164 JEE MAIN

Case 1 : When source is moving towards æ v rel ö


stationary listener 38. (d) w rod = w point = ç
è r ÷ø
,
æ v ö vrel represents the velocity of one point w.r.t.
apparent frequency h ' = h ç ÷
è v - vs ø other.
3v - v
æ 340 ö = 506 Hz = and ‘r’ being the distance between
= 500 ç ÷ r
è 336 ø them.
Case 2 : When source is moving away from 2v
the stationary listener =
r
æ v ö æ 340 ö 39. (b) When either of A or B is 1 i.e. closed then
h" = h ç ÷ = 500 ç ÷ = 494 Hz lamp will glow.
è v + vs ø è 344 ø In this case, Truth table
In case 1 number of beats heard is 6 and in
case 2 number of beats heard is 18 therefore Inputs Output
frequency of the source at B = 512 Hz.
A B Y
34. (b) In the given equation [r] = [b][x];
\[b] =[r]/[x]. But r is mass per unit length and 0 0 0
x is distance, therefore [b]=ML–1/L=ML–2T0
35. (a) Work done in going from a distance r 1 to a 0 1 1
distance r2 away from centre of the earth, 1 0 1
by a body of mass m, is,
W = GMm (1/r1 – 1/r2), 1 1 1
For our case we should have
1/2 mv2 = GMm [ (1/Re) – (1/10Re)] This represents OR gate.
= (GMm/Re) ´ (9/10) 40. (c) Here the number of molecules is same.
v = Ö[(2GM/Re) ´ ( 9/10)] = Ö(9/10) ´ escape Hence,
velocity
T1 + T2 200 + 400
= Ö(9/10) ´ 11.1 km/s = 10.6 km/s Tfinal = = = 300 K
36. (a) The angle for which the ranges are same is 2 2
complementary. l
Let one angle be q, then other is 90° – q 41. (c) As we know, time period, T = 2p
g
2u sin q 2u cos q When additional mass M is added then
T1 = , T2 =
g g l + Dl
TM = 2p
g
4u 2 sin q cos q
T1T2 = =2R
g l + Dl 2
TM = æ TM ö l + Dl
l or ç ÷ =
è T ø l
u 2 sin 2 q T
(Q R = )
g 2
æ TM ö Mg
or, ç =1+
Hence it is proportional to R. è T ÷ø Ay
37. (d) mg = 2TL Þ pr2 Ldg = 2TL Þ pr2 dg = 2T..
é Mgl ù
êQ Dl = Ay ú
This relation is independent of L.
ë û
Solutions-Mock Test -4 MT-165

1 éæ TM ö
2 ù
A h
\ = êç ÷ - 1ú le 2m e eV 2m P e4V
y ëêè T ø ûú Mg \ = =
lP h 2me eV
42. (b) More the initial temperature more is the rate
2m P e4V
of cooling. Hence, T3 > T2 > T1
or mP
The rate of cooling decreases with decrease = 2
me
in temperature difference between body and
surrounding. m0 I m Isin q
47. (c) B= (sin q + sin q) = 0
43. (c) 1 1 2 3C + 2C a 4p a 2pa
= + =
C ¥ C ¥ +C C Ca ( C a + C ) B at any point on Y axis is inversely
proportional to a.
æ -1 + 3 ö
2C ¥ 2 +2CC ¥ -C 2 = 0 ® C ¥= Cç a Y
è 2 ÷ø
44. (a) From the graph, it is clear that for the same q q
value of load, elongation is maximum for
P O
X
wire OA. Hence OA is the thinnest wire –a/2 I a/2
among the four wires 48. (b) Power of source = EI = 240 × 0.7 = 166
45. (b) Resistance between P and Q
140
5 Þ Efficiency = Þ h = 83.3%
r´ r 166
æ r rö 6 = 5r
rPQ = r P ç + ÷ =
è 3 2ø 5 11 V0 100 V0 100
r+ r
6 49. (d) i R = = = 5 , iL = = = 10
R 20 X L 10
Resistance between Q and R
r 4 V0 100
´ r and i C = = =5
r r 4
rQR = P (r + ) = 2 3 = r XC 20
2 3 r 4
+ r 11
2 3 Current, i = i 2R + (iC - i L ) 2 = 52 + 52 = 5 2 amp.
Resistance between P and R 50. (a) Electric field intensity at the centre of the
r 3 disc.
´ r s
r ær ö 3 2 3
rPR = P ç + r÷ = = r E= (given)
3 2 è ø r 3 2 Î0
+ r 11
3 2 Electric field along the axis at any distance
Hence, it is clear that rPQ is maximum x from the centre of the disc
46. (d) Energy in joule (E) æ ö
= charge × potential diff. in volt s x
E' = ç1 - ÷
ç
2 Î0 ÷
Eelectron = qeV and Eproton = qp 4V è x2 + R 2 ø
h h From question, x = R (radius of disc)
de-Broglie wavelength l = =
P 2mE æ ö
\ E ' = s ç1 - R
÷
h h 2 Î0 çè R + R2
2 ÷
ø
le = lP =
2m e eV and 2m P e4V
s æ 2R - R ö
(Q qe = qP) = ç ÷
2 Î0 çè 2R ÷ø
EBD_7206
MT-166 JEE MAIN

4 Radioactivity at T1 is R1 = l N1,
= E
14 Radioactivity at T2 is R2 = l N2
\ % reduction in the value of electric field \ Number of atoms decayed in time
(T1–T2) = (N1–N2)
æ 4 ö
çè E - E÷ø ´ 100 R1 - R2 ( R1 - R2 )T
14 1000 =
= = % ; 70.7% =
l 0.693
E 14
i.e., a (R1 – R2)T
51. (a) f max = mmg, a max = mg. 57. (a) On comparing the given equation to
If A is the amplitude a max r
E = a0iˆ cos (wt – kz)
= Aw2 = 4p 2 AV 2 = mg . w = 6 × 108,
mg 2p w
Therefore, A = 2 2
. k= =
4p V r c
52. (c) Total time taken to travel distance d is :
w 6 ´ 108
k= = = 2 m -1
d d æ n +n2 ö d c 3 ´ 108
+ = dçç 1 ÷=
÷ n ;
2n 1 2n 2 è 2n 1 n 2 ø eff 58. (d) In the graph given, slope of curve 2 is
greater than the slope of curve 1.
3
n 2 = 3n 1 Þ n eff = n1 æ gP ö æ gP ö
2 çè ÷ø > çè ÷ø Þ g 2 > g1
V 2 V 1
53. (b) On the screen, we have four amplitudes pair
wise coherent. g He > g O2
(A1 + A2) + (A3 + A4) = A12 + A34 Since, g monoatomic > g diatomic
However, if A 12 and A 34 have equal Hence, curve 2 corresponds to helium and
magnitude because of random phase of A12 curve 1 corresponds to oxygen.
and A34, no fringes will be seen.
59. (c) O
nh h
54. (a) mvr = , l= ;
2p mv q Tcosq
nh
q
Using the two concept we get, mvr = q C Tsinq q
2p
x Fe
(where n = 1)
mg
1´ h
2p r = ....(1);
mv In equilibrium, Fe = T sin q
h mg = T cos q
l= ....(2)
mv
Fe q2
2pr h ´ mv 1 tan q = =
Divide (2) by (1), = = = 1:1 mg 4p Î0 x 2 ´ mg
l mv ´ h 1
55. (b) The process of changing the frequency of
a carrier wave (modulated wave) in q2
accordance with the audio frequency signal \ x=
4p Î0 tan q mg
(modulating wave) is known as frequency
modulation (FM). Electric potential at the centre of the line
0.693
56. (d) 1. l = 1/ 2 2. R = lN t V=
kq
+
kq
= 4 kmg / tan q
t x/2 x/2
Solutions-Mock Test -4 MT-167

60. (a) W = MB (cos q1 - cos q2 ) log x log y log z


(b) Let = = =k
b-c c-a a -b
= MB (cos 0° - cos 60°)
Þ log x = k (b - c), log y = k(c - a ),
1 MB
= MB(1 - ) =
2 2 log z = k(a - b)
MB
\ t = MB sin q = MB sin 60° = 3 = 3W \ x a . y b . z c = p k[ a (b -c))+ b (c - a )+ c(a - b)]
2
MATHEMATICS =p
k ( 0)
=1
61. (c) Given equation is where p is any arbitrary base of the log.
(p – q) x2 + (q – r) x + (r – p) = 0 (c) Given expression
By using formula for finding the roots = log xyz xy + log xyz yz + log xyz zx
2
- b ± b - 4ac = log xyz ( xy . yz . zx)
viz : , we get
2a
2 = log xyz ( x 2 . y 2 . z 2 )
( r - q) ± ( q - r ) - 4( r - p)( p - q)
x=
2( p - q) = 2 log xyz ( xyz) = 2 ´ 1 = 2

(r - q) ± (q + r - 2 p) r - p 64. (c) Set A = {a1, a2, ....., a20} has 20 distinct


Þ x= = ,1 elements.
2( p - q) p-q
We have to select 5-element subset.
62. (b) (a 2 + b 2 + c 2 ) p 2 - 2(ab + bc + cd ) p
\ Number of 5-element subsets = 20C5
+ b2 + c2 + d 2 £ 0
According to question
Þ (a 2 p 2 - 2abp + b 2 ) + (b 2 p 2 - 2 bcp + c 2 )
+ (c 2 p 2 - 2cdp + d 2 ) £ 0 5 4 (
20 C = 19 C .k
)
Þ (ap - b)2 + (bp - c) 2 + (cp - d) 2 £ 0 20! æ 19! ö
Þ = k. ç
Þ ap - b = 0, bp - c = 0 & cp - d = 0 5! 15! è 4! 15!÷ø
20
b c d Þ = k Þk=4
Þ = = Þ a, b, c and d are in G.P 5
a b c
Also ad = bc 65. (b) cos36°cos42°cos78°
63. (c) = cos36°cos(60° – 18°)cos(60° + 18°)

(a) log( a + 2 b) =
1
2
log( a + 2 b) 2 =
5 +1
(cos 2
60° - sin 2 18° )
4
1 2 2
= log(a + 4b + 4ab) é ö ù
2
2 æ 5 + 1ö æ
=ç ê1 - 5 -1 ú
÷ ê 4 çè 4 ÷ø ú
1 è 4 ø
= log(12ab + 4ab) ë û
2
1 æ 5 + 1ö 1 æ 5 + 1 ö æ 5 + 1 - 2 5 ö
= log( 2 4 . ab )
2
= ç ÷ -ç ÷ç ÷
è 4 ø 4 è 4 øè 16 ø
1
= (4 log 2 + log a + log b)
2
EBD_7206
MT-168 JEE MAIN

( )( ) 32 x 22 y
2
æ 5 + 1ö 5 +1 5 -1
+ = 32 + 22
=ç ÷- 3sec q 2 tan q
è 16 ø 64

Þ 3 x cos q + 2 y cot q = 32 + 22 ...(1)


é
( )

5 +1 ê 5 -1 ú
= 1- Similarly, Equation of normal to the
16 ê 4 ú
ëê ûú x2 y2
hyperbola - at point Q (3 sec f,
32 22
5 +1 é 4 - 6 + 2 5 ù 1
= ê ú= 2 tanf) is
16 ë 4 û 8
32 x 22 y
66. (c) The given expression is equal to + = 32 + 22
3sec f 2 tan f

cos(cos -1 x + sin -1 x + sin -1 x) Þ 3 x cos f + 2 y cot f = 32 + 2 2 ...(2)


æp ö 1 p p
= cos ç + sin -1 x ÷ = - sin(sin -1 x) = - x = - Given q + f = Þ f = - q and these
è2 ø 5 2 2
passes through (h, k)
-1 p
[Using cos x + sin -1 x = ] \ From eq. (2)
2
æp ö æp ö
p p
3 x cos ç - q÷ + 2 y cot ç - q÷ = 32 + 22
67. (b) a+b+ g = Þ a + g = – b. è2 ø è2 ø
2 2
Þ 3h sin q + 2k tan q = 32 + 2 2 ...(3)
æp ö
so that cot ( a + g ) = cot ç - b÷
è2 ø and 3h cos q + 2k cot q = 32 + 22 ...(4)
Comparing equation (3) & (4), we get
cot a cot g - 1 1
Þ = 3h cos q + 2 k cot q = 3h sin q + 2 k tan q
cot a + cot g cot b
3h cos q - 3h sin q = 2k tan q - 2k cot q
Þ cot a cot g - 1 = 2 Þ cot a cot g = 3.
3h(cos q - sin q) = 2k (tan q - cot q)
(since cot a + cot g = 2cot b)
3h(cos q - sin q)
68. (d) Let the coordinate at point of intersection
of normals at P and Q be (h, k) (sin q - cos q)(sin q + cos q)
Since, equation of normals to the = 2k
sin q cos q
h y p e r b o l a
-2k (sin q + cos q)
or, 3h = ...(5)
x2 y2 a 2 x b2 y sin q cos q
- = 1 At point (x1, y1) is +
a 2 b2 x1 y1 Now, putting the value of equation (5) in
= a2 + b2 therefore equation of normal to eq. (3)

x2 y2
the hyperbola - = 1 at point P (3 -2k (sin q + cos q)sin q
32 22 + 2k tan q = 32 + 22
sin q cos q
secq , 2 tanq) is
Solutions-Mock Test -4 MT-169

Þ 2k tan q - 2 k + 2k tan q = 13 72. (d)


(a) We have | AB | = | A | | B |
-13
–2k = 13 Þ k = Also for a square matrix of order 3,
2
Hence, ordinate of point of intersection | kA |= k 3 | A | because each element of the
-13 matrix A is multiplied by k and hence in this
of normals at P and Q is case we will have k3 common
2
69. (b) Let tanq 1 , tanq 2 be the roots of the \ | 3AB |= 33 | A || B |= 27( -1)(3) = -81
equation 2tan2q – 4tanq + 1 = 0. Thus
(b) Since A is invertible, therefore A–1 exists
tanq1+ tanq2 = 4/2 = 2; tanq1tanq2 = 1/2.
Now tan(q1+ q2) = [(tanq1 + tanq2)/ (1 – and AA-1 = I Þ det( AA-1 ) = det( I )
tanq1tanq2)]
Þ det(A ) det(A -1 ) = 1
= 2/[1 – (1/2)] = 4.
1 - wn 1
70. (c) We know, 1 + w + w 2 + ... + w n -1 = Þ det(A -1 ) =
1- w det(A)
æ np ö æ np ö
But wn = cos ç ÷ + i sin ç ÷ (c) (A + B) 2 = (A + B)(A + B)
è nø è nø
2 2
= cos p + i sin p = - 1 = A + AB + BA + B
2 2
p p p = A + 2AB + B if AB = BA .
and 1 - w = 2 sin 2 - 2i sin cos
2n 2n 2n 73. (d) Given system of equations can be written
in matrix form as AX = B where
æ pöé p pù
= - 2i sin ç ÷ ê cos + i sin ú
è 2n ø ë 2n 2n û æ1 2 3ö æ6ö
A = ç 1 3 5 ÷ and B = ç 9 ÷
Thus, 1 + w + w 2 + ... + w n -1 ç2 5 a÷ çb÷
è ø è ø
Since, system is consistent and has
æ cosp pö
iç -i sin ÷ infinitely many solutions
2 è 2n 2nø
= = \ (adj. A) B = 0
æ pöé p pù p
-2i sinç ÷ êcos +i sin ú sin
è 2nø ë 2n 2nû 2n æ 3a - 25 15 - 2a 1 ö æ 6 ö æ 0 ö
ç a - 6 -2 ÷ ç 9 ÷ = ç 0 ÷
Þ ç 10 - a
p p è - 1 -1 1 ÷ø çè b ÷ø çè 0 ÷ø
i cos sin
= 2n - i 2 2n = 1 + i cot (p / 2n) Þ – 6 – 9 + b = 0 Þ b = 15
p p
sin sin and 6(10 – a) + 9(a – 6) – 2(b) = 0
2n 2n Þ 60 – 6a + 9a – 54 – 30 = 0
71. (a) C1 (1, 0); C2 (0, –2) Þ 3a = 24 Þ a = 8
Hence, a = 8, b = 15.
r1 = 1 + 15 = 4, r2 = 4 - 3 = 1
ì – x + 1, x < 1
74. (d) f (x) = | x – 1| = í
C1C 2 = 1 + 4 = 5 î x – 1, x ³ 1
Consider f (x2) = (f (x))2
r1 - r2 = 3 Þ C1C 2 < r1 - r2
If it is true it should be " x
Hence, C2 lies inside C1.
EBD_7206
MT-170 JEE MAIN

\ Put x = 2 76. (a) We have,


LHS= f (22) = |4 – 1| = 3
RHS = ( f (2))2 = 1 é cos2 q cosq sinqùé cos2 f cosfsinfù
\ (a) is not correct AB= ê 2 úê 2 ú
Consider f (x + y) = f (x) + f (y) êëcosqsinq sin q úûêëcosf sinf sin f úû
Put x = 2, y = 5 we get
f (7) = 6; f (2) + f (5) = 1 + 4 = 5 écos 2 q cos 2 f + cos q cos f sin q sin f

\ (b) is not correct êë cos q sin q cos 2 f + sin 2 q cos f sin f
Consider f (| x |) = | f (x) |
Put x = – 5 then f (| –5 |) = f (5) = 4 cos 2 q cos f sin f + cos q sin q sin 2 f ù
ú
| f (– 5) | = | – 5 – 1| = 6 cos q cos f sin q sin f + sin 2 q sin 2 f úû
\ (c) is not correct.
Hence (d) is the correct alternative. écos q cos f cos q sin fù
= cos( q - f) ê ú
75. (d) Let a = tan q and b = tan f ë sin q cos f sin q sin f û

\ sin -1 é
2a ù -1 é 2 tan q ù Since AB = 0, \ cos(q - f) = 0
êë1 + a 2 úû = sin êë 1 + tan 2 q úû
p
\ (q - f) is an odd multiple of
= sin -1 [
sin 2q] = 2
77. (c) Let f image of goh image, fogoh = F (x) = f
2q = 2 tan -1 a [goh (x)]
é 2b ù é 2 tan f ù = f [g ( x + 3)] = f (cos x + 3)
and sin -1 ê 2ú
= sin -1 ê 2 ú
ë1 + b û ë1 + tan f û 2
F(x) =
-1 cos x + 3 + 1
= sin [ sin 2f] = 2f = 2 tan b
-1

Now x + 3 ³ 0 and -1 < cos x + 3 £ 1


é 2a ù
Thus, sin -1 ê = 2 tan–1 a and
ë 1 + a 2 úû
D
fogoh f
-1 é
2b ù
sin ê = 2 tan–1 b
ë 1 + b2 úû A
C
A>–3 goh
é 2a ù g
\ 2 tan –1 x = sin -1 ê h
ë 1 + a 2 úû B

é 2b ù
+ sin -1 ê x + 3 ¹ (2n - 1) p, n Î N
ë1 + b2 úû
= 2 tan–1 a + 2 tan–1 b 78. (c) Since | x | is not diff. at x = 0
Þ tan x = tan–1a + tan–1b
–1
|x – 1|
a+b
tan–1 x= tan–1
1 - ab 1
a+b
\ x=
1 - ab
x
1
Solutions-Mock Test -4 MT-171

Þ | x - 1 | is not diff at x = 1. Þ log (1 + y) = log (2 + sin x) + log C


Þ 1 + y = C (2 + sin x)
x n | x | in n times diff. at x = 0 Given y(0) = 2
3
Þ ( x - 1) 2 | x - 1 | is twice diff. at x = 1
Þ 1 + 2 = C[2 + sin 0] Þ C =
2
but not thrice diff. at x = 1 æ pö
Now, y çè ÷ø can be found as
1 2
79. (c) f(x) = is discontinuous at x =1.
x -1
3æ pö 9
( x - 1) 2 1+y= ç 2 + sin ÷ Þ 1 + y =
2è 2ø 2
(gof) (x) = g(f(x)) = - , which
( 2x - 1)( x - 2)
7
is not defined at x = 1/2, 2. Þ y=
Hence the set of points where (gof) (x) is 2
discontinuous is {1/2, 1, 2} æ pö 7
Hence, y çè ÷ø =
m
n+r
m
n+ r 2 2
80. (a) å Cn = å Cr
r =0 r =0 83. (c) Differentiating w.r.t.x, we get
1 1
(Q n + r Cn =n + rCn + r - n )
= 2f ( x )f ¢(x )
f ( x ) (f (x )) 2
= nC0 + n + 1C1 + n + 2C2 + n + 3C3 +......+ n +
mC 1 1
m Þ f ' (x) = Þ f ( x) = x + C
Using, nC0 = 1 = n + 1C1 2 2
= (n+1C0 + n + 1C1) + n + 2C2 + n + 3C3 +....+ where C is a constant.
n + mC
m 2a f (x)
Using, nCr + nCr+1 = n+1Cr+1 84. (c) I=ò dx ... (i)
0 f (x) + f (2a - x)
= (n + 2C1 + n + 2C2) + n + 3C3 +.....+ n + mCm
Using this again and again, we are left with f (2a - x)
2a
= n + mCm – 1 + n + mCm ÞI=ò dx ... (ii)
f (2a - x) + f (x)
0
= n + m + 1Cm = n + m + 1Cn + 1
Adding (1) and (2).
1
81. (b) As x ® ; {x + 1} ® {1 + 1 / 3} ® 1 / 3 2a f (x) + f (2a - x)
dx = [ x ]0
2a
3 2I = ò
0 f (x) + f (2a - x)
1 1
Similarly {x + 2} ® as x ® Þ I =a
3 3
p/4
Þ lim f (x) = lim
x - 1/ 3
=1 85. (d) I = ò (x | x |+ sin 3 x + x tan 2 x + 1) dx
x ®1/ 3 x ®1/ 3 x - 1/ 3 p/4 ¯ ¯ ¯
82. (c) Given differential equation is odd f odd f odd f
( 2 + sin x ) . dy = cos x p/4 p
(1 + y ) dx I= ò dx =
-p / 4 2
which can be rewritten as
dy cos x a
= dx
1 + y 2 + sin x ò
[Q f (x)dx = 0, as f (x) is an odd function]
Integrate both the sides, we get -a

dy cos x dx
ò =ò
1+ y 2 + sin x
EBD_7206
MT-172 JEE MAIN

86. (b) (1 – x – 2x2)6 = (1 + x)6 (1 – 2x)6


= 1+ a1x + a2x2 + …….. + a12x12 1 a b
Putting x = 1/2, we have 0 = 1 + a1/2 + a2/22 89. (a) Given, in DABC 1 c a = 0
+ a3/23 + a4/24 + ……..+ a12/212 .....(1) 1 b c
Putting x = –1/2, we have
1 = 1 – a1/2 + a2/22 – a3/23 + a4/24 – ….. + Þ 1(c2 – ab) – a(c – a) + b(b – c) = 0
a12/212....(2) Þ a2 + b2 + c2 – ab – bc – ca = 0
Adding (1) and (2), we have Þ 2a2 + 2b2 + 2c2 – 2ab – 2bc – 2ca = 0
1 = 2 (1 + a2/22 + a4/24 + …..+ a12/212) Þ (a2 + b2 – 2ab) + (b2 + c2 – 2bc)
Þ a2/22 + a4/24 + a6/26 …..+ a12/212 = –1/2 + (c2 + a2 – 2ca) = 0
Þ (a – b)2 + (b – c)2 + (c – a)2 = 0
x 2 y2
87. (b) y 2 = 4x & + =1 Here, sum of squares of three members can
8 2
be zero if and only if a = b = c
Equation of tangent to above curves are Þ DABC is equilateral.
respectively.
Þ ÐA = ÐB = ÐC = 60°
2 1
y = mx + and y = mx + 8m 2 + 2 \ sin 2 A + sin 2 B + sin 2 C
m

1 = (sin 2 60° + sin 2 60° + sin 2 60°)


Comparing = 8m 2 + 2
m
2
2 2 æ 3ö 9
Þ m (8m + 2) = 1 = 3´ ç =
ç 2 ÷÷ 4
è ø
seeing the options
90. (c) x = tanA, y = tanB, – z = tanC. Then (x + y –
1 z) = –xyz.
m=± satisfy the equation
2 Þ tanA + tanB + tanC = tanA tanB tanC
1 Þ A + B + C = p Þ 2A + 2B = 2p – 2C
Þy=± x ± 2 Þ 2y = ± x ± 4 Þ tan(2A + 2B) = tan(2p – 2C) = – tan2C
2
Þ tan2A + tan2B + tan2C
i.e. 2y = x + 4 & x + 2y + 4 = 0
= tan2A.tan2B.tan2C
88. (b) Let the point be ( x1, y1).
Therefore y1 = (x1 – 3)2 ...(i) 2 tan A 2 tan B 2 tan C
Þ + +
\ Now slope of the tangent at ( x1, y1) is 1 - tan A 1 - tan B 1 - tan 2 C
2 2
2(x1 – 3), but it is equal to 1.
2 tan A 2 tan B 2 tan C
= . .
Therefore, 2(x1 – 3) = 1 Þ x1 = 7 1 - tan A 1 - tan B 1 - tan 2 C
2 2
2
Put the value of tanA , tanB, tanC, we get
2
æ7 ö 1
y1 = ç – 3÷ = . 2x 2y 2z
è2 ø 4 Þ + -
2 2
1- x 1- y 1- z 2
æ 7 1ö
Hence the point is ç , ÷ .
è 2 4ø 8xyz
=-
(1 - x )(1 - y 2 )(1 - z 2 )
2
Mock Test-5
ANSWER KEY
1 (b) 16 (b) 31 (c) 46 (c) 61 (b) 76 (a)
2 (b) 17 (a) 32 (b) 47 (b) 62 (d) 77 (d)
3 (a) 18 (a) 33 (a) 48 (c) 63 (d) 78 (b)
4 (d) 19 (c) 34 (b) 49 (d) 64 (a) 79 (c)
5 (b) 20 (b) 35 (b) 50 (b) 65 (a) 80 (d)
6 (c) 21 (b) 36 (a) 51 (d) 66 (a) 81 (a)
7 (b) 22 (b) 37 (c) 52 (d) 67 (a) 82 (c)
8 (d) 23 (b) 38 (d) 53 (b) 68 (d) 83 (c)
9 (b) 24 (c) 39 (c) 54 (a) 69 (b) 84 (a)
10 (c) 25 (a) 40 (c) 55 (a) 70 (c) 85 (c)
11 (a) 26 (a) 41 (b) 56 (d) 71 (c) 86 (c)
12 (b) 27 (c) 42 (a) 57 (d) 72 (c) 87 (a)
13 (b) 28 (d) 43 (b) 58 (c) 73 (d) 88 (c)
14 (d) 29 (a) 44 (b) 59 (b) 74 (c) 89 (d)
15 (b) 30 (a) 45 (a) 60 (d) 75 (b) 90 (a)

Solutions

CHEMISTRY 5. (b) The more the reduction potential, the more


is the deposition of metals at cathode. Cation
1. (b) Correct order of increasing basic strength having E° value less than – 0.83 V (reduction
is NH3 > PH3 > AsH3 > SbH3 > BiH3 potential of H2 O) will not deposit from
2. (b) (i) n = 5 means l = 0, 1, 2, 3, 4 aqueous solution.
since m = + 1 6. (c) Number of p-bonds
hence total no. of electrons will be
O O
= 0(from s) + 2(from p) + 2(from d) + 2(from || ||
f) + 2(from g) H 2S2 O7 = HO - S - O - S - OH 4
=0+2+2+2+2=8 || ||
O O
(ii) n = 2, l = 1, ml = –1, ms = –1/2 represent
2p orbital with one electron O
3. (a) Aluminium has greater affinity for oxygen ||
and the reaction is highly exothermic. H 2SO4 = HO - S - OH 2
||
4. (d) Among alkaline earth metals, barium and O
radium h ave the tendency to form ××
peroxides. H 2SO3 = HO - S - OH 1
||
O
EBD_7206
MT-174 JEE MAIN

7. (b) DH = nCpDT = 0 means DH constant. 14. (d) In reaction (d) oxidation number changes
æ vf ö from + 4 in NO2 to + 3 in HNO2 and + 5 in
DS = nR ln ç ÷ ³ 0 DS increases. HNO3
è vi ø 15. (b) In neutral and alkaline medium
8. (d) Reaction between diborane and alkene are
carried out in dry ether under an atmosphere MnO 4- + 2H 2 O + 3e - ¾¾
® MnO 2 + 4OH -
of N2 because B2H6 and the products are In acidic medium:
very reactive. The products further treated MnO 4- + 8H + + 5e - ¾¾
® Mn 2 + + 4H 2 O
with alkaline H2O2 to convert into alcohols.
alkaline C
B2 H6 + 6C2 H 4 ¾¾
® B(C2 H4 )3 ¾¾¾¾
®
H 2O 2
reactive 16. (b) Ca , 6.4g of CaC2 contain p-electron
3CH3CH 2OH + H3BO3
C
9. (b) For fcc,
= 4NA
2a a 17. (a)
r= = = 0.3535a
4 2 2
given a = 361 pm CH2=CH – CH2– CH = CH2 O3 HOOC– CH–
2
COOH
+ 2HCOOH
r = 0.3535 × 361
= 128 pm Zn / H2O
2HCHO+OHC–CH– CHO
10. (c) Ions I and IV are the same (trans), with mirror
2 Reduction
plane through en groups.
18. (a) Because of the formation of the most
2- -1 2-
+
11. (a) Cr O 2 Cl2 ; Cr = +6; MnO 2 ; Mn = +4 stable carbonium ion, C6 H5 - C H 2
(Highest oxidation state) 19. (c) Let atomic masses of A and B be a and b
-1 -1 amu respectively
[Ni Cl 4 ]2 -; Ni = + 2; [Co Cl 4 ]2 - ;Co = +2
\ Molar mass of AB2 = (a + 2b) g mol–1
-1 -1 and Molar mass of AB4 = (a + 4b) g mol–1
[Fe(CN)6 ]3- Fe = +3; [Cu(CN) 4 ]2 - Cu = +2
For compound AB2
12. (b) The concentration of electrolyte, which DTb = Kb´ WB ´ 1000/ WA ´ MB
ionises in water shall be more although 80% 2.3 = 5.1 ´ 1 ´ 1000/ 20.0 ´ (a + 2b)....I
of it dimerises
For compound AB4
13. (b) Hg2 Cl 2 + 2 NH 4 OH ¾¾
® 1.3 = 5.1 ´ 1 ´ 1000/ 20.0 ´ (a + 4b)....II
(A) Solving (I) and (II), a = 25.49 b = 42.64
[H 2 N - Hg - Cl + Hg] ¯ + NH 4 Cl + 2 H 2 O 20. (b) 2NaI + 2 NaNO 2 + 4 CH 3COOH
(Black)
¾
¾® I 2 + 2NO +
HgCl 2 + 2 NH 4 OH ¾¾
® Hg(NH 2 )Cl+
(B) (White) 4 CH 3COONa + 2 H 2 O
NH 4Cl + H 2 O The colour of CCl4 layer turns purple due
to liberated I2.
HgCl2 + 2KI ¾¾
® HgI 2 ¯ +2KCl 21. (b) For bcc lattice, number of atoms per unit cell = 2
(Red) Now

HgI 2 + 2KI(excess) ¾¾
® K 2 (Hg I 4 ) n´M 2 ´ 100
d= =
-8
a ´ No (4 ´ 10 cm)3 ´ 6.02 ´ 1023
3

= 200/38.528 = 5.19 g/cc


Solutions-Mock Test -5 MT-175

22. (b) Entiomers of C4H10O are


H
OH HO
| |
H3CH 2C - C - CH3 H 3C - C - CH 2 CH 3 O O
| | HO
H
H
OH NH2
| HO
H 2SO 4
CH 3 CH 2- C H - CH 3 ¾¾ ¾¾®
A OH
CH 3 CH = CH - CH 3 + CH 2 = CHCH 2 CH 3
Major ( B ) Minor ( C ) O O
H2O
1
23. (b) In general t1/ 2 of reaction µ NH2
(a 0 )n -1 O–H

For a second order reaction, OH

1 1 1
t1/ 2 = = = = 10 min
K(a 0 ) n -1 K(a 0 ) 0.5 ´ 0.2 O O

H –H2O
O NH2
H +H2O
O O O OH
24. (c)

O NH2 25. (a) pOH = pK b + log


[salt]
[base]
OH
O O = - log 10 -10 + log 1 = 10; pH = 14 - 10 = 4
26. (a) Amines are basic due to the presence of a
HCl (g) / CCl4
O NH2
lone pair of electrons on nitrogen atom. The
lone pair can be easily donated.
OH
Mechanism : 27. (c) Sandmeyer’s reaction

NH2
HO HO O
NaNO2
HCl
O NH2

+ –
OH NH2 Cl Cl

CuCl
HO
OH O
+
+H
NH2
+ HO
–H
OH
EBD_7206
MT-176 JEE MAIN

28. (d) The strongest oxidising agent is one which 6´2


has maximum tendency to gain electrons, Hence, R AB = = 1.5W
6+2
i.e. whose E°Red is maximum
29. (a) C6H5Br + NaO – CH2 – CH = CH2 ¾¾ V = IR Þ 60 = I Þ I = 4A
® 15
C6H5 – O – CH2 – CH = CH2 + NaBr
3´3 9
Allyl phenyl ether [Q R eq = = = 1.5W]
30. (a) Alkyl or Aryl cyanide react with grignard 3+ 3 6
reagent to form ketones 33. (a) Here, Imax = I1 + I 2 + 2 I1 I2 cos 0º
CºN
Imin = I1 + I2 + 2 I1 I 2 cos180º
Ether
+C6H5MgBr
\ Imax + Imin = 2(I1 + I2 )
O
C 1 ´ 0 + 1´ PQ + 1´ PR PQ + PR
34. (b) xcm = =
+
H 3O 1+ 1+1 3
C6 H5– C=NMgBr
and ycm = 0
C6H5
35. (b) For a diamagnetic material, the value of µr
or C 6 H 5 - C = O + MgBrNH 2 is less than one. For any material, the value
|
of Îr is always greater than 1.
C6H5
36. (a) Let us consider a spherical shell of radius x
PHYSICS and thickness dx.
31. (c) Person’s speed walking only is Charge on this shell
1 "escalator" æ5 xö 2
dq = r.4px 2 dx = r0 ç 4 - R ÷ .4px dx
60 second è ø
Standing the escalator without walking the \ Total charge in the spherical region from
centre to r (r < R ) is
1 "escalator"
speed is r
40 second æ5 xö
q = ò dq = 4 pr0 ò ç - ÷ x 2 dx
Walking with the escalator going, the speed è4 Rø
0
add.
1 1 15
So, the person’s speed is + =
60 40 120 dx
"escalator"
x
second
120
So, the time to go up the escalator t =
5
= 24 second.
32. (b) 6W and 2W are in parallel combination
é 5 r3 1 r 4 ù
- . ú = pr0r 3 æç - ö÷
5 r
B
= 4 pr0ê .
A 2W 6W 1.5W ëê 4 3 R 4 ûú è3 Rø
6V 3W 6V 3W 6V 3W \ Electric field at r,
1.5W 1.5W 3W
1 q
E= . 2
4p Î0 r
Solutions-Mock Test -5 MT-177

39. (c) Inside the sphere, V is constant and is equal


1 pr0 r 3 æ 5 r ö
= . ç - ÷ to that on the surface of the sphere. Outside
4pÎ0 r 2 è 3 R ø
the sphere it comes out to be, V = –Gm/r
r0 r æ 5 r ö i.e. |V| µ 1/r. Hence the graph (c) is correct.
= 4Î ç 3 - R ÷ 40. (c)
0è ø
41. (b) Number of undecayed atom after time t2 ;
Mg
37. (c) = P0 N0
A = N 0 e -lt2 ...(i)
3
g g
P0V0 = PV
Number of undecayed atom after time t1;
Mg = P0A
2N0
= N 0 e -lt1
P0 Ax0 g = PA( x0 - x) g 3
...(ii)

g
P0 x0 -lt 1
P= From (i), e 2 =
g 3
( x0 - x )
æ 1ö
Piston Þ–lt2 = loge çè 3 ÷ø ...(iii)

2
x From (ii) – e -lt2 =
3
x0 Cylinder
containing æ 2ö
ideal gas Þ–lt1 = loge çè 3 ÷ø ...(iv)
Let piston is displaced by distance x Solving (iii) and (iv), we get
t2 – t1 = 20 min
æ P0 x0g ö
Mg - ç ÷ A = Frestoring 42. (a) Rate of cooling µ surface area and for a
çè ( x - x )g ÷ø given mass, surface area of sphere is
0
minimum.
æ x0g ö 43. (b) Work-done (W) = P0V0
P0 A ç1 - ÷ = Frestoring According to principle of calorimetry
çè ( x - x )g ÷ø
0 Heat given = QAB = QBC
[ x0 - x » x0 ] = nCVdTAB + nCPdTBC
3 5
gP0 Ax = (nRTB - nRTA ) + (nRTC - nRTB )
F=- 2 2
x0 3 5
= (2P0 V0 - P0 V0 ) + (4P0 V0 - 2 P0V)
\ Frequency with which piston executes 2 2
SHM. 13
= P0 V0
1 gP0 A gP0 A2 2
1
f = = Thermal efficiency of engine (h)
2p x0 M 2p MV0
W 2
= = = 0.15
38. (d) Apparent wt. Q given 13
= Real wt.–Upthrust
1
= 12 – (1000 ´ 10- 6 ) ´103 ´10 = 12 - 5 = 7 N
2
EBD_7206
MT-178 JEE MAIN

44. (b) Q The E.M. wave are transverse in nature


i.e.,
r r
k ´E r q i2
= =H …(i)
mw i1
r r P
r
where H = B dl
m
r r
k ´H r q
and = -E … (ii)
r we r r
r m 0 i1 cos q
k is ^ H and k is also ^ to E
r r r r r B=
2p r
(directed perpendicular to
or In other words X || E and k || E ´ B
the plane of paper, inwards)
45. (a) For conservation of momentum, we have, The force exerted due to this magnetic field
m1v1 = (m1 + m2) v or v = m1/(m1 + m2) v1 on current element i2 dl is
1 1 dF = i2 dl B sin 90°
Now the loss of energy = m v 2– \ dF = i2 dl
2 1 1 2
é m 0 i1 cos q ù m0
(m1+ m2)v2 ê 2 p r ú = 2 pr i1 i2 dl cos q
\ Fraction of energy lost ë û
49. (d) Let a1 = a, I1 = a12 = a2
1 1 a2 = 2a, I2 = a22 = 4a2
m1v12 - (m1 + m 2 )v 2 I2 = 4I1
= 2 2
Ir = a12 + a22 + 2a1a2 cos f
1
mv
2 1 1 = I1 + I 2 + 2 I1 I2 cos f
= 1 – [(m1+m2) / m1] ´ ( v2/v12) Ir = I1 + 4 I1 + 2 4 I12 cos f
= 1 – [(m1+m2)/m1]×[ m1v12/ [(m1+m2)2]×(1/
v12) Þ Ir = 5I1 + 4 I1 cos f … (1)
= 1 – [ m1/[(m1+m2)] = m2 /(m1+m2) 2
Now, I max = ( a1 + a2 ) = ( a + 2a ) 2 = 9 a 2
46. (c) I = I0 sin ( wt + f ) ; I
Imax = 9I1 Þ I1 = max
E = E0 sin ( wt + f) ; 9
Substituting in equation (1)
æ p pö
I = 4 sin wt ; E = 100 sin ç wt + + ÷ 5 Imax 4 I max
è 2 3ø Ir = + cos f
9 9
p p 5p I
Phase diff. between I and E = + = Ir = max [ 5 + 4 cos f]
3 2 6 9
f BA I é f ù
47. (b) Mutual inductance = = Ir = max ê 5 + 8 cos2 - 4 ú
I I 9 ë 2 û
[ MT -1Q -1 L2 ] Imax é fù
[Henry] = = ML2 Q -2 Ir = ê1 + 8 cos 2 ú
[QT -1 ] 9 ë 2û
48. (c) Magnetic field due to current in wire 1 at 50. (b) Using conservation of momentum,
point P distant r from the wire is v
m1v1+ = m1 1 + mv
m i 3
B = 0 1 [ cos q + cos q ] m1v1
4p r Þ m1v1 - = mv
3
Solutions-Mock Test -5 MT-179

(v = 5gl to complete motion along vertical 54. (a) A maximum frequency deviation of 75 kHz
circle) is permitted for commercial FM broadcast
stations in the 88 to 108 MHz VHF band
2m1v1 2 m1v1
= mv Þ v = 55. (a) A + d = i + i ' Þ i ' = A + d – i = 30 + 30 – 60 = 0°
3 3 m
56. (d) As l is increased, there will be a value of l
3 m above which photoelectrons will be cease
or v1 = 5gl
2 m1 to come out so photocurrent will become
51. (d) From the figure it is clear that range is zero. Hence (d) is correct answer.
57. (d) K.E. gained by charged particle of charge q
required when accelerated under a pot. diff. V will be
Ek = qV;
u 2 sin 2q (10)2 sin(2 ´ 30°) For a given V, E µ q.
R= = =5 3
g 10 For proton, deutron and a-particle, the ratio
u of charges is 1 : 1 : 2.
30° 13.6Z 2
Range R 58. (c) E= ; According to question,
n2
10m

10m

Tower 13.6Z2 13.6Z2


- = 47.2
4 9

æ 5ö
52. (d) The induced emf is Þ 13.6Z2 ç ÷ = 47.2
r r è 36 ø
-d f d ( B. A) - d ( BA cos 0º )
e= =- =
dt dt dt Þ Z 2 = 25 Þ Z = 5
× × × ×
59. (b) As S2 is open, hence C1 & C3 are in series,
also C2 & C4 are in series combination.
× × × ×
3 3
Q= ×V = × 12 = 9mC;
× l
V
× × × 4 4

× × × × 1´ 3
C'1 = = 3 / 4F
X 1+ 3
× × × × C1
dA d (l ´ x ) 2´ 4 4
\ e = –B = -B C'2 = = F
dt dt 2+ 4 3
dx
\ e = - Bl = - Blv
dt
53. (b) Given, current gain of CE amplifier b = 69, C2
IE = 7 mA
IC B 12V
or = 69
IB
b 69 I C Charge on C'1 = 9mC;
We know that, a = = =
1 + b 70 I E
4
69 69 Charge on C'2 = ×12 = 16mC
IC = I E ´ = ´7 3
70 70
Collector current, IC = 6.9 mA Hence, charge on C1 and C3 is 9mC, as both
are in series combination.
EBD_7206
MT-180 JEE MAIN

60. (d) Yc ´ ( DLc / Lc ) = Ys ´ (DLs / Ls ) p


Þ x= and y = 0
2
æ 1´ 10 -3 ö æ DL ö
Þ 1´1011 ´ çç ÷ = 2 ´1011 ´ ç s ÷ p
1 ÷ è 0.5 ø Thus x + 2y = k Þ =k
è ø 2
0.5 ´10-3 (x 2 - 1) dx
\ DLs = = 0.25 mm 63. (c) I=ò
2 æ x 2 + 1ö
(x 4 + 3x 2 + 1) tan -1 ç ÷
Therefore, total extension of the composite è x ø
wire (Dividing Num. and Den. by x2)
= DLc + DLs æ 1ö
= 1 mm + 0.25 mm = 1.25 mm çè1 - 2 ÷ø dx
x
I=ò
MATHEMATICS æ 2 1ö -1 æ 1ö
çè x + 3 + 2 ÷ø tan çè x + x ÷ø
x
1 dt
61. (b) m( x) = , which is discontinous at x = 1 =ò
x -1
( t + 1) tan
2 -1
t
1 1
f (u) = 2 = , æ 1 æ 1ö ö
u + u - 2 (u + 2) (u - 1) çè where t = x + x Þ dt = çè1 - 2 ÷ø dx ÷ø
which is discontinous at u = – 2, 1 x

1 1 æ x 2 + 1ö
when u = – 2, then = -2 Þ x = = log | tan -1 t | + C = log tan -1 ç ÷ +C
x -1 2 è x ø

when u = 1, then
1
=1 Þ x = 2 x2 + 1
Þ f (x) =
x -1 x
Hence given composite function is 64. (a) f ( x ) = Pe 2 x + Qe x + Rx
1
discontinous at three points, x = 1, and 2. Þ f ¢( x ) = 2Pe 2 x + Qe x + R
2
62. (d) Let y = cos (x + y)
Þ 31 = 2Pe 2 log 2 + Qelog 2 + R
dy æ dy ö
Þ = - sin ( x + y ) ç1 + ÷ ...(1) Þ 8P + 2Q + R = 31 ......... (i)
dx è dx ø
Also, 0 = P + Q ......... (ii)
Now, given equation of tangent is
log 4 39
x + 2y = k & ò0 (f (x) - Rx)dx =
2
-1
Þ Slope = log 4 39
2 Þ ò0 (Pe2x + Qex )dx =
2
dy -1
So, = put this value in (1), we get éP ù
log 4
39
dx 2 Þ ê e 2x + Qe x ú =
-1 ë2 û0 2
æ 1ö
= - sin ( x + y ) ç1 - ÷
2 è 2ø P 2log 4 P 39
Þ e + Q elog 4 - - Q =
Þ sin (x + y) = 1 2 2 2
p p Þ 15P + 6Q = 39 ......... (iii)
Þ x+ y = Þ y= -x Solving (i), (ii) and (iii), we get P = 5, Q = – 6,
2 2 R= 3
p
Now, - x = cos (x + y)
2
Solutions-Mock Test -5 MT-181

r rr rr r
65. (a) Any tangent to the ellipse
x2
+
y2
= 1 at
r c (c.b) r c b.c a
Þr= -
r
a = –
( )
2 2
a b p p2 p p2
x cos q y sin q 68. (d) For f (x) to be defined, we must have
P(a cos q , b sin q ) is + =1
a b
Y x - 1 - x 2 ³ 0 or x ³ 1 - x2 > 0
P( 1
\ x2 ³ 1 – x2 or x ³ .
2
B aco
sq , 2
b si Also, 1 – x2 ³ 0 or x2 £ 1.
nq
) æ 1 ö æ 1 ö
X 1
O Now, x ³ Þ çè x - ÷ø çè x +
2
A ÷ ³0
2 2 2ø
1 1
Þ x£- or x ³
2 2
It meets co-ordinate axes at A (a sec q , 0) Also, x2 £ 1 Þ (x – 1) ( x + 1) £ 0
and B (0, b cosecq) Þ –1 £ x £ 1
1 1
Thus, x > 0, x ³ and x2 £ 1
2
\ Area of DOAB = ´ a sec q × b cosec q
2 2
ab é 1 ù
Þ (area) D = ÞxÎê ,1ú
sin 2q ë 2 û
For area to be min, sin 2 q should be max. aX + b
69. (b) Let Y = . Then
and we know max value of sin 2 q = 1 c
\ Dmin = ab sq. units. 1 a
66. (a) The curve of y (x2 + 4a2) = 8a3 is symmetrical Y = (a X + b ) Þ Y - Y = ( X - X )
c c
about y-axis and cuts it at A (0, 2a). Tangent
1 a2 1
at A is parallel to x-axis. x-axis is asymptote. Þ å(Y - Y)2 = 2 å(X - X)2
This curve meets x2 = 4ay N c N
Where, Therefore S.D. of Y
x2
=
8a 3
Þ x 4 + 4a 2 x 2 - 32a 4 = 0 a2 1 2 a2 2 a
4a x 2 + 4a 2 = å (X - X) = s = s
c2 N c2 c
Þ (x 2 - 4a 2 )(x 2 + 8a 2 ) = 0 Þ x = ± 2a 70. (c) Given system of equations
\ Required
é 2a 8a 3 2a x
2 ù x sin 3q – y + z = 0 ù
area = 2 ê ò dx - ò are homogeneous system of
0 x 2 + 4a 2 0 4a
dx ú x cos 2q + 4 y + 3z = 0 úú
ëê ûú linear equation
2 x + 7 y + 7z = 0 úû
A(0, 2a)
Since system has non-trivial solution
(–2a, a) (2a, a) sin 3q –1 1
cos 2q 4 3 = 0
O \
2 7 7
a2
= (6p - 4). Þ sin 3q [28 – 21] + 1 [7 cos 2q – 6] + [7 cos 2q
3
r rr r r – 8] = 0
67. (a) p r + ( r .b ) a = c ...(1)
rr rr rr rr Þ 3 sin q – 4 sin3 q + 2(1 – 2 sin2 q) – 2 = 0
p( r .b) + ( r.b)(a.b) = c.b
Þ sin q (4 sin2 q + 4 sin q – 3) = 0
rr
r r c.b rr Either sin q = 0 or 4 sin2 q + 6 sin q – 2 sin q – 3= 0
Þ r.b = , since a.b = 0 , putting in (1),
p
EBD_7206
MT-182 JEE MAIN

Þ (2 sin q – 1) (2 sin q + 3) = 0
1ì 1 ü
1 3 í1 - n ý
sin q = ,sin q ¹ – 2
=n- î
\ [Q sin q > – 1] 2 þ
2 2 = n - 1 + 2-n
1
p 1-
2
\ q = np or q = np + (–1)n 6
75. (b) Let f (x) = 3x10 – 7x8 + 5x6 – 21x3 + 3x2 – 7
é (–1)n ù f ¢ (x) = 30x9 – 56x7 + 30x5 – 63x2 + 6x
Þ q = p ên + ú. f ¢ (1) = 30 – 56 + 30 – 63 + 6
ë 6 û = 66 – 63 – 56 = – 53
71. (c) A = {1, 2, 3}, B = {4, 5}
Þ (A × B)n (B × A) = f f (1 – a ) – f (1)
Consider lim
Then A Ç B = f a®0 a3 +3a
A × B = {(1, 4), (1, 5), (2, 4), (2, 5), (3, 4), f ¢ (1– a )(–1)–0
(3, 5)} = lim (By using
a®0 3a 2 +3
B × A = {(4, 1), (4, 2), (4, 3), (5, 1), (5, 2), L’hospital rule)
(5, 3)} f ¢(1 – 0)(–1) – f ¢ (1) 53
(A × B) Ç (B × A) = f = = =
3(0)2 + 3 3 3
72. (c) f ( x) = x 2 [ x]
Y
76. (a)
F
f b H
a
g
e d
c
–1 O 1 X B

y = –x2 a + e + f + g = 285, b + d + f + g = 195


c + d + e + f = 115, e + g = 45, f + g = 70,
d + g = 50
a + b + c + d + e + f + g = 500 – 50 = 450
ìï- x 2 , -1 < x < 0
Þ f ( x) = í As in previous question, we obtain
ïî 0, 0 £ x <1 a + f = 240, b + d = 125, c + e = 65
a + e = 215, b + f = 145; b + c + d = 165
73. (d) y = e x + sin x a + c + e = 255; a + b + f = 335
dy dx 1 Solving we get
= e x + cos x Þ =
x b = 95, c = 40, a = 190 , d = 30, e = 25, f = 50
dx dy e + cos x
Diff. w.r.t y and g = 20
Desired quantity = a + b + c = 325
d2x - 1[e x - sin x ] dx
= ´
dy 2 (e x + cos x ) 2 dy 77. (d) We have f ( x ) = 4 x3 - 7, x Î R.
(sin x - e x ) f is one-one. Let x1, x2ÎR and f (x1) = f(x2).
=
x 3
(e + cos x) Þ 4x13 - 7 = 4x 2 3 - 7 Þ 4x13 = 4x 2 3
1 3 7 15
74. (c) + + + + ... Þ x13 = x 2 3 Þ x13 - x 2 3 = 0.
2 4 8 16
æ 1 ö æ 1 ö æ 1ö æ 1ö Þ ( x 1 - x 2 ) ( x1 2 + x 1 x 2 + x 2 2 ) = 0.
= ç1 - ÷ + ç1 - ÷ + ç1 - ÷ + ç1 - ÷ + ....
è 2 ø è 4 ø è 8 ø è 16 ø éæ 3x 2 ù
2
x ö
Þ ( x 1 - x 2 ) êç x 1 + 2 ÷ + 2 ú = 0 .
êëè 2 ø 4 ú
û
Solutions-Mock Test -5 MT-183

Þ x1 – x2 = 0, because the other factor is p


non-zero. Þ - cot -1 ( x + 1) = 2np ± tan -1 x Put n = 0
2
Þ x1 = x2 \ f is one-one.
Þ p / 2 - cot -1 (x + 1) = ± tan -1 x = tan -1 (± x)
f is onto. Let k ÎR any real number.
Þ p / 2 = tan -1 (± x ) + cot -1 ( x + 1)
1/ 3
æk+7ö 1
f ( x ) = k Þ 4x 3 - 7 = k Þ x = ç ÷
è 4 ø Þ x + 1 = ± x Þ 2x + 1 = 0 ; x = –
2
81. (a) Given, k sin q + cos 2 q = 2k – 7
1/ 3
æ k + 7ö Þ k sin q + 1 – 2 sin2 q = 2k – 7
Now ç Î R, because k ÎR and
è 4 ÷ø Þ 2sin2 q – k sin q + (2k – 8) = 0
For the existence of real roots, discriminant ³ 0.
é æ k + 7 ö 1/ 3 ù
3
é æ k + 7 ö 1/ 3 ù Þ k2 – 4 × 2 (2k – 8) ³ 0 Þ (k – 8)2 ³ 0, which is
f êç ÷ ú = 4 êç ÷ ú –7 always true.
êë è 4 ø úû êë è 4 ø úû
k-4
Roots of the quadratic equation are , 2,
æ k + 7ö 2
=4ç -7 = k
è 4 ÷ø but sin q ¹ 2.
1/ 3 k-4
æk+7ö \ sin q = but - 1 £ sin q £ 1
\ k is the image of ç ÷ 2
è 4 ø k-4
Þ -1 £ £1 Þ -2 £ k -4 £ 2
\ f is onto. 2
\ f is a bijective function. Þ 2£k£6
78. (b) (1, 1) satisfies g( x) = ax + b \ a + b = 1 1 - cos 3 x
82. (c) lim
(2, 3) satisfies g( x) = ax + b \ 2a + b = 3 x ® 0 x sin x cos x
Solving the two equation, we get a= 2, b = –1 (1 - cos x) (1 + cos x + cos 2 x)
It can be checked that other ordered pairs = lim
satisfy g(x) = 2x – 1 x ®0 x sin x cos x
q r æ xö
2sin 2 ç ÷
79. (c) We have a + b = - p , ab = p = lim
è 2ø
´
(1 + cos x + cos2 x)
x ®0 æ xö æ xö cos x
Now the given equation x.2sin ç ÷ cos ç ÷
è 2ø è 2ø
a ( x - b) 2 + b ( x - a ) 2 = 0
æ xö
sin ç ÷
Þ (a + b) x 2 - 4 ab x + ab ( a + b ) = 0 è 2 ø 1 + cos x + cos 2 x 1 3
= lim ´ = ´3 = .
x ®0 æ x ö æ xö 2 2
æ qö r r æ qö 2ç ÷ cos ç ÷ cos x
è 2ø è 2ø
Þ çç - ÷÷ x 2 - 4 x + çç - ÷÷ = 0
è p ø p p è pø
-1 æ 2 x ö
83. (c) y = sin çç ÷÷
2
Þ pqx + 4prx + rq = 0 ------(i) è 1+ x2 ø
Since a and b have opposite signs, therefore dy 1 2(1 + x 2 ) - 4 x 2
= .
p and r must have opposite signs.
Þ pq and rq must have opposite signs
dx
1-
4x 2 (1 + x )
2 2

Þ roots of equation (i) have opposite signs (1 + x ) 2 2

80. (d) sin (cot–1(x + 1)) = cos (tan –1x) 2(1 - x 2 ) 1 2(1 - x 2 )
= . =
Þ cos( p / 2 - cot -1 ( x + 1)) = cos(tan -1 x) (1 - x 2 ) 2 1 + x
2
| 1 - x 2 | (1 + x 2 )
2 2
If | x | < 1 Þ x < 1, then 1–x > 0
EBD_7206
MT-184 JEE MAIN

dy 2 é x
-1 2 (2 - 1)
ù é 2 x +1 - 2 x ù
\ | 1- x 2 | = 1- x 2 Þ = 87. (a) y = tan ê ú = tan -1 ê ú
dx 1 + x 2 x x +1 x x +1
ëê1 + 2 .2 ûú ëê1 + 2 .2 ûú
If | x | > 1 Þ x > 1, then 1 –x2 < 0
2

= tan -1 ( 2 x +1 ) - tan -1 (2 x )
2 2 dy 2
\ | 1- x |= x -1 Þ =-
dx 1+ x 2 dy 2 x +1 log 2 2 x log 2
Þ = -
dy dx 1 + 2 2 ( x +1) 1 + 2 2 x
Obviously does not exist for x2 = 1 or | x | = 1
dx æ dy ö æ2 ö æ 3ö
\ç ÷ = (log 2) ç - 1÷ = log 2ç - ÷
é æ 1 öù è dx ø x =0 è5 ø è 5ø
84. (a) f {f [f ( x)]} = f êf ç ÷ú
ë è 1 - x øû a a2 1+ a3
æ ö 88. (c) b b2 1 + b3
ç ÷ x -1 ö
÷ = f æç
1
= fç ÷ c c2 1 + c3
ç1- 1 ÷ è x ø
ç ÷
è 1- x ø = (1 + abc)(a - b)(b - c)(c - a ) = 0 ....(1)
æ 1 ö a2
\ f (x) is not defined for x = 1; f ç ÷ is not 1 a
è1- x ø
defined for x = 0. also, 1 b b 2 ¹ 0, since A, B, C are not
\ f {f [f(x)]} is discontinuous at x = 0 and 1 i.e.,
1 c c2
there are two points of discontinuity.
85. (c) Consider cos 255° + sin 195° Coplanar = (a - b)(b - c)(c - a ) ¹ 0 ...(2)
= cos (270° – 15°) + sin (180° + 15°) From (1) & (2), abc = –1
= – sin 15° – sin 15° 89. (d) A and B will agree in a certain statement if
both speak truth or both tell a lie. We define
æ 3 -1 ö æ 3 -1 ö following events
= – 2 sin 15° = -2 çç ÷÷ = - çç ÷÷
è 2 2 ø è 2 ø E1 = A and B both speak truth Þ P(E1) = xy
E2 = A and B both tell a lie Þ P (E2) = (1 – x )
dy y æyö
86. (c) Given, = - cos 2 ç ÷ (1 – y)
dx x èxø E = A and B agree in a certain statement
dy dv Clearly, P(E / E1 ) = 1 and P(E / E 2 ) = 1
Putting, y = vx so that = v+x
dx dx
The required probability is P(E1 / E).
dv
We get, v + x = v - cos 2 v Using Baye’s theorem
dx
P(E1 / E )
dv dx dx
Þ 2
=- Þ sec 2 v dv = - P(E1 )P(E / E1 )
cos v x x =
Integrating, we get P(E1 )P(E / E1 ) + P(E 2 )P(E / E 2 )
æyö xy.1 xy
tan v = – ln x + ln c tan ç ÷ = - ln x + ln c = =
èxø xy.1 + (1 – x )(1 – y).1 1 – x – y + 2 xy
90. (a) | x1z1 – y1z2 |2 + | y1z1 – x1z2 |2
This passes through æç1, p ö÷ Þ ln c =1
è 4ø = | x1z1 |2 + | y1z2|2 – 2Re(x1y1z1z2)
+ | y1z1 |2 + | x1z2 |2 + 2Re (x1y1z1z2)
æ eö
\ y = x tan -1 ç log ÷ = x12 | z1 |2 + y12 | z2 |2 + y12 | z1 |2 + x12 | z2 |2
è x ø
= x12 | z1 |2 + y12 | z2 |2 + y12 | z1 |2 + x12 | z2 |2
= 2(x12 + y12) (42) = 32(x12 + y12)
Mock Test-6
ANSWER KEY
1 (d) 16 (b) 31 (c) 46 (b) 61 (a) 76 (c)
2 (c) 17 (b) 32 (b) 47 (a) 62 (b) 77 (b)
3 (c) 18 (b) 33 (c) 48 (b) 63 (d) 78 (a)
4 (a) 19 (d) 34 (c) 49 (b) 64 (a) 79 (a)
5 (c) 20 (b) 35 (c) 50 (d) 65 (a) 80 (b)
6 (b) 21 (c) 36 (d) 51 (c) 66 (b) 81 (b)
7 (b) 22 (a) 37 (c) 52 (c) 67 (a) 82 (c)
8 (c) 23 (d) 38 (a) 53 (d) 68 (b) 83 (d)
9 (c) 24 (d) 39 (a) 54 (d) 69 (c) 84 (c)
10 (b) 25 (c) 40 (d) 55 (d) 70 (d) 85 (d)
11 (a) 26 (a) 41 (a) 56 (d) 71 (c) 86 (b)
12 (a) 27 (b) 42 (b) 57 (c) 72 (d) 87 (d)
13 (a) 28 (c) 43 (b) 58 (d) 73 (b) 88 (b)
14 (c) 29 (a) 44 (b) 59 (d) 74 (d) 89 (d)
15 (a) 30 (c) 45 (b) 60 (c) 75 (d) 90 (a)

Solutions
CHEMISTRY 1
–249 = 433 + ´ 492 - 2x
1. (d) When mixture of two liquids boil at a lower 2
temperature than either of them, it shows Þ x = 464kJ mol-1.
positive deviation from Raoult’s law. 5. (c) n = 3, l = 2 means 3d orbital
2. (c) The conformers are interconvertible.
3. (c) The correct order of acidic strength of the
given species in +2 +1 0 –1 –2
i.e. in an atom only one orbital can have
HSO3 F > H 3 O + > HSO 4 - > HCO3- the value ml = + 2
(iv) (ii) (iii) (i)
6. (b) The more is the value of equilibrium
(i) < (iii) < (ii) < (iv) constant, the more is the completion of
It corresponds to choice (c) which is correct reaction or more is the concentration of
answer. products i.e. the order of relative strength
would be
1 OH– > CN– > H2O
4. (a) H 2 (g) + O2 (g) ® H 2 O(g) ΔH = – 249
2 7. (b) Z-3-amino-3-nitro-2-methyl propenoic acid.
Let the bond enthalpy of O - H is x. Then The senior –NO2 gp and – COOH gp are
DH = SB.E. of reactant - S B.E.of product on the same side.
EBD_7206
MT-186 JEE MAIN
8. (c) CH3 OCH2 Cl is a 1° halide but still conc. H SO
hydrolysis takes place through 2C2 H5OH ¾¾¾¾¾¾
2 4 ® C H O.C H
2 5 2 5
- H 2O
Diethyl ether
+
SN 1 mechanism because CH 3O - C H 2 is Thus, (A) is ethanol and (B) is sodium
stabilized by resonance ethoxide.
+ 16. (b) Claisen self condensation is given by esters
+ which contain at least one hydrogen on an
CH 3O CH 2Cl CH3– O – CH 2 CH 3– O = CH2
9. (c) Electron withdrawing group stabilises the alpha carbon atom (e.g., CH3COOC2H5).
benzene ring due to delocalisation of Compound, C6H5.COOC2H5 will not give
charge. this reaction. All remaining compounds will
give this reaction.
– CH3 and – CH2OH are electron donating
17. (b) The step involved is a precipitation step.
group and hence decrease the stability of
benzene ring,– OCH3 is weaker electron Increasing the Cl - concentration will
withdrawing group than – COCH3. Hence
reduce the concentration of Ag + in
– COCH3 group stabilize the phenoxide ion
solution. Remember Ksp = éë Ag + ùû [ Cl - ]
more at p-position.
10. (b) Ag2S + 2NaCN Na2S + 2AgCN
AgCN + NaCN ¾
¾® Na[ Ag ( CN ) 2 ] and the source of the Cl - is irrelevant.
1 Thus increased concentration of Cl - must
11. (a) M.P. µ (HCl having more covalent
f
result in decreased concentration of Ag +
character)
M.P. µ Lattice energy (Ionic compd.) in order to maintain the solubility product
LiF is ionic having highest Lattice energy. constant. Addition of Ag2 SO4 would
12. (a) AgX + 2 Na2S2O3 ® Na3[Ag(S2O3)2]+ NaX probably be counterproductive, since
thiosuphate Sodium argento (soluble complex) additional Ag + is being added, and
K f ´ w ´ 1000 probably all will not be recovered.
13. (a) Molar mass of solute = 18. (b) Reaction of alcohols with Lucas reagent
DTf ´ W
proceeds through carbocation formation,
Given : Kf = 1.86, w = 1.25 g, W = 20 g, SN1 mechanism.
DTf = 273 – 271.9 = 1.1K Further 3° carbocations (from tertiary
Therefore, molar mass of solute alcohols) are highly stable thus reaction
1.86 ´ 1.25 ´ 1000 proceeds through SN1 mechanism.
= = 105.7
1.1 ´ 20
19. (d) CH2 = CH2 ¾¾¾
HOCl
® ClCH 2 CH2 OH
14. (c) +4
H 2SO3 ( aq ) + Sn 4+ ( aq ) + H 2 O ( l ) ¾¾ ¾¾¾¾®
aq.NaHCO3
® HOCH2 CH 2OH
+6 20. (b) All alkali metals dissolve in liquid ammonia
Sn 2+ ( aq ) + HSO 4- ( aq ) + 3H +
giving deep blue solution.
Hence H2 SO 3 is the reducing agent
21. (c) N2Cl4 has two pyramidal nitrogen atoms
because it undergoes oxidation.
with sp3 hybridisation in each. This yields
15. (a) 2C 2 H 5 OH + 2 Na ¾
¾® 2C 2 H 5 ONa + H 2 ­ a non-planar molecule. PCl5 involves sp3d
Ethanol Sod. ethoxide
hybridisation. SbF 6 – and SiF 6 2– both
involve sp3d2 hybridisation.
(A) (B)
Solutions-Mock Test-6 MT-187

22. (a) According to van der Waal's equation for 27. (b)
one mole of gas 28. (c) Hydroxylamine and hydrazine, both do not
æ a ö have carbon, hence NaCN will not be
çççP ∗ 2 ÷÷÷ (V , b) < RT formed in Lassaigne’s extract leading to
è V ø
negative test for nitrogen.
a
at very high pressure P == 29. (a) Tollen's reagent also oxidises
V2 a-hydroxyketones, hence positive result is
a observed in such case.
So, is negligible.
V2 O OH
|| |
O O
|| ||
P (V – b) = RT Ag(NH 3 ) +2
R - C- CH - R ' ¾¾¾¾ ® R - C - C- R '+ Ag ¯
H2 O
PV – Pb = RT Silver mirror

on dividing RT on both sides. 30. (c) In (c), sulphate ion is present outside the
Pb coordination sphere so it can form white
[ Z < 1∗ compressibility factor.. ppt of BaSO4 with BaCl2 (aq).
RT
23. (d) The appearance of colour in alkali metal
PHYSICS
halides is generally due to F-centres. F
centers in NaCl produce a band of optical
1 k
absorption towards the blue end of visible 31. (c) Frequency of spring (f ) = = 1 Hz
spectrum, thus the colour appears yellow 2p m
under transmitted light. On the other hand, k
Þ 4p 2 =
KCl with F-centers appears magenta, and m
KBr appears blue.
o
m = 1 kg
24. (d) Given : E Mg 2+ / Mg = -2.36V

Mg 2+(aq) + 2e - ® Mg (s) If block of mass m = 1 kg is attached then,


k = 4p2
Now, identical springs are attached in
parallel with mass m = 8 kg. Hence,
0.059 [Mg2+ ]
E = Eo + log keq = 2k
Mg2+/ Mg Mg2+/ Mg 2 [Mg]
Given [Mg2+] = 0.01 M; [Mg] = 1 1 k´2 1
F= = Hz
Hence, 2p g 2
0.059
E = -2.36 + log( 0.01) = -2.42V
Mg 2 + / Mg 2 k
25. (c) For electrophilic substitution reaction, the 8 kg
order of reactivity among the given
compounds is follows : s
C6H5OCH3 > benzene > C6H5NO2 32. (b) E= or
Î0
26. (a) KI + AgNO3 (slight excess) ¾¾
® AgI + KNO3
s = E Î0 = 100 ´ 8.85 ´ 10 -12 = 10 -9
¾® Ag + + NO 3- ;
AgNO 3 ¾
But the direction is downward, which
AgI ( s ) + Ag + ¾
¾®[ AgI ]Ag + means the charge is negative.
Note: Practically, near earth, E » 100 V / m ,
s = –10–9 C/m2 and Q = –106 C.
EBD_7206
MT-188 JEE MAIN
33. (c) The magnetic field at a point on the axis of a \ Differential above term and equating it
circular loop at a distance x from centre is, to zero.
m 0i a 2 b
B= \ 2aw - b = 0 Þ w = .
2a
2( x 2 + a 2 )3 / 2
m i (200 - T1 ) (T - T )
B' = 0 39. (a) Q = KA = (2K)A 1 2
2a L L
B.( x 2 + a2 )3/ 2 1
\ B' = =(1.5K) A (T2-18) ×
a3 L
54(53 ) 200 - T1= 2T1 – 2T2 = 1.5 T2- 27,
Put x = 4 & a = 3 Þ B ' = = 250 µT Solving T1 = 116ºC and T2= 74ºC
3´3´ 3
40. (d) 4i 1 = 5i 2
u 2 sin 2 q
34. (c) H= or H µ u 2 5
2g \ i1 = i2
4
DH Du i1 = current through 4 W
= 2. = 2(+2%) = 4% increased i2 = current through Ammeter
H u
5 ´ 4 20
é 1 Total R = =
1 1 ù 9 9
35. (c) Wave number = RZ 2 ê - ú
l êë n1 n 2 úû
2

10 ´ 9 9 5 9
1 I= = Þ i 2 + i 2 = Þ i 2 = 2A
Þlµ 20 2 4 2
Z2 41. (a) When positive terminal connected to A
By question n = 1 and n 1 = 2 then diode D1 is forward biased, current,
Then, l1 = l2 = 4l3 = 9l4 2
I= = 0.4 A
36. (d) Angular momentum J = mv R 5
It shows that angular momentum increases When positive terminal connected to B
as the perpendicular distance R increases then diode D2 is forward biased, current,
along line BC.
37. (c) mg – T = ma 2
I= = 0.2 A
T 360 10
\ a = g - = 10 - = 4m / s 2 42. (b) Let pole strength = m
m 60
l
T \ M = m ´ l when l' =
2
ml M
M' = m ´ l' = = = 40 units
a 2 2
®

T
43. (b) According to theorem of perpendicular
axes, moment of inertia of triangle (ABC)
mg I0 = kml 2 ..... (i)
mg
BC = 1
1 Moment of inertia of a cavity DEF
38. (a) X=
2
a w 2 - bw + c mæ lö
IDEF = K ç ÷
At resonance X becomes maximum and 4 è 2ø
aw 2 - bw + c becomes minimum. =
k
ml 2
16
Solutions-Mock Test-6 MT-189

From equation (i), Final image distance L = v0 + ue = 20


I0 1 1 1
IDEF = for objective = +
16 1 v0 u0
Moment of inertia of remaining part
I0 15I0 1 1 1
Iremain = I0 - = for eye piece = - + or
16 16 fe 25 u e
r r
44. (b) f = B. A ; f = BA cos wt 1 1 1
+ =
df wBA 5 25 u e
e=- = wBA sin wt ; i = sin wt
dt R
25
2 \ue = cm
æ wBA ö
2 6
Pinst =i R=ç ´ R sin 2 wt
è R ÷ø
\ v o = 20 - u e
T
ò Pinst ´ dt u e = 20 -
25 95
=
0
Pavg = 6 6
T
ò dt 1 1 95 89
0 \ = 1- = 1- =
T
u0 v0 6 95
2
2 ò sin wtdt
95
=
(wBA) 0 1 ( wBA) 2 \u0 =
R T = 89
2 R
ò dt 1 ì hc
0 ü
47. (a) V= í - fý
é eîl þ
(w B pr 2 ) 2 pr 2 ù
\ Pavg = êA = ú
8R ëê 2 ûú Put l1 = 3 ´10 -7 m, V1 = 1.85 ,
1 q
45. (b) Electric Intensity = ´ l 2 = 4 ´ 10 -7 m, V2 = 0.82
4 p Î0 r 2

1 1 in the above equation respectively


or qmax = r2E = × 2.5 ×
9 9
9 ´ 10 9 ´10 hc
f = 1.216 ´ 10 -19 J = Þ l 0 = 5451Å
2.5 × 3 × 106 l0
or qmax = 2 × 10–3 coulomb
48. (b) Let V be volume of the load, and let its
46. (b) 20 cm relative density be r then

u0 v0 ue FL VrgL
Y= = . ......(1)
Al a Al a
when the load is immersed in liquid, the net
weight = weight - upthrust
F' L (Vrg - V ´ l ´ g )L
objective \Y = = ......(2)
Al w Al w
Ve=25 cm Eye piece

Equating (1) and (2), we get r = r - l


la lw
EBD_7206
MT-190 JEE MAIN

la lD
which gives r = =b ....(ii)
la - l w d
Multiplying equation (i) and (ii) we get,
Distance travelled on pitch scale
49. (b) Pitch = b
Number of rotation y=
4
53. (d) Internal energy is a state function hence in
2mm
= = 0.5 mm a cyclic process change in internal energy
4
is zero.
Least count = 54. (d) From the law of conservation of momentum
Pitch we know that,
Number of division on circular scale m1u1 + m2u2 + .... = m1v1 + m2v2 + ....
Given m1 = m, m2 = 2m and m3 = 3m
0.05mm and u1 = 3u, u2 = 2u and u3 = u
= = 0.01 mm ®
50
Let the velocity when they stick = v
bt Then, according to question,

50. (d) As we know, E = E0 e m Y
b15 2m, 2u

15 = 45e m
[As no. of oscillations = 15 so t = 15sec] j sin 60°
b15 j
1 – 60°
= e m m, 3u X
3 P 60° i i cos 60°
Taking log on both sides (–j) sin 60°
b 1
= ln3 –3m, u
m 15
51. (c) Given: Amplitude of electric field,
E0 = 4 v/m
()
m × 3u î + 2m × 2u
Absolute permitivity,
e0 = 8.8 × 10–12 c2/N-m2 ( -ˆi cos 60° - ˆjsin 60°) + 3 m × u
Average energy density uE = ?
( -ˆi cos 60° + ˆjsin 60°) = (m + 2m + 3m) v
®
Applying formula,
1 ˆ æ ö ˆ
Average energy density uE = e0 E 2 Þ 3muiˆ - 4mu i - 4mu ç 3 ˆj÷ - 3mu i
4
2 è 2 ø 2
1
Þ uE = ´ 8.8 ´ 10 -12 ´ (4)2 æ 3 ˆö ®
4 + 3mu çç j ÷÷ = 6m v
= 35.2 × 10–12 J/m3 è 2 ø
2 æ Df ö p 3 3 ˆ ®
52. (c) 2I0 = 4I0 cos ç ÷ here, Df = Þ muiˆ - muiˆ - muj = 6m v
è 2 ø 2 2 2
2p l ®
But, Df = Dx so, Dx = 1 3 ˆ
l 4 Þ - muiˆ - muj = 6m v
2 2
dy l
= ....(i) ®
D 4 Þ v =
12
(
u ˆ
- i - 3jˆ )
Solutions-Mock Test-6 MT-191

55. (d) Gravitational force provides the necessary ur 2 2


centripetal force. OA = | r1 | = ( 2) + ( 2) = 4 = 2units.

mv 2 GmM GM The distance of point B(2, 0) from the


\ = also g = 2 origin,
( R + x) ( R + x) 2
R
uur 2 2
OB = | r2 | = (2) + (0) = 2 units.
mv 2 æ GM ö R 2 n!
\ = mç
( R + x) è R 2 ÷ø ( R + x )2 r !( n - r ) ! 1 Q
Now, potential at A, VA = .
4p Î0 (OA)
mv 2 R2
\ = mg
( R + x) ( R + x) 2 1 Q
Potential at B, VB = .
1/ 2 4p Î0 (OB)
gR 2 æ gR 2 ö
\ v2 = Þ v=ç ÷
R+ x è R + xø \ Potential difference between the points
A and B is given by
56. (d) As the surrounding is identical, vessel is
identical time taken to cool both water and 1 Q 1 Q
liquid (from 30°C to 25°C) is same 2 minutes, VA – VB = . - .
4p Î0 OA 4p Î0 OB
therefore
æ dQ ö æ dQ ö Q æ 1 1 ö Q æ 1 1ö
çè ÷ø =ç ÷ = -
dt water è dt ø liquid çè ÷ø = çè - ÷ø
4p Î0 OA OB 4p Î0 2 2

(mw C w + W)DT (ml Cl + W) D T Q


or, = = ´ 0 = 0.
t t 4p Î0
(W = water equivalent of the vessel) 58. (d) Here, R = 4 kW = 4 × 103 W
or , m w C w = m l C l Vi = 60 V
Zener voltage Vz = 10 V
mWCW
\ Specific heat of liquid , Cl = RL = 2 kW = 2 × 103 W
ml
VZ 10
Load current, IL = = = 5 mA
50 ´ 1 RL 2 ´ 103
= = 0.5 kcal / kg
100
Vi - VZ
Current through R, I =
Y R
60 - 10 50
= = = 12.5 mA
57. (c) A(Ö2,Ö2) 4 ´ 10 3
4 ´ 103
®
r1 Fom circuit diagram,
I = IZ + IL
O X Þ 12.5 = IZ + 5
(0,0) ®
r2 B (2,0) Þ IZ = 12.5 – 5 = 7.5 mA

The distance of point A ( )


2, 2 from the
A 0.9 km
H
B I
origin, 59. (d) ENGINE L
C L
EBD_7206
MT-192 JEE MAIN
Let after 5 sec engine at point C Thus required period
AB BC = LCM of 2π & 5π = 10π
t= +
330 330 63. (d) Given ax2 + bx + c = 0
0.9 ´ 1000 BC Þ ax2 = –bx – c
5= + Now, consider
330 330
\ BC = 750 m y = 4ax2 + 3bx + 2c
Distance travelled by engine in 5 sec = 4 [–bx – c] + 3bx + 2c
= 900 m – 750 m = 150 m = – 4bx – 4c + 3bx + 2c
Therefore velocity of engine = – bx – 2c
Since, this curve intersects x-axis
150 m \ put y = 0, we get
= = 30 m/s .
5sec –bx – 2c = 0 Þ – bx = 2c
60. (c) 9.25 days is equal to three half-lives for Tl-
-2c
201. The fraction remaining is then : Þx=
1/2 × 1/2 × 1/2 = 1/8. Thus 1/8 of 80 mCi b
remains. Thus, given curve intersects x-axis at
exactly one point.
MATHEMATICS
64. (a) Since, sec (q – f), secq and sec (q + f) are in
61. (a) The probability of A winning in a game = P A.P.,
\ 2 secq = sec (q – f) + sec (q + f)
6 1
(A) = = 2 cos ( q + f) + cos ( q - f)
12 2 Þ =
cos q cos ( q - f ) cos ( q + f )
The probability of B winning in a game = P (B) Þ

4 1
=
( )
2 cos 2 q - sin 2 f = cos q [ 2cos q cos f]
=
12 3
Þ cos 2 q (1 - cos f ) = sin 2 f = 1 - cos 2 f
\ Reqd. probability = P (A Ç B Ç A) + P (B Ç A Ç B) f
Þ cos 2 q = 1 + cos f = 2 cos2
= P (A).P(B).P(A) + P (B).P(A).P(B) 2
f
\ cos q = 2 cos
1 1 1 1 1 1 5 2
= × × + × × = f
2 3 2 3 2 3 36 But given cosq = k cos
2
62. (b) Period of sinx = 2π \ k= 2
y
Þ period of sin 3 x = 2π dt
65. (a) x = ò
0 1+ t2
period of sin 3 x = π
1 dy
Þ 1= .
3 x 1+ y 2 dx
Þ period of sin = 2π
2
é y ( x)
period of cos5 x = 2π dI(x)
êQ If I( x) = ò f (t ) dt , then = f {y ( x)}.
Þ period of cos5 x = π ê dx
ë f ( x )

5 x
Þ period of cos = 5π
5
Solutions-Mock Test-6 MT-193

{ } { }
Adding all terms, we get,
d d ù
y ( x ) - f {f( x )} . f( x ) ú
dx dx û x å n 2 - n < {[12 x] + [22 x] + [32 x] + …

dy + [ n2 x]} £ x å n 2
= 1 - y2
dx
Dividing each term by n3,
d2 y 1 dy y
Þ = .2y . = . 1+ y2 = y æ 1 öæ 1ö
x ç1 + ÷ç 2 + ÷
2 2 dx 1+ y2
dx 2 1+ y we get , è n øè nø 1
-
6 n2
66. (b) | a1 x + a2x2 + … + anxn | £ | a1 | | x |+| a2 | | x
|2 [12 x ] + [2 2 x] + ..... + [ n 2 x]
+ | a3 | | x | 3 + ……. + | an | | x | n <
n3
£ 2 [| x | + | x |2 + ….. + | x |n ] [Q a n < 2]
æ 1 öæ 1ö
xç1 + ÷ç 2 + ÷
2| x| 2| x| 2 1 è n øè nø
£
= [1- | x | n ] < < 6
1- | x | 1- | x | 3 1- | x |
Let n ® ¥, we get,
1 3
Therefore, | a1x + ….. + anxn | < 2. . =1
3 2 x [12 x] + [2 2 x ] + .... + [ n 2 x] x
( 2) - 0 < Lt £
\ | a 1x + ….. + anxn |
< 1 for all 6 n ®¥ n3 3
n . – 1 < a1x + ….. + anxn < 1 for all n
[12 x ] + [2 2 x ] + ..... + [n 2 x ] x
Þ 1 + a1x + a2x2 + … + anxn > 0 for all n Þ lim =
n ®¥ 3 3
n
Þ 1 + a1x + .. + a nxn ¹ 0 for all n
r
-
p
p 69. (c) T r +1 = 5 C r ( x 2 ) 5-r æç c ö÷ = 5 C r x10 -3r c r
67. (a) Since, e 2 <q< èxø
2
\ 10 –3r = 1Þ 3r =9 Þ r = 3
p
- p
\ log e 2 < log q < log coeff. of x = 5 C3c 3 = 10c 3
2
p p p (p + q)!
i.e., – < log q < log <1 < 70. (d) Sp = coeff. of xp = p+qC
p=
2 2 2 p!q!
é p p pù p+ q
Cq
êQ 2 < e \ log 2 < log e = 1 and 1 < 2 ú and Sq = coeff. of xq =
ë û

p p (p + q)!
\– < log q < \ cos (log q) > 0 = , \ Sp = Sq
2 2 q!p!
But 0 < cos q < 1, \ log ( cos q) < log 1 = 0 71. (c) Number of ways of making 3 sets of 10 balls
i.e., log (cos q) < 0. Hence, cos (log q) > having 2, 3 and 5 balls
10!
and further
log ( cos q) 2!.3!.5!
68. (b) Since 12 x – 1 < [12 x] £ 12 . x they can be given to 3 persons one each in
22 x – 1 < [22 x] £ 22 x 3! ways.
32 x – 1 < [32 x] £ 32 x ………………
n2 x – 1 < [n 2 x] £ n2 x
EBD_7206
MT-194 JEE MAIN
74. (d) Operate C1+C2 +C3 ,
10!
Total number of ways = ´ 3! we get,
2!3!5!
æ dy ö x + 1 + w + w2 w w2
72. (d) Given, sin 2 x çè - tan x ÷ø - y = 0
dx x + 1 + w + w2 x + w2 1 =0
dy y x + 1 + w + w2 1 x+w
or, = + tan x
dx sin 2 x
dy
or, - y cosec2 x = tan x ...(1)
x w w2
dx
Now, integrating factor (I.F) = e ò
- cosec2 x Þ x x + w2 1 =0
x 1 x+w
or, I.F =
1
- log|tan x|
e 2 = e
log ( tan x ) -1
[Q 1 + w + w2 = 0]
1
= = cot x
tan x 1 w w2
Now, general solution of eq. (1) is written as 2
Þ x 1 x+w 1 =0Þx =0
y (I. F.) = ò Q(I.F.) dx + c 1 1 x+w
\ y cot x = ò tan x . cot x dx + c
l m n
\ y cot x = ò 1.dx + c
75. (d) Lines are concurrent if m n l =0
\ y cot x = x + c n l m
73. (b) From cosine and sine formula, we have
l+ m+n m n
b2 + c2 - a 2 Þ l+ m+n n l = 0 (C1 ® C1 + C2 + C3 )
cos A = and l+ m+n l m
2bc

sin A sin B sin C 1 m n


= = = k.
a b c Þ (l + m + n) 1 n l =0
Given, in any DABC, 1 l m
sinB Þ (l + m + n )( mn + nl + lm - l 2 - m 2 - n 2 ) = 0
cos A =
2sin C Þ l + m + n = 0; l 2 + m 2 + n 2 - lm - mn - nl = 0
From the above given formula, we have \ l2 + m2 + n2 = lm +mn + nl
sin B = bk, sin C = ck
3 4 2 3 4 2
bk b 76. (c)
\ cos A = = 5 8 2 = 2 4 0
2 ck 2 c
Put the value of cos A in the formula, which x y 2 x -5 y -8 0
gives
= 2 (2y – 16 – 4x + 20 ) = 2 (2y – 4x + 4 )
b b2 + c 2 - a 2 \ given determinant = 0 Þ 2y – 4x + 4 = 0
=
2c 2bc Þ 2x – y – 2 = 0 which represents st. line.
Þ c2 = a 2 Þ c = a
Solutions-Mock Test-6 MT-195

77. (b) Centre of the given circle is (1,2) and its æ pö p


81. (b) sin x – cos x = 1 Þ sin ç x - ÷ = sin Þx
radius = 1 + 4 + 20 = 5 . Since the radii of è 4ø 4
the two circles are equal, therefore these p p p p
will touch externally and the point of contact – = np + (–1)n Þ x = np + (–1)n +
4 4 4 4
will lie mid - way between the two centres . r r r r r r
If (h,k) is the centre of the circle, then 82. (c) [A - B B - C C - A]
r r r r r r
h +1 k+2 = [(A - B) ´ (B - C)]. (C - A)
= 5, =5 \ h = 9, k = 8 r r r r r r r r r
2 2 = [A ´ B - A ´ C - 0 + B ´ C].(C - A)
\ its equation is ( x –9)2 + ( y – 8)2 = 52 r r r r r r
i.e., x2 + y2 – 18x – 16y + 120 = 0 = [A ´ B].C - [B ´ C].A
r rr r rr
= [A BC] - [A BC] = 0 (Q [ABC] = [BCA])
x2 y 2 r r
78. (a) Ellipse is + =1 83. (d) Vector A = ˆi + ˆj + k,
ˆ B = 4iˆ + 3jˆ + 4kˆ
16 3
r
Now, equation of normal at (2, 3/2) is And C = ˆi + aˆj + bkˆ are linearly
16 x 3 y 1 1 1
- = 16 - 3
2 3/ 2 independent, if 4 3 4 = 0
13
Þ 8x – 2y = 13 Þ y = 4 x - 1 a b
2
Operate C3®C3 – C1 ; C2®C2 – C1
13
Let y = 4 x - touches a parabola 1 0 0
2
y2 = 4ax. we get, 4 -1 0 =0
We know, a straight line y = mx + c touches 1 a -1 b -1
a parabola y2 = 4ax if a – mc = 0
Þ–(b–1)=0Þb=1
æ 13ö r
\ a - ( 4) ç - ÷ = 0 Þ a = – 26 Again, | C | = 1 + a 2 + b 2 = 3
è 2ø
Þ 1 + a2 + b2 = 3 Þ 1 + a2 + 1 = 3
Hence, required equation of parabola is \ a2 = 1 Þ a = ± 1
y2 = 4 (– 26)x = – 104 x Hence, a = ± 1 , b = 1
79. (a) I study or I fail = p Ú q 84. (c) Lt [f (x ) + g ( x ) + h (x )]
x ®3
Now, ~ (p Ú q) º~ p Ù (~ q)
é 2 x -3 2(2 x + 1) ù
Hence, negation of 'I study or I fail' is I do = Lt ê + - ú
x ®3 ë x - 3 x + 4 x 2 + x - 12 û
not study and I do not fail.
é 2 x + 8 + x 2 - 6x + 9 - 4 x - 2 ù
æ 3p ö = Lt ê ú
80. (b) sin 4 ç - a ÷ = cos 4 a,
è 2 ø x ®3ê
ë x 2 + x - 12 úû
sin 4 ( 3p + a) = sin4 a,
x 2 - 8x + 15 (x - 3)( x - 5)
æp ö = Lt = Lt
sin 6 ç + a ÷ = cos6 a, sin6 (5p – a) = sin6 a x ®3 2
x + x - 12 x ®3 ( x + 4)( x - 3)
è2 ø
\ given quantity x -5 3-5 2
= Lt = =-
= 3[cos4a + sin4 a] – 2 [cos6 a + sin6 a] x ®3 x + 4 3+ 4 7
= 3 [1 – 2sin2 a cos2a] –2 [1 – 3 sin2 acos2a] 85. (d) We know that every quadratic equation has
=1 exactly two roots which are either, both real
or both are imaginary. So, any quadratic
EBD_7206
MT-196 JEE MAIN
equation has neither exactly one real root
nor booth roots are always real.
86. (b) Here c = ar , e = ar 2 and d = bs, f = bs2

a b 1
1
Then area of triangle, A = c d 1
2
e f 1 16
16 3/ 2
x
1 = 4 ´ 16 – ò x dx = 64 - .
= [(cf – ed ) + (be – af ) + (ad – ac )] 0
3
2
2 0
1
= [abrs2 – abr2s+abr2 – abs2 + abs – bar ] 2 128 64
2 = 64 - (4) 3 = 64 - = sq.units
3 3 3
1
= ab [ rs (s –r ) + (r 2 –s2) + (s – r )] 3p / 2 5p / 6 p
2 89. (d) ò [ 2 sin x]dx = ò [ 2 sin x]dx + ò [ 2 sin x ]dx
p/2 p/2 5p / 6
1
= ab (s –r) [ rs – s– r + 1 ]
2 7p / 6 3p / 2
+ ò [2 sin x ]dx + ò [ 2 sin x ]dx
1 p 7p / 6
= ab (s –r ) (s –1) (r –1 )
2 5p / 6 p 7p /6
87. (d) Let P ( x1, y1 ), Q ( x2,y2 ) be two points on = ò 1 dx + ò 0 dx + ò (-1) dx
p/2 5p / 6 p
1
the curve y = x + . Let î be unit vector
x 3p / 2
along x-axis and + ò (-2) dx
7p / 6
Let ĵ be a unit vector along y-axis. Then
uuur uuur æ 5p p ö

æ
- + 0 + çp -
7 p ö æ 7 p 3p ö
÷ + 2ç - ÷=-
p
OP = x1ˆi + y1ˆj, OQ = x 2ˆi + y 2ˆj è 6 2 ÷ø è 6 ø è 6 2 ø 2
uuur 90. (a) L f ¢(1)
Since, OP × ˆi = 1 and OQ.î = 1 and
f ( x ) - f (1) ax 2 + b - a - b
OQ.î = -1 = Lt = Lt
x ®1 x -1 x ®1 x -1
\ x1 = 1 and x2 = –1 Þ y1 = 2 and y2 = –2 ;
a ( x 2 - 1)
\ OP = î + 2 ĵ; OQ = - î - 2 ĵ = Lt = Lt a ( x + 1) = 2a Rf ¢(1)
x ®1 x - 1 x ®1

\ 2OP + 3OQ = 2i + 4 j - 3i - 6 j = -i - 2 j ; f ( x) - f (1) bx 2 + ax + c - a - b


= Lt = Lt
x ®1 x -1 x ®1 x -1
Hence, 2OP + 3OQ = 1 + 4 = 5 c
= Lt [ b( x + 1) + a ] +
x ®1 x -1
88. (b) y = 4 meets the parabola y2 = x at A, if
16 = x c
= 2b + a + = 2b + a if c = 0
\ A is (16, 4) x -1
Reqd. area = Area of rect. OMAC – Area Since, f is diff. at x = 1\ 2 a = 2 b + a
OMA Þ a = 2 b. Thus, result holds if a = 2 b, c = 0.
Mock Test-7
ANSWER KEY
1 (d) 16 (c) 31 (c) 46 (b) 61 (d) 76 (c)
2 (c) 17 (c) 32 (c) 47 (a) 62 (a) 77 (c)
3 (a) 18 (a) 33 (d) 48 (a) 63 (c) 78 (b)
4 (c) 19 (a) 34 (c) 49 (d) 64 (d) 79 (c)
5 (d) 20 (d) 35 (b) 50 (a) 65 (a) 80 (a)
6 (c) 21 (c) 36 (c) 51 (a) 66 (c) 81 (d)
7 (a) 22 (c) 37 (c) 52 (d) 67 (b) 82 (a)
8 (d) 23 (b) 38 (b) 53 (d) 68 (c) 83 (a)
9 (b) 24 (c) 39 (b) 54 (c) 69 (a) 84 (c)
10 (b) 25 (b) 40 (c) 55 (a) 70 (d) 85 (d)
11 (c) 26 (c) 41 (b) 56 (b) 71 (b) 86 (c)
12 (b) 27 (b) 42 (a) 57 (a) 72 (a) 87 (a)
13 (c) 28 (b) 43 (a) 58 (b) 73 (d) 88 (c)
14 (b) 29 (c) 44 (a) 59 (b) 74 (a) 89 (b)
15 (b) 30 (a) 45 (c) 60 (a) 75 (a) 90 (d)

Solutions
CHEMISTRY HA + NaOH ¾¾
® NaA + H 2 O
1. (d) (a) CH2Cl CH ® Na + + A -
NaA ¾¾ (ionized)
| KOH
¾¾¾¾ ® ||| ;
CH2Cl alcohol CH [H + ][A - ]
Ka =
CH3Cl CH [HA]
| KOH
¾¾¾¾ ® ||| ; hence true Given, pH = 6, [H+] = 1 × 10–6
alcohol
CH CH
(b) Both are position isomers K a [Acid]
[H + ] =
(c) Since, they are isomers, precentage [Salt]
of C, H and Cl in both will be same.
CH2Cl CH2OH [Salt] K 10-5
(d) = a = = 10 :1
|
hydrolysis
| [Acid] [H + ] 10-6
CH 2Cl ¾¾¾¾¾ ® CHOH;
3. (a) According to the kinetic theory of gases,
CH3 CH3 the average velocity of the molecules in
| hydrolysis
| the gas is given by the expression
CHCl 2 ¾¾¾¾¾ ® C H(OH) 2
CH3 8RT
v= where T is the absolute
- H 2O
| pM
¾¾¾ ¾® C HO
temperature and R is the gas constant. Thus
Hence, statement (d) is wrong.
the average velocity can be taken as
2. (c) ˆˆ† H + + A –
HA ‡ˆˆ proportional to the square root of the
(Unionized, weak acid and common ion absolute temperature. Hence the ratio of
effect)
EBD_7206
MT-198 JEE MAIN

the average velocity at 200ºC to that at 50ºC H


|
HCl
273 + 200 (ii) CH 3 - C = O + C 2 H 5OH ¾¾
¾®
will be equal to which is 1.21.
273 + 50
H
4. (c) Compounds Nature |
HCl
KO2 Superoxide CH3 - C - OC 2 H 5 ¾¾
¾®
| C 2H 5OH
BaO2 Peroxide OH
Hemiacetal
SiO2 Oxide
H
CsO2 Superoxide |
5. (d) In a ‘fcc’ crystal atoms are located at the CH3 - C (OC 2 H 5 ) 2
-Acetal -
centre of the 6 faces and at the 8 corners.
12. (b) The temperature of 383° K is equal to
On each face their is 1 atom which is shared
110°C. Although the salts will increase the
by 2 cells. Hence, the no. of atoms/ unit cell
boiling point of water, it should boil at or
= 6/2 = 3 below this temperature.
Again the corner atom is shared by 8 other 13. (c) FeO is gangue and SiO2 is flux to form slag
cells. Hence no. of atoms =8/8 = 1 FeSiO3.
No. of atoms/unit cell = 1 + 3 = 4
6. (c) Moissan boron, which is an amorphous FeO ∗ SiO ¾¾
↑ FeSiO3
2
low purity boron, is made by reduction of Basic impurity acidic flux slag
boric oxide with magnesium. Boron
14. (b) Since each of (i), (ii) and (iii) are hexa-
obtained has 86% purity & is amorphous.
coordinated, in the case of (ii), one of the
7. (a) Addition of a catalyst to a reaction mixture
chlorines (chloride ions) is coordinated to
has the effect of lowering the activation
the central cobalt ion and in (iii), two such
energy of the reaction by changing the
chlorides are coordinately linked. Thus, the
path or mechanism of the reaction. The
ionisable chlorides in (i) is three in (ii) it is
reaction rate increases manifold. However,
two and in (iii) it is only one.
the equilibrium constant and the enthalpy
Primary valency means the valency of the
(DH) of the reaction are unaffected.
complex cation.
8. (d) No. of atoms of hydrogen in 0.046 g of
15. (b) Polyethylene or Polyethene,
alcohol
0.046 [ CH 2 - CH 2 ] n ,
= ´ 6 ´ 10 23 ´ 6 is made from a single monomer, it is a
46
= 1 × 10–3 × 6 × 1023 × 6 = 3.6 × 1021 homopolymer.
9. (b) Since the gas B turns CuSO4 solution blue, 16. (c) Enzyme inhibition can be either reversible
it can be NH3. or irreversible. In case of irreversible
Since formula of the given compound A is inhibition the inhibitor dissociates very
M3N, A is either lithium or sodium nitride. slowly from its target enzyme because it
Of the two, Li3N is most likely since it is a
has become tightly bound to the enzyme
stable, very high melting compound.
10. (b) He is less soluble in blood. either covalently or non-covalently. Some
11. (c) (i) irreversible inhibitors are important drugs
like Penicilline and Aspirin. Thus "drug
O H induced poisonin g" may bound
|| |
(i)CH3 MgBr
CH3 - C ¾¾¾¾¾¾ ® CH3- C - OH irreversibly to the active site of the enzyme.
| (ii)H3O + | 17. (c) More the s - character, more is the stability
H CH3 of the carbanion. hence the correct order is
2-Propanol sp > sp2 > sp3.
Solutions-Mock Test-7 MT-199

18. (a) N1 V1 = N2 V2 (c) Energy change of 1 mol, Ice (s), at


N NaOH = M NaOH = 0.164 0°C ® 1 mol, Ice (s), –5°C
Total DH
Þ 25 × N = 32.63 × 0.164
= CP [H2O (l)] DT + DH freezing +
32.63 ´ .164 CP [H2O (s)] DT
N= = 0.214 N
25 = (75.3 J mol–1 K–1) (–5)K
But N H 2SO4 = 2 ´ M H 2SO 4 + (–6 × 103 J mol–1)
+ (36.8 J mol K–1) (–5) K
–1
Normality 0.214
Þ M= = = 0.107 M DH = – 6.56 kJ mol–1 (exothermic
2 2 process)
19. (a) The total numberof electrons in the
So, DH = 6.56 kJ mol–1.
molecular species given, respectively are
17, 16 and 18. Write down the electronic
n2
configuration of the molecular species and 25. (b) r = 0.529 ´
observe the number of electrons in Z
antibonding orbitals which are respectively r = 211.6 pm = 2.11 Å
are 7, 6 and 8.
20. (d) Amongst the sets, it is (d) that is not n2
Þ 0.529 ´ = 2.11Å
arranged in the order of decreasing Z
reactivity towards electrophilic
n = 2 (Balmer series).
substitution. Carboxylic acid group in the
26. (c) The secondary amines react with HNO2 to
case of benzoic acid deactivates the
give the oily nitroso derivative. Amongst
benzene nucleus towards electrophilic
the options, (c) is the secondary amine.
substitution reaction and thus it can not
27. (b) In Kjeldahl¢s method of estimation of
have higher reactivity than phenol which
nitrogen, the nitrogen present in most of
has a highly activating phenolic OH group.
the organic compounds is quantitatively
21. (c) Deviation from ideal gas behaviour is
converted into ammonium sulphate. The
greater, when the pressure is higher and
(NH4)2 SO4 so obtained is decomposed
the gas is closed to its liquefaction point
with excess of NaOH solution to give NH3
or its critical temperature. Thus, the
which is absorbed in an excess of standard
conditions of –100ºC and 4 atm pressure
HCl or H2SO4 and residual mineral acid is
among the sets given causes maximum
then titrated with standard NaOH solution.
deviation.
Thus, option (b) is the correct choice.
22. (c) Be(OH)2 is amphoteric that means it can
28. (b) NaCl is a salt of strong acid and strong
react with both acids and alkalies
base hence on dissolution will give neutral
23. (b) Ionization energy increases along a period
solution.
but reduces down a group. Electron affinity
NaCl + AgNO3 ® AgCl ¯ + HNO3
increases along a period from left to right
and deceases on moving down in a group. V ´ 45 (800 - V) ´ 20 800 ´ 29.875
24. (c) In order to calculate the enthalpy change 29. (c) + =
100 100 100
for H2O at 5°C to ice at –5°C, we need to
calculate the enthalpy change of all the 9V V
transformation involved in the process. Þ + 160 - = 239 Þ V = 316 mL
20 5
(a) Energy change of 1 mol, H2O (l), at
5°C ® 1 mol, H2O (l), 0°C 30. (a) FeCl3 (aq)+ 3NaOH (aq) ¾¾
®
(b) Energy change of 1 mol, H2O (l), at limiting (Excess amount)
reagent Not behave as
0°C ® 1 mol, H2O (s) (ice), 0°C limiting reagent
Fe(OH)3 (s) + 3NaCl (aq.)
EBD_7206
MT-200 JEE MAIN

weight in g æ pt ö
Moles of Fe(OH)3 = M. weight of Fe(OH) 35. (b) y = 2 sin ç + f ÷
3 è2 ø
2.14 g
= = 0.02 mol. dy p æ pt ö
107 g/mol velocity of particle = 2 ´ cosç + f ÷
dt 2 è 2 ø
1.0 mole of Fe(OH)3 is obtained from = 1.0
mole of FeCl3 d2y p2 æ pt ö
0.02 mole of Fe(OH)3 will be obtained from acceleration =- sinç + f ÷
dt 2 è2 ø
= 0.02 mole FeCl3
No. of moles 0.02 mole p2
Molarity = = = 0.2 M Thus, a max =
Volume in L 0.1 L 2
PHYSICS 36. (c) Escape velocity ve = 2gR
31. (c) Velocity of wave v = nl thus, it doesn’t depend on mass.
v 37. (c) Let us consider a thickness dx of wire. Let
where n = frequency of wave Þ n =
l it be at a distance x from the centre O.
v1 396
n1 = = = 400 Hz
l1 99 ´ 10 -2
dx
v 396
n2 = 2 = = 396 Hz
l 2 100 ´ 10-2 x

no. of beats = n 1 –n2 = 4 a


O
b
3rv
32. (c) v rms =
mass of the gas

5 50
33. (d) Speed, u = 60 ´ m/s = m/s
18 3 N
5 100 Number of turns per unit length =
d = 20m, u' = 120 ´ = m/s b-a
18 3 \ Number of turns in thickness dx =
Let declaration be a then (0) – u2 = –2ad
2
or u2 = 2ad … (1) N
dx
b-a
and (0)2 – u'2 = –2ad' or u '2 = 2 ad '
…(2) Small amount of magnetic field is produced
(2) divided by (1) gives, at O due to thickness dx of the wire.
d' m 0 NI dx
4 = Þ d ' = 4 ´ 20 = 80m \ dB =
d 2 (b - a) x
34. (c) Force, F = 1 q1q 2 On integrating, we get,
4pe 0 r 2 b m 0 NI dx m NI
B = òa = 0
q1.q 2 2 b-a x 2 (b - a )
Þ e0 =
4pFr 2 b dx m0 NI
So dimension of e0
òa x
=
2 (b - a )
[loge x] ba
[ AT ]2
= = [ M -1L-3T 4 A 2 ]
[ MLT -2 ][ L2 ] m 0 NI b
B= loge
2 (b - a) a
Solutions-Mock Test-7 MT-201

38. (b) The specific resistance (r) is determined


by the formula
C
XpD 2
r=
4L h
where symbols have their usual meaning. d
39. (b) For translational motion, B
mg – T = ma .....(1)
For rotational motion, A q R
a R
T.R = I a = I ....(2)
O
R
m R

m
(h + R)2 = d 2 + R 2
mg
Solving (1) & (2),
Þ d 2 = ( h + R )2 - R 2
mg mg 2 mg 2 g
a= = 2
= =
æ I ö mR 3m 3
çè m + 2 ÷ø m+ d = ( h + R )2 - R 2
R 2R 2
40. (c) We know that V/T = constant
d = h2 + 2hR
V + DV V DV 1
\ = or =
T + DT T V DT T d = 5002 + 2 ´ 6.4 ´ 10 6 = 80 km
nh
41. (b) l = ,| E |µ Z 2 / n2 ; n = 3 +Q –Q
2p 44. (a)
Þ lH = lLi and |EH| < |EL i|
a
æ T ö c
42. (a) h = çç1 - c ÷ ´100
÷
è TH ø
–Q a Q
æ T ö Electric potential at centre
Þ 70 = ç1 - c ÷ ´ 100
è 1000 ø 1 Q 1 Q
= +
T 4pe 0 2a 4pe 0 2a
0.7 = 1 - c
1000
1 Q 1 Q
T - - =0
\ c = 0.3 or Tc = 300K. 4pe 0 2a 4pe 0 2a
1000 45. (c) r = 5 cm. = 5×10–2m
43. (a) Let d is the maximum distance, upto it the –5 2
objects BE = 0.5 × 10 W/m
we know that field due to coil at
From DAOC

OC 2 = AC 2 + AO2 r=5cm
EBD_7206
MT-202 JEE MAIN

48. (a) Let the body be depressed by distance x


m 0I
centre B = from its equilibrium position. The extra
2r upthrust created is x rAg which applies to
it annuals the earth’s megnetic field whole body. If a be acceleration created
m I rA
So, 0 = 0.5 ´ 10 -5 then, xrAg = mga Þ a = x
2r m
Since, acceleration a x. So it is equation of
2 R ´ 0.5 ´ 10 -5 5 S.H.M.
I= = A = 0.4 A
m 4p
2 rA m Ta 1
46. (b) This is a problem based on constraint So, w = Þ T = 2p ;
m rA A
motion. The motion of mass M is constraint
with the motion of P and Q. Let AN = x, NO Stress
49. (d) Young’s modulus, Y =
Strain
dz or stress = Y. strain
= z. Then velocity of mass is . Also, let
dt or strain = Stress / Y
dl Fl Dl Fl A Y
or Dl = ; 1 = 11 . 2 2
OA = l . then =U YA Dl 2 A1Y1 F1l1
dt
From DANO, using pythagorous theorem l1= l2 & Y1 = Y2 , F1 = F2

\ x2 + z2 = l2 Dl1 pr2 2 4r 2
Þ = = =4
Here x is a constant. Dl 2 pr12 r2
Differentiating the above 50. (a) When springs are in parallel, then
A x N B m 2p K1 + K 2
T = 2p Þ =w=
K1 + K 2 T m
Z
l 51. (a) Given: Radius of air bubble,
P U qq U r = 0.1 cm = 10–3 m
Q Surface tension of liquid,
O S = 0.06 N/m = 6 × 10–2 N/m
M
Density of liquid, r = 103 kg/m3
equation w.r.t to t Excess pressure inside the bubble,
rexe = 1100 Nm–2
dz dl Depth of bubble below the liquid surface,
0 + 2z = 2l Þ zvM = lU
dt dt h=?
2s
l U U As we know, rExcess = hrg +
Þ vM = U= = r
z z / l cos q
2 ´ 6 ´ 10 -2
æ zö Þ 1100 = h × 103 × 9.8 +
çèQ cos q = ÷ø 10 -3
l Þ 1100 = 9800 h + 120
Þ 9800h = 1100 – 120
0.61l l
47. (a) We know that Χq < < 980
4 R Þ h= = 0.1 m
9800
The minimum distance between them 52. (d) When capacitance is taken out, the circuit
is LR.
R 9.46≥1015 ≥10≥ 0.61≥ 600≥10,9 \ tan f =
wL
l< 0.61≥l < R
9 0.3
11
= 1.15 × 10 m 1 200
Þ wL = R tan f = 200 ´ =
Þ 1.115 × 108 km. 3 3
Solutions-Mock Test-7 MT-203

Again , when inductor is taken out, the 1


circuit is CR. F ( Px ) = mu 2 - 0
2
1
\ tan f = é1 ( 2n - 1) ù = 1 mu 2
w CR or, P ( Fx ) = P ê mu 2 ú
1 1 200 ë 2 n2 û 2
Þ = R tan f = 200 ´ =
wc 3 3 n2
\ P=
2n - 1
2
2 æ 1 ö 56. (b) F = qE = mg (q = 6e = 6× 1.6 × 10–19)
Now, Z = R + ç - wL ÷
è w C ø mass m
Density (d) = =
volume 4 3
æ 200 200 ö
2 pr
= (200) 2 + ç - ÷ = 200 W 3
è 3 3ø m
or r 3 =
4
Power dissipated = Vrms I rms cos f pd
3
Vrms R æ Rö æ qE ö
= Vrms . . çQ cos f = ÷ Putting the value of d and m ç = and
Z Z è Zø
–7
è g ÷ø
solving we get r = 7.8 × 10 m
V 2rms R (220)2 ´ 200 220 ´ 220 æ - tö
R
= 2 = 2 =
Z (200) 200 57. (a) I = I o ç1 - e L ÷
çè ÷ø
= 242 W
53. (d) Magnetic field between the plates in this (When current is in growth in LR circuit)
case is zero.
æ - ´2 ö
5
Eæ - tö
54. (c) Lateral magnitude = v/u; R
5
Mag. along axis = ç1 - e L ÷ = ç1 - e 10 ÷
R çè ÷ø 5 çè ÷ø
dv u 2
= = = 1 if v = u , \ u = 2 = (1 – e–1)
du v 2
55. (a) Let u be the initial velocity of the bullet of 58. (b) line 1 line 2 line 3
mass m. 5m F I
After passing through a plank of width x, 3W 9W
its velocity decreases to v.
8.0 V 16.0 V
4 4 u(n - 1) I1 I2
\ u–v= or, v = u - =
n n n
If F be the retarding force applied by each In steady state capacitor is fully charged
plank, then using work – energy theorem, hence no current will flow through line 2.
By simplyfing the circuit
1 1 1 1
Fx = mu2 – mv2 = mu2 – mu2
2 2 2 2
3W 9W
é 2ù
( n - 1) 2 ê1 - ( n - 1) ú
2 1
= 2 mu ê
n2 ë n2 ú
û 8.0 V 16.0 V

1 æ 2n - 1 ö
Fx = mu 2 ç ÷ Hence resultant potential difference across
2 è n2 ø resistances will be 8.0 V.
Let P be the number of planks required to V 8.0 8
stop the bullet. Thus current I = = =
Total distance travelled by the bullet be- R 3+ 9 12
fore coming to rest = Px 2
Using work-energy theorem again, or, I= = 0.67 A
3
EBD_7206
MT-204 JEE MAIN

59. (b) Pressure of light on totally reflecting


Þ l 2 x 2 y 2 - 2lxy 2 + y 2 + l 2 x 2 - 2lx + 1
surface
2I = l 2 x 2 y 2 + 2lx 2 y + x 2 + l 2 y 2 + 2ly + 1
P= (C = velocity of light)
C
F 2I Þ l 2 ( x 2 - y 2 ) - 2l( xy 2 + x 2 y + x + y ) = 0
P= =
A C Þ l 2 ( x + y )( x - y ) - 2l [ xy( x + y) + ( x + y )] = 0
2IA 2 ´ 12 ´ 1.5 ´ 10 –4
ÞF= = Þ l ( x + y) [ l ( x - y ) - 2 xy - 2] = 0
C 10 –4 ´ 3 ´ 108
8 ´ 15 ´ 10 -8 Þ ( x + y) [ l ( x - y) - 2 xy - 2] = 0
= = 12 ´ 10 -8 = 1.2 ´ 10 -7 N
10 Þ l ( x - y ) - 2 xy - 2 = 0
60. (a) Total momentum will be conserved.
Initial momentum = Final momentum 2 xy + 2 xy + 1 l
Þ =l Þ =
M.v = m ´ 0 + (M - m)v ' x- y x- y 2
Mv
\v' = æ dy ö æ dy ö
M-m çè x + y ÷ ( x - y ) - ( xy + 1) ç1 - ÷
ø è dx ø
dx
MATHEMATICS Þ =1
( x - y )2
61. (d) Given f(x) = tan–1 (sin x + cos x) This is the first order differential equation
1 dy
f ¢(x) = .(cos x - sin x) and clearly degree of is 1. Hence
1 + (sin x + cos x) 2 dx
æ 1 1 ö degree of the differential equation is 1.
2. ç cos x - sin x ÷
= è 2 2 ø 63. (c) [(t -1 - 1) x + (t -1 + 1)-1 x -1 )]8
1 + (sin x + cos x) 2
-1 8
æ pö éæ 1 ö æ1 ö 1 ù
2 cos ç x + ÷
è 4ø = ê ç - 1÷ x + ç + 1 ÷ ú
\ f ¢(x) = êëè t ø è t ø x úû
1 + (sin x + cos x)2
Let Tr + 1 be the term independent of x, then
if f ¢ (x) >0 then f(x) is increasing function.
8- r -r r
Hence f(x) is increasing, if æ1 ö æ1 ö æ1ö
Tr +1 = 8Cr ç - 1÷ x 8- r . ç + 1 ÷ ç ÷
p p p 3p p èt ø èt ø è xø
- < x+ < Þ - <x<
2 4 2 4 4 8- r -r
Hence, f(x) is increasing when æ1 ö
8 æ1 ö 8 -2 r
= Cr ç - 1÷ . ç + 1÷ . x
æ p pö è t ø è t ø
x Îç - , ÷
è 2 4ø \ 8 - 2r = 0 Þ r = 4
62. (a) \ T5 is the term independent of x and
4 -4
1 + x 2 + 1 + y 2 = l( x 1 + y 2 – y 1 + x 2 ) æ1 ö æ1 ö
T5 = 8C4 ç - 1÷ . ç + 1÷
Þ 1 + x 2 (1 + ly ) = 1 + y 2 (lx - 1) èt ø èt ø
4 -4
8 æ1- t ö æ1 + t ö
1 + x2 lx - 1 = C4 ç ÷ .ç ÷
Þ = è t ø è t ø
1+ y 2 ly + 1
4 4
8 æ1- t ö 8. 7 . 6.5 æ 1 - t ö
x2 + 1 l 2 x 2 - 2l x + 1 = C4 ç ÷ = ç ÷
Þ = è1+ t ø 4 . 3 . 2 . 1è 1+ t ø
y2 + 1 l 2 y 2 + 2l y + 1
4
Þ ( y 2 + 1)(l 2 x 2 - 2lx + 1) æ 1- t ö
= 70. ç ÷
è 1+ t ø
= ( x 2 + 1)(l 2 y 2 + 2ly + 1)
Solutions-Mock Test-7 MT-205

64. (d) Equation of the tangent at the point ‘q’ is 26


x sec q y tan q Þ a + b – 2ab = ...(i)
- =1 49
a b
15
Þ P = (a cos q, 0) and Q = (0, – b cot q) And P(not A and not B) =
49
Let R be (h, k) Þ h = a cos q, k = –b cot q
k -b -bh 15
Þ = Þ sin q = and Þ P (not A) × P(not B) =
h a sin q ak 49
h 15
cos q = Þ (1 – a) × (1 – b) =
a 49
By squaring and adding,
15
b2 h2 h2 Þ 1 – b – a + ab =
+ =1 49
a2k 2 a2 34
Þ a + b – ab = ...(ii)
Y 49
From (i) and (ii),
R Q 42
a+b= ...(iii)
49
P O X 8
and ab =
49
42 42 4 ´ 8
(a – b)2 = (a + b)2 – 4ab = ´ -
49 49 49
b2 a2 a2 b2 196 14
Þ +1 = Þ - =1 = \ a–b= ...(iv)
2 2 2 2 2401 49
k h h k
Now, given eq n of hyperbola is From (iii) and (iv),
x2 y 2 4 2
- = 1 Þ a 2 = 4, b 2 = 2 a= ,b=
4 2 7 7
Hence probability of more probable of the
a2 b2 =1
4
-
2
\ R lies on - = 1 i.e., 2 4
x y 2 x 2 y2 two events =
7
65. (a) Let the probability of occurrence of first
2
event A, be ‘a’
i..e., P(A) = a 66. (c) ò
I = [ x 2 ]dx
\ P(not A) = 1 – a 0
And also suppose that probability of The function [x2] varies as follows between
occurrence of second event B, P(B) = b, x = (0, 2)
\ P(not B) = 1 – b
Now, P(A and not B) + P(not A and B) ì0 if 0 £ x 2 < 1, or 0 £ x < 1
ï
2
=
26
[ x ] = ïïí1 if 1 £ x < 2 or 1 £ x < 2
2
49 2
ï2 if 2 £ x < 3 or 2 £ x < 3
26 ï 2
Þ P(A) × P(not B) + P(not A) × P(B) = ïî3 if 3 £ x < 4 or 3 £ x < 2
49
1 2 3 2
26
Þ a × (1 – b) + (1 – a) b =
49 ò
Þ I = 0.dx + ò 1.dx + ò 2.dx + ò 3.dx
0 1 2 3
EBD_7206
MT-206 JEE MAIN

= 0 + ( 2 - 1) + 2( 3 - 2 ) + 3(2 - 3 ) 71. (b) y=x


2

= 2 -1+ 2 3 - 2 2 + 6 - 3 3 2
x=y
= 5- 2 - 3
ax + b a b
67. (b) Let y = Þ y= x+
c c c
a b Solving, y = x2 and x = y2
Þ y = Ax + B, where A = and B =
c c y = y4 or y (y3 – 1) = 0 Þ y = 0 or y = 1
So, y = Ax + B and hence \ Point of intersection are (0, 0) & (1, 1)
y - y = Ax + B - (Ax + B) = A(x - x) 1

2
Þ ( y - y) = A ( x - x ) 2 2 ò
To find the shaded area, A = ( x - x 2 ) dx
0
1
Þ å ( y - y) 2
=A 2
å (x - x ) 2
2
ë û
1é x3 ù 2 1 1
= é x 3/ 2 ù - ê ú = - =
2 2 2 3 0 ê 3 ú
ë û0 3 3 3
Þ ns y = A ( n s x ) Þ s y = A s x
72. (a) P (A Ç B)
68. (c) a + b = 3; ab = a ; g + d = +12 ; gd = b P(B)
a, b, g, d are in increasing G.P.. P(A) P (A Ç B Ç C)

b = ax , g = ax 2 , d = ax 3
P (B Ç C)
a + b = a + ax = 3 = a(1 + x) .....(1) P (A Ç C)
P(C)
g + d = ax 2 + ax 3 = 12 = ax 2 (1 + x) .....(2)
\ Probability that atleast one of the events
3 a(1 + x ) A, B, C exists is given by the shaded region.
Divding =
12 a x 2 (1 + x ) Req.
prob. = P(A) + P(B) + P(C) - P(A Ç B)
1 1
or = 2 or x = 2 - P(B Ç C) - P(C Ç A) + P(A Ç B Ç C)
4 x
1 1 1 1 5
Þ b = 2a and a + 2a = 3 Þ a = 1 and b = 2 = + + -0-0- + 0 =
4 4 4 8 8
\a = ab = 2 r
-r æ 1 ö
2 2 3 3 73. (d) Tr +1 = 6 C r x 6 çç ÷÷ = 6Cr(x)6 – r – 2r
g = ax = 1´ 2 = 4; d = ax = 1´ 2 = 8 è x2 ø
\ b = g d = 4 ´ 8 = 32 For coefficient of x 6, 6–r –2r = 6, or r = 0
69. (a) Consider the example: Let A = {1, 2, 3}, This means the term is the first term.
R = {(1, 1), (1,2)} and S = {(2, 2), (2, 3)} Þ T1 = 6 C0 x 6 = 1.x 6
Clearly R and S are transitive relations on A. Þ coefficien t of x 6 = 1
R È S = {(1, 1), (2, 2), (1, 2), (2, 3)} 74. (a) For k = 0,
R È S is not transitive as (1,3) ÏRÈS.
70. (d) I = ò log 2x dx = ò log 2x.1.dx
Using Integration by parts
2
I = log 2x. x - ò 2x .ò1.dx –1 1

1
= x log 2x - ò x . xdx + c = x log 2x – x + c it is obvious from the given
Solutions-Mock Test-7 MT-207

interval that graph will be 79. (c) n = 7


increasing from – 1 to 1 Prob. of getting any no. out 1, 2, 3, … 9 is p
Similar graphs can be obtained = 9/15
for all values of k. \ q = 6/5
75. (a) It can be also solved by comparing P (x = 7) = 7C7 p7q0
dy [Binomial distribution]
with the linear equation + Py = Q
dx æ9ö
7
æ 3ö
7
ò1.dx =ç ÷ =ç ÷
The integrating factor, I.F. = e = ex è 15 ø è 5ø
Therefore, y I.F = ò 2e2x .I.F + C 80. (a) For x ³ a , the equation becomes
x2 – 2a(x–a) – 3a2 = 0
y.e x = ò 2e2x .e x + C
Þ x = (1 + 2 )a , (1 - 2 )a
2 2e 2 x for x £ a , the equation becomes
y.e x = 2ò e3x + C = e3x + C Þ y = + ce - x x2 – 2a[–(x–a)] – 3a2 = 0
3 3
a+x a-x a-x Þ x 2 + 2ax - 5a 2 = 0
Þ x = -(1 + 6)a, (-1 + 6)a
76. (c) D = a-x a+x a-x = 0
This shows (-1 + 6) a is one of the roots.
a-x a-x a+x
81. (d) n(A) = 1000, n(B) = 500, n(A Ç B) ³ 1,
3a - x a - x a - x n(A È B) = p
n(A È B) = n(A) + n(B) – n(A Ç B)
Þ D = 3a - x a + x a - x , p = 1000 + 500 – n (A Ç B)
3a - x a - x a + x 1 £ n(A Ç B) £ 500
Hence p £ 1499 and p ³ 1000
C1 ® C1 + C2 + C3 1000 £ p £ 1499
1 a-x a-x 82. (a) y = log2 {log 2 ( x)} = log 2 {log e x. log 2 e}
= (3a - x) 1 a + x a - x = 0 Þ y = loge{logex.log2e}.log2e
1 a-x a+x Þ
dy d
= log 2 e [log e {log e x. log 2 e}]
Using R 2 ® R 2 - R1 and R 3 ® R 3 - R1 dx dx
dy 1
1 a-x a-x Þ = log 2 e. .
dx log e x. log 2 e
Þ D = (3a - x) 0 2x 0 =0
d
0 0 2x (log e x. log 2 e)
dx
or, 4x 2 (3a - x ) = 0 Þ x = 0 or 3a dy 1 1 log 2 e
Þ = log 2 e =
77. (c) We know that centroid divides the median dx log e x x x ln x
in the ratio 2 : 1.
83. (a) For f (x) to be continuous, lim f ( x ) = f (0 )
x®0
Radius of the circle = 2 ´ length of median
3 f (0) = k
2 lim sin 3x
= ´ 3a = 2a x ® 0 f (x) =lim
x ®0 = =
3 sin x
Centre of the (given) circle is C(0, 0).
Therefore the equation of the circle sin 3x

(x – 0)2 + (y – 0)2 = (2a)2 Þ x2 + y2 = 4a2 = Lim 3x = 3 é lim sinq ù
78. (b) Its contropositive is ‘sum of digits of n is x ®0
sin x êëQx ®0 q = 1úû
not divisible by 9’ x
Þ n is not divisible by 9
Þk=3
EBD_7206
MT-208 JEE MAIN

84. (c) The function breaks at x = 0 and multiples æp ö


of x. Hence the function is differentiable at 88. (c) sin -1 (1 - x ) = ç - sin -1 x ÷ - sin -1 x
è 2 ø
all other points as the function is p
continuous at all these pts. (Q cos–1x = - sin -1 x )
2
p
sin -1 (1 - x) =
- 2 sin -1 x
2
Taking sum of both sides
æp ö
At x = 0, for f (x) to be continous 1 - x = sin ç - 2 sin -1 x ÷ = cos( 2 sin -1 x )
è2 ø
lim f (0 - ) = f (x = 0) = lim f (0 + )
x® 0 x® 0 = cos 2q, where sin -1 x = q
f (x) = 0 at x = 0 1 - x = 1 - 2 sin 2 q = 1 - 2 x 2 or x(1 – 2x) = 0
RHL = lim sin( x + h ) = sin h > 0 1
x ®0 or x = 0,
2
L. H. L. = lim sin(x - h ) = sin (-h ) < 0 89. (b) Let us assume an arbitary mean a = 155.
x ®0
Hence, not differentiable at x = 0 Following table is constructed :
Similarly, f (x) is not differentiable at all X i - 155
multiples of p, i.e, n p where n = 0, 1, 2.....
Xi fi ui =
5
u i2 fiui f i u i2
p/3 140 4 –3 9 –12 36
cos x + sin x
85. (d)
ò 1 + sin 2x
dx 145
150
6
15
–2
–1
4
1
–12
–15
24
15
0
155 30 0 0 0 0
p/3 160 36 1 1 36 36
cos x + sin x
= ò sin 2 x + cos 2 x + 2 sin x cos x
dx 165
170
24
8
2
3
4
9
48
24
96
72
0
175 2 4 16 8 32
p/3 p/3 Total 125 77 311
cos x + sin x p
= ò (cos x + sin x ) 2
dx = ò dx = 3 \
0 0
æ 2ö
86. (c) The equation of the pair of tangents is given 2
Variance, s = c 2ç
ç
å fi u i2 - æç å fi u i ö÷ ÷
by SS1 = T2
n ç n ÷ ÷
ç è ø ÷ø
(3x 2 + 2 y 2 - 5)(3.12 + 2.2 2 - 5) = (3x.1 + 2 y.2 - 5) 2 è
9x2 – 4y2 – 24xy + 40y + 30x – 55 = 0 90. (d) Line is ^ to 3x + y =3
further angle, q between them can be
found by using 1
\ Slope of line, m =
3
2 h 2 - ab 2 (12)2 - (9)(-4)
tan q = = x
9 + ( -4) Equation is, y = mx + c = +c
a+b 3
2 180 12 5 \q = tan -1 12 2
= = , It passes through (2, 2) Þ 2 = +c
5 5 5 3
x-5 4
87. (a)
2
> 0 Þ x2 + 5x – 14 < x – 5 Þ c=
x + 5x - 14 3
Þ x2 + 4x – 9 < 0 x
Þ a = – 5, – 4, – 3, – 2, – 1, 0, 1 Þ y- = 4 / 3 Þ 3y - x = 4
3
a = – 5 does not satisfy any of the options
a = – 4 satisfy the option (a) a2 + 3a – 4 = 0 \ y-intercept = 4 / 3
Mock Test-8
ANSWER KEY
1 (c) 16 (d) 31 (c) 46 (a) 61 (c) 76 (b)
2 (a) 17 (b) 32 (c) 47 (b) 62 (c) 77 (b)
3 (d) 18 (a) 33 (c) 48 (c) 63 (c) 78 (b)
4 (d) 19 (b) 34 (c) 49 (a) 64 (b) 79 (d)
5 (b) 20 (d) 35 (b) 50 (d) 65 (c) 80 (a)
6 (b) 21 (c) 36 (a) 51 (c) 66 (a) 81 (d)
7 (a) 22 (b) 37 (c) 52 (c) 67 (c) 82 (d)
8 (a) 23 (d) 38 (a) 53 (a) 68 (a) 83 (b)
9 (c) 24 (c) 39 (a) 54 (c) 69 (a) 84 (c)
10 (a) 25 (c) 40 (c) 55 (a) 70 (b) 85 (b)
11 (c) 26 (c) 41 (b) 56 (a) 71 (a) 86 (c)
12 (b) 27 (a) 42 (b) 57 (b) 72 (b) 87 (c)
13 (d) 28 (c) 43 (a) 58 (d) 73 (a) 88 (c)
14 (d) 29 (d) 44 (c) 59 (b) 74 (a) 89 (b)
15 (a) 30 (d) 45 (a) 60 (b) 75 (b) 90 (d)

Solutions
CHEMISTRY D - Silicates are widely used in ion-exchange
beds in domestic and commercial water
Pm purification, softening, and other
1. (c) Density (r) =
RT applications.
(1 bar = 0.987 atm)
3. (d) H2O. Because of hydrogen-bonding the
4 ´ 0.987 atm ´ 28 g / mol intermolecular forces increase. Hence
r N2 =
R ´ 300 K boiling point increases.
Let the molar mass of gas be x 4. (d) According to Huckel’s rule of aromaticity
2 ´ 0.987 atm ´ x compound having (4n + 2)p electrons is
rgas = aromatic. In option (a), (b), (c), compounds
R ´ 300 K
have 6p e– while (d) has 5p.
Given rgas = r N ´ 2
2 z2
5. (b) For Hydrogen like species E varies as
2 ´ 0.987 atm ´ x 4 ´ 0.987 atm ´ 28 g / mol n2
= ´2
R ´ 300 K R ´ 300 K 2 p 2 me 4 z 2 z2
2
\ x = 112 g/mol. i.e. E n = - k . = E´
h2 n2 n2
2. (a) A - Silica gel packets are used to absorb moisture
and keep things dry i.e. as drying agent. (z = 1, n = 1 for H-atom);
B - Silicon is a semiconductor and is used \ for e- in Li2+ in 2nd orbit we have
in transistors. z = 3; n = 2
C - Silicone is used as sealant. 9
\ En = E ´
4
EBD_7206
MT-210 JEE MAIN

6. (b) In K3 [Fe (CN)6] {[Fe (CN)6]5– C º CH


\ y – 6 = – 3, y = 3}the oxidation state of + CH3– CH2MgBr
12. (b)
Fe is 3
\ in
C º CMg Br
OH
C2H6 +
(NH3)4Co Co(NH3)2 (SO4)X
OH
4×0 +1×3 +1×(–1)+ 1×(–1)+ 1×3 + 2×0–2x = 0 O
or 6 – 2 – 2x = 0 ; 2x = 4 ; or x = 2 ||
-
13. (d) O - S - CF3 is the best leaving group as
7. (a) Phenol has activating (electron releasing) ||
–OH group and bromine water supplies Br+ O
ion easily, hence under such conditions it is the weakest base
reaction does not stop at monobromo or 14. (d) 9 O3 + 2 I 2 ¾¾
® I 4O 9 + 9O 2
dibromo stage but a fully brominated
(2,4,6,-tribromophenol) compound is the æ 1 ö
final product. 15. (a) E = 0.0591 logç ÷ ;
ç [H + ] ÷
è ø
OH OH
pH = 3 \ [ H + ] = 10 -3 \ E = 0.177 volts
Br Br
+ 3 Br2
16. (d) NH3 + HCl ¾¾
® NH 4 Cl
moles of HCl = 0.2 M × 25 × 10–3 L = 0.005
Br moles HCl (total consumed)
2, 4, 6-Tribromophenol
moles of NH3 = 0.2 M × 50 × 10–3 L = 0.01
moles HCl
O
||
Na / alcohol
excess NH3 = 0.01 – 0.005 = 0.005 moles
8. (a) R - C - OR ' ¾¾ ¾ ¾¾® RCH 2 OH + R ' OH
1 mole ammonia = 1 mole NH4Cl
0.005 NH3 = 0.005 NH4Cl
NH 3Cl
+ – +
NH3 NO3
– Total volume
= VHCl + VNH3 = 25 + 50 = 75 mL
9. (c) AgNO3
+ AgCl
(White ppt.) 0.005 mole
[NH3 ] = [NH 4 Cl] = = 0.066 M
p-Chloroaniline does not give white ppt. 75 ´ 10 -3 L
with AgNO3.
10. (a) Lewis acid is the substance which accept [NH 4 Cl]
pOH = pK b + log
electrons. Here I2 is accepting electrons. [NH3 ]
11. (c) All ortho para directors other than
HALOGENS activate the ring to [0.066]
pOH = 4.75 + log
electrophilic substitution while all meta [0.066]
directors deactivate the ring to further
pOH = 4.75
electrophillic substitution, because halogen
can both withdraw and release electrons pH = 14 – pOH Þ pH = 9.25
through inductive effect and resonance
respectively.
Solutions-Mock Test-8 MT-211

H Cl (d) Homoaromatic (6pe–)


C
+
17. (b) aq. Na2CO3 Na

C
H Cl
18. (a) Osmotic pressure, p = CRT ; No delocalization of electron in whole ring
6 7.45 Unstable.
n CH 3COOH = , n KCl =
60 74.5
Since KCl ionises. Therefore its effective 21. (c) 2 AgBr + Na 2 S 2 O 3 ¾¾® Ag 2 S 2 O 3 + 2 NaBr;
conc., in solution increases.
19. (b) According to Le Chatlier ’s principle Boil
Ag 2S 2O 3 ¾¾¾® Ag 2S + SO 3
increase in pressure will increase the Black ppt.
reaction in the direction in which lesser no. 22. (b) The reagent is BaCl2 which imparts green
of moles are formed to offset the increase colour to flame. BaCl 2 forms chromyl
in pressure. In reverse reaction, one chloride (which is red in colour), when
gaseous mole is formed as compared to two treated with K2Cr2O7 and conc. H2SO4.
in forward direction. Hence reverse direction
is favoured. 2BaCl2 + K 2Cr2O7 + 3H 2SO4
20. (d) (a) Antiaromatic (4pe–) ¾¾
®K2SO4 + 2BaSO4 + 2CrO2Cl2 + 3H2O
Chromyl chloride
(red gas)
+
Na 23. (d) Each of them decompose aqueous solution of
aq. Na2CO3
NaHCO3. Recall that when at least two NO2–
groups are present in ortho and para positions
with respect to phenolic –OH group, it becomes
Unstable. highly acidic and gives effervescences of CO2
with aq. NaHCO3 solution.
(b) Antiaromatic (4pe–) 24. (c) The nitration of aniline is difficult to carry
+ out with nitrating mixture since –NH2 group
Na get oxidised which is not required. So the
aq. Na2CO3
amino group is first protected by acylation
to form acetanilide which is then nitrated to
Unstable. give p-nitro acetanilide as a major product.
25. (c) Equiv. mass of
(c) Aromatic (6pe–) molar mass molar mass
MnO 4- = = .
7-2 5
aq. Na2CO3 Equiv. mass of
molar mass molar mass
C 2 O 24 - = =
+ 2( 4 - 3) 2
Na
Meq. of KMnO4= 50 × 5 × 0.04 = 10 = meq
Stable. of H2C2O4 = 50 × 2 × 0.1 = 10. Hence (3).
EBD_7206
MT-212 JEE MAIN

8 6 PHYSICS
26. (c) Each cell, Cu atoms + =4
8 2
31. (c) TV g -1 = constant
12
Ag atom = =3 g -1
4 T1V1g -1 = T2V2
1
1 1 æV ö 2
Au atom = = 1. Þ T (V ) 2 =T2 ç ÷
1 è 4ø
Hence, formula, Cu4Ag3Au é Vù
K E æ1 1 ö êëQ g = 1.5, T1 = T ,V1 = V and V2 = 4 úû
27. (a) ln 1 = a ç , - ÷
K2 R è T1 T2 ø 1
æ 4V ö 2
E æ 310 - 300 ö \ T2 = ç ÷ T = 2T
ln 4 = a ç ÷ èV ø
8.314 è 310 ´ 300 ø
32. (c) According to the question,
E æ 310 - 300 ö
2ln 2 = a ç ÷ nh nh
8.314 è 310 ´ 300 ø 2pr = nl = =
p mv
0.693 ´ 2 ´ 8.314 ´ 300 ´ 310 nh nh
Ea = = 107.2 kJ/mol or mvr = or mv =
10 2p 2pr
.. mv2
OH OTs F = qv B =
28. (c) TsCl r
mv nh
or, qB = =
r 2pr.r
+ H

nh
or, r2 =
29. (d) A(g) ¾¾
® A (l) 2pqB

DH = DU + Dn g RT
nh
or, r=
Given, DH = –3RT 2pqB
Here
Dng = np – nr = 0 – 1 = –1 i.e., r µ n1/2
DH = DU - RT 33. (c) Let l be length of the organ pipe. Difference
between successive resonating
Þ -3RT = DU - RT
frequencies.
Þ -3RT + RT = DU Þ -2RT = DU
V
| DH |>| DU | = 595 – 425 = 170 =
2l
Conc. HCl (for both open and closed pipes);
30. (d) 2H3AsO4 ∗ 5H2S ¾¾¾¾¾
↑ As2S5 ∗ 8H2O
2 moles 1 mole Now, V = 170 Þ l = 340 = 1 m
1 mole ½ mole 2l 2 ´ 170
34. (c) Both fall with equal acceleration g, have
\ number of moles of H3AsO4
equal displacements in time t; therefore
35.5 extension = 0.
= < 0.25 .
142 1 1 1 1
\ number of moles of As2S5 35. (b) = + + ;
T T1 T2 T3
0.25
= < 0.125 mole. T » T3 (smallest half-life)
2
Solutions-Mock Test-8 MT-213

1 1 finally fB starts decreasing


» Þ T » T3 (smallest half-life) \ emf is positive and increasing.
T T3
Only graph (c) shows these characteristic.
36. (a) I1 = 4I0
41. (b) d m = 2 ´ 64 ´105 ´ 40 + 2 ´ 64 ´105 ´ 50
I2 = Aaverage ;
= 103 2 ´ 64 ´ 4 + 103 2 ´ 64 ´ 5
= [ I0 + I0 + 2 I0 I0 cos f]ev = 2I0
I 4I = 8 ´ 103 ( 8 + 10) = 48 km
\ 1 = 0 =2
I 2 2I 0 42. (b) Length of the rod = observed reading -
zero error
1 1 1 uf 60 ´ 24 = (Main scale division + Vernier scale
37. (c) + = Þv= = = 40 cm
u v f u -f 36 division x L.C.) - Zero error
Further differentiation gives, = (6.4 + 8 × 0.01) – (–0.04)
1 du 1 dv = 6.4 + 0.08 + 0.04 = 6.52 cm
+ =0
u 2 dt v 2 dt 43. (a) Given: F = 100 kN = 105 N
2 2 Y = 2 × 1011 Nm–2
dv v du æ 40 ö
Þ =- 2 = -ç ÷ ´ 9 = -4 cm / s l0 = 1.0 m
dt u dt è 60 ø radius r = 10 mm = 10– 2 m
38. (a) ( ) (
e s T14 - x 4 = e s x 4 - T24 ; ) From formula, Y =
Stress
Strain
T14 - x 4 = x 4 - T24
Stress F
Þ Strain = =
or 2 x 4 = T14 + T24 ; Y AY
1/ 4 1/ 4 105 105
éT4 + T4 ù é 300 4 + 200 4 ù = =
x=ê 1 3.14 ´ 10-4 ´ 2 ´ 1011
2
2
ú =ê
2
ú = 263.8 K pr 2Y
êë úû êë úû 1
39. (a) The potential difference along slabs =
628
V0 = E1 d1 + E2 d2 + E3 d3 1
Therefore % strain = ´100 = 0.16%
sd1 sd 2 sd 3 628
= + +
Î0Î1 Î0Î2 Î0Î3 44. (c) Yes, the person can catch the ball when
horizontal velocity is equal to the horizontal
s é 5 ´ 10-3 3 ´ 10 -3 2 ´ 10 -3 ù
= ê + + ú component of ball’s velocity, the motion of
Î0 êë 2 3 5 úû ball will be only in vertical direction with
s respect to person for that,
= (3.9 ´10 -3 ) = 351
Î0 vo
= vo cos q or q = 60°
s 351 2
= = 9 ´10 4 V / m
Î0 3.9 ´ 10 -3 45. (a) From Einstein's photoelectric equation
Electric field intensity in slab A
hc
K.E.l < ,f ...(i)
s 9 ´ 10 4 l
E1 = = = 4.5 ´ 104 V / m
Î0Î1 2 (for monochromatic light of wavelength l)
40. (c) Initially, fB increases as magnet approaches where f is work function
the solenoid hc
K.E.l / 2 < , f ...(ii)
\ e = – ve and increasing in magnitude. l/2
When magnet is moving inside the (for monochromatic light of wavelength l/2)
solenoid, increase in fB slow down and From question,
EBD_7206
MT-214 JEE MAIN

hc æ hc ö
K.E.l / 2 < 3(K.E.l ) Þ , f < 3çç , f÷÷÷ 51. (c) We know that, v =
dx
Þ dx = v dt
l/2 çè l ø dt
2hc hc x t
, f < 3 , 3f
l l Integrating, ò dx = ò v dt
hc hc 0 0
Þ 2f < \f=
l 2l t
46. (a) The output at C corresponds to A NAND or x = ò (v0 + gt + ft 2 ) dt
0
B or A × B = C
t
47. (b) The figure of merit of a galvanometer is the é gt 2 ft 3 ù
current required to produce unit deflection = êv0 t + + ú
ëê 2 3 úû
in the galvanometer. Mathematically, 0

K = I/q, where k is the figure of merit of the gt 2 ft 3


or, x = v0 t + +
galvanometer. 2 3
Initial amount g f
48. (c) Left amount = ; T = n×t½ At t = 1, x = v0 + + .
(2)n 2 3
9 = n 1 × t½ ......(1) 52. (c) Fractional decrease in kinetic energy of
18 = n2 × t½ ......(2) mass m
2 2
IO IO æ m2 - m1 ö æ 2 - 1ö
In case (1) = ......(3); = 1 - 1 -
1/ 2 çè m + m ÷ø = çè ÷
3
(2)9 / t 2 1 2 + 1ø
In case (2), 2
IO IO æ 1ö 1 8
I= = ....(4) = 1 – ç ÷ =1- =
1/ 2 2 è 3ø 9 9
(2)18 / t æ 9 / t1 / 2 ö
ç (2 ) ÷ Percentage loss in energy
è ø
I0 I0 8
From eq. (3) & (4), we get I = = = ´ 100 ; 90%
(3) 2 9 9
53. (a) The correct plotting of the base current vs
12345
49. (a) Concept involved l = Å; From base voltage (i.e., input characteristics of a
V
common emitter transistor) is shown by (a)
1
above l¥
V 54. (c) P
Hence, X rays of largest wavelength can
T cos q
be generated by applying lowest potential
q T
i.e.,10kV. s
50. (d) If F = 20N, 10 kg block will not move and it F = Eq = q
e0K
will not press 5 kg block So N = 0.
T sin q

mg

F s
10kg 5kg T sin q = .q .... (i)
e0 K

Breaking T cos q = mg .... (ii)


Breaking
strength strength
µ×10×g=40 µ×5×g=20
Solutions-Mock Test-8 MT-215

Dividing (i) by (ii),


2
of spherical shell = R)
sq 3
tan q =
e 0 K . mg
v2 sin 2q
\ s µ tan q Horizontal range AB =
g
55. (a) Volume of removed sphere
2
3 æ 2gh ö
4 æRö 4 æ 1ö çç ÷÷ sin(2 ´ 30°)
Vremo = p ç ÷ = pR 3 ç ÷ 1+ k2 / R2
3 è2ø 3 è 8ø =è ø
= 2.08 m
g
Volume of the sphere (remaining)
58. (d) Acceleration
4 3 4 3æ1ö r æRö
Vremain = pR - pR ç ÷ ç ÷q 0
3 3 è8ø F 3 Î0 è 2 ø rq 0 R
a= = =
4 3æ7ö m m 6m Î0
= pR ç ÷ rq R R
3 è8ø \ v 2 = 2a ( R / 2) = 2. 0 .
6m 2
Therefore mass of sphere carved and 2
1 1 q 0 rR
1 K.E. = mv 2 =
remaining sphere are at respectively M 2 12 Î0
8 2 R eq 2 + 3 R eq
7 59. (b) R eq = 1 + = ;
and M. 2 + R eq 2 + R eq
8
2 R eq + R 2 eq = 2 + 3 R eq ;
Therefore, gravitational force between these
two sphere,
R 2 eq - R eq - 2 = 0 Þ R eq = 2W
GM m
F= 60. (b) As 1g of steam at 100°C melts 8g of ice at 0°C.
r2 10 g of steam will melt 8× 10 g of ice at 0°C
7M 1 Water in calorimeter = 500 + 80 + 10g = 590g
G ´ M
8 8 7 GM 2 MATHEMATICS
= =
(3R )2 64 ´ 9 R 2
61. (c) We have the equation
41 GM 2 y2 + xy + px2 – x – 2y + p = 0
;
3600 R 2
We know any general equation
56. (a) WBC = PDV = nRDT = –nRT0
WCA = 2nRT0 ln2 ax2 + by2 + 2hxy + 2gx + 2fy + c = 0 ... (1)
nR rT0 represents two straight lines if
DQBC = n CP DT = ;
g -1 abc + 2fgh – af 2 – bg2 – ch2 = 0 ... (2)
2ln 2 - 1 é WCA ù On comparing given equation with (1), we get
Hence, effeciency = =ê ú
g /( g - 1) ë DQ BC û
1 1
57. (b) Velocity of the tennis ball on the surface of a = p, b = 1, h = , g = - , f = – 1, c = p
2 2
the earth or ground Put these value in equation (2)
2gh 1 1
v= ( where k = radius of gyration p × 1 × p + 2 × – 1 × – × – p × (–1)2 – 1
k2 2 2
1+ 2 2 2
R æ 1ö æ1ö
× ç - ÷ - p´ç ÷ = 0
è 2 ø è 2ø
EBD_7206
MT-216 JEE MAIN

64. (b) Let


2 1 1 p
Þ p + - p- - = 0 æ 2p ö æ 4p ö
2 4 4 x cos q = y cos ç q + ÷ = z cos ç q + ÷=k
è 3 ø è 3 ø

2 5p 1 k æ 2p ö k æ 4p ö k
Þp - + =0 cos q = ; cosç q + ÷ = ; cosç q + ÷=
4 4 x è 3 ø y è 3 ø z

Þ 4p2 – 5p + 1 = 0 Þ (4p – 1) (p – 1) = 0 æ 2p ö æ 4p ö
Þ cos q + cosç q + ÷ + cosç q + ÷
1 è 3 ø è 3 ø
Þ p = 1,
4 k k k
= + + = 0 Þ xy + yz + zx = 0
62. (c) L f ¢(0) = h ® 0 f (0 - h) - f (0)
lim x y z
-h
50

1 - 1 - h2
65. (c)
å 50 C r (2x - 3) r (2 - x) n -r
= lim x =0
h ®0 -h
=50 C 0 ( 2 x - 3) 0 ( 2 - x ) n + 50 C1 (2 x - 3)1 (2 - x ) n -1
1 - 1 - h2 1 + 50 C 2 (2 x - 3) 2 (2 - x ) n -2 ...... + 50 C50 ( 2x - 3) n
= lim ´
h®0 -h
1+ 1- h2 = ( x - 1)50 =50 C 0 x 50 -50 C1 x 49 + .....
-1 -1 Coeff. of x25 = -50 C 25 .
= lim =
h ®0 2 66. (a) The common root must satisfy
1 + 1- h2
( x 3 + 5 x 2 + px + q ) - ( x 3 + 7 x 2 + px + r ) = 0
Rf¢(0)=
i.e. - 2x 2 + q - r = 0
2
f (0 + h) - f (0) 1- 1- h Sum of common roots = 0
lim = lim
h®0 h h ®0 h Þ x1 = -5, x 2 = -7 .
1 1 a a
= lim = 67. (c) Let the roots be , , a, ar , ar 2
2 r
h ®0 2 r
1 + 1- h2
æ 1 1 ö
Therefore, f(x) is not differentiable at x = 0. Þ aç + + 1 + r + r 2 ÷ = 40
2 r
èr ø
Since L f ¢(0) and R f ¢(0) are finite therefore, f(x)
1æ 1 1 2ö
is continuous at x = 0. Also, ç 2 + + 1 + r + r ÷ = 10
aèr r ø
Hence f (x) is continuous but not differentiable at Now product of roots = – s
x = 0.
63. (c) Length of focal chord will be a5 = –s Þ| s |=| a |5 = 32
1 25 1 5 68. (a) We can decide denomination in 13 C1 ways.
t 2 +1 + +1 = Þ t + = ± Þ t = ±2 .
t 2 4 t 2 Then 3 cards out of this denomination in
Since slope is positive. Therefore t = 2. 4
C3 ways and remaining card in 48 ways.
æ1 ö So no. of ways
The end points will be (4, 4), ç , -1÷ . = 13 C1 ´4 C3 ´ 48 = 52 ´ 48 ways .
è4 ø
4
Slope = .
3
Solutions-Mock Test-8 MT-217

69. (a) ( a + k b ).( a - k b ) = 0 Þ| a |2 - k 2 | b |2 = 0


ex 2 cos 2x 2x sec 2 x
| a |2 |a | 2 74. (a) D '(x) = ln(1 + x) cos x sin x
k2 = Þk = ± =±
| b |2 |b| 5 cos x 2 ex - 1 sin x 2

70. (b) We have, f (x) = exp ( 5x - 3 - 2 x2 ) ex sin 2x tan x 2


5 x - 3- 2 x 2 +
1
- sin x cos x
i.e. , f (x) = e
1+ x
For Domain of f (x), 5 x - 3 - 2 x 2 should cos x 2 ex - 1 sin x 2
be +ve.
ex sin 2x tan x 2
i.e., 5x - 3 - 2 x2 ³ 0
+ ln(1 + x) cos x sin x
Þ 2 x2 - 5x + 3 £ 0
-2x sin x 2 ex 2x cos x 2
(By taking –ve sign common)
Þ 2 x( x - 1) - 3( x - 1) £ 0 = B + 2Cx + ....
Þ (2 x - 3)( x - 1) £ 0 1 2 0 1 0 0 1 0 0
Þ 2x - 3 £ 0 or x -1³ 0 Put x = 0, B = 0 1 0 + 1 0 1 + 0 1 0 = 0
3 1 0 0 1 0 0 0 1 0
Þ x£ or x ³1
2
3 é 3ù 1
\ 1£ x £ i.e., x Î ê1, ú 75. (b) sec(A - B) =
2 ë 2û cos(A - B)
3
Hence, domain of the given function is [1, ]. 1
2 =
cos A cos B + sin A sin B
71. (a) Total no. of ways in which 2 cards can be
chosen = 52 C 2 = 1326 . 1 c2
= =
b a a b 2ab
Total no. of ways in which 2 cards of spades . + .
c c c c
can be drawn (13 in no.) = 13 C 2 = 78 .
2
Therefore required probability Now tan(A - B) = sec (A - B) - 1
78 1 c4 c 4 - 4a 2 b 2
= = . = -1 =
1326 17 4a 2 b 2 4a 2 b 2
72. (b) f(1). = 3, f ¢(1). = –2, g(1). = –1, g ¢(1). = 4 | a 2 - b2 |
=
g (x )f (a ) - g(a )f (x ) 2ab
lim
x®a x-a
g ' ( x )f (a ) - g (a )f ' ( x ) 76. (b) 1 - iz
= lim =| w |= 1 (given)
x ®a 1 z -i
= g ' (a )f (a ) - g (a )f ' (a ) = 4.3 - (-1)(-2) = 10
1 - iz z |z |
Þ =1 Q 1 = 1
dy z-i z2 | z2 |
73. (a) = sin x + 2 x
dx
Þ| 1 - iz |=| z - i |
On integrating,
Þ| 1 - i(x + iy) |=| (x + iy) - i |
2x 2
y = – cos x + = x 2 - cos x + c .
2
EBD_7206
MT-218 JEE MAIN

Þ| (1 + y) - ix |=| x + i ( y - 1) | x 2 -x 2
y= + & y= -
Þ [(1 + y) 2 + (- x) 2 ] = [x 2 + ( y - 1) 2 ] 3 3 3 3
On putting y = 0, from both equations we
Þ (1 + y) 2 + x 2 = x 2 + ( y - 1) 2
get x = –2
Þ 1 + 2y + y 2 + x 2 = x 2 + y2 - 2y + 1 Þ h = –2.
79. (d) Let base = b
or, 4y = 0 \ y= 0
From this it is known that the locus of z, is
y = 0, which is a real axis. Therefore, z is
situated on real axis.
77. (b) Let the point A represents the complex h
number z, B represents wz and C represents h2 - b 2
w z.
w & w are complex cube roots of unity
2p b
clearly wz means rotation of z by and
3
Altitude (or perpendicular) = h2 - b2
2p
w2z (= wz) means rotation of wz by . 1
3 Area, A = × base × altitude
2
y 1
= ´ b ´ h2 - b2
B (w, z) 2
A (z)
dA 1 é 2 - 2b ù
Þ = h - b2 + b .
db 2 ê ú
2 h2 - b2 û
x
ë
O
D 1 é h 2 - 2b 2 ù
= ê ú
2 êë h2 - b 2 úû

C (w, z) dA h
Put =0, Þ b=
db 2
2p
\ Ð AOB = Ð BOC = Ð COA = also 1 h h2 h2
3 Maximum area = ´ ´ h2 - =
OA = OB 2 2 2 4
= OC = |z|. That is the D ABC is equilateral.
1 dx
Now AC = 2AD = 2 (OA cos 30°) 80. (a) Let I = ò 1 + sin x dx = ò x
3 2 tan
= 2 |z| = 3 |z| 1+ 2
2 x
1 + tan 2
3 3 3 2 2
Area of D ABC = (side)2 = |z|
2 2 æ 2 xö x
78. (b) Equation of tangent to circle x2 + y2 = 1is ç 1 + tan ÷ dx sec 2 dx
è 2ø
y = mx ± 1 + m 2 . This also touches the ò x
1 + tan 2 + 2 tan
x
= ò 1 + tan 2 x + 2 tan x
2

circle (x – 2)2 + y2 = 4. 2 2 2 2

2m ± 1 + m 2 x 1 x
\ =2Þm=±
1 Substitute tan = t Þ sec 2 dx = dt
3. 2 2 2
1+ m2
x
Therefore common tangents are Þ sec 2 dx = 2 dt .
2
Solutions-Mock Test-8 MT-219

Then Since this circle is passing through A(p, q)


2 dt dt -1 \ p 2 + q 2 + 2 gp + 2 fq + c = 0 ....(2)
I=ò = 2ò =2 +C
2
1 + t + 2t (1 + t ) 2 (1 + t) Circle (1) touches x-axis,
-2 \ g2 - c = 0 Þ c = g2.
= +c
x From (2), we have
1 + tan
2 p 2 + q 2 + 2 gp + 2 fq + g 2 = 0 ....(3)
x Let the other end of diameter through (p,
-1 tan
2 2 q) be (h, k), then
= 1- + (c - 1) = +b
x x h+ p k+q
1 + tan tan + 1 = - g and =-f
2 2 2 2
Where b = c – 1, a new constant Put in (3)
x
1 - tan
=- 2 + b = - tanæç p - x ö÷ + b æ h + pö
2
æ k + qö æ h + p ö
x p2 + q 2 + 2 p ç - ÷ + 2q çè - ÷ +ç ÷ =0
1 + tan è4 2ø è 2 ø 2 ø è 2 ø
2
Þ h2 + p 2 - 2hp - 4kq = 0
æx pö \ locus of (h, k)
= tan ç - ÷ + b . Clearly
è2 4ø
is x 2 + p2 - 2 xp - 4 yq = 0
p
a = - and b Î R Þ ( x - p )2 = 4qy
4
ì1 - [ x] é2 4 5 ù
ï
81. (d) f (x) = í 1 + x , x ¹ -1 and f (x) 83. ê
(b) Given that A= ê 4 8 10 úú
ïî1 , x = -1 êë- 6 - 12 - 15úû

ì1 ,x < 0
ï 2 4 5
=í 1 - x
ïî1 + x , x ³ 0 |A|= 4 8 10
-6 -12 -15
ì1 ,x < 0
ï 1 Order of given matrix = 3
ì1 ,x < 0 ï1 ,0 £ x <
2 and |A| = 2(–120 + 120) – 4(–60 + 60) + 5(–48
ï1 - 2 x ï
f (2x) = í [ ] Þ f ( 2 x) = í 1 + 48) = 0
,x > 0 £ x £1
ï1 + [ 2 x ] ï0 ,
2 \ Rank of A < 3
î ï 1 3 i.e. Rank of A either 0, 1 or 2
ï- ,1 £ x <
î 3 2 Now, Applying R 2 ® R 2 - 2R1 and
1 R 3 ® R 3 + 3R1
Þf (x), for all values of x where x < a
2 é2 4 5ù
1
continuous function and for x = and x Þ = A ê0 0 0ú
ê ú
2 êë 0 0 0 úû
= 1 f (x) be a discontinous function.
82. (d) Let the variable circle be Since the equivalent matrix in echelon form
has only one non-zero row, therefore, rank
x 2 + y 2 + 2 gx + 2 fy + c = 0 ....(1) of the matrix A = 1, i.e., r (A) = 1
EBD_7206
MT-220 JEE MAIN

r r r r r r Hence, the required plane is


84. (c) (iˆ ´ a × b )iˆ + ( ˆj ´ a × b ) ˆj + (kˆ ´ a × b )kˆ
r r r r r r y(a ' b - ab' ) + z(a ' c - ac' ) + (a ' d - ad' ) = 0
= (iˆ × a ´ b )iˆ + ( ˆj × a ´ b ) ˆj + (kˆ × a ´ b )kˆ
r r r r r r or y(ab '- a ' b) + z(ac '- a ' c) + (ad ' - a ' d) = 0
(Q a ´ b × c = a × b ´ c )
r r r r r r
= (a ´ b )iˆ + ( a ´ b ) ˆj + (a ´ b )kˆ 89. (b) x 2 - y2
r = tan ea = k (a const.)
= ar ´ b x 2 + y2

85. (b) z1 + z2
2
+ z1 - z2
2 Þ kx 2 - x 2 = ky 2 + y2
(1 - k) 2
Þ (1 - k)x 2 = (1 + k)y 2 Þ y 2 = x
= z1 2 + z2 2
+ 2 z1 z2 + z1 + z2
2 2
- 2 z1 z2 (1 + k)
dy æ 1 - k ö x y 2 x y
= 2 éê z1 + z2 ùú
2 2 2 2
= 2 z1 + 2 z2 ë û
Þ =ç ÷ = . =
dx è 1 + k ø y x 2 y x
86. (c) Now there are only two possibilities :-
Either head appears odd no. of times = event A 90. (d) Let the equation of AB is
Or head appears even no. of times = event B x –1 y – (–1) z – ( -10)
Since both are mutually exclusive and = = =k
2 –3 8
1 Let L be the foot of the perpendicular drawn
equally likely. Therefore P(1). = P(2). = .
2 from P(1, 0, 0).
1 P(1, 0, 0)
Therefore required probability = .
2
p p
87. (c) - £ sin -1 x £
2 2
p p p p
- £ 3 sin -1 a £ Þ - £ sin -1 a £
2 2 6 6
A L B
æ-pö p -1 1
sin ç ÷ £ a £ sin Þ £a£ . \ L =(2k + 1, –3k – 1, 8k – 10).
è 6 ø 6 2 2
Now, direction ratio of PL = (2k, – 3k – 1, 8k
88. (c) Equation of a plane through the line of – 10) and direction ratio of AB = (2, – 3, 8)
intersection of given plane is Since, PL is perpendicular to AB
ax + by + cz + d + l (a ' x + b' y + c' z + d' ) = 0 \ 2(2k) – 3(–3k – 1) + 8(8k – 10) = 0
Þ (a + la' )x + (b + lb' )y + (c + lc' )z + (d + ld' ) = 0 2(1 - 1) + (–3)(0 + 1) + 8(0 + 10)
It is parallel to y = 0, z = 0 Now, k =
(2) 2 + (–3) 2 + (8) 2
i.e., x-axis whose direction ratios are 1, 0, 0
0 – 3 + 80 77
\ 1(a + la ' ) + 0(b + lb' ) + 0(c + lc' ) = 0 = = =1
4 + 9 + 64 77
a \ Required co-ordinate
Þl=- .
a' = L = (2 + 1, –3 – 1, 8 – 10) = (3, – 4, –2).
Mock Test-9
ANSWER KEY
1 (b) 16 (c) 31 (a) 46 (a) 61 (b) 76 (b)
2 (c) 17 (b) 32 (a) 47 (a) 62 (c) 77 (c)
3 (b) 18 (a) 33 (b) 48 (a) 63 (a) 78 (a)
4 (d) 19 (a) 34 (d) 49 (b) 64 (b) 79 (d)
5 (a) 20 (a) 35 (d) 50 (d) 65 (d) 80 (c)
6 (c) 21 (a) 36 (c) 51 (a) 66 (b) 81 (b)
7 (a) 22 (a) 37 (d) 52 (a) 67 (b) 82 (b)
8 (d) 23 (c) 38 (c) 53 (a) 68 (d) 83 (d)
9 (c) 24 (a) 39 (c) 54 (a) 69 (a) 84 (b)
10 (c) 25 (d) 40 (b) 55 (b) 70 (c) 85 (a)
11 (b) 26 (a) 41 (d) 56 (c) 71 (c) 86 (b)
12 (d) 27 (d) 42 (b) 57 (d) 72 (a) 87 (b)
13 (d) 28 (d) 43 (c) 58 (a) 73 (a) 88 (d)
14 (b) 29 (c) 44 (a) 59 (d) 74 (c) 89 (d)
15 (b) 30 (a) 45 (c) 60 (b) 75 (b) 90 (b)

Solutions
CHEMISTRY 2. (c) It is 2- pentyne, CH3CºCCH2CH3 , which
being a non-terminal alkyne, will not give
1. (b) The energy involved in the conversion of
ppt. with ammonical Cu2Cl2 solution.
1 3. (b)
Cl2 (g) to Cl–° (aq) is given by
2
1 ( K .E.)1 n - n0 (1.5 ´ 1015 - 1 ´ 1015 ) 1
DH = D diss H °Cl2 + D eg H °Cl + D hyl H °Cl = 1 = =
2 ( K .E.)2 n 2 - n0 (2.0 ´ 1015 - 1 ´ 1015 ) 2
Substituting various values from given
4. (d) Given e = 1.60 × 10–19 C
data, we get
d = 9.17 × 10–11 m
DH = æç ´ 240 ö÷ + (–349) + (–381) kJmol –1
1 From m = e × d
è2 ø m = 1.60 × 10–19 × 9.17 × 10–11
= (120 – 349 – 381) kJ mol–1 = – 610 kJ mol–1 = 14.672 × 10–30
i.e., the correct answer is (b) % ionic character
Observed dipole moment
=
Dipole moment for 100%
ionic bond
EBD_7206
MT-222 JEE MAIN

6.104 ´ 10-30
= ´ 100 HCl
MnS ¾¾¾
® MnCl2 ¾¾¾¾
NaOH
® Mn(OH)2 ¾¾¾
® MnO(OH)2
[O]
14.672 ´ 10 -30 (Brown colour)
= 41.5% K4 [Fe(CN)6]

5. (a) To prepare unsymmetrical alkanes,


Corey – House reaction is most important. No reaction

dry
R - X + R ¢2 CuLi ¾¾¾
® R - R ¢ + R ¢Cu + LiX The above reaction path is confirming the
ether presence of Zn2+ ion.
6. (c) For a cube as given, the Cl– ions are at the 9. (c) The thermal stability of carbonates is in the
corners and one each in the face centre i.e. following order.
it is a ccp structure.
Li2 CO3 < Na 2 CO3 < K 2CO3 < Rb 2CO3 < Cs 2 CO3
For a ccp structure 4r– = Ö2 a, The face
diagonal = Ö2a Li 2CO3 is considerably less stable and
On the face diagonal there are only Cl– ions decompose readily.
\ 4r– = Ö2 a or r– = 1.414 ´ 400/4 = 141.4pm Li 2 CO 3 ¾¾
D
® Li 2 O + CO 2
10. (c) Only secondary amines give nitroso amines
on reaction with nitrous acid. This reaction
a/2 is known as Libermann’s nitroso reaction.
11. (b) Q 143.5 g of AgCl = 36.5 HCl
a/2
36.5 ´ 0.4305
0.4305 g AgCl = g HCl
7. (a) u rms = 3RT / M , i.e. u rms a 1 / M 143.5

M H 2 < M CH 4 < M NH3 < M CO2 36.5 ´ 0.4305 0.4305


geq of HCl = =
143.5 ´ 36.5 143.5
8. (d) In presence of HCl
ZnS ü HCl ZnCl 2 0.4305 1000
ý ¾¾¾® Normality of HCl = × = 0.15
MnSþ MnCl2 143.5 20
CoSü HCl
ý ¾¾¾® No reaction Normality of H 2SO 4 = 0.6 – 0.15 = 0.45
NiS þ
Strength of H 2SO 4 = 49 × 0.45 = 22.05 g/L
HCl NaOH
ZnS ¾¾¾
® ZnCl2 ¾¾¾¾
® Zn(OH)2
white ppt
12. (d) For the given reaction K = [CO2] because
concentrations of solid CaCO3 and CaO
K4[Fe(CN)6]
have been taken as unity, since [CaCO3] or
[CaO] do not come in the expression there
Zn2[Fe(CN)6] will be no effect of addition of CaCO3.
bluish white ppt
Solutions-Mock Test-9 MT-223

13. (d) 50% NaOH


2 C6 H5 CHO + NaOH ¾¾¾¾¾®
OCH3 O OCH3
C6 H5COONa + C6 H5CH 2 OH
AlCl 3
+ O Friedal-craft
acetylation 19. (a) Octahedral complexes result from d2sp3
COOH
O (inner orbital) and sp3d2 (outer orbital)
Succinic
anhydride
O hybridisation eg. [Ni(NH3)6]2+.
Zn – Hg Tetrahedral complexes are formed by sp3
conc. HCl
hybridisation. e.g. Ni(CO)4
OCH3 and square planar complexes are formed by
dsp2 hybridisation. e.g. [Ni(CN)4]2–.
20. (a) sp-hybridized carbon atom is more
electronegaive than sp2 hybridized carbon
O O which in turn is more electronegative than
(Clemmenson's reduction)
sp3 hybridized carbon. More
P P
14. (b) HO OH . electronegative atom accommodates the
OH O OH
negative charge more easily.
[H4P2O7] tetrabasic 21. (a) Hall-Heroult process is used in extraction
of Al from alumina.
15. (b) Br
O NOH
CHCH2Br Cº CH
22. (a) H2NOH H2SO4
alc. KOH
NaNH2

A
H O
N
CºCCH2CH3
NaNH2
Caprolactum
CH 3CH2 Cl

B 23. (c) Co exists as Co 3+ in [CoF 6 ] 3– . The


configuration of Co 3+ is 3d 6 or
a0 a0 0.08 3d
16. (c) a = n , 2n = = , Thus 2n = 8 or
2 a 0.01
n = 3; T = 3 ´ t1/2 = 3 ´ 10 = 30 min The configuration of CoF63– is
17. (b) x/m = kp1/n, Taking log 3d
Log x/m = log k + (1/n) log p.
\ slope = 1/n
18. (a) Aldehydes containing no a-hydrogen The unpaired electrons in the d-orbitals do
atom on warming with 50% NaOH or KOH not tend to pair up because F– is weak
undergo disproportionation i.e. self ligand. (Ligands which cause a small degree
oxidation - reduction known as Cannizzaro’s of crystal-field splitting are said to be weak.)
reaction. Hence, this complex is spin-free.
EBD_7206
MT-224 JEE MAIN

OH
1. K2CO3 3d 4s 3p
24. (a)
OH

OH Fe(CN)64 - and Co(NO2 )36-

3d 4s 3p
s 2 3
O X X X X X X d sp
Less stable
26. (a) sp-hybrid C is more electronegative than
sp2 hybrid C which is more electronegative
O
2. CH3I than sp3 hybrid C. As the hybrid state of a-
+ C changes from sp3 to sp, the acid strength
tends to increase from X to Z.
OH
Resonance stabilized 27. (d) In equation (i) Fe2(SO4)3 and in equation
(ii) Fe2(SO4)3 on decomposing will form
oxide instead of Fe.
OCH3
The correct sequence of reactions is
O ,heat CO,600°C
Fe ¾¾¾¾
2 ® Fe3 O 4 ¾¾¾¾¾ ®
OH
CO, 700°C
FeO ¾¾¾¾¾
® Fe
25. (d) In Fe(CN)36- the oxidation number of Fe = +3
28. (d) Cp = q/T ;
and so it has a 3d5 configuration
Heat required = No. of moles × Cp×T
In Fe(CN)64 - the oxidation number of Fe = +2
= (144 × 25 × 10)/(14.4 × 2) = 1250J
and so it has a 3d6 configuration.
29. (c)
In Co(NO2 )63- the oxidation number of
– +
Co = +3 and so it has a 3d6 configuration. NH2 OH NH 2 O Na

CHCl3
Fe(III) + +
aq. KOH
3d 4s 3p A B Organic layer Aq. layer

NH2 NC
Fe(CN)36-
CHCl3
3d 4s 3p KOH – C2H 5OH
2 3
X X X X X X d sp A Unpleasant smell
(Organic layer)

Fe (II) and Co (III)


Solutions-Mock Test-9 MT-225


O OH OH
COO

I1= 3A 3mF
CCl4 C E F I = 15/6
+ A + – 2
KOH
r2= 2 W
Aq. layer (B)
COO
– 18V
15V
5W 4W
r1= 1 W
30. (a) DG° = -nFE°
– +
B D 2mF G H
(i) Fe 2+ + 2e- ¾ ¾
® Fe; E 0 = - 0.47 V;
3+ -
® Fe2+ ; E 0 = + 0.77 V;
(ii) Fe + e ¾ ¾
Current in circuit ABCD =
(iii)Fe3+ + 3e - ¾ ¾
® Fe
E 18
= = 3A
R + r 5 +1
(i) DG 0 = -nFE 0 = -2(-0.47)F = 0.94F
15
(ii) DG 0 = -nFE 0 = -1(+0.77)F = -0.77F Current in circuit EFGH = = 2.5A
2+ 4
(iii) On adding : DG 0 = +0.17 F P.D across 5W = 3 ´ 5 = 15V
0
DG 0 = -nFE 0 ; E for 4 ´ 25
P.D across 4W = = 10 V
3+ DG0 0.17F 10
(Fe ® Fe) =
¾¾ = = -0.057 V
- nF -3F The two capacitors are in series
PHYSICS combination. (+ve part is connected with –
ve part)
31. (a) Given, l A = 8l, l B = l
3´ 2 6 ;
NA Ceq = = mF
NB = 3+ 2 5
e
6 mF
-l B t e -l A t \Q = ´ 25V = 30mC .
Þ N oe = No 5
e
e -lt = e -8lt e-1 P
-lt -8lt -1
e =e 33. (b) wall
Comparing both side powers 45°
O 4m A 6m
-lt = -8lt - 1
As ball is projected at an angle 45° to the
1
t=– horizontal therefore Range = 4H
7l 10
1 or 10 = 4H Þ H = = 2.5 m
The best possible answer is t = 4
7l (Q Range = 4 m + 6 m = 10m)
E 15 u 2 sin 2 q
32. (a) Q = CV; I1 = ; I2 = Maximum height, H =
R +r 6 2g
H ´ 2g 2.5 ´ 2 ´10
\ u2 = = = 100
2 2
sin q æ 1 ö
ç ÷
è 2ø
EBD_7206
MT-226 JEE MAIN

or, u = 100 = 10 ms -1 38. (c) Since particle is moving undeflected


Height of wall PA So, q E = qvB
2
1 g(OA) E 104
= OA tan q - ÞB= = = 103 wb / m 2
2 u 2 cos2 q V 10
1 10 ´ 16 39. (c) Torque at angle q
= 4- ´ = 2.4 m
2 1 1 l w,a
10 ´ 10 ´ ´ t = Mg sin q.
2 2 2
34. (d) mg sin q = ma Also t = la Q
Q
\ a = g sin q l
\ l a = Mg sin q
where a is along the inclined plane 2
\ vertical component of acceleration is g 2 é Ml 2 ù
Ml l Q I =
sin2 q .a = Mg sin q ê rod ú
3 2 ë 3 û
\ relative vertical acceleration of A with
respect to B is la sin q 3g sin q
Þ =g \ a=
g 3 2 2l
g (sin 2 60 - sin 2 30] = = 4.9 m/s2
2 gP g ´ 105
40. (b) v= Þ 330 × 330 = Þ g = 1.4
35. (d) Weknowthat escapevelocityve andorbital velocity d 1.3
vo are related as, ve = Ö2.vo or ve =1.414 vo.
g = 1.4 for diatomic gas molecules, so the
Thus for the satellite to escape from its orbit, degree of freedom is 5.
% increase in the velocity required is Hence the option (b) is correct.
= [(ve – vo) / vo] ´ 100 = ((Ö2 vo – vo) / vo] 41. (d) E = s ´ area ´ T 4 ; T increases by a factor
´ 100 = 41.4%
3
36. (c) Water fills the tube entirely in gravity .
2
less condition i.e., 20 cm. 1
37. (d) DH = mL = 5 × 336 × 103 = Qsink Area increases by a factor .
4
Qsink T 42. (b) Core of acceptance angle
< sink
Qsource Tsource q = sin -1 n12 - n 22

Tsource
[ Qsource < ´ Qsink 43. (c) Nuclear fission equation
Tsink
135
Energy consumed by freezer 92 U + 0 n1 ¾¾
® 56 Ba141 + 36 Kr92

[ w output < Qsource , Qsink + 3 0 n1 + Q(energy)


Hence particle x is neutron.
æT ö
< Qsink ççç source ,1÷÷÷ F F l T2
çè T
sink ø÷ 44. (a) Surface tension, T = = . .
l l l T2
Given: Tsource < 27°C ∗ 273 < 300K,
T2
Tsink < 0°C ∗ 273 < 273 k (As, F.l = K (energy); = V -2 )
2
l
æ 300 ö÷ Therefore, surface tension = [KV–2T–2]
ç ,1
Woutput = 5×336×103 ççè 273 ø÷÷ =1.67×105J
Solutions-Mock Test-9 MT-227

45. (c) This figure can be shown as 50. (d) Time taken for travelling vertical distance of
4.9m with zero initial velocity = Ö(2h/g)
+2q =Ö2´4.9/9.8= 1 sec. In this time, distance
A +2q travelled with horizontal velocity v should
be at least 6.2 m. Thus minimum velocity v is
3 v ´ 1 = 6.2
a a
2 or v = 6.2m/s.
51. (a) Given wave equation is y(x,t)
= e( - ax +bt + 2 ab xt )
2 2
B C – 2q
a /2
–q D –q
-[( ax )2 + ( b t ) 2 + 2 a x . b t ]
3 = e
Hence q × r1 = 2q × a = 3qa
2 - ( ax + bt ) 2
=e
hc
46. (a) eV = –W 2
l æ ö
b ÷
- çè x + tø
According to question, e a
=
hc It is a function of type y = f (x + vt)
eV = –5.01
l b
Þ Speed of wave =
12400 a
Þ eV = – 5.01, 52. (a) In the given system all four gate is NOR
2000
gate
Þ eV = 6.2 –5.01 eV = 1.2 eV = 1.2 Volt Truth Table
47. (a) Velocity is maximum when K.E. is maximum
A B (y ' = A + B) y '' = (A + y ') y ''' = (A + y '') y = y ''+ y '''
For minimum. P.E.,
0 0 1 0 0 1
dV 0 1 0 1 0 0
= 0 Þ x 3 - x = 0 Þ x = ±1
dx 1 0 0 0 1 0
1 1 0 0 0 1
1 1 1
Þ Min. P.E. = - =- J
4 2 4
A B y
K.E.(max.) + P.E.(min.) = 2 (Given) i.e.,
0 0 1
1 9
\ K.E.(max.) = 2 + = 0 1 0
4 4
1 0 0
1 2 1 1 1
K.E.max . = mvmax .
2
53. (a) T in general = 2pÖ(m/k); Half the cycle on
1 2 9 3 one side has period pÖ(m/k) and the other
Þ ´ 1 ´ vmax . = Þ vmax. =
2 4 2 half pÖ(m/2k)
48. (a) Power dissipated = (Erms) (Irms) cosq æ Dö lD D b
54. (a) S = (m - 1) t ç ÷; b= or =
Hence, power dissipated depends upon è 2d ø 2d 2d l
phase difference. æbö
or S = (m - 1) tç ÷ or 5b
49. (b) t = MB sin q (q = 90°) èlø

B1 t1 10 -6 ´ b
t = MB Þ = = (1.5 – 1) × 6 ×
B2 t2 l
.5
(since magnetic moment is same) Þ l = 5 ´ 10- 6 ´ 6 = 6 × 10–7 m = 6000Å
EBD_7206
MT-228 JEE MAIN

55. (b) Radius of circular path followed by electron violet is one which has frequency more
is given by, than green. Hence answer is violet.
58. (a) Given M = 8 × 1022 Am2
mu 2meV 1 2m d = Re = 6.4 × 106m
r= = = V
qB eB B e m0 2M
Earth’s magnetic field, B = .
4p d3
B2 r 2 e
Þ V= = 0.8V 4 p ´ 10 -7 2 ´ 8 ´ 1022
2m = ´ @ 0.6 Gauss
4p (6.4 ´ 106 )3
For transition between 3 to 2.
59. (d) It is E + 4 p P for a long cavity || to E ; it is
æ 1 1 ö 13.6 ´ 5
E = 13.6 ç - ÷ = = 1.88eV
è 4 9ø 36 4pP
E+ for a spherical cavity and
Work function = 1.88 eV – 0.8 eV = 1.08 eV 3
» 1.1eV different for other shapes.
56. (c) At equilibrium, pressure is same, temp. and Df (W2 - W1 )
60. (b) =
molecular weight do not change (given). Dt t
Using ideal gas eq. P1V1 = n1RT1 Rtot = ( R + 4 R )W = 5R W
V m nd f -n(W2 - W1 )
From above, V is constant, 1 = 1 ; i= =
m V 2 m 2
Rtot dt 5 Rt
(Q W2 & W1 are magnetic flux)
V1 + V2
Add 1 on both sides, we get =
V2 MATHEMATICS
m1 + m 2
61. (b) We have
m2
V m 2m 2 r ´a + r ´b = a ´b + b ´a = 0
Þ V +2V = 2
=
m + 2m
=
3
1 2 m +
1 m2 Þ r ´ (a + b ) = 0

57. (d) In first case : Þ r = l (a + b ) = l (–2 î – ˆj + 2 k̂ )

1 1 1 4 -1 1
f1 = = RZ 2 ( - ) = RZ 2 ( ) Þ r̂ = (–2 î – ĵ + 2k̂ )
l1 4 16 16 3
62. (c) Let l1, m1, n1 and l2, m2, n2 be the d.c of line
3 1 and 2 respectively, then as given
= RZ 2 = .18RZ 2 l1 + m1 + n1 = 0
16
(green light) and l2 + m2 + n2 = 0
and l12 + m12 – n12 = 0 and
1 1 1 l22 + m22 – n22 = 0
In second case : f 2 = = RZ 2 ( - )
l2 4 25 (Q l + m + n = 0 and l2 + m2 – n2= 0)
Angle between lines, q is
2 21
or RZ ´ = .21RZ 2 cos q = l1l2 + m1m2 + n1n2 ...(1)
100
As given l2 + m2 = n2 and l + m = – n
Hence f2 > f1 Among the options, only Þ (– n)2 – 2lm = n2 Þ 2lm = 0 or lm = 0
Solutions-Mock Test-9 MT-229

So l1m1 = 0, l2m2 = 0 r r r
Equating the coefficient i , j, k , we get
If l1 = 0, m1 ¹ 0 then l1m2 = 0
If m1 = 0, l1 ¹ 0 then l2m1 = 0 1 = p + 4q,
If l2 = 0, m2 ¹ 0 then l2m1 = 0 a = p + 3q and b = p + 4q.
If m2 = 0, l2 ¹ 0 then l1m2 = 0
Also l1l2 = 0 and m1m2 = 0 From first and third, b = 1.
l2 + m2 – n2 = l2 + m2 + n2 – 2n2 = 0 Now | rc | = 3 Þ1+ a + b =3
2 2

1 Þ a 2 = 1, \ a = ± 1.
Þ 1 – 2n2 = 0 Þ n = ±
2
Hence , a = ± 1, b = 1.
1 1
\ n1 = ± , n2 = ± p+q p-q
2 2 66. (b) cosecq = , sin q =
p-q p+q
1 cos q =
\ cos q = q = 60° (acute angle)
2
2
63. (a) If A will be final winner æ p - qö 2 pq
± 1 - sin 2 q = 1 - ç ÷ =
è p + qø ( p + q)
Þ (A beats B ) (B beats C)
+ (A beats C) (C beats B) æ p qö
cot ç + ÷
è 4 2ø
2 2 æ 1 - 2 öæ 1 - 2 ö 4 1 5
= ´ +ç ÷ç
3 3 è 3 øè 3 ø
÷ = + =
9 9 9
p q q
64. (b) We have, P È Q cot cot - 1 cot - 1
4 2 = 2
=
p q q
= {x Î R : f ( x ) = 0 or g (x ) = 0} cot + cot cot + 1
4 2 2
= {x Î R : f ( x) g (x ) = 0}
q q
(a) represents the set for which either f (x) = 0 cos - sin
2 2
and g (x) = 0 or f (x) = – g (x) = q q
cos + sin
(c) represents the set for which f (x) and g 2 2
(x) both zero. On rationalizing denominator, we get
r r r
65. (d) If a, b, c are linearly independent vectors, æ q qöæ q qö
cos - sin cos + sin
then rc should be a linear combination of ç 2 2 ÷ ç 2 2÷
ç q q÷ç q q÷
r r çè cos + sin ÷ø çè cos + sin ÷ø
a and b . 2 2 2 2
r r r
Let c = pa + qb cos q
=
q 2q q q
( )
2
i.e. ˆi + aˆj + bkˆ = p ˆi + ˆj + kˆ + sin + cos + 2sin cos
2 2 2 2

(
q 4iˆ + 3jˆ + 4kˆ )
EBD_7206
MT-230 JEE MAIN

cos q 2 pq / ( p + q ) pq Y

x
t an
= = =

y=
1 + sin q ( p - q) p

y=
1+

co
p+q

tx
q X
= O p/4 x = p/2
p
p/4 p/2
67. (b) The displacement of x for all S.H.M. is given = [ log sec x ] 0 + [ log sin x ] p / 4
by

dx = log 2 + log 2 = 2 log 2 = log 2


x = a cos (nt + b) Þ = - na sin (nt + b)
dt
70. (c) sec x. tan x
d2x
ò 1 + sec x dx = ò
sec x. sec x - 1
dx
Þ = - n 2 a cos (nt + b)
dt 2 =ò
tan x
dx
sec x - 1
d2 x (Put sec x = t Þ sec x tanx dx = dt)
Þ 2
= -n 2 x
dt
dt y dy
2
= òt t -1
= 2ò
2
( y + 1) y
(where y2 = t – 1)
d x
Þ 2
+ n 2x = 0
dt
= 2 tan -1 y + C = 2 tan -1 sec x - 1 + C
68. (d) Seeing the options we can assume that
2
\ f ( x ) = tan -1 x and g (x) = sec x - 1.
ò e (log x + ax ) cos( bx 2 + c)dx

P' ( x )
K æ bö 71. (c) y2 = P(x) Þ 2 yy1 = P' (x ) Þ y1 =
= eax cos ç bx 2 + c - tan -1 ÷ + A
2

è 2y
2
a +b 2 aø

Put b = 0 & a = 1 2 yP' ' ( x ) - P' ( x ).2 y1


\ y2 =
4y2
Þ ò xe x cos c dx = Kex cos(c) + A
2 2

cos c P' ( x )
2 ò
Þ 2xe x dx = Ke x cos c + A 2 yP ' ' ( x) - P' ( x).
2 2

y
=
4y 2
cos c x
Þ e = Ke x cos c + A
2 2

2
2 y 2 P' ' ( x) - [P' (x )]2
Þ y2 =
2 1 4y3
Comparing coeff. of e x cos c , we get K =
2
2P(x )P' ' (x ) - [P' ( x)]2
=
69. (a) Required area 4y 3

p/4 p/2
= ò0 tan x dx + ò
p/4
cot x dx
Solutions-Mock Test-9 MT-231

Þ 2 y3 y 2 =
1
2
[
2P( x )P' ' (x ) - {P ' (x )}2 ] It meets co-ordinate axes in points
A (a, 0, 0), B(0, b, 0), C(0, 0, c).

d 3 1 2 P( x ) P ' ' ' ( x ) + 2P ' ( x ) P' ' ( x ) ù æa b cö


Þ2 ( y y 2 ) = éê The centroid of DABC is ç , , ÷
dx 2ë - 2P ' ( x ) P' ' ( x ) úû è 3 3 3ø

d æç 3 d 2 y ö÷ a b c
Þ2 y = P( x ) P' ' ' ( x ) Þ = a, = b, = g
dx çè dx 2 ÷ø 3 3 3
Þ a = 3a, b = 3b, c = 3g
æ 1- x ö
72. (a) f ( x ) = log ç ÷ Hence the required plane is
è 1+ x ø
x y z x y z
+ + = 1 i.e., + + = 3 .
æ 1+ x ö æ 1- x ö 3a 3b 3g a b g
\ f ( - x ) = log ç ÷ = - log ç ÷ = -f ( x )
è 1 - x ø è 1+ x ø 74. (c) The given equation reduces to
\ It is an odd function ( x - 1) 2 y 2
- = 1 . Thus a2 = 9, b2 = 3
2 2
(b) f ( x) + f (-x) = log ( x + x +1) + log(-x + x +1) 9 3
b2 a2 (e 2
ì
î
ü
(
= log í( x + x 2 + 1) (- x + x 2 + 1 ý = log x 2 + 1 - x 2
þ
) Using = – 1), we get
2
3 = 9(e 2 - 1) Þ e = .
= log (1) = 0 \ f (–x) = – f (x) 3
\ It is an odd function
75. (b) The equation of the ellipse is
x x
(c) f(x)= x + +1; 9( x + y - 2) 2 + 16(x - y) 2 = 144 ....(1)
e –1 2
Differentiating (1) with respect to x, treating y as
x constant,weget, 18( x + y - 2) + 32( x - y) = 0 ....(2)
–x x - xe x
f(-x)= –x
- +1 = x
- +1
e -1 2 1- e 2 Differentiating (1) with respect toy, treating x as constnat,
we get , 18( x + y - 2) - 32( x - y) = 0 ....(3)
ì x x ü ìï -xe x x üï
f(x)-f(-x)= í + +1ý - í - +1ý Solving (2) and (3), we get the coordinates of the
î e x -1 2 þ ïî 1-e x 2 ïþ centre (1, 1).
76. (b) Let S º x2 – 4y
x(1 - e x ) Since the point (2a, a) lies inside the parabola,
+ x = -x + x = 0
ex - 1 \ S (2a, a) = 4a2 – 4a < 0
or a (a – 1) < 0 ...(1)
\ f(–x)=f(x) Also, the vertex A (0, 0) and the point (2a, a) are
Hence, it is an even function on the same side of the line y = 1 (the equation of
(d) f (x) = e2x + sinx; latus rectum)
So, a – 1 < 0 i.e. a < 1 ...(2)
f (–x) = e–2x + sin (–x) = e–2x – sinx ¹ ±f(x)
73. (a) Let the equation of the required plane be
x y z
+ + = 1 ...(i)
a b c
EBD_7206
MT-232 JEE MAIN

Y
Þ 9ax +1 , 9 bx +1 , 9cx +1 are in G.P..

S (0, 1)
[See the properites of A.P & G.P.]

y=1 p
sin (2k - 1)x - sin(2k - 3)x
(2a, a) 79. (d) a k - a k -1 = ò dx
0
sin x

X p p
A 2sin 2 (k - 1) x ù
= ò 2 cos 2 (k - 1)x dx = ú =0
0
2 (k - 1) û0
From (1) and (2), we have a (a – 1) < 0
or 0 < a < 1 for k = 2, 3, 4
Þ a1 = a2 = a3 = ..............
D bc bc (b + c - a )
77. (c) r = = = Þ the sequence is a constant sequence.
s a + b + c (b + c + a ) (b + c - a )

bc (b + c - a) bc (b + c - a)
80. (c) å a(b - c) = 0 , then both roots of equation
= = [ S i n c e is 1.
{ (b + c )2 - a 2 } 2bc
c(a - b)
Product of roots = 1 =
2
a =b +c ] 2 2 a(b - c)
C
Þ ab – ac = ca – bc

2ac
2ac = bc + ab Þ b=
a+c
Hence, roots are in H.P.
A B
b+ c-a z1 + z3 z 2 + z 4
\r = 81. (b) Since z1 + z3 = z2 + z4 Þ =
2 2 2
D (z4) C (z3)
2 1 1
78. (a) = +
c+ a b+ c a+ b

2 b+ a+ c
=
( b + c) ( a + b)
A (z1) B (z2)
Þ 2 ab + 2b + 2 ac + 2 bc
æ Midpoint of ö æ Midpoint of ö
Þ ç =
= 2 bc + 2 ac + c + 2 ab + a è z1 and z3 ÷ø çè z 2 and z 4 ÷ø
Þ 2b = a + c Þ Quadrilateral ABCD is a parallelogram.
\ a , b, c, are in A.P. Also, | z1 – z3 | = |z2 – z4 |

Þ ax, bx , cx, are in A.P.

Þ ax + 1, bx + 1, cx + 1, are in A.P.
Solutions-Mock Test-9 MT-233

85. (a) The number of ways to give image to each


æ Distanceö æ Distance ö element of E in F is 2.
i.e. ç Between ÷ = ç Between ÷ \ the total number of ways to give images
ç ÷ ç ÷
è z1 and z3 ø è z 2 and z 4 ø to the elements of E in F = 2 × 2× 2× 2.
But in two of them all the elements of E have
Þ Length of Diagonals are equal. the same image 1 or the same image 2 (the
\ The parallelogram ABCD is a rectangle mapping being into in these two cases).
\ the number of onto functions = 24 – 2 = 14
æ z1 - z 2 ö 86. (b)
Now, arg ç z - z ÷ i.e. angle between z1,
è 3 2ø (a) We have
f (x) = 6x – 1, x Î R.
p
z2 and z3 (taken in order) is ± which is Let f (x1) = f(x2), x1, x2 Î R
2
Þ 6x1–1 = 6x2 –1 Þ 6x1 = 6x2 Þ x1 = x2.
angle between sides AB and BC (see figure)
\ ‘f’ is one-one.
82. (b) f(x) = ex and g(x) = sin–1x and h (x)= f (g(x))
(b) We have f (x) = x2 + 7, x ÎR.
-1
Þ h(x) = f(sin–1 x) = e
sin x f(–2) = (–2)2+ 7 =11, f (2) = (2)2 + 7 = 11
\ The images of distinct elements –2 and 2 of R
-1
esin x are equal.
Þ h'(x) = \ ‘f’ cannot be one-one.
1 - x2
f (x) = x3, x Î R.
h '(x) 1 Let f (x1) = f(x2), x1, x2 Î R
Þ =
h(x) 1 – x2 Þ x13 = x 2 3 Þ x13 – x 2 3 = 0
83. (d) f (x) = sin|x| - |x|
Þ ( x1 - x 2 ) ( x1 2 + x1 x 2 + x 2 2 ) = 0
f ( x ) x =0 - = -sinx + x Þ f ¢(x) = -cosx + 1
Þ x1 - x 2 = 0 Þ x1 = x 2 ,
f ( x ) x =0+ = sinx - x Þ f ¢(x) = cos x - 1 because the other factor cannot be zero
\ ‘f’ is one-one
Þ Lim- f (x )= Lim+ f ( x ) = 0
x ®0 x ®0
2x + 1
(d) We have f ( x) = , x ÎR - {7}.
- x2 -1 æx ö
x-7
84. (b) f (x) = 4 + cos çè - 1÷ø + log(cos x)
2
Let f ( x1 ) = f ( x2 ), x1 , x2 Î R - {7}.
æx ö
f (x) is defined if – 1 £ ç - 1÷ £ 1 and cos 2x1 + 1 2x 2 + 1
è2 ø Þ =
x> 0 x1 - 7 x2 - 7
x p p Þ - 15x1 = -15x 2 Þ x1 = x 2 .
or 0£ £ 2 and - < x <
2 2 2
\ f is one-one.
p p
or 0 £ x £ 4 and - < x < -1
2 2 é 1 ù
87. (b) ê
f ( x) = x + e
2 2- x ú
and f (2) =k
é pö ê ú
\ x Î ê 0, ÷
ë 2ø ëê ûú
EBD_7206
MT-234 JEE MAIN

If f (x) is continuous from right at x =2 then dA dl db


Given : = -5 , = 2, = -3
lim f ( x ) = f ( 2 ) = k dt dt dt
x ®2 + We know, A = l × b
-1 dA db dl
é 1 ù Þ = l. + b. = -3l + 2b
Þ lim x + e x ú
ê 2 2 - dt dt dt
x®2+ ê ú Þ – 5 = – 3l + 2b.
ë û When b = 2, we have
= k Þ k = lim f ( 2 + h ) 9
h ®0 – 5 = – 3l + 4 Þ l = = 3m
3
-1
é 1 ù 90. (b) The equation c ( y + c) 2 = x3 ...(i)
ê 2 2 -( 2 + h ) ú
Þ k = lim ê( 2 + h ) + e ú has one arbitrary constant, so the equation
h ®0 must be of the first order. Differentiating (i)
êë úû
we get
-1 dy
Þ k = lim é4 + h 2 + 4h + e -1/ h ù 2c ( y + c ) = 3x 2 ...(ii)
h ®0 ë û dx
-1 2
1 æ dy ö
Þ k = é 4 + 0 + 0 + e-¥ ù Þk= . Þ 4c 2 ( y + c )2 ç ÷ = 9 x 4 ...(iii)
ë û 4 è dx ø
é1 2 1 2 2 4 3 2 9 ù Divide (iii) by (i), we get
ê 2 sec 2 + 2 sec 2 + 2 sec 2 ú 2
88. (d) lim ê n n n n n n ú æ dy ö
4c ç ÷ = 9 x Þ c =
9 x
n®¥ ê 1 2 ú è dx ø
+.... + sec 1ú 4 æ dy ö 2
êë n û çè ÷ø
dx
r r2 Put the value of c in (ii), we get
is equal to lim 2
sec2 =
n®¥ n n2 é ù
2 ê ú
1 r r 9 x ê 9 x ú æ dy ö 2
lim . sec2 2 2× y + ç ÷ = 3x
n®¥ n n n 4 æ dy ö 2 ê 4 æ dy ö 2 ú è dx ø
Þ Given limit is equal to value of çè ÷ø ê çè ÷ø ú
dx ë dx û
1
integral ò x sec 2 x 2 dx é ù
0
ê ú
9 x ú æ dy ö
Þ 9x ê y + = 6x 2 ç ÷
1
1
1
1 ê 4 æ dy ö 2 ú è dx ø
2 x sec2 x 2dx = ò sec2 t dt [put ê ç ÷ ú

or
2 êë è dx ø úû
0 0
2
x =t æ dy ö 27
2
æ dy ö
3

Þ 2xdx = dt] Þ 3y ç ÷ + x = 2x ç ÷
è dx ø 4 è dx ø
1 1 1
= (tan t ) = tan1 . 2 3
2 0 2 æ dy ö æ dy ö
89. (d) Let A be the area, b be the breadth and l be Þ 12 y ç ÷ = 8 x çè ÷ø - 27 x
è dx ø dx
the length of the rectangle.
Mock Test-10
ANS WER KEY
1 (d) 16 (b) 31 (b) 46 (c) 61 (c) 76 (d)
2 (d) 17 (a) 32 (a) 47 (b) 62 (d) 77 (d)
3 (a) 18 (c) 33 (b) 48 (c) 63 (d) 78 (b)
4 (b) 19 (b) 34 (c) 49 (a) 64 (b) 79 (b)
5 (c) 20 (c) 35 (b) 50 (d) 65 (c) 80 (a)
6 (b) 21 (d) 36 (d) 51 (b) 66 (a) 81 (c)
7 (d) 22 (d) 37 (b) 52 (a) 67 (d) 82 (b)
8 (a) 23 (b) 38 (c) 53 (c) 68 (c) 83 (c)
9 (c) 24 (c) 39 (c) 54 (b) 69 (d) 84 (a)
10 (b) 25 (d) 40 (c) 55 (b) 70 (d) 85 (a)
11 (c) 26 (a) 41 (c) 56 (a) 71 (a) 86 (b)
12 (d) 27 (b) 42 (a) 57 (a) 72 (d) 87 (d)
13 (c) 28 (a) 43 (d) 58 (c) 73 (c) 88 (b)
14 (b) 29 (a) 44 (c) 59 (a) 74 (c) 89 (c)
15 (a) 30 (c) 45 (a) 60 (d) 75 (a) 90 (d)

Solutions
CHEMISTRY n1 = 3, n2 = 4

1. (d) This type of isomerism occurs when both 1 æ1 1 ö 1 R7


= Rz 2 ç 2 - 2 ÷ Þ =
cation and an ion are complex. The l è 3 4 ø l 36
isomerism is caused by the interchange of
1
ligands between the two complex ions of R=
A
the same complex. Examples are
(i) [Co(NH 3 ) 6 ] [Cr(CN) 6 ] and 1
´7
[Cr(NH3)6][Co(CN)6] 1 A 1 7 36 A
= Þ = Þl=
(ii) [Co(NH3 ) 6 ] [Cr(C 2 O4 )3 ] and l 36 l 36 A 7
[Cr(NH3)6][Co(C2O4)3] 3. (a)
2. (d) For Lyman series (shortest wavelength) CH2 COO, CH 2 –
n1 = 1, n2 = ¥ | + 2H2O || + 2CO2 + H2 + 2OH
CH 2 COO, CH2
1 æ 1 1 ö anode cathode
= Rz 2 ç 2 - 2 ÷ Total equivalent of
l ç ÷
è n1 n2 ø
C2 H 4 ∗ CO 2 ∗ H 2 < 0.2 ∗ 0.2 ∗ 0.2 < 0.6
1 æ1 1 ö 1 0.2 0.2 0.2
Þ = 12 R ç - ÷ Þ = R ∗ ∗ <0.4
Total moles of gases =
A è 1 ¥ ø A 2 1 2
st
Longest wavelength = 1 line nRT 0.4≥ 0.0821≥ 273
V< < < 8.96 L.
P 1
EBD_7206
MT-236 JEE MAIN

4. (b) Atomic mass of carbon is less and hence


no. of moles will be more.
5. (c) W2 = 4g, V = 1000 mL = 1 L, p = 6.0 ´ 10–4 H H Cl
atm Cl3C – C = O +
T = 300K, R = 0.0821 Latm. K–1 mol–1 Trichloro H Cl
acetaldehyde
p = W2´RT/(M2V)
\ M2 = 4 ´ 0.0821 ´ 300/ (6 ´ 10–14 ´ 1)
= 16.42 ´ 104 = 1.6 ´ 105 Cl
Cl3C – CH
6. (b) NH2CONH2 + NH2 –NH2 ¾¾®
NH2NHCONH2 + NH3 Cl
DDT
(Semicarbazide)
12. (d) [Zn(NH3)4]2+, Ni(CO)4 and [Cd(CN)4]2– all
7. (d) H 2C - CH - CH 2 form tetrahedral structures where the central
| | |
CN CN CN atom uses sp3 hybridisation.
Whereas [Cu(NH 3 ) 4 ] 2+ uses dsp 2
3-Cyano-1, 5-pentadinitrile
hybridisation and forms square planar
8. (a) The linking of identical atoms with each other structure. Although Cu should use both
to form long chains is called catenation. tetrahedral (sp3) and square planar (dsp2)
However, this property decreases from hybridisations but the later one is confirmed
carbon to lead. Decrease of this property is through X-ray analysis.
associated with M-M bond energy which 13. (c) Diallyl ether has double bond which shows
decreases from carbon to lead. addition reaction with KMnO4 (change its
9. (c) Polyacrylonitrile (PAN), acrilan or orlon colour) while di-n-propyl ether does not
react with KMnO4.
CN CN 14. (b) Mn2+ (3d 5) is more stable then Mn 3+ (3d 4).
| | due to half filled orbitals.
n CH 2 = CH ¾¾
® (CH 2 - CH)-n
Mg
Acrylonitrile orlon 15. (a) CH3CH2CH2Cl ¾¾¾
®CH3CH2CH2MgCl
ether
Grignard reagent
It is hard used in preparing cloths, carpets.
(1) HCHO
D ¾¾¾¾¾
+ ® CH3 - CH 2 - CH 2 - CH 2 - OH
10. (b) 3Mg+ N 2 ¾¾® Mg3 N 2 ; (2) H3O Primary alcohol
X Y
16. (b) Total pressure of mixture of gases A,B & C
Mg 3 N 2 + 6H 2 O ¾
¾® 3Mg (OH ) 2 + 2 NH 3 ­ = 10atm.
( colourless )
Partial pressure of A = (No. of moles of
CuSO 4 + 4 NH 3 ¾
¾® [Cu ( NH 3 ) 4 ] SO 4 A ´ 10)/ 10
Blue complex or (No. of moles of A ´ 10)/ 10 = 3
11. (c) Chloral on reaction with chlorobenzene in Hence, no. of moles of A = 3
the presence of a catalytic amount of \ Partial pressure of B = (No. of moles of
sulphuric acid forms DDT B ´ 10)/ 10
(dichlorodiphenyl Trichloro ethane). (since partial pressure of B = 1atm)
\ No.of moles of B = 1
Now the no. of moles of C = 10 – ( 3+1) = 6;
1 mole of C weighs = 2 g.
\ 6 mole of C will weigh = 2 ´ 6 = 12
Solutions-Mock Test-10 MT-237

17. (a) Whenever dehydration can produce two


different alkenes, major product is formed OH O
according to Saytzeff rule i.e. more ||
C LiCu (CH )
substituted alkene (alkene having lesser H ¾¾¾¾¾

number of hydrogen atoms on the two H H 2O


C2 H 5
doubly bonded carbon atoms) is the major
product.
Such reactions which can produce two or CH 3 O
more structural isomers but one of them LiAlH
H ¾¾ ¾¾4®
in greater amounts than the other are H O 2
called regioselective ; in case a reaction is C2 H5
100% regioselective, it is termed as
regiospecific.
CH3
In addition to being regioselective, alcohol
dehydrations are stereoselective (a 5 1
2
reaction in which a single starting 6 3 OH
4
material can yield two or more C2 H 5
stereoisomeric products, but gives one of
them in greater amount than any other). 21. (d) As3+ and Cd2+ are the radicals of group II,
whereas Ni2+ and Zn2+ are the radicals of
group IV. The solubility product of group
C6H 5 , CH 2 , CH, CH , CH 3 ¾¾¾¾¾¾
2 4↑ Conc. H SO IV radicals is higher as compared to group
| | II. NH4OH increases the ionisation of H2S
OH CH 3 by removing H+ of H2S as unionisable
water.
H H H CH(CH3)2
C=C + C=C H2S 2H+ + S2–; H + + OH - ¾¾® H 2 O
C6H5 CH(CH3)2 C6H5 H Thus, excess of sulphide ions are present
cis trans which leads to the precipitation of all the
(minor) (major)
four ions.
18. (c) ˆˆ† 3Zr 4 + + 4PO 43-
é Zr3 ( PO 4 ) ù ‡ˆˆ Note : HCl decreases ionisation of H2 S
ë 4û whereas NH4OH increases the ionisation
3S 4S
Ksp = (3S)3 (4S)4 of H2S.
= 27S 3 × 256 S 4 22. (d) The maximum concentration of SO42– will
be same as the solubility of M2SO4 ;
= 6912 S 7.
If solubility is x mol L–1 M2SO4
1/7
æ K sp ö 2M+ + SO42–
\ S=ç
è 6912 ÷ø Ksp =4x , therefore x = (Ksp /4) 1/3
3

[SO4–2] = x = (Ksp /4) 1/3


log 1.837 = [(3.2 ´ 10–5)/4 ] 1 / 3
19. (b) (1.5)n = 1.837, n = =1 . 5
log1.5 = 2 ´ 10 M–2
23. (b) The reaction proceed through
O Markownikoff’s rule by ionic mechanism.
NaOH, H O
20. (c) ¾¾ ¾ ¾ ¾
2¾®
+
H D The initial carbonium ion (CH 3CH 2 C H 2 )
rearrange into more stable
+
(CH3 C HCH3 ) carbonium ion.
EBD_7206
MT-238 JEE MAIN

24. (c) Solid potassium dichromate when heated 29. (a)


with concentrated sulphuric acid and a
soluble chloride gives orange red vapours CH2
of a volatile oily liquid CrO2Cl2
CH +
K2Cr2O7 + 4NaCl + 6H2SO4
CH (A)
¾® 2KHSO4 + 4NaHSO4 + 2CrO2Cl2
chromyl chloride CH2 2NBS
Br
25. (d) –NO2 group is electron withdrawing, and
alc. KOH
hence III and IV are strong acids whereas – –2HBr
CH3 is electron releasing group, hence I is
Tetralin (C) Br
less acidic than the others. (B)
–2H2 Pd
26. (a) Fe(OH)3 is positive sol. K4[Fe(CN)6] will
provide [Fe(CN)6]4– for coagulation.

27. (b) Of the two possible carbocations, benzylic


type (II) is more stable than the I. Naphthalene

+
CH3CHCH2 OH ......(I) 30. (c) Emetal = (Wt of metal ´ 96500)/No of coulombs
(22.2 ´ 96500)
+ = » 59.5
CH3 CH2CH OH ......(II) (2 ´ 5 ´ 60 ´ 60)

177
Oxidation no. of metal = = +3
hence bromination occurs on 3rd C atom 59.5
PHYSICS
28. (a) When pH = 14 [H + ] = 10 -4 and [OH - ] = 1 M
31. (b) Wavelength of monochromatic green light
= 5.5 × 10–5 cm
Ksp = [Cu 2 + ] [OH - ]2 = 10 -19
Power
-19 Intensity I =
10 Area
\ [Cu 2+ ] = = 10 -19
- 2
[OH ] 100 ´ ( 3 /100 ) 3
= = Wm -2
4p ( 5 )
2 100 p
The half cell reaction
Now, half of this intensity (I) belongs to electric
Cu 2+ + 2e - ¾
¾® Cu field and half of that to magnetic field, therefore,
0 .059 1 I 1
E = E° – log = e0 E 02 C or E 0 = 2I
2 [ Cu 2+ ] 2 4 e0 C

0.059 1 æ 3 ö
= 0.34 – log -19 = – 0.22 V 2´ç p÷
2 10 è 100 ø
=
æ
ç
1 ö
9 ÷
è 4p ´ 9 ´10 ø
´ 3 ´108 ( )
Solutions-Mock Test-10 MT-239

6 k k
= ´ 30 = 7.2 MA1 = ( M + m) A2
25 M m+M
\ E 0 = 2.68 V / m
æ k ö
m1 \ çV = A M ÷
32. (a) For inelastic collision v ' < v è ø
(m1 ∗ m 2 )
1 v Q A1 M = A2 M + m
< v<
(1 ∗ 1) 2
n ↑ v(H) Before 1 A m+M
\ A = M
v 2
(n)(H) ↑ After
2 35. (b) The resultant gravitational force on each
Loss in K.E. particle provides the necessary centripetal
1 1 æ v ö2 1 force.
= mv 2 , (2m) çç ÷÷÷ < mv2
èç 2 ø
C
2 2 4
K.E. lost is used to jump from 1st orbit to fAC
2nd orbit L L
DK.E. = 10.2ev O
Minimum K.E. of neutron for inelastic F
collision m
A m D B
1
mv 2 < 2≥10.2 < 20.4 eV \ mv2/r = Ö (F2 + F2 + 2F2cos 60°) = Ö3 F
2
But r = (Ö3 L/2) ´ (2/3) = 1 / Ö3
33. (b) Acceleration when there is no friction is a =
\ mv2Ö3 / L = Ö3GM2 / L2 Þ v =Ö(GM/L)
gsinq and acceleration when friction is there is
36. (d) Let the charges on spheres of radii r and R
a¢ = (g sin q – mg cos q)
be q1 and q2 respectively. Then, Q = q1+q2.
1 2 The potential at the centre will be :
We know that s = ut + at , under
2
1 q1 q 2
condition of u = 0, gives, t = Ö(2s/a). V= ( + )
Therefore, t'/ t = Ö(a / a' ). 4pe 0 r R
For our case we get n = Ö[g sin q /(g sin q – mg As surface densities are equal, hence
cos q)]
Þ n 2 = g sin q /(g sin q - mg cos q) q1 q2 æ R2 ö
s= = or q 2 = q ç
1ç 2 ÷
÷
= 1/(1 – m cot q) 4pr 2 4p R 2 è r ø
But q = 45°
Þ cot q = 1 \ n2 = 1 /(1– m) or m = 1 – (1/n2) 1 q1 R 1 q
V= ( + q1 ) = ´ 1 (r + R )
34. (c) The net force becomes zero atthe mean 4pe o r r 2 4pe o r 2
point.
Therefore, linear momentum must be R2
Now, q1 + q2 = Q or q1 + q1 =Q
conserved. r2
\ Mv1 = (M + m)v2 q1 Q Q æ R+r ö
Þ 2
= 2 2
;V= çè 2 2 ÷ø
r R +r 4pe0 R +r
EBD_7206
MT-240 JEE MAIN

vR = v2 + v2 + 2v 2 cos q 8 3 9
37. (b) Þ 10 ´ : ´ 19 : 7 ´
10 10 10
v
qv Þ 8 : 5.7 : 6.3
q 41. (c) Consider a small element dx of radius r,
2R
r< x∗R
L
R

q
= 2v2 (1 + cos q) = 2v cos x
2
r
38. (c) Magnetic moment of the hydrogen atom, dx

when theelectron isin nth excited state, i.e.,


n ' = (n + 1)
L
As magnetic moment M n = In A = in (prn2 )

mz 2 e5 3R
i n = eVn =
4e02 n 3 h 3 Mg

At equilibrium change in length of the wire


n2h2 æ 1 ö
rn = 2 2 ç
k= ÷ 1
Mg dx
4p kzme è 4p Î0 ø
ò dL < ò é 2R ù2
Solving we get magnetic moment of the 0 pê x ∗ Rú y
hydrogen atom for n th excited state ëê L ûú
Taking limit from 0 to L
æ e ö nh
Mn ' = ç ÷
è 2m ø 2p
é ù
39. (c) Speed on reaching ground
u ê ú
ê ú
Mg ê 1 Lú MgL
v= u 2 + 2 gh ΧL < ê, ≥ ú<
p y ê é 2Rx ù L 2R ú 3pR 2 y
H ê ê ∗ Rú ú
ê ú
Now, v = u + at ë ëê L ûú 0 û

Þ u 2 + 2 gh = -u + gt The equilibrium extended length of wire =


L + DL
Time taken to reach highest point is
MgL æ 1 Mg ö÷
u < L∗ < L çç1 ∗ ÷
t= ,
3p R Y
2 çè 3 p YR 2 ø÷
g
42. (a) We know that
2
u + u + 2 gH nu T2 300 - 250 50 1
Þ t= = (from h = 1- = = =
g g T1 300 300 6
question)
Þ 2gH = n(n –2)u2 Q2 1 Q - 750
or h = 1 - Þ = 1
40. (c) Emissive power = aÎ Q1 6 Q1
a1 Î1 : a2 Î2 : a3Î3 Q1 = 6Q1 –4500
Þ –5Q1 = -4500 Þ Q1 = 900 Cal
Solutions-Mock Test-10 MT-241

43. (d) Kinetic energy of each molecule, dh


47. (b) Let the rate of falling water level be -
3 dt
K.E. = K B T
2 Initially at t = 0 ; h = h A
In the given problem, t=t;h=0
Temperature, T = 0°C = 273 K æ dh ö
Height attained by the gas molecule, h = ? Then, A ç - ÷ = pa 2 .v
è dt ø h

3 819K B A
K.E. = K B ( 273) = dt = - dh
a

2 2 2
pa 2 gh
K.E. = P.E. [Q velocity of efflux of
819K B liquid v = 2 gh ]
Þ = Mgh
2 Integrating both sides
819K B t 0
or h = A -1 2
ò dt = - òh dh
2Mg 0 2 g pa 2 h
hc 0
44. (c) In X-ray tube, l min = A é h1 2 ù
eV [t] t
0
=- . ê ú
2 g pa 2 ëê 1 2 ûú h
æ hc ö
In l min = In ç ÷ - InV 2A h
è eø t=
Clearly, log lmin versus log V graph pa 2 g
slope is negative hence option (c) correctly 48. (c) B due to AOB and COD are ^ to each
depicts. 2
®
other. Hence, net B = B12 + B 22
45. (a) E = E 0 iˆ + 2E 0 ˆj m 2 2 1/ 2
B = 0 ( I1 + I 2 ) ( ^ to plane ABCD)
Given, E 0 = 100N / c 2pa
®
So, E = 100iˆ + 200ˆj
Radius of circular surface = 0.02 m
2 22
Area = pr = ´ 0.02 ´ 0.02
7
-3
= 1.25 ´10 ˆi m [Loop is parallel to Y-Z
2

plane] 49. (a)


Now, flux (f) = EA cosq d dI NMI
E= ( NMI ) Þ E = NM Þ E =
( ) -3
= 100iˆ + 200ˆj .1.25 ´ 10 ˆi cos q° [q = 0°] dt dt T
= 125 × 10–3 Nm2/c E MI
Emf induced per unit turn = =
= 0.125 Nm2/c N t

46. (c) Let r be the internal resistance of the cell. 1 1


50. (d) At resonance Lw = , w=
Terminal potential difference (V) < EMF of Cw LC
a cell (E). E
Current through circuit i =
E - V æ E - Vö R
V = E – Ir or r = =ç R
I è V ÷ø Power dissipated at Resonance = i2R
EBD_7206
MT-242 JEE MAIN

51. (b) x = at 3 and y = bt 3 Þ 10 ´ 106 = 9(N max )1/ 2


2
dx dy æ 10 ´ 106 ö
vx = = 3at 2 and v y = = 3bt 2 Þ N max = ç 12 -3
÷ ; 1.2 ´ 10 m
dt dt ç 6 ÷
è ø
58. (c) After 8 sec., the counting rate falls to
\ v = v x2 + v2y = 9a 2t 4 + 9b2t 4
4
100 1 æ1ö
= = ç ÷ th Part ;
= 3t 2 a 2 + b2 1600 16 è 2 ø
52. (a) nlast = n first + ( n - 1) ´ d Time period =
8
sec. = 2 sec.
4
Þ 3n = n + ( 56 - 1) ´ 4 Therefore, after 6 seconds, the counting rate
3
or 2n = 55 ´ 4 Þ n = 110Hz
should be æç ö÷ th part of 1600, i.e.
1
53. (c) No. of electrons reaching the collector, è2ø
96 1
nC = ´ 1010 = 0.96 ´ 1010 ´ 1600 = 200 .
100 8
nE ´e 1
Emitter current, I E = 59. (a) U = CV2
t 2
From this, we get U µ V2
nC ´ e
Collector current, I C =
t
\ Current transfer ratio, U
U
I n
a= C = C =
0.96 ´ 1010
= 0.96
IE nE 1010
V2 V
l D 6.5 ´ 10 –7
54. (b) b = = ´ 1 = 0.65 × 10–3 = 60. (d) 30 Divisions of vernier scale coincide with
2d 10-3 29 divisions of main scales
0.65 mm 29
The distance between fifth bright fringe Therefore 1 V.S.D = MSD
30
from third dark fringe = 2.5 b = 2.5 × .65 = Least count = 1 MSD – 1VSD
1.63 mm. 29
55. (b) Graph [A] is for material used for making = 1 MSD - MSD
30
permanent magnets (high coercivity) 1
Graph [B] is for making electromagnets and = MSD
30
transformers.
1
56. (a) By displacement method, the size of object = ´ 0.5° = 1 minute.
30
is given by
MATHEMATICS
O= I1 ´ I 2 ;
n +4
61. (c) C r - (n C r +3.n C r -1 +3.n C r - 2 + n C r -3 )
\ A = A1 ´ A 2
57. (a) The critical frequency of a sky wave for = n + 4 C r - (n C r +n C r -1 ) + (n C r -1 + n C r -2 )
reflection from a layer of atmosphere is given
+ (n C r -1 + n C r - 2 ) + ( n C r - 2 +n C r -3 )
by f c = 9(N max )1/ 2
n+4
= C r - n+1C r +n+1C r-1+n+1C r-1+n+1C r-2
Solutions-Mock Test-10 MT-243

= n + 4 C r - n + 2 C r + n + 2 C r -1 Þ cos B - cos A = cosC - cosB


Þ 2cos B = cos A + cos C
= n + 4 C r - n +3 Cr =n +3 C r -1 Þ secA, secB, secC are in H.P.
62. (d) a -d + a + a + d =9 Þ a =3 66. (a) Any tangent to the parabola y2 = 4ax is
a
(a - d) 2 + a 2 + (a + d) 2 = 35 y = mx + ...(1)
m
Þ 18 + 2d 2 + 9 = 35 Þ d 2 = 4 Þ d = ±2 æ aö
meets x2 = 4by where x2 = 4b ç mx + ÷
n è m ø
S = ( 2a - 2d + nd - d )
2 or mx2 – 4bm2x – 4ab = 0 ...(2)
Taking d = –2, we get If the line (1) be a tangent to the second
(Q d = 2 will not give any option) parabola, then roots of (2) must be equal.
The condition for this is
n
S = [6 - 2n + 6] = 6n – n2 a
2 16b2m4 + 16abm = 0 or m3 = -
63. (d) Let a' = a/(a – 1), b' = b/(b – 1) b
Þ a = a'/(a'– 1), b = b'/(b' – 1) 1/ 3
æaö a1/ 3
Þ 1/a = (a' – 1)/a', 1/b = (b' – 1)/b' \ m= -ç ÷ =-
Then equation whose roots are 1/a, 1/b, is èbø b1 / 3
x2 – (1/a + 1/b)x + 1/ab = 0 Substituting this value of m in (1), we get
Þ x2 – [(a' – 1)/a' + (b' – 1)/b'] x + [(a' – 1) (b' –
a1/ 3 æ b1 / 3 ö
1)] / a'b' = 0 y= - x + aç- ÷
Þ a'b'x2 – [2a'b' – (a' + b')]x + a'b' – (a' + b') b1 / 3 ç a1 / 3 ÷
è ø
+1=0 i.e. xa1/3 + yb1/3 + a2/3 b2/3 = 0
Þ bx2 – (a + 2b)x + a + b + 1 = 0 which is the required equation of the
64. (b) f (q) = cos 2 (cos q) + sin 2 (sin q) common tangent.
67. (d) Let the point of contact be (h,k); equation
Now, put q = p /2 & 0
of chord of contact is T = 0
f (0) = cos 2 1 + 0 = cos 2 1 Þ xh – yk – 12 = 0 ………..(1)
Equation of common chord C1 – C2 = 0
f (p / 2) = 1 + sin 2 1
Þ 5x – 3y –10 = 0 ………. (2)
Since, sin 2 1 > cos 2 1 (1) and (2) represent the same line
Þ h/k = k/(–3)= –12/(–10)
p p p
[Q sin q > cos q " < q< &1> ] Þ h = 6.k = –18/5
4 2 4
2 68. (c) The equation of chord having (x1,y1) as its
Now, seeing the options 1 + sin 1 is greater
mid-point is
than all other options.
(xx1/a2)+ (yy1/b2 )– k = (x12/a2) + (y12/b2)
65. (c) tanA/2, tanB/2, tanC/2 are in A.P.
–k
Þ tanA/2 – tanB/2 = tanB/2 – tanC/2
Þ (xx1/a2) + (yy1/b2 ) = (x12/a2) + (y12/
sin A / 2 sin B / 2 sin B / 2 sin C / 2 b2)...(1)
Þ - = -
cos A / 2 cos B / 2 cos B / 2 cos C / 2 It makes equal intercepts on the axes when
Þ cos(C/2)sin(A/2 – B/2) = cosA/2 sin (B/ (a2/x1) = (b2/y1) Þ (x1/a2) – (y1/b2 ) =0
2 – C/2) Hence the locus of (x1, y1) is (x/a2) = (y/b2 )
EBD_7206
MT-244 JEE MAIN

69. (d) R = {(x, y) : x, y Î N and x2 – 4xy + 3y2 = 0}


é4 2 2ù
Now, x2 – 4xy + 3y2 = 0 -1 AdjA 1 ê ú
\A = = - 5 0 5ú
Þ (x – y) (x – 3y) = 0 | A | 10 ê
êë 1 - 2 3úû
\ x = y or x = 3y
\ R = {(1, 1), (3, 1), (2, 2), (6, 2), (3, 3), Comparing , we get a = 5
(9, 3),......}
(4 - 3sin x - 2 cos2 x )
1/(2 sin x -1)
72. (d) lim
Since (1, 1), (2, 2), (3, 3),...... are present in x® p / 6
the relation, therefore R is reflexive.
sin x -1
Since (3, 1) is an element of R but (1, 3) is é ù
not the element of R, therefore R is not lim ê {1 + (2sin x - 1)(sin x - 1)} ú = e -1/ 2
x® p / 6 ê 1 ú
symmetric êë (2sin x - 1)(sin x - 1) úû
Here (3, 1) Î R and (1, 1) Î R Þ (3, 1) Î R
(6, 2) Î R and (2, 2) Î R Þ (6, 2) Î R | x - 1|
73. (c) We have f (x) =
For all such (a, b) Î R and (b, c) Î R x2
Þ (a, c) Î R
Hence R is transitive. ì -x + 2
ïï 2 , x ³1
=í x
r r -1
70. (d) det (Mr) = = 2r - 1 ï 1- x , x < 1
r -1 r
ïî x 2
2007 2007
Clearly, f (x) is not differentiable at x = 0 and x = 1.
å det (M r ) = 2 å r - 2007 So, by definition, these are two of the critical
r =1 r =1
points. For points other than these two, we have
2007 ´ 2008
= 2´ - 2007 = (2007) 2 ì -x + 2
2
ï 3 , x >1
f ' (x) = ïí x
é1 - 1 1 ù ï 1- x , x < 1
ê ú ïî x 2
71. (a) A = ê2 1 - 3ú
ëê1 1 1 úû
Clearly, f ' (x) = 0 at x = 2. So x = 2 is also a critical
Cofactors of various entries are points. Hence, f (x) has three critical points, viz.
4, - 5, 1; 2, 0, - 2; 2, 5, 3 0, 1 and 2.

| A |= 1´ 4 + (-1) ´ -5 + 1´1 = 10 æ 3x 2 - 1 ö
74. (c) y = tan -1 ç ÷
ç 3x - x 3 ÷
è ø
é 4 -5 1 ù
Cofactor Matrix C = ê 2 0 -2ú p æ 3x - x 3 ö p
ê ú = - tan -1 ç ÷ = + 3 tan -1 x
êë 2 5 3 úû 2 ç 3x 2 - 1 ÷ 2
è ø
dy 3
é4 2 2ù =
T ê ú dx 1 + x 2
\ Adj A = C = ê- 5 0 5ú
êë 1 - 2 3úû æ x 2 -1 ö p æ 2 ö
u = sin -1 ç ÷ = - cos -1 ç x - 1 ÷
ç x2 +1÷ 2 ç x 2 +1 ÷
è ø è ø
Solutions-Mock Test-10 MT-245

p Similarly, one can show that


=- + 2 tan -1 x p 2m+1
2
ò 0 cos x dx = 0
du 2 78. (b) Since, f ' (x) = f (x), therefore, f (x) = aex.
=
dx 1 + x 2 Since, f (0) = 2, therefore, f (x) = 2ex.
dy dy du 3 ex
= = \ I = 2ò dx
dx du dx 2 3 + 8e x
Put ex = t, \ dt = ex dx
75. (a) Let f(x) = ax 2 + bx + c
dt 1
\ I = 2ò Þ I = log(3 + 8e x ) + C
ax3
bx 2 3 + 8t 4
g ( x) = ò f ( x ) = + + cx 79. (b) y = | cos x – sin x |
3 2
a b Y
\ g(1) = + + c = 2a + 3b + c = 0
3 2 f(x) = cos x g(x) = sin x
g(0) = 0 = g(1)
\ g(x) is continuous apply Rolle’s
theorem
Þ g( x) = 0 for some value x Î (0,1) X
O p/4 p/2
Þ g' (x ) = f ( x) = ax 2 + bx + c = 0
has a root in (0, 1)
76. (d) Let z1 = 1 + i and z2 = 1 – i
z2 1 - i (1 - i ) (1 - i ) p /4
= = =-i
z1 1 + i (1 + i ) (1 - i ) Required area = 2 ò (cos x - sin x ) dx
0
æz ö p/4
2 + 3ç 2 ÷ = 2 [ sin x + cos x ]0
2 z1 + 3z2 è z1 ø = 2 - 3i
=
2 z1 - 3z2 æ z ö 2 + 3i é 2 ù
2-3ç 2 ÷ =2ê - 1ú = (2 2 - 2) sq. units
è z1 ø ë 2 û
80. (a) Any vector (x, y, z) can be represented as a
2 z1 + 3z2 2 - 3i 2 - 3i linear combination of three non coplanar
= =
2 z1 - 3z2 2 + 3i 2 + 3i vectors.
r r r
OP = xa + yb + zc
é z1 | z |ù
êQ = 1 ú r r r r
ë z2 | z2 | û But P is (3, 2,1) Þ OP = 3a + 2b + c
4+9 Now if coordinate axis changes then the
= =1
4+9 vector is represented by
p r r r r r r r r r
77. (d) I = ò sin n x cos2m +1 x dx x(a + b + c ) + y (a - b + c ) + 2(a + b - c )
r r r
0
p = 3a + 2b + c
= ò sin n (p - x).cos2m+1 (p - x) dx
0 r r r
Comparing coeff of a, b & c , we get
p
= -ò sin n x cos 2m+1 x dx = -I Þ I = 0 x + y + z = 3, x - y + z = 2, x + y - z = 1
0
EBD_7206
MT-246 JEE MAIN

On solving, we get 83. (c) x + 2 = 6 Þ x = 4, Its converse is x = 4 Þ x


+ 2 = 6 i.e. if x = 4 then 4 + 2 = 6
3 1
x= , y = ,z =1
2 2 84. (a) y
Hence the new co-ordinates of point P
æ3 1 ö
are ç , ,1÷
è2 2 ø
81. (c) If the given lines are coplanar, then a line
which is normal to the plane in which they
x
lie, will be perpendicular to all the three
given lines. Let the direction cosines of the Since, x1 < x 2 Þ f (x1) < f (x 2 ) if
normal be l, m, u.
\ ll1 + mm1 + un1 = 0, ll2 + mm2 + un2 = x1 ,x 2 ,---,x n are in ascending order
0, Þ f(x1 ),f(x 2 ),---f(x n ) are also in
ll3 + mm3 + un3 = 0. Eliminating l, m, u from
ascending order.
these equations,
If M is the median along x, then f (M) is the
l1 m1 n1 median along y
we get, l 2 m2 n2 = 0 . 85. (a) The required event occurs if two sixes are
observed in the first seven throws and a six
l3 m3 n3 is observed on the eighth throw. If p is the
82. (b) Let OP be the normal from probability that a six shows on the die, the
number of throws n is 7, and X is the number
O to the plane of times a six is observed, then X ~ B(7,p).
of DABC, let l, Therefore the required probability equals
m, n be the
direction P(X = 2) times the probability of getting a
cosines of OP six on the eighth throw, i.e., it equals
Let s be the 2 5
area of DABC. æ1ö æ5ö æ1ö
Therefore, ( 7 C2 p2 q5 )(p) = (7 C2 ) ç ÷ ç ÷ ç ÷
è6ø è6ø è6ø
OAC = Projection of DABC on zx plane =
s.m. 7
C2 (55 )
But OAC is a right angled triangle, so that =
68
1
area of DOAC = a.c. 86. (b) Let us consider
2 r
a = a1$i + a2 $j + a3kˆ
1 ac
\ ac = s.m. Þ m = . Similarly, in DOBC, r
2 2s b = b1$i + b2 $j + b3kˆ
r
l=
bc
and in D OAB, n =
ab c = c1i$ + c2 $j + c3 kˆ
2s 2s then as per question
Since, l2 + m2 + n2 = 1 r r r r
é l ar + b l 2 b lcr ù = é ar b + cr b ù
( )
2 2
æ ac ö æ bc ö æ ab ö
2 ë û ë û
\ ç ÷ +ç ÷ +ç ÷ =1
è 2s ø è 2s ø è 2s ø
l ( a1 + b1 ) l ( a2 + b2 ) l ( a3 + b3 )
1
Þs= b 2 c 2 + c 2a 2 + a 2 b 2 Þ l 2 b1 l 2 b2 l 2b3
2
lc1 lc2 lc3
Solutions-Mock Test-10 MT-247

a1 a2 a3 1 5
Þ P( A) + = [from (i)]
= b1 + c1 b 2 + c2 b3 + c3 6P ( A ) 6
b1 b2 b3 Þ 6 [P(A)]2 – 5P (A) + 1 = 0
Þ (2P (A) – 1) (3P (A) – 1) = 0
a1 + b1 a2 + b2 a3 + b3 1 1
4
Þ P(A) = ,
Þl b1 b2 b3 2 3
c1 c2 c3
æ 1ö 1 1
88. (b) B ç 5, ÷ Þ n = 5, p = , q =
è 2ø 2 2
a1 a2 a3
= b1 + c1 b2 + c2 b3 + c3 æ 1ö 1 1
B ç 7, ÷ Þ n = 7, p = , q =
è 2ø 2 2
b1 b2 b3
Since X and Y are independent events
( R1 - R2 ), ( R2 - R3 ) ÞX+Y=3
Þ X = 0, Y = 3; X = 1, Y = 2; X = 2, Y = 1; X
a1 a2 a3 a1 a2 a3 = 0, Y = 3
Þ l 4 b1 b2 b3 = c1 c2 c3 \ P (X + Y = 3)
c1 c2 c3 b1 b2 b3 5 7 5 7
æ 1ö 7 æ1ö 5 æ1ö 7 æ1ö
= 5C0 çè ÷ø . C3 çè ÷ø + C1 çè ÷ø C2 ç ÷
è 2ø
2 2 2
Þ l 4 = -1
Hence l has no real values. æ 1ö
5
æ1ö
7
æ1ö
5
87. (d) Since A and B are two independent events +5 C2 ç ÷ 7
C1 ç ÷ + 5C3 ç ÷
è 2ø è2ø è2ø
\ P(A Ç B) = P(A) P(B)
7
7 æ1ö 55
1 C0 ç ÷ = .
Þ P (A) P(B) = è2ø 1024
6
89. (c) We know that,
(given) ...(i)
and P (neither of A nor B) = 1 1 1
tan -1 + tan -1 + tan -1 + + ......
1 1+ 2 1 + 2´ 3 1 + 3´ 4
P (A È B) =
3
1 1
tan -1 + tan -1 + ...... +
1 2 1 + (n - 1)n 1 + n(n + 1)
Þ P ( A È B) = 1 - P ( A È B) = 1 - =
3 3
1 n + 19
We know that tan -1 = tan -1
1 + (n + 19) (n + 20) n + 21
P(A È B) = P(A) + P(B) - P(A Ç B)

2 1
Þ = P ( A ) + P ( B) - n -1 1 1
3 6 Þtan -1 + tan -1 + tan -1
n +1 1 + n(n + 1) 1 + ( n + 1) ( n + 2)

2 1 5
Þ P(A) + P(B) = + = 1 n + 19
3 6 6 + ...... + = tan -1
1 + (n + 19) (n + 20) n + 21
EBD_7206
MT-248 JEE MAIN

1 1 90. (d) Given expansion can be written as


Þ tan -1 + tan -1 + ...... +
1 + n(n + 1) 1 + (n + 1)(n + 2) n
æ x -1 ö n
ç x ÷ .(1 - x ) = (–1) x (1 – x)
n –n 2n
1 -1 n + 19 n -1 è ø
= tan - tan -1
1 + ( n + 19) ( n + 20) n + 21 n +1
Total number of terms will be 2n + 1 which
æ 1 ö -1 æ 1 ö is odd ( Q 2n is always even)
Þ tan-1 ç 2 ÷ + tan ç 2 ÷ + ...... +
è n + n +1ø è n + 3n + 3 ø 2n + 1 + 1
\ Middle term = = (n + 1) th
1 2
tan -1
1 + ( n + 19) ( n + 20)
Now, Tr +1 = nCr (1)r x n - r

æ n + 19 n - 1 ö
-1 ç n + 21 - n + 1 ÷ 2n
Cn × x 2 n - n
= tan ç ÷ So, = 2 n Cn .( -1)n
çç 1 + n + 19 ´ n - 1 ÷÷ n
x .( -1) n
è n + 21 n + 1 ø
Middle term is an odd term. So, n + 1 will
20
= tan -1 =S be odd.
n2 + 20n + 1 So, n will be even.
20 \ Required answer is 2nCn.
-1
\ tan S = 2
n + 20n + 1

You might also like